You are on page 1of 343

Integral

Kokeboken
4
3
2
1

sin(x )
2 2
dx
=
4
sinh2(x)

1
+ log(log x) dx = x log(log x) + C
log x
Z

cos(x2) + sin(x2) dx =

X
xs1
1
dx
=
(s)
= (s)(s)
s
ex 1
n
n=1

log
0

(1 + sin x)1+cos x
1 + cos x

dx = 0

Del I
.
.
.
.
.
.

.
.
.
.
.
.

.
.
.
.
.
.

.
.
.
.
.
.

.
.
.
.
.
.

.
.
.
.
.
.

.
.
.
.
.
.

.
.
.
.
.
.

.
.
.
.
.
.

.
.
.
.
.
.

.
.
.
.
.
.

.
.
.
.
.
.

.
.
.
.
.
.

.
.
.
.
.
.

.
.
.
.
.
.

.
.
.
.
.
.

1
2
4
6
6
6
7

2.1 Introduksjon II . . . . . . . . . . . . .
2.2 Bevis . . . . . . . . . . . . . . . . . . .
2.2.1 Den antideriverte . . . . . . . .
2.2.2 Delvis integrasjon . . . . . . . .
2.2.3 Substitusjon . . . . . . . . . . .
2.3 Symmetri og nyttige sammenhenger .
2.4 Periodiske funksjoner . . . . . . . . . .
2.5 Diverse substitusjoner . . . . . . . . .
2.5.1 Weierstrass substitusjon . . . .
2.5.2 Euler substitusjon . . . . . . . .
2.6 Brker og kvadratrtter . . . . . . . .
2.6.1 Brker . . . . . . . . . . . . . .
2.6.2 Kvadratrtter . . . . . . . . . .
2.7 Delvis integrasjon . . . . . . . . . . . .
2.7.1 Delvis kanselering . . . . . . .
2.7.2 Eksponentialfunksjonen . . . .
2.7.3 Tabell og reduksjonsformler . .
2.8 Trigonometrske funksjoner . . . . . . .
2.9 Logaritmiske funksjoner . . . . . . . .
2.10 Ulike tips og knep . . . . . . . . . . . .
2.10.1 Rekursjoner og funksjonsflger
2.10.2 Nyttig funksjonallikning . . . .
2.10.3 Integral par . . . . . . . . . . .
2.11 Oppgavesammling II . . . . . . . . . .
2.11.1 Integral . . . . . . . . . . . . .
2.11.2 Oppgaver . . . . . . . . . . . .

.
.
.
.
.
.
.
.
.
.
.
.
.
.
.
.
.
.
.
.
.
.
.
.
.
.

.
.
.
.
.
.
.
.
.
.
.
.
.
.
.
.
.
.
.
.
.
.
.
.
.
.

.
.
.
.
.
.
.
.
.
.
.
.
.
.
.
.
.
.
.
.
.
.
.
.
.
.

.
.
.
.
.
.
.
.
.
.
.
.
.
.
.
.
.
.
.
.
.
.
.
.
.
.

.
.
.
.
.
.
.
.
.
.
.
.
.
.
.
.
.
.
.
.
.
.
.
.
.
.

.
.
.
.
.
.
.
.
.
.
.
.
.
.
.
.
.
.
.
.
.
.
.
.
.
.

.
.
.
.
.
.
.
.
.
.
.
.
.
.
.
.
.
.
.
.
.
.
.
.
.
.

.
.
.
.
.
.
.
.
.
.
.
.
.
.
.
.
.
.
.
.
.
.
.
.
.
.

.
.
.
.
.
.
.
.
.
.
.
.
.
.
.
.
.
.
.
.
.
.
.
.
.
.

.
.
.
.
.
.
.
.
.
.
.
.
.
.
.
.
.
.
.
.
.
.
.
.
.
.

.
.
.
.
.
.
.
.
.
.
.
.
.
.
.
.
.
.
.
.
.
.
.
.
.
.

.
.
.
.
.
.
.
.
.
.
.
.
.
.
.
.
.
.
.
.
.
.
.
.
.
.

.
.
.
.
.
.
.
.
.
.
.
.
.
.
.
.
.
.
.
.
.
.
.
.
.
.

.
.
.
.
.
.
.
.
.
.
.
.
.
.
.
.
.
.
.
.
.
.
.
.
.
.

.
.
.
.
.
.
.
.
.
.
.
.
.
.
.
.
.
.
.
.
.
.
.
.
.
.

11
12
13
13
15
17
18
30
40
47
50
53
53
56
60
61
63
65
69
76
80
80
84
88
90
90
91

.
.
.
.
.
.
.
.
.
.
.
.
.

.
.
.
.
.
.
.
.
.
.
.
.
.

.
.
.
.
.
.
.
.
.
.
.
.
.

.
.
.
.
.
.
.
.
.
.
.
.
.

.
.
.
.
.
.
.
.
.
.
.
.
.

.
.
.
.
.
.
.
.
.
.
.
.
.

.
.
.
.
.
.
.
.
.
.
.
.
.

.
.
.
.
.
.
.
.
.
.
.
.
.

.
.
.
.
.
.
.
.
.
.
.
.
.

.
.
.
.
.
.
.
.
.
.
.
.
.

.
.
.
.
.
.
.
.
.
.
.
.
.

.
.
.
.
.
.
.
.
.
.
.
.
.

.
.
.
.
.
.
.
.
.
.
.
.
.

.
.
.
.
.
.
.
.
.
.
.
.
.

.
.
.
.
.
.
.
.
.
.
.
.
.

103
104
105
105
106
107
108
108
111
121
125
129
129
131

1.1 Brk . . . . . . . . . . . . .
1.2 Trigonometriske funksjoner
1.3 Oppgavesammling I . . . .
1.3.1 Integral . . . . . . .
1.3.2 Integral . . . . . . .
1.3.3 Oppgaver . . . . . .

.
.
.
.
.
.

.
.
.
.
.
.

.
.
.
.
.
.

.
.
.
.
.
.

.
.
.
.
.
.

Del II

Del III
3.1 Introduksjon III . . . . . . . . . . . .
3.2 Viktige konstanter . . . . . . . . . .
3.2.1 EulerMascheroni konstanten
3.2.2 Catalans Konstant . . . . . .
3.2.3 GlaisherKinkelin konstanten
3.3 Spesialfunksjoner . . . . . . . . . . .
3.3.1 Gulv og tak-funksjonene . . .
3.3.2 Gammafunksjonen . . . . . .
3.3.3 Betafunksjonen . . . . . . . .
3.3.4 Digamma-funksjonen . . . . .
3.3.5 Polygamma-funksjonen . . .
3.3.6 Riemann zeta funksjonen . .
3.3.7 Hurwitz zeta function . . . .

.
.
.
.
.
.
.
.
.
.
.
.
.

ii

3.4

3.5

3.6
3.7
3.8
3.9
3.10

3.3.8 Polylogaritmen . . . . . . . . . . . .
3.3.9 Dilogaritmen . . . . . . . . . . . . .
3.3.10 Dilogaritmen . . . . . . . . . . . . .
3.3.11 Elliptiske Integral . . . . . . . . . . .
Transformasjoner . . . . . . . . . . . . . . .
3.4.1 Laplace transformasjonen . . . . . .
3.4.2 Fourier-transformasjon . . . . . . . .
3.4.3 Mellin transformasjonen . . . . . . .
3.4.4 Landens transformasjon . . . . . . .
3.4.5 Cauchy-Schl
omilch transformasjonen
3.4.6 Diverse transformasjoner . . . . . .
Diverse applikasjoner . . . . . . . . . . . . .
3.5.1 Gulv og tak-funksjoner . . . . . . . .
3.5.2 Itererte integral . . . . . . . . . . . .
Derivasjon under integraltegnet . . . . . . .
Uendelige rekker . . . . . . . . . . . . . . .
Dobbel Integraler . . . . . . . . . . . . . . .
Kompleks Integrasjon . . . . . . . . . . . . .
3.9.1 Typer integraler . . . . . . . . . . . .
Oppgavesammling III . . . . . . . . . . . . .
3.10.1 Integral . . . . . . . . . . . . . . . .
3.10.2 Oppgaver . . . . . . . . . . . . . . .

.
.
.
.
.
.
.
.
.
.
.
.
.
.
.
.
.
.
.
.
.
.

.
.
.
.
.
.
.
.
.
.
.
.
.
.
.
.
.
.
.
.
.
.

.
.
.
.
.
.
.
.
.
.
.
.
.
.
.
.
.
.
.
.
.
.

.
.
.
.
.
.
.
.
.
.
.
.
.
.
.
.
.
.
.
.
.
.

.
.
.
.
.
.
.
.
.
.
.
.
.
.
.
.
.
.
.
.
.
.

.
.
.
.
.
.
.
.
.
.
.
.
.
.
.
.
.
.
.
.
.
.

.
.
.
.
.
.
.
.
.
.
.
.
.
.
.
.
.
.
.
.
.
.

.
.
.
.
.
.
.
.
.
.
.
.
.
.
.
.
.
.
.
.
.
.

.
.
.
.
.
.
.
.
.
.
.
.
.
.
.
.
.
.
.
.
.
.

.
.
.
.
.
.
.
.
.
.
.
.
.
.
.
.
.
.
.
.
.
.

.
.
.
.
.
.
.
.
.
.
.
.
.
.
.
.
.
.
.
.
.
.

.
.
.
.
.
.
.
.
.
.
.
.
.
.
.
.
.
.
.
.
.
.

131
132
135
136
140
140
146
147
147
149
152
155
156
156
157
164
167
176
177
194
194
194

Konvergens . . . . . . . . . . .
analysens fundamentaltheorem.
Konvergens . . . . . . . . . . .
Funksjonalanalyse . . . . . . .
Bohr-Mullerup Theoremet . . .

.
.
.
.
.

.
.
.
.
.

.
.
.
.
.

.
.
.
.
.

.
.
.
.
.

.
.
.
.
.

.
.
.
.
.

.
.
.
.
.

.
.
.
.
.

.
.
.
.
.

.
.
.
.
.

.
.
.
.
.

201
203
211
215
223
226

Del IV
A.1
B.1
C.1
C.2
C.3

Del V

.
.
.
.
.

.
.
.
.
.

.
.
.
.
.

.
.
.
.
.

.
.
.
.
.

.
.
.
.
.

.
.
.
.
.

229
5.1 Kortsvar . . . . . . . . . . . . . . . . . . . . . . . . . . . . . . . . 230
5.30 Langsvar . . . . . . . . . . . . . . . . . . . . . . . . . . . . . . . . 238

1.1

1 Brk

BRK

trekke sammen eller forTidligere har du mtt oppgaver som ber deg om a
enkle brk-uttrykk
1x
x
1x x1
x x+1
3x 1
+
=
+
= 2
1+x
x1
1+x x1
x1 x+1
x 1
g
dele eller spalte brken? Som ett
Hva om vi nsker a
a andre veien? Alts
aa
vise at
eksempel kan du se om du klarer a
x2

2
1
1
=

1
x1
x+1

trekke sammen hyresiden. 1 Det finnes mange m


trekke sammen
uten a
ater a
brker p
a og for en enda grunndigere gjennomgang kan kalkulus anbefales eller

BESTEMTE

INTEGRAL KAN VRE LUNEFULLE

Ett gjennomg
aende tema i dette heftet er at integrasjon er en kunst og i
del II skal vi se at bestemte integral kan bli beregnet p
a forunderlige m
ater.
prve ut flere metoder for a
bestemme ett integral.
Oppfordringen fr des blir a
Selv om du ser en lsning som du vet vil virke, ta en pause. Tenk. Er det noen
andre metoder som kan virke, er de lettere, mer elegant enn min? Ett eksempel
er flgende integral
Z
I=
1

4x
dx
x2 1

ha lest forrig
Flere vil etter a
aende seksjon begynne med delbrksoppspalting.
Flott! Men hva om teller hadde vrt x2 + 1, hva kunne en gjort da? Svaret
er jo substitusjon, og detaljene skal leser f
a kose seg med alene. Ett litt tffere
eksempel som vi kommer tilbake til senere er flgende
Eksempel 1.1.1. Bestem verdien av flgende integral
Z
1
I=
dx
x(1
+
x2 )
1
En ser ikke noen umidelbar smart substitusjon s
a en naturlig vei er delbrksoppspalting.
Det finnes mange m
ater, men for eksempel
1
1 + x2 x2
1
x
=
=

2
x(1 + x )
x(1 + x2 )
x
1 + x2
bestemme integralet av siste del, bruker vi
Integrasjonen blir n
a enklere. For a
substitusjon
Z
Z
1
x
1
2x dx
1
I=

dx
=
log
x

= log x log(1 + x2 ) + C
2
2
x
1+x
2
1+x
2
1 Som

liten frsteklasse tass p


a gymnaset var jeg dypt interessert i temaet og holdt p
a i flere uker
dele brker og lage regler. Det var frst det p
ret at skolen begynte med teorien.
med a
aflgende a

1 Brk

gjre noen sm
Dette uttrykket er rett. Men vi velgere a
a algebraiske omskrivninger ved hjelp av logaritmereglene log a + log b = log ab. Grunnen til dette vil
snart bli klar2




Z
1
1
1
x2
1
=
+C
dx =
log
log 1
x(1 + x2 )
2
1 + x2
2
1 + x2
Hvor det blant annet ble brukt at log x = 21 log x2 og
1
sette inn grensene v
1 1+x
are f
ar vi endelig
2 . Ved a
Z

x2
1+x2

(x2 +1)1
1+x2


 
1
1
1
dx
=
log
1

x(1 + x2 )
2
1 + x2
1




1
1
1
1
log 1
=
lim log 1
2 x
1 + x2
2
1+1

Siden 1/x 0 n
ar x s
a vil 1/(1 + x2 ) 0 n
ar x . Alts
a g
ar frste del
mot log 1 = 0.
Z
1
1
1
1
1
I=
dx =
log(1 0) log
=
log 2
2)
x(1
+
x
2
2
2
2
1
og vi er ferdige. I siste overgang brukte vi at log(1/2) = log 1 log 2 = log 2.
Finnes det en enklere metode? Ja, heldigvis. Men den krever et bedre falkeye
se. Vi begynner i stedet med den noe uvante substitusjonen y 7 1/x. Dett
for a
er det samme som at x 7 1/y slik at dx = dy/y 2 . N
ar x s
a vil y 0
siden y = 1/x. Og tilsvarende x > 1 gir y = 1/x = 1. Dermed s
a f
as
Z
1

1
dx =
x(1 + x2 )

Z
1

dy
1
1
=
1/y(1 + (1/y)2 ) y 2
2

Z
0

2y
dy
1 + y2

bruke en a
penbar substitusjon. Tanken er at selv om
Problemet lses s
a ved a
det ikke ser ut som en metode vil fre frem, kan det vre lurt a gi den en sjangse
like vell. Substitusjonen x 7 1/y kalles gjerne den inverse substitusjonen, eller
en bijectiv substitusjon. Den er mye brukt p
a bestemte integral med grenser 1
og .

2 Prv

fortsette a
beregne integralene uten disse algebraiske krumspringene. Hva skjer?
a

1.2

2 Trigonometriske funksjoner

TRIGONOMETRISKE FUNKSJONER

For en diskusjon om Rieman-integrerbarhet og ulike typer integraler henvises


leses til Appendix 1. Her blir og temaet om konvergens av integraler studert
gjennomg
aende.
Proposisjon 1.2.1. Vi har at (arctan x)0 =
t

Z
0

1
1+x2

og

1
dx = arctan t
1 + x2

(1.1)

for alle t 0. Videre s


a er
Z
0

1
dx =
1 + x2

1/t

1
dx = arctan t
1 + x2

(1.2)

Bevis. Vi beviser frste del av proposisjonen mens siste del overlates til leser,
se oppgave. Derivasjonen av arctan x kan vises ved implisitt derivasjon. La
y = arctan x, da er tan y = x. Derivasjon gir
d
d
tan y =
x
dx
dx
d
dy
(tan y)
=1
dy
dx
 dy
1 + (tan y)2
=1
dx
1
dy
=
dx
1 + (tan y)2
d
1
arctan x =
dx
1 + x2
d
d dy
=
. I siste linje kan
dx
dy dx
f
vi benytte oss av at y = arctan x og tan y = x for a
a det nskede resultatet.
Derivasjonen av tan x kan for eksempel fres
I andre linje ble kjerneregelen benyttet, alts
a at

dy sin x
(sin x)0 cos x sin x (cos x)0
(sin x)2
(cos x)2
dy
tan x =
=
=
+
dx
dx cos x
(cos x)2
(cos x)2
(cos x)2
Som viser at (tan x)0 = 1 + (tan x)2 , siden sin x/ cos x = tan x per definisjon.
integrere (arctan x)0 = 1/(1 + x2 ) fra null til t f
Ved a
as
t

Z
0

dx
=
1 + x2

Z
0

h
it
(arctan x)0 dx = arctan x = arctan t
0

Som flger fra at siden tan 0 = 0 s


a er arctan 0 = 0. Alternativt kunne vi integrert
uttryket via substitusjonen x = tan y. Da blir

d
= (tan y)0 dx = 1 + tan2 (y) dy = (1 + x2 ) dy
dx

2 Trigonometriske funksjoner

Alts
a er dx/(1 + x2 ) = dy. Grensene blir arctan 0 = 0 og arctan t. Slik at
integralet kan skrives som
t

Z
0

1
dx =
1 + x2

arctan t

dy = arctan t
0

Dette fullfrer frste del av beviset. Siste delen av beviset overlates til leser, se
oppgave 4.
Korollar 1.2.1.
Z
Z

Z
1

1
dx =
dx =
2
2
1+x
1+x
4
Z1
1
1

dx =
dx =
1 + x2
1 + x2
2
0

Bevis. Frste overgang faller direkte ut av likning (1.2), med t = 1. Da har vi


Z

1/1

1
dx =
1 + x2

1
dx = arctan 1 =
2
1+x
4

Andre overgang flger fra likning (1.1). Verdien av arctan 1 vet vi, siden tan /4 =
1 s
a er arctan 1 = /4.

Oppgaver
1. a) Bevis siste halvdel av proposisjon (1.2.1), alts
a likning (1.2).
b) Vis videre at integralene under
t

Z
0

log x
dx =
1 + x2

1/t

log x
dx ,
1 + x2

(1.3)

holder for alle t > 0. Bestem flgende integral


Z
log x
dx ,
1
+ x2
0
benytte deg av likning (1.3).
ved a
Z
2. Bruk substitusjonen u 7 tan x til a bestemme I =

tan x)n+2 + tann x dx

Bestem integralet
Z

/4

tan9 x + 5 tan7 x + 5 tan5 x + tan3 x dx

bruke resultatet ovenfor.


ved a

3 Oppgavesammling I

1.3

O P P G AV E S A M M L I N G I

1.3.1

INTEGRAL

1.3.2

INTEGRAL
x2 + 3x
dx
x2

Z
1.

sin(x) dx
Z

dx dx
x2

18.
1

x
dx
x2 + 1

3.
0

4.
3

Z
6.

20.
Z
21.

4 x dx

1/2

Z
7.

(2x 1)

50

dx

Z
23.

Z
8.

x dx

Z
24.

1
dx
x ln x

9.

Z
25.

Z
10.

sin x cos x dx

e1

Z
12.

Z
x
dx
ex

13.
14.

en

ln x dx n R

15.

x a dx

3/4

dx
+ 1 x2


ln 1 + x2 x dx

x2
dx
x2 + 4x + 8

1
dx
x ln(x)n

42.

x3 ex dx

ex+e dx

ln(x)3 dx

46.

30.

x2

e1

41.

45.

x + 2x + 2
dx
x1

ey dx

29.

dx

x2

x2
dx
1

44.

sin x cos x dx
Z

39.

43.

dx

sin 2x
dx
sin x

27.

28.

x2

x1
dx 40.
x2 + 2x + 7

/4

loge (a)2

ex + 1
dx
ex 1


ln 1 + x2 x dx

11.

dx
x (1 + x )

38.

1+e
x
dx
e +x

26.

sin(1/x)e

x!

dx


x+1
dx
x

37.

ln x
dx
x

22.

Z 
36.

1
dx
1+ x

1
x

2x ex dx

35.

xex
dx
(x + 1)2

1
dx
1x

5.


x ln

x ln(x + 1) dx
0

x
dx
x+1

19.

dx

sin(2x)esin(x) dx

33.
34.

ln 2

ln

/6

sin(x) dx
Z

Z
32.

17.

Z
2.

/3

x+1
dx
(x + 2)4

47.

(m x)(x n) dx
n

Z
16.

log10 (x) dx

31.

p
x3 1 x2 dx

Z
48.

e2

dx
16

3.3 Oppgaver
Z

Z
50.
Z

x 2
dx
x4

57.

sin(2x)
dx
sin x

58.

49.

/3n

51.
0

4a
dx
x2 a2

52.
Z

54.
Z

/2

55.
/6

Z
56.

1.3.3

1
dx
tan x

x2 + 1
dx
x(x2 + 3)

Z
67.

xn ln x dx

Z
68.

log5 (x)
dx
log25 (x)

Z
69.

24
dx
x4 x2

62.

dx
2x ln x

66.

2ln x dx

xn ln x dx

x2 sin(x)
dx
2
0
Z q

60.
x + x dx
61.

65.

eln(x +1)
dx
x+1

59.

esin(x) ecos(x) dx

53.

1
dx
x +x

1
dx
x ln x x

tan(nx) dx
Z

x2
Z

70.

Z
63.

cos(sin(x)) cos x dx

Z
71.

Z
64.

x+x


2

x2 x
dx
2x + 1
ex/2
dx
1 + ex

 1
ln x 2

ln(x2 )

dx

e3(x+ln 3) dx
Z 5
x x+1
dx
72.
x3 + 1

O P P G AV E R

1. Vis at integralet
Z

a+2

K=

sin x + 1 dx ,
a

er konstant dersom a er et reellt tall. Kan resultatet forklares geometrisk?


2. La D betgne omr
adet avgrenset av linjene y = sin x, x = sin y og y = 2+x.
Bestem arealet av D Hint: En god tegning gjr ofte susen
3. Vi har flgende integral
ea

Z
I=
1

1
dx ,
x log x + ax

hvor a er et reellt tall. Drft integralet for tilfellene a < 0, a = 0 og a > 0,


4. Bestem flgende integral
r

Z
1+

x1
x+1

!2
dx

gange ut parentesen.
Hint: Integralet kan lses b
ade med og uten a

3 Oppgavesammling I
5. Vis at
Z


22 x2 1 e2x dx = e2 e2

hvor og henholdsvis er minste og strste lsning av x(x 1) = 1.


R a x 2
dx = 12
6. Finn ett reelt tall a som tilfredstiller 0 e +e
2
5
Z x
log t
7. For x > 0, la f (x) =
dt. Finn funksjonen f (x) + f (1/x) og vis at
1 1+t
f (e) + f (1/e) = 1/2.
Z 2
Z
Z
r2
8. Bestem flgende integral
d
sin d
2 dr
(1 + r2 )
0
0
0
9.
10. La f (x) vre en funksjon slik at
1) f (x) er integrerbar for x [0, 3]
Z 1
2)
f (x) dx = 1.
0

3) f (x) =

1
2

f (x) for alle x [0, 3]. Bestem integralet


3

f (x) dx
0

11. Det oppgis at


Z

dx
=
og
x2 + a2
a

dx
(x2

2
a2 )

2a3

bestemme
hvor a > 0 er en positiv konstant. Bruk dette til a
Z
x2
2 dx .
2
(x + 1)
Integralene ovenfor kan fritt benyttes, og det oppgis og at integralet ovenfor konvergerer.
12. Vis at integralet
Z
0

f (x)
x

er invariant (er det samme) om f byttes ut med f (cx), hvor c er en konstant.


se p
13. Ved a
a integralet mellom f (x) = sin x og g(x) = cos x kan vi f
a et
omr
adet som likner p
a en bart (se 345). Bestem arealet av barten.

14. M
alet med denne oppgaven er a bevise hva som er strst av log 2 og 1/ 2 .
Og de er nrmere enn hva enn skulle tro ved frste yenkast.

3.3 Oppgaver

a) Vis at ulikheten
2
1+ 2
1
2 >
1
+
t
t+1+ 2
holder for alle 0 < t < 1.
b) Den geometriske aritmetriske ulikheten er definert som
xy

> xy
log x log y
studere fider x > 0. Bevis ulikheten, Her kan det vre fordelaktig a
gur (1.1).

1/x

ab

x
b

Figur 1.1: Den aritmetriske geometriske ulikheten

benytte
c) Bestem hva som er strst av log 2 og 1/ 2 . En st
ar fritt til a
tidligere oppgaver.

gi leser en pustepause og for a


la
Denne siden er med hensikt blank, for a
forfatter sl
ass mot dinosaurer.

II

12

II

1 Introduksjon II

2.1

INTRODUKSJON II

I denne delen vil vi hovedsaklig dykke dypere ned i tidligere beskte temaer,
men ogs
a lre noen nye eksotiske teknikker for a beregne spesielt h
arete integral.
Det som forventes av leseren p
a dette tidspunktet er en grundig forst
aelse for
de elementre integrasjonsteknikkene som ble gjennomg
att i del I. For a kunne
lse mange av integralene som kommer videre m
a disse elementre teknikkene
sitte helt ut til fingerspissene.
Hvor i forrige del fleste integralene kunne bli lst via et steg En smart
substitusjon, en delbrkoppspalting, en frekk faktorisering osv vil vi se frem bli lst. Mange av lsningene vil
mover at integralene krever flere steg for a
virke merkelige og noen vil virke som de er tatt rett ut av det bl
a. Men husk at
bak hvert skritt som frer mot en lsning ligger et stort maskineri.
Ofte er grunnen til at vi bruker en s
a merkelig substitusjon eller delvis inte ha sett
grasjon enkel. Det fungerer. Fremmgangsm
aten blir en blanding av a
liknende problem fr, og prve en rekke velkjente triks og knep. Og det er ogs
a
m
alet med denne delen; og lre bort alle disse merkelige substitusjonene og
beregne integral, slik at du selv kan anvende dem. For etter a

metodene for a
ha sett det samme smarte trikset nok ganger, blir det ikke lengre et triks men en
nyttig teknikk du selv kan bruke.
se studere n
Et nkkelkonsept i denne delen er a
ar en kan gjre flgende
omskrivning
Z
Z
Z
f (x) dx =
g(x) dx a f (x) dx .
S

Metoden for a komme fra venstre side til hyre side av likningen er ikke spesielt
viktig. Det kan vre en lur substitusjon, en brutal delvis integrasjon eller noe
helt annet. Poenget er at likningen ovenfor kan lses med hensyn p
a integralet
over S 1 , en beregner alts
a integralet indirekte. I mange tilfeller vil dette og
vre eneste metoden som fungerer da det eksisterer funksjoner som ikke har
lse integraelementre antideriverte2 . I denne delen blir ogs
a andre m
ater a
anvende symmetri, til smart bruk av delvis
ler inderekte p
a studert, alt fra a
integrasjon.

1 S
a
2 En

fremt a 6= 1, da ender en opp med at integralet over D er null.


R
sier gjerne at en funksjon har en elementr antiderivert dersom f dx kan uttrykkes ved elemen-

Rb

tre funksjoner. Selv om en funksjon ikke har en elementr antiderivert kan likevel
f (x) dx
a
eksistere og vre elementr. Dette er noe som skaper hodebry for mange matematikkere.

II

2.1 Den antideriverte

2.2

13

BEVIS

I denne delen skal vi se nrmere p


a noen geometriske bevis for b
ade delvis
integrasjon, og hvorfor den antideriverte gir oss arealet under en funksjon. Tilsutt bevises ogs
a substitusjon mer formelt. Det formelle beviset for analysens
fundamentaltheorem spares til appendixet, da det er noe langt. For den ivrige
leser se theorem (B.1.1).

2.2.1

DEN

ANTIDERIVERTE

a F (x) + C
Lemma 2.2.1. Anta at F 0 (x) = f (x) da vil ogs
en vilk
arlig konstant.

0

= f (x) hvor C er

a [a, b],
Theorem 2.2.1. La f vre en kontinuerlig funksjon som tar reelle verdier p
og la F vre den antideriverte til f for x [a, b]. Da er
Z

f (t) dt = F (b) F (a) .


a

Vi venter som sagt med et formelt bevis for FTC, men i korte trekk kan det
formelle beviset kokes ned til flgende fire linjer
F (b) F (a) =
=
=

n
X
k=1
n
X
k=1
n
X
k=1
Z b

[F (xk ) F (xk1 )]

Teleskoperende sum

(2.1)

F 0 (xk )[xk xk1 ]

Middel verdi theoremet

(2.2)

Riemann sum

(2.3)

F 0 (xk )xk
F 0 (x) dx

(2.4)

Merk at theorem (2.2.1) krever at f er begrenset og kontinuerlig p


a intervallet.
Dette er den klassiske definisjonen av en Rieman-integrerbar funksjon. En kan
utvide integralbegrepet til langt mer ville funksjoner, men da holder ikke lengre
det p
aflgende beviset vann. Beviset under med andre ord langt mer en intuisjon
enn et formelt bevis.
Bevis. Dette beviset tar utgangspunkt i figur (2.2) og baserer seg p
a visuell fremstilling. Anta at det eksisterer en eller annen funksjon A(y) som beskriver arealet
under f (x) fra et punkt a til ett eller annet punkt y > a. Det rde omr
adet i figur(2.1) er eksempelvis A(b). N
a utvider vi omr
adet litt med en faktor h. Arealet
av det bl
a omr
adet kan da skrives som
bl
a = A(b + h) A(b) .
Ut i fra figur (2.1) s
a vil det bl
a omr
adet alltid ligge mellom det grnne rektangelet og det oransje. Dette kan beskrives ved flgende ulikhet
h f (b) A(b + h) A(b) h f (h + b) .

14

II

2 Bevis
y

f (h + b)
f (b)

A(b)
x
a

b+h

Figur 2.1: Grafen til en vilk


arlig funksjon f (x)
Siden arealet av et rektangel er lik grunnlinjen ganget med hyden. Grunnlinjen
her blir G = (h + b) b = h, og hyden er f (x). Trikset er n
a at siden h > 0 s
a
kan en fritt dele ulikheten p
ah
f (b)

A(b + h) A(b)
f (h + b) .
h

(2.5)

Videre lar en h 0. Visuelt kan en se p


a dette som at vi gjr det bl
a omr
adet
mindre og mindre. Helt tilsvarende som n
ar en utledet formelen for den deriverte.
lim f (b) lim

h0

h0

A(b + h) A(b)
lim f (h + b) .
h0
h

se pent ut. Det midterste leddet kan en kjenne igjen som


Dette begynner a
definisjonen av den deriverte
f 0 (x) = lim

h0

f (x + h) f (x)
.
h

Frste leddet blir f (b), og siste leddet blir f (b). Da har en alts
a skvist A0 (x)
f (b) A0 (b) f (b) .
Dette medfrer at A(x) er en antiderivert til f (x)! Siden A0 (x) = f (x), s
a betyr
det at A(x) = F (x) hvor F 0 (x) = f (x). Funksjonen A kan dermed skrives som
A(b) = F (b) + C ,
hvor C er en eller annen konstant lemma (2.2.1). V nsker n
a a bestemme denne
ukjente konstanten. Merk at per definisjon s
a er A(a) = 0, slik at
0 = F (a) + C C = F (a) .

II

2.2 Delvis integrasjon

15

Arealet under funksjonen f (x) fra x = a til x = b kan da skrives som


A(b) = F (b) F (a) ,
skrive dette som
og som kjent er det mer vanlig a
Z

f (x) dx = F (b) F (a) .


a

Merk at beviset tar utgangspunkt i at f (x) er stigende p


a intervalet, hadde funksjonen vrt konstant kunne en brukt akkuratt samme argument. Hadde funksjonen i stedet vrt synkende ville eneste forskjell vrt at ulikhetstegnene i
likning (2.5) vrt snudd.
Beviset her krever at f er en kontinuerlig, men at funksjonen er strengt positiv
er ikke ndvendig. Med litt mer spissfindighet kan en fre et liknende argument
for en funksjon som er negativ. Integrasjon holder i langt mer generelle former
dekke alle spesialtilfeller, men gi a
en viss
enn dette, men poenget er her ikke a
intuisjon om hvordan derivasjon og integrasjon henger sammen.

2.2.2

D E LV I S

INTEGRASJON

Formelen for delvis integrasjon er gitt under


Z
Z
u dv = uv u du
integrere produktregelen
Tidligere har vi utledet denne formelen ved a
(uv)0 = u0 v + uv 0
utlede formelen ovenfor p
I denne delen skal vi se p
a en mer visuell m
ate a
a,
uten bruk av derivasjon. Forh
apentligvis vil denne metoden vre mer intuitiv, i
vre en alternativ fremgangsm
tillegg til a
ate.
uttrykke arealet av det
Beviset tar utgangspunkt i figur (2.2) og ide`en er a
gr
a omr
adet p
a to ulike m
ater. P
a den ene siden kan arealet under u(v), fra A
til bli beskrevet ved flgende integral
Z v2
ABB 0 A0 =
u dv
v1

Tilsvarende s
a kan ogs
a arealet begrenset av punktene ABB 0 A0 bli beskrevet
av den inverse funksjonen v(u) langs u-aksen som
Z u2
CBB 0 C 0 =
v du
u1

Arealet av det gr
a omr
adet er likt summen av CBB 0 C 0 og ABB 0 A0 . P
a
den andre siden kan det gr
a omr
adet - mer formelt polygonet avgrenset av
CBAA0 B 0 C 0 - uttrykkes som differansen mellom OA0 B 0 C 0 og OABC.
OABC = u1 v1 og OA0 B 0 C 0 = u2 v2

16

II

2 Bevis
v
u2

u1

u(v)

B0

C
B

A0

A
v1

v2

Figur 2.2: En vilk


arlig funksjon i uv-koordinatsystemet

ta differansen s
Ved a
a kan det gr
a omr
adet uttrykkes som
h iu2 ,v2
CBAA0 B 0 C 0 = OA0 B 0 C 0 OABC = u2 v2 u1 v1 = uv
u1 ,v1

sammenlikne de to uttrykkene vi har for det gr


Ved a
a omr
adet s
a er
h

uv

iu2 ,v2
u1 ,v1

v2

u2

u dv +

v du

v1

(2.6)

u1

som ikke er noe annet en den kjente formelen for delvis integrasjon med leddene
stokket om. Noen vil kanskje stusse p
a grensene ovenfor, men husk at b
ade u
og v begge er funksjoner av en underliggende variabel si x. Spesifikt s
a er
u(x1 ) = u1 , v(x1 ) = v1 og u(x2 ) = v2 , v(x2 ) = v2 . S
a likning (2.6) kan skrives
Z

x=x2

x=x1

h ix=x2 Z
u dv = uv

x=x1

x=x2

v du .

(2.7)

x=x1

som er akkuratt formen for delvis integrasjon med grenser. Merk at beviset tok
utgangspunkt i at funksjonen u(v) var en-til-en eller bijektiv.
Dette er ikke noe ndvendig krav, som vi s
a n
ar vi utledet formelen tidligere.
Uten for mye bevis s
a kan beviset ovenfor utvides til kurver som ikke er en-tilen, s
a lenge u og v er funksjoner kun avhengig av en underliggende variabel.
Integralene vil fortsatt kunne beregnes, men n
a vil noen av omr
adene vre
negative.

II

2.2.3

2.3 Substitusjon

17

SUBSTITUSJON

I starten n
ar en lrer substitusjon vil en ofte missbruke notasjonen. Vi skriver
du
gjerne u = f (x)
= f 0 (x) du = f 0 (x) dx. Men hva skjer egentlig i
dx
den siste overgangen? Vi arbeider jo tross alt med differensialer og ikke brker.
Husk p
a at substitusjon egentlig bare er kjerneregelen baklengs. Akkuratt
som delvis integrasjon essensielt er produktregelen baklengs. Essensen av kjerneregelen er
dy
dy du
=
,
dx
du dx
Grunnen til at vi foretrekker Leibniz notasjonen og skriver deriverte som forhold
er at notasjonen er veldig hintende (suggestive). Ofte n
ar ting ser ut som de
kanselerer s
a gjr de virkelig det. Ofte oppfrer disse differensialene seg som
brker for eksempel dy/ dx = 1/ dx/dy. Men a rettferdigjre at en kan behandle
differensialer som brker er en egen gren innenfor matematikk som kalles for
infinitesmal kalkulus.
La du/ dx = u0 (x), substitusjon kan da skrives direkte som
Z
Z
Z
Z
du
g(u(x))u0 (x)dx = g(u(x))
dx = g(u) du = g(t) dt
dx
Hvor det er den andre overgangen om m
a rettferdigjres. Det som ser ut som
en kanselering av dx i andre overgang er direkte knyttet til kjerneregelen. Nota understreke kanseleringen.
sjonen er med andre ord valgt for a
Theorem 2.2.2. La I = [a, b] R vre et intervall og la u(x) vre en til en
funksjon samt kontinuerlig deriverbar. Anta at g er kontinuerlig p
a I, da er
Z b
Z u(b)
g(u(x))u0 (x) dx =
g(t) dt .
a

u(a)

Bevis. Vi definer funksjonen F som flger


Z x
F (x) =
g(t) dt
0

Vi studerer s
a funksjonen F (u(x)) og dens deriverte.
dF du
F (u(x))0 = F 0 (u(x))u0 (x) =
du dx
Z u(x)
d
=
g(u(t)) dt u0 (x) = g(u(x))u0 (x)
du 0
Hvor siste overgang flger fra frste del av analysens fundamentaltheorem
integerere F (u(x))0 = g(u(x))u0 (x) fra a til b s
(B.1.1). Ved a
a har vi
Z b
Z b
du
dx.
F (u(b)) F (u(a)) =
g(u(x))u0 (x) dx =
g(u(t))
dx
a
a
Selvsagt vet vi ogs
a at
Z

u(b)

F (u(b)) F (u(a)) =
u(a)

og dette fullfrer beviset.

u(b)

g(t) dt =

g(u) du
u(a)

18

II

3 Symmetri og nyttige sammenhenger

2.3

SYMMETRI OG NYTTIGE SAMMENHENGER

se p
M
alet med denne delen er a
a hvordan integral og funksjoner kan forenkles ved hjelp av symmetri. Det frste vi skal se p
a er definisjonen av en
symmetrisk funksjon.
Definisjon 2.3.1. En funksjon f kalles symmetrisk omkring c dersom

f (x c) = f (x c)
En funksjon f kalles anti-symmetrisk omkring c dersom

f (x c) = f (x c)
Dette m
a selvsagt holde for alle x. Dersom det bare holder for x C hvor
C = [a, b] er et endelig interval, s
a sier en at f er henholdsvis odde eller like
over C
Definisjon 2.3.2. En funksjon f kalles for en likefunksjon eller jevnfunksjon
dersom funksjonen er symmetrisk omkring origo.
f (x) = f (x)
En funksjon f kalles odde eller for en oddefunksjon dersom funksjonen er antisymmetrisk omkring origo.
f (x) = f (x)
Som igjen m
a holde for alle x.
En grafisk fremstilling av odde og like funksjoner er vist i figur (2.3). Ek2
sempelvis s
a er x2 , cos x, ex og |x| like funksjoner. Tilsvarende s
a er x3 , sin x,
2
xex og x eksempel p
a odde funksjoner.
En sammlebetegnelse for odde og like funksjoner er paritet. S
a pariteten
2
2
til ex er like, partiteten til xex er odde, og ex har ingen paritet. Odde og

(a) Et plot av to odde funksjoner

(b) Et plot av to like funksjoner

Figur 2.3: Pictures of animals


like funksjoner har mange nyttige egenskaper hvor de viktigste egenskapene er

II

3 Symmetri og nyttige sammenhenger

19

sammlet i tabell (2.1). Her er O en forkortelse for en odde funksjon, og L er en


forkortelse for en likefunksjon. Videre s
a er E enten odde eller like, og E har
motsatt paritet av E. S
a dersom E er like s
a er E odde. Uttrykket E E = O
betyr det samme
a er
 som at O L = O og L O = O. Gitt to funksjoner f og g s
f g = f g(x) .
Tabell 2.1: Noen egenskaper til odde og like funksjoner.
E+E
E/E
E E
EL

=
=
=
=

EE
E/E
OO
LE

E
L
O
L

=
=
=
=

L
O
O
L

Fra figur (2.3) virker det som arealet av en likefunksjon er likt p


a hyre og
venstre side av origo. Tilsvarende for en oddefunksjon, bare at omr
adene n
a har
motsatt fortegn.
Dette stemmer faktisk, men fr vi viser det la oss ta et litt annet eksempel
med hensyn p
a symmetri.
Eksempel 2.3.1. Se p
a funksjonen
f (x) = x
vises at arealet under f (x) og f (a x) grafisk. Alts
Det nskes a
a
Z a
Z a
f (x) dx =
f (a x) dx
0

Ved a se p
a figur figur(2.4) s
a er det rimelig apenbart at omr
adet under funksjo-

x
Figur 2.4: f (x) og f (a x) i samme figur.
nene er like. Arealet under x fra 0 til a er en trekant med grunnlinje a og hyde
a dermed s
a er
Z a
a2
x dx =
2
0
Tilsvarende for a x er dette og en trekant med grunnlinje a og hyde a s
a
Z a
2
a
x dx =
2
0

20

3 Symmetri og nyttige sammenhenger

II

Eksemepelet ovenfor var veldig konstruert og pedantisk. Men det stiller et


viktig sprsm
al, gjelder dette for alle funksjoner? Svaret er heldigvis ja, og kan
generaliseres til flgende proposisjon
Proposisjon 2.3.1. Anta at f er en vilk
arlig funksjon, og a,b er to reelle tall da er
Z b
Z b
f (a + b x) dx =
f (x) dx .
a

Bevis. Igjen kan dette visualiserer med at f blir rotert omkring linjen x = (a +
se p
a
b)/2 og det er derfor logisk at arealet er uforandret. Begynner med a
venstre side av likheten
Z b
f (x) dx
a

bruke substitusjonen x 7 a + b u s
Ved a
a er du = dx. Videre er x = a
u = b, og x = d u = a s
a
Z b
Z a
Z b
f (x) dx =
f (a + b u) dx =
f (a + b u) du
a

bytte integrasjonsvariabelen tilbake til x.


og beviset fullfres ved a
Dette enkle beviset frer direkte til at
Korollar 2.3.1. Gitt at f er en symmetrisk funksjon omkring c, g er en antisymmetrisk funksjon omkring c da er
Z c+a
Z c+a
Z c+a
f (x) dx = 2
f (x) dx og
g(x) dx = 0 dx ,
(2.8)
ca

ca

der at a er en positiv, reell konstant. Spesielt dersom c = 0 s


a er
Z a
Z a
Z a
f (x) dx = 2
f (x) dx og
g(x) dx = 0 dx ,
a

(2.9)

gitt at f er en likefunksjon, og g er odde.


Som det overlates leser a vise. Her er det nok a putte inn i proposisjon (2.3.1)
og benytte seg av definisjonen av en odde og like-funksjon. Dette er og svrt
logisk om en betrakter figur (2.3).
Langt i fra alle funksjoner har symmetriske egenskaper, for eksempel s
a er ex
hverken like eller odde. Men
ex =

ex + ex
ex ex
+
= sinh x + cosh x
2
2

Slik at ex kan skrives som en odde og en likefunksjon. Faktisk s


a kan enhver
funksjon f skrives som en en odde og en likefunksjon
f (x) =

f (x) f (x)
f (x) + f (x)
+
2
2

(2.10)

Hvor frste del er en like, og andre del en odde. Fra denne observasjonen kan
en vise

II

3 Symmetri og nyttige sammenhenger

21

Proposisjon 2.3.2. La f vre en vilk


arlig funksjon da er
Z a
Z a
f (x) + f (x) dx
f (x) dx =
a

Bevis. Legg merke til at


Z a
Z a
Z a
f (x) + f (x)
f (x) f (x)
=
f (x) + f (x) og
=0
2
2
a
0
a
Dette flger fra korollar (2.3.1) siden frste frste funksjon er like og andre
integrere likning (2.10) fra a til a f
funksjon er odde. Ved a
as da
Z a
Z a
f (x) dx =
f (x) + f (x) dx ,
a

som nsket. Alternativt kan en og dele integralet p


a midten og benytte substitusjonen x 7 y i integralet fra a til 0.
Proposisjon 2.3.3. La f vre en integrerbar funksjon p
a intervalet [0, 2a], hvor
a R er en positiv konstant. Da holder
Z 2a
Z a
Z a
f (x) dx =
f (x) dx +
f (2a x) dx
0

for alle funksjoner f .


Korollar 2.3.2. La f vre en vilk
arlig funksjon da er
Z a
Z 2a

2 f (x) dx hvis f (2a x) = f (x)


f (x) dx =
0

0
0
hvis f (2a x) = f (x)
hvor a er reell konstant.
Bevis. Beviset flger direkte fra proposisjon (2.3.3) og nesten rett fra korollar (2.3.1) med et snedig skifte av koordinater. Geometrisk beskriver integralet
en funksjon som er odde eller like omkring x = a. Vi deler frst integralet p
a
midten og bruker deretter substitusjonen x 7 2a u p
a det siste integralet.
Z 2a
Z a
Z 2a
f (x) dx =
f (x) dx +
f (x) dx
0

2a2a

f (2a u) du ,

f (x) dx +

=
0

2aa

hvor grensene blir u = 2a 2a = 0 og u = 2a a = a. Bruker vi n


a at
Ra
Rb
f
(x)
dx
=

f
(x)
dx
kan
integralet
skrives
som
b
a
Z

2a

f (x) + f (2a x) dx .

f (x) dx =

(2.11)

sette inn henholdsvis f (2a x) = f (x) og


Dette beviser proposisjon (2.3.3). A
f (2a x) = f (x) fullfrer beviset for korollar (2.3.2). Som vi skal se snart
flger likning (2.11) ogs
a rett fra (2.13) i lemma (2.3.1).

22

II

3 Symmetri og nyttige sammenhenger


se hvor nyttig symmetriegenskaper kan vre
La oss ta et eksempel for a

Eksempel 2.3.2. Putnam 1987: B1


p
Z 4
log(9 x) dx
p
p
J=
log(9 x) + log(3 + x)
2
N
a er a + b x = 6 x s
a ved a benytte seg av proposisjon (2.3.1) f
ar en direkte
p
p
Z 4
Z 4
log(9 (6 x)) dx
log(3 + x) dx
p
p
p
p
=
log(9 (6 x)) + log(3 + (6 x))
log(3 + x) + log(9 x)
2
2
ta gjennomsnittet av uttrykkene f
Meget pent. Ved a
ar en at
p
p
Z
log(9 x)
log(3 + x)
1 4
p
p
p
+p
dx
J=
2 2
log(9 x) + log(x + 3)
log(x + 3) + log(9 x)
Dermed forenkler integranden dramatisk og en st
ar igjen med
p
Z 4p
Z
log(9 x) + log(x + 3)
1
1 4
(4 2)
p
p
J=
dx =
=1
1 dx =
2 2
2
2
log(9 x) + log(x + 3)
2
Merk at teknikken som ble brukt her kan enkelt generaliseres til flgende to
korollar
Korollar 2.3.3. La f (x) vre en vilkr
alig funksjon p
a (a, b) da er
Z b
f (x)
ba
dx =
,
f
(a
+
b

x)
+
f
(x)
2
a
gitt at a og b er to reelle konstanter og at integralet konvergerer.
For a sikre seg konvergens holder det at f (a + b x) + f (x) 6= 0 p
a intervalet.
Beviset er i seg selv enkelt, og en kan bruke nyaktig samme fremgangsm
ate
vise, men i praksis er
som i eksempelet ovenfor. I seg selv er korollaret enkelt a
bruke. For eksempel s
det vanskeligere a
a er
Z 1
Z
1
x3 + x + 1
x3 + x + 1
1
dx =
dx =
.
2
3
3
2
0 4 3x + 3x
0 [(x 1) + (x 1) + 1] + x + x + 1
Men a se denne overgangen krever at en har en liten algebra-trollmann i magen.
tippe at denne teknikken fungerer og se om theoSom regel er det enklere a
lse
rem (2.3.1) kan benyttes. P
a enkle integrander kan korollaret brukes til a
integraler ved inspeksjon.
Lemma 2.3.1.
Z

f (x) dx =
a

1
2

f (x) + f (a + b x) dx

(2.12)

f (x) + f (a + b x) dx

(2.13)

f (x) + f (a + b x) dx

(2.14)

=
a

Z
=

Gitt at a,b er to reelle tall og c = (a + b)/2 er gjennomsnittsverdien av a og b.

II

3 Symmetri og nyttige sammenhenger

23

Bevis. Frste likning faller rett ut fra (2.3.1), siden integralet over f (x) og f (a +
se neste overgang kan en snitte integralet i to
b x) er like store. For a
b

Z
f (x) dx =

Z
f (x) dx +

(2.15)

f (x) dx ,
c

med c = (a + b)/2. Ved a bruke substitusjonen x 7 a + b u p


a siste integral f
as
b

f (x) dx

f (x) dx =
a

Zac

f (a + b u) du
c

f (x) + f (a + b x) dx .

=
a

Grensene blir u = (a + b) c = c og u = (a + b) b = a, og en byttet tilbake til x


som integrasjonsvariabel i siste linje. Dersom en i stedet bruker substitusjonen
x 7 a + b u p
a frste integral i (2.15) f
ar en
Z

f (x) dx =
a

Z
f (a + b x) dx +

f (x) dx
c

f (a + b x) + f (x) dx .

=
c

som var den siste likningen som skulle vises.


kjre trigonometriske funksjonene v
La oss rette fokus mot de kjente a
are.
Vi vet allerede at

2
Z 2
Z 2
2 sin(2x)
1 cos(2x)
dx =
=
sin2 (x) dx =
2
4
0
0
0
Der det ble benyttet at sin2 x cos2 x = cos 2x, og enhetsformelen cos2 x +
sin2 x = 1. Dette er et integral som dukker mye opp b
ade i fysikk og flervariabel
analyse, men hva med integralet av cos2 (x), kan vi bruke resultatet p
a noen
se p
m
ate? Ved a
a figur (2.5) s
a er cos2 x og sin2 x samme funksjon, bare at
cosinus er forskjvet med en halv periode alts
a


sin
x = cos x
2
Der virker da logisk at arealet under sinus og cosinus er det samme, gitt at en
integrerer over en hel periode. Dette viste du vel allerede fra fr, men ved a sette
inn i proposisjon (2.3.1) f
as direkte at
Z

/2

/2

cos(x)2 dx

sin(/2 x) dx =

sin(x) dx =
0

/2

Interessant, siden arealet under av funksjonene er like s


a er
Z
0

/2

sin(x)2 =

1
2

Z
0

/2

sin(x)2 + cos(x)2 dx =

1
2

/2

1 dx =
0

24

II

3 Symmetri og nyttige sammenhenger

x
/2

3/2

Figur 2.5: Her er cos x den stipplede funksjonen, og sin x er den heltrukne.

1 y
x
0

3
2

Figur 2.6: f (x) og f (a x) i samme figur.


Stilig! Teknikken fungerer ikke bare p
a alle intervall, men alle multipler av /2.
Z
0

n/2

sin(x)2 =

Z
0

n/2

cos(x)2 =

1
2

n/2

1 dx =
0

n
4

Men dette er kanskje kjent fra fr? La oss angripe problemet p


a en mer grafisk
m
ate. Ved a se nrmere p
a intervalet 0 til . Legg merke til symmetrien fra x = 0
til x = /2 i figur (2.6). Funksjonen f (x) = sin(x)2 deler kvadratet avgrenset av
y = 0, y = 1 og x = 0, x = /2 i nyaktig to p
a figuren. Slik at arealet under f
er nyaktig halvparten av arealet til kvadratet. Alts
a
Z /2
1 

sin2 x dx =
=
2 2
4
0
Dette er helt tilsvarende som a bestemme arealet av en trekant, den er nyaktig
halvparten av kvadratet.
f
For a
a samme resultat som fr kan en dele opp integralet i n kvadrat hver
med areal /2, og nyaktig det samme argumentet kan og benyttes p
a cos(x)2 .
Hittil har det bare vrt h
andvifting, hvordan kan en vre sikker p
a at fremgangsm
aten er rett? Svaret finner vi i flgende theorem
Theorem 2.3.1. Anta f er kontinuerlig p
a [a, b] og at f (x) + f (a + b x) er
konstant for alle x [a, b] da er


Z b

ba
a+b
=
f (a) + f (b)
f (x) = (b a)f
2
2
a

II

3 Symmetri og nyttige sammenhenger

25

Bevis. Beviset her tilegnes Roger Nelsen [? , s39-41], og er mildt sagt pent. Merk
at teoremet kan vises utelukkende ved a betrakte figur(2.7). Men litt algebratre-

f (a) + f (b)

f (a)

a+b

f (x)

f (b)

x
a

(a + b)/2

Figur 2.7: Bevis for teoremet uten ord.


beregne integralet benyttes lemma (2.3.1) og
ning har aldri skadet noen. For a
likning (2.13) Da er


Z b
Z c
b+a
a K
f (x) dx =
f (x) + f (a + b x) dx =
2
a
a
Siden f (x)+f (a+bx) = K, hvor K er en konstant. For a bestemme konstanten
kan hvilken som helst x [a, b] benyttes. Velges x = a eller x = b f
as
K = f (a) + f (b)
Derimot om en velger f (x) = c f
as

K = 2f

a+b
2

Integralet kan dermed skrives som




Z b

ba
a+b
f (x) = (b a)f
=
f (a) + f (b)
2
2
a
som var det som skulle vises.
En ser at om f (x) = sin2 x eller f (x) = cos2 x s
a er f (x) + f (/2 x) = 1
konstant s
a
Z /2
Z /2

/2 0 

f (0) + f (/2) =
sin2 x dx =
cos2 x dx =
2
4
0
0
som stemmer med tidligere resultater. La oss ta et noe vanskeligere eksempel
avslutningsvis

26

II

3 Symmetri og nyttige sammenhenger

Eksempel 2.3.3. Putnam 1987: B1


/2

dx

1 + tan(x)

Dette er en av de vanskeligste oppgavene som har vrt gitt p


a en Putnam eksa tegne funksjonen f
men3 . Ved a
ar en noe som likner p
a figur (2.8). Allerde n
a

y
1
0.8
0.6
0.4
0.2
0

x
0

/4

/2

Figur 2.8: Grafen til funksjonen 1/ tan(x)

burde en kjenne igjenn teknikken. Omr


adet under funksjonen er halvparten av
rektangelet. Slik at det forventes at
Z

/2

dx

1 + tan(x)

1 

=
.
2 2
4

vre sikre p
For a
a dette sjekkes det at f (x) f (a + b x) er konstant. N
a er
tan(/2 x) =

sin(/2 x)
cos x
=
= 1/ tan(x)
cos(/2 x)
sin x

slik at

f (x) + f (/2 a) =
=
3 Putnam

1 + tan(x)
1

1 + tan(x)

+
+

1
1

+ 1/ tan(x) 2

tan(x) 2

+ tan(x) 2

tan(x)

tan(x)

er en den mest prestisjefylte matematikkonkuransen for studenter i USA og Kanada. Konkurransen best
ar av 12 sprsm
al, med maks poengsum 120. Oppgaven som er vist her klarte bare
f
23 av 2043 deltakare a
a 3 eller mer poeng p
a

II

3 Symmetri og nyttige sammenhenger

27

Dermed s
a er f (x) + f (a + b x) konstant for alle x [0, /2]. Arealet under
f (x) er dermed nyaktig halvparten av kvadratet med hyde 1 og bredde /2.
Fra theorem (2.3.1) har en alts
a at


Z /2
dx
/2 0

f (0) + lim f (x) =


.
=
2
2
4
x/2
1
+
tan(x)
0
Der f (0) = 1 og den siste grensen blir 0, siden tan(x) g
ar mot uendelig n
ar
x /2 dermed vil 1/ tan x 0.
I oppgaven ovenfor var figuren en god indeksjon p
a symmetrien. La oss
avslutningsvis ta et eksempel som i utgangspunktet virker motstriende mot tidligere resultater.
Eksempel 2.3.4. Her skal vi studere flgende integral
Z
A=

/2 p

1 + cos2 x dx

La som definere funksjonen f som


f (x) =

1 + cos2 x

Ved a se p
a figur(2.9) ser det ut som at integralet fyller opp nyaktig halvparten
av det stiplede omr
adet. S
a antakelsen om at arealet er

1.5

0.5

x
0

/4

/2

Figur 2.9: Den rde kurven viser funksjonen f (x) = 1 + cos2 x .

f (0) + f (/2)
A =
2


1+ 2
=
4

virker ikke urimelig. Dessverre s


a er
g(x) = f (x) + f ( x) =

1 + cos2 x +

1 + sin2 x

28

3 Symmetri og nyttige sammenhenger

II

se p
ikke konstant p
a intervallet. Ved a
a den deriverte kan en se at funksjonen

duver sakte
i
intervalet,
eksempelvis
s
a er g(0) = g(/2) = 1 + 2 , mens

vre konstant, men det


g(/4) = 6 . S
a f (x) f ( x) er akk s
a nre a
hjelper dessverre ingenting. Ved direkte utregning s
a er faktisk

Z /2 p
1+ 2
2
1 + cos x dx 1.9100 >
1.8961
4
0
S
a selv om det ser ut som funksjonen har pen symmetri, m
a en teste om f (x) +
2
f (a+bx) er konstant. Problemet
i
denne
oppgaven
er
at
1+cos
(x) tilfredstiller

f (x) = f (a + b x), mens x ikke gjr det. En konvolusjon av en symmetrisk


og ikke symmetrisk funksjon er dessverre ikke symmetrisk.

Addendum: Integralet som ble studert i denne oppgaven ikke er mulig a

beregne analytisk og betegnes som et elliptisk integral av andre grad. Men a


vise dette overlates som en artig oppgave til leser.
Oppgaver
1. La vre en reell konstant, og f (x) en kontinuerlig funksjon. Hvilke
egenskaper m
a f ha for at
Z
Z
f (x) dx
=
f (x) dx
x
1 +
0
skal gjelde? Bestem ogs
a
Z

dx
(1 +

x2 )(1

+ ex )

2. Vis at
Z


log

(1 + sin x)1+cos x
1 + cos x


dx = 0 ,

3. Bestem konstantene A,B,C og D slik at


Z
Z
Z 1
Z
dx
dx
dx
dx
=
A
=
B
=
C
.
2
2
2
1
+
x
1
+
x
1
+
x
1
+
x2
1
0
0

4.

Z 2p
p
x2 x + 1 x2 3x + 3 dx
0

5. Flgende transformasjon
Z
Z

xR(sin x, cos2 x) dx =
R(sin x, cos2 x) dx
2
0
0
er nyttig i flere sammenhenger. Vis at den stemmer der R(x) er en rasjonell funksjon uten singulariter p
a intervalet. Gjelder identiteten for
R(cos x, sin2 x) ?

II

3 Symmetri og nyttige sammenhenger

29

6. Et komplett elliptisk integral av andre grad kan skrives p


a fomen
Z
E(k) =

/2 p

1 k 2 sin2 x dx

a) Vis at integralet
Z

/2 p

1 + cos2 x dx

kan uttrykkes via E(x).


beregne et
La oss ta en liten digresjon til numerisk integrasjon. For a
integral kan en dele opp intervalet i n deler og konstruere et trapes p
a
hver del. Da ender en opp med at
Z
T (n) =

f (x) dx =
a

n1
X

h
f (a) + f (b) + h
f (xk )
2
k=1

hvor n er antall interval, h = (b a)/n, og xk = a + hk. En vre grense


for feilen i metoden er gitt som
0 Error

(b a)3
M
2n2

Hvor M er den strste verdien |f 00 ()| har p


a intervallet.
vre helt sikker p
b) Bestem hvor mange interval n som trengs for a
a
at

Z /2 p
(1 + 2 )
1 + cos2 x dx >
4
0

30

2.4

II

4 Periodiske funksjoner

PERIODISKE FUNKSJONER

I forrige seksjon ble det sett nrmere p


a integralene av sin x, og cos x og begge to er eksempler p
a periodiske funksjoner. En enkel definisjon p
a en periodisk
funksjon er flgende
Definisjon 2.4.1. La f (x) vre en funksjon, og la T vre en konstant. Dersom
f (x + T ) = f (x)
holder for alle x, s
a kalles f for periodisk. En sier ogs
a at f har en periode p
a T.
Alts
a dersom funksjonen gjentar seg selv, ved jevne interval s
a er funksjonen
periodisk. Noen ganger kalles periodiske funksjoner ogs
a for sykliske funksjoner.
merk at periodiske funksjoner ikke trenger a oppfre seg like pent som sin x eller

y
t

Figur 2.10: Illustrasjon av en periodisk funksjon


cos x. Fra definisjonen ovenfor flger det at 1 har periode . Siden f (x) = 1, s
a
er f (x + ) = 1 for alle x.
Faktisk s
a har konstante funksjoner, alle perioder4 .
Proposisjon 2.4.1. Dersom f (x) = f (x + p) og g(x) = g(x + p) for alle x, hvor
p 6= 0, og
F (x) = f (x) + g(x) og

G(x) = f (x)g(x)

(2.16)

da er b
ade har b
ade F og G periode p.
beregne F (x + p) og G(x + p), men merk at p ikke
Dette er rett frem med a
vre den minste perioden funksjonen har.
trenger a
Eksempel 2.4.1. Definer f (x) = 1 sin x og g(x) = sin x, da har b
ade f og g
periode 2 siden sin(x + 2) = sin x cos 2 + cos x sin 2 = sin x. Merk at
F (x) = f (x) + g(x) = 1
Sik at F (x) har alle perioder, ikke bare 2.
4

definere periodiske funksjoner som f (x + T ) = f (T ), hvor


En kan unng
a dette problemet ved a
betrakte konstanter som periodiske
T er ulik null. Men i mange situasjoner er det mer naturlig a
funksjoner.

II

4 Periodiske funksjoner

31

Eksempel 2.4.2. Definer f (x) = cos x i sin x og g(x) = cos x + i sin x, da har
igjen b
ade f og g en minste periode p
a 2. Merk at
G(x) = f (x) g(x) = cos2 x i2 sin2 x = cos2 x + sin2 x = 1
Sik at F (x) igjen har alle perioder5 , ikke bare 2.
Theorem 2.4.1. La f (x), vre en funksjon med periode T hvor T 6= 0, og la k
vre en vilk
arlig konstant da er
a+T

b+T

Z
f (x) dx =

b
kT

Z
f (x) dx = k

(2.17)

f (x) dx

0
b+nT

0
b

f (x) dx + (n m)

f (x) dx =
a+mT

(2.18)

f (x) dx

Z
a

f (x) dx .

2T
a

(2.19)

T +a

Figur 2.11: Illustrasjon av likning (2.17). Omr


adet fra a til a + T , er like stort
som arealet over en periode alts
a fra T til 2T .

Korollar 2.4.1. Dersom R(x, y) er en rasjonell funksjon s


a er
Z

R(sin x, cos x) dx =

R(sin x, cos x) dx

Bevis. Dette flger direkte fra likning (2.17) med a = 0 og b = og T =


2.
sette m, a, b = 0, og
Merk at likning (2.18) flger direkte fra (2.19) ved a
la b = a, m = 0, n = 1 s
n = k. Tilsvarende kan frste likning vises ved a
a
Z

a+1T

Z
f (x) dx + (1 0)

f (x) dx =
a+0T

Z
f (x) dx =

f (x) dx
0

Ved a heller sette b = a f


ar vi samme resultat, og integralene m
a derfor vre like.
bevise siste likning om en frst har vist de to foreg
Men det er enklere a
aende.
Her bevises bare (2.17) og (2.18) mens siste likning overlates som en velse til
leser.
5 Her

innfre denne vil kanskje noe se p


er i2 = 1 den s
akalte imaginre enheten. A
a som juks.
Droppes derimot i blir G(x) = cos 2x som b
ade har periode 2 og .

32

II

4 Periodiske funksjoner

vise at begge integralene er likt integralet over 0 til


Bevis. Det er noe enklere a
T . Uten tap av generalitet la c a b, da er
Z c+T
Z T
Z T +c
f (x) dx =
f (x) dx +
f (x) dx
c

T
c

Z
f (x) dx +

=
c

Z
f (x + T ) dx =

f (x) dx
0

Hvor substitusjonen u = x T ble brukt i andre overgang og f (x + T ) = f (x)


siden f har periode T . Dette medfrer at
Z a+T
Z T
Z b+T
f (x) dx =
f (x) dx =
f (x) dx
a

som nsket. Intuitivt beskriver dette en horisontal forflyttning, og om funksjonen


er periodisk vil denne forlyttningen bevare arealet.
Siden k er et naturlig tall kan en dele opp integralet
kT

f (x) dx =
0

k1
X Z (n+1)T
n=0

f (x) dx =

nT

k1
XZ T
n=0

f (u + nT ) du

(2.20)

der substitusjonen x 7 u + nT ble benyttet i siste overgang. Via induksjon er


vise at f (x + nT ) = f (x) holder for alle n Z. Argumentet holder
det enkelt a
for n = 1, og anta s
a at f (x + pT ) = f (x) for en eller annen p. N
a er

f x + (p + 1)T = f (x + T ) + pT ) = f (x + T ) = f (x)
Her ble induksjonshypotesen brukt f (x0 + pT ) = f (x0 ) med x0 = x + T , og i siste
overgang at f er periodisk. Resultatet flger ved induksjon og ved innseting f
as
Z

kT

f (x) dx =
0

k1
XZ T
n=0

f (x + nT ) dx =

k1
XZ T
n=0

Z
f (x) dx = k

f (x) dx ,
0

som var det som skulle vises. Dette fullfrer beviset.


Likning (2.18) beskriver en samling av k like integral av typen i likning (2.17),
siden integralet over hver hele periode er like stort. En kan og kombinere resultatene, for eksempel s
a f
as
Z a+kT
Z b+T
f (x) dx = k
f (x) dx
a

bruke likning (2.17) og (2.18). Setter enn n = m inn i (2.19) f


ved a
ar en
Z b
Z b+nT
=
f (x) dx =
f (x) dx
(2.21)
a

a+nT

hvor n selvsagt kan vre negativ. Igjen gir dette mening siden om en integrerer
over (2T, 3T ) eller (T, 2T ) spiller ingen rolle da funksjonen er periodisk, en st
ar
skifte integralet slik det passer seg.
alts
a fritt til a
ta en rekke sm
I stedet for a
a eksempler, sammler vi alt i et magnum opus
eksempel nedfor. Merk en del av teorien fra avsnitt (2.3) vil og bli brukt

II

4 Periodiske funksjoner

33

Eksempel 2.4.3.
Z

53/4

dx
(1 +

25/4

2sin x ) (1

2cos x )

7
4

Legg merke til at om en ansl


ar integralet numerisk, f
ar en I 5.4978, s
aI
a at verdien av integralet
1.75 = 7/4. Dette gir oss i hvertfall en god gjetning p
er rett. La f (x) betegne integranden n
a er f (x + 2) = f (x) slik at integranden
y
1
0.8
0.6
0.4
0.2
x
k

k + 2

k + 4

k + 6 k + 7

Figur 2.12: Grafen til f , hvor k = 25/4.


er periodisk. Dette kan en enten se ved innsettning eller at integranden best
ar
skrive om grensene kan
av sin x og cos x ledd som begge har periode 2. Ved a
integralet skrives som
Z

5/4+62

I=

5/4

f (x) dx =
/4+32

f (x) dx + 3
/4

f (x) dx
0

hvor (2.19) ble brukt, og f (x) igjen betegner integranden.


I praksis er omskrivningen at integralet deles opp i to deler, delen som best
ar
av hele perioder, og delen som best
ar av halve perioder. Eg fra k til k + 6, og
k + 6 til k + 7 i figur (2.12).
Kall integralene for henholdsvis A og B, merk at
Z

dx
=
sin
x
(1 + 2
) (1 + 2cos x )

Z
=
0

du
2sin u 2cos u

 sin u cos u
2
1 + 2sin(u+) 1 + 2cos(u+) 2
2sin x 2cos x dx
(1 + 2sin x ) (1 + 2cos x )

via substitusjonen x 7 u + , her ble ogs


a sin(u + ) = sin u og cos(u + ) =
cos u benyttet. Dette resultatet f
ar en bruk for om en deler B p
a midten
Z
B=3

f (x) dx + 3
0

Z
f (x) dx = 3

1 + 2sin x 2cos x dx
,
(1 + 2sin x ) (1 + 2cos x )

34

4 Periodiske funksjoner

II

Det kan vises at f (x) + f (a + b x) er konstant [Oppgave] slik at theorem (2.3.1)


kan brukes. Da er


a+b
1 + 2sin(/2) 2cos(/2)
3

 =
B = 3(b a)f
= 3
sin(/2)
cos(/2)
2
2
1+2
1+2
Helt tilsvarende kan A beregnes s
a
Z 3/4
Z 5/4
dx
dx
A=
+
sin
x
cos
x
sin
x
(1 + 2
) (1 + 2
)
) (1 + 2cos x )
/4
3/4 (1 + 2
bruke substitusjonen u = x /2 s
Ved a
a kan siste integral skrives som
Z 5/4/2
Z 3/4
du
2sin x dx

 =
(1 + 2sin x ) (1 + 2cos x )
1 + 2sin(u+/2) 1 + 2cos(u+/2)
3/4/2
/4
bruke dette kan A
Siden sin(u + /2) = sin u og cos(u + /2) = cos u. Ved a
skrives som
Z 3/4
Z 3/4
dx
1 + 2sin x
dx
=
A=
sin
x
cos
x
(1 + 2
) (1 + 2
)
1 + 2cos x
/4
/4
beregne integralet kan igjen benytte seg av theorem (2.3.1) siden g(x) +
For a
g(a + b x) er konstant.
Z 3/4

dx
=
=
A=
cos
x
cos(/2)
1+2
2 1+2
4
/4
som fr. Her betegner da selvsagt g = 1/(1 + 2cos(x) ). Oppsumert s
a er
Z 53/4
3

7
dx
=
+
=
sin x ) (1 + 2cos x )
(1
+
2
2
4
4
25/4
vise.
som var det en nsket a
En noe mer intuitiv fremgangsm
ate a vise A p
a er a heller bruke likning (2.13)
fra lemma (2.3.1).
Z 3/4
Z /2
Z /2
dx
1
1

A=
=
+
=
dx =
cos
x
cos
x

cos
x
1+2
1+2
4
/4
/4 1 + 2
/4
Hvor cos(/4 + 2/4 x) = cos x. Om en ikke er s
a glad i benytte slike knep
heller, kan integralet ogs
a beregnes enda mer elementrt.
Z 3/4
Z /2
Z 3/2
dx
dx
dx
=
+
cos
x
cos
x
1+2
1 + 2cos x
/4
/4 1 + 2
/2
Z /2
Z /4
dx
2cos(x)
=

dx
cos x
cos(x)
/4 1 + 2
/2 1 + 2
Z /2
Z /2 cos x
2
dx
dx
+
,
=
cos x
cos x 1 + 2 cos x
1
+
2
2
/4
/4
der substitusjonen u 7 /2 x ble brukt. Helt tilsvarende regning kan a brukes
beregne B elementrt.
for a
se p
I dette eksempelet studeres en del spesialtillfeller av theoreme ved a
a
funksjoner med ulike egenskaper

II

4 Periodiske funksjoner

35

Proposisjon 2.4.2. La n Z/{0} vre et heltall ulikt null og la f vre en funksjon.


Dersom
1) at f er en funksjon med periode T , f (x + T ) = f (x) s
a er
Z T
Z T
f (nx) dx =
f (x) dx
0

2) at f er en vilk
arlig funskjon slik at integralene konvergerer s
a er
Z /2
Z /2
f (sin 2x) dx =
f (sin x) dx
0
Z /2

(2.22)

(2.23)

f (cos 2x) dx =
0

1
2

/2

f (cos x) + f ( cos x) dx

(2.24)

3) at f er en likefunksjon, f (x) = f (x) s


a er
Z

/2

/2

f (sin nx) dx =
0
Z /2

f (sin x) dx

(2.25)

f (cos x) dx

(2.26)

0
Z /2

f (cos nx) dx =
0

I tilfellet hvor f er odde, f (x) = f (x) holder likning (2.25) og (2.26)


generellt bare for n = 1, 2.
heller studere
Beviset overlates til leser se oppgave (7). Vi kommer til a
symmetriegenskapene mellom sin x og cos x.
Proposisjon 2.4.3. La m, p, q Z, da er
Z (sk)/2
sin(mx)n dx = 0 og

(rk)/2

(q+k/2)

cos(mx)n dx = 0

(pk/2)

n
ar n 2N 1 og r, s 2Z + 16 . Alts
a er r 2j + 1, s 2i + 1 hvor j, i N. Det
kreves i tillegg at dersom i og j har samme paritet m
a k vre like og og tilsvarende
for p og q. Dersom i og j eller p og q har ulik paritet s
a kan k Z velges fritt.
Beviset her uttelates men er logisk om en betrakter figuren. Beviset ville vrt
a dele opp integralene halveis, og drftet de ulike tilfellene for i, j, k, m, n, p, q, r, s
men for leserens mentale helse utellates dette. Derimot frer proposisjonen direkte til
Korollar 2.4.2.
Z

2n
2k+1

cos(mx)

dx =

cos(mx)2k+1 dx = 0

(2.27)

sin(mx)2k+1 dx = 0

(2.28)

0
2n

Hvr n, m, k N.
6 Notasjonen

her betyr at n er odde. Dette kan og skrives som n {2u 1 : u N}, denne
notasjonen tar dog opp noe mer plass. Tilsvarende for 2Z 1, merk disse mengdene er identiske.

36

II

4 Periodiske funksjoner

Lemma 2.4.1. La R vre en rasjonell funksjon da er


Z

R(cos x, sin x) dx

R(sin x, cos x) dx =
0

Spesielt s
a er
Z

f (cos x) dx .

f (sin x) dx =
0

Hvor K = /2 eller K = 2.
vise likheten for K = /2 er trivielt. Ved a
bruke den kjre substitusjoBevis. A
nen y 7 /2 x s
a er
/2


R cos(/2 y), sin(/2 y) dy

R(sin x, cos x) dx =
/2

/2

R(cos x, sin x) dx ,

=
0

som nsket. La oss n


a studere tilfellet K = 2 som er hakket mer spennende.
frst bruke den samme substitusjonen y 7 /2 x s
Ved a
a er
Z

3/2


R sin(/2 x), sin(/2 y) dy .

R(sin x, cos x) dx =
/2

Integralet kan forenkles ved a snu grensene og benytte som fr at sin(/2 y) =


dele integralet ved x = 0.
cos y og cos(/2 y) = cos y. Neste steg blir a
Z

/2

R(cos y, sin y) dy +

R(cos y, sin y) dy

3/2

En naturlig fortsettelse er a benytte substitusjonen y 7 2+u eller likning (2.19)


med n = m = 1,a = 3/2,b = /2 og T = 2 p
a frste integralet. Uansett blir
Z

Z
R(cos u, sin u) du +

/2

/2

Z
R(cos y, sin y) dy =

R(cos x, sin x) dx
0

Som var det som skulle vises. Nyaktig samme bevis som ovenfor kan brukes p
a f (sin x), noe enklere er bare se at R(cos x, sin x) = f (cos x) om R ikke
inneholder noen sinus ledd.
Dette vil si at R(x, y) er en symmetrisk funksjon. Som betyr at i ethvert
bytte om disse
integral som kun best
ar av cos x og sin x ledd, st
ar en fritt til a
Z
0

3 cos x
dx =
2 sin x cos x + 3

Z
0

3 sin x
dx ,
2 cos x sin x + 3

s
a fremt en integrerer over en kvart, eller hel periode. Lemmaet her kan brukes
vise et noe mer generelt resultat
til a

II

4 Periodiske funksjoner

37

Proposisjon 2.4.4. La k vre en konstant da er


Z k/2
Z k/2
R(cos x, sin x) dx =
R(sin x, cos x) dx ,
m/2

m/2

gitt at k, m 0 (mod 4) eller k, m 1 (mod 4).


Siste del betyr at k og m alltid er p
a formen 4n eller 4n + 1 for et heltall n
vise
og ikke ndvendigvis like. Slik at vi kan godt velge k = 4 + 1 og m = 4. A
dette tar liten plass n
ar vi allerede har vist lemma (2.4.1)
Bevis. Vi kan uten tap av generalitet sette m = 0 igjennom hele beviset. Siden
vise proposisjonen for de negative verdiene
integralet kan deles ved x = 0 og a
er akkuratt likt som for de positive. Tar frst tilfellet for k = 4n, da er
Z 2n
Z 2
R(cos x, sin x) dx = n
R(cos x, sin x) dx
0

=n

2n

R(sin x, cos x) dx =
0

R(sin x, cos x) dx ,
0

der frste og siste overgang flger fra likning (2.18) og andre overgnag flger
fra lemma (2.4.1). Tilsvarende for tilfellet n
ar k = 4n + 1 s
a er
Z 2n+/2
Z 2
Z /2
R(cos x, sin x) dx = n
R(cos x, sin x) dx +
R(cos x, sin x) dx ,
0

hvor likning (2.18) ble brukt. En har vist symmetrien til begge integralene p
a
bruke dette har en dermed
hyresiden i lemma (2.4.1). Ved a
Z 2n+/2
Z 2
Z /2
R(cos x, sin x) dx = n
R(sin x, cos x) dx +
R(sin x, cos x) dx
0

0
2n+/2

R(sin x, cos x) dx ,

=
0

som var det som skulle vises. Dette fullfrer beviset.


bruke denne symmetrien til a
lse integral er ikke rett frem. Men den er
A
vise at ulike trigonometriske integral er like. Vi kan etablere
svrt nyttig for a
liknende identiteter for
Proposisjon 2.4.5. .
La f (x) vre en begrenset funksjon p
a intervalet og la m,n vre heltall. Da er
Z n
Z 2n
2
f (cos x) dx =
f (cos x) dx
(2.29)
Z

m
n/2

2m
n

Z
f (sin x) dx =

2
m/2

f (sin x) dx

(2.30)

gitt at n, m er p
a formen n, m 0 (mod 4) eller n, m 1 (mod 4).
Beviset for dette overlates til leser, og kan vises p
a samme m
ate som de
to tidligere proposisjonene. Merk at proposisjonen holder selv om m = 4p og
n = 4l + 1. La oss ta en svak generalisering av Riemann-Lebesgue lemmaet

38

II

4 Periodiske funksjoner

Lemma 2.4.2. (Riemann-Lebesgue lemma) La f (x) vre en kontinuerlig deriverbar7 funksjon p


a (a, b). La g(x) vre en funksjon med periode T . Da er
Z
lim

1
f (x)g(nx) dx =
T

Z
g(x) dx

f (x) dx .
a

gjre beviset enklere


Lemmaet holder under svakere betingelser, men for a
holder dette for oss
Bevis. La oss frst se p
a integralet over g(x), dette kan skrives som
!
Z T
Z x
1
g(x) dx + h(x) = xA + h(x) .
g(t) = x
T 0
0
Her betegner A integralet, og h er en eller annen kontinuerlig funksjon med
periode T . Frste overgang kommer fra likning (2.18), hvor k = x/T . For at
dette skal gjelde m
a x/T vre et heltall , dette ordnes via funksjonen h(x) som
derivere begge sider s
er g(x) integrert over et passelig interval. Ved a
a er
g(x) = A + h0 (x) ,
slik at g(nx) = A + h0 (nx). Her trengtes ikke produktregelen og benyttes p
a
frst del, da A er en konstant og ikke avhengig av x. Settes dette inn i den
opprinnelige likningen f
as
Z

Z
f (x)g(nx) dx = A

f (x) dx +
a

f (x)h0 (nx) dx .

vise at det siste integralet g

M
alet er n
aa
ar mot null, n
ar n g
ar mot . Ved a
bruke delvis integrasjon kan integralet skrives som
b

f (x)h(nx)
n

f (x)h (nx) dx =
a

b

n
a

f 0 (x)h(nx) dx .

Siden f , f 0 og h(x) er kontinuerlige p


a (a, b) medfrer dette og at de er vre
begrenset, som igjen betyr at integralet konvergerer. Hyresiden er dermed et
reelt tall s
a hyresiden g
ar mot null n
ar n vokser. Dette medfrer
Z
n

Z
f (x)g(nx) dx = A

lim

f (x) dx + 0 =
a

1
T

Z
g(x) dx

f (x) dx ,
a

som var det som skulle vises.

7 kontinuerlig

deriverbar er en faglig betegnelse som betyr at f er kontinuerlig, og at dens deriverte


ogs
a er kontinuerlig

II

4 Periodiske funksjoner

39

Oppgaver
7.
a) Lett oppvarming. Vis at dersom f har periode T , alts
a f (x + T ) = f (x)
s
a er
Z T
Z T
f (nx) dx =
f (x) dx ,
0

gitt at n = Z/{0} alts


a at n er et vilk
arlig heltall ulikt 0. Bevis ogs
a
resten av proposisjon (2.4.2).
8. Bevis at det kun eksisterer to n N slik at
Z /2
Z /2
d
d
I(n) =
=
2 + sin n
2 + sin
0
0

2 3

Tilsvarende bestem n
ar I(n) =

9. Vis at
Z

b+nT

f (x) dx + (n m)

f (x) dx =
a+mT

f (x) dx
0

hvor igjen a, b, T er reelle tall, f (x + T ) = f (x) for alle x, og n, m Z.


10. Bestem integralet
Z

71/2

I=


log

23

(1 + sin x)1+cos x
1 + cos x


dx .

a periodiske funksjoner, og i denne oppgaven ser en


11. Tidligere har en sett p
nrmere p
a spesialtilfellet der integranden i tillegg er odde eller like.
La f (x) vre en likefunksjon, og la g(x) vre odde. B
ade f og g har periode T = 2L, hvor L er en naturlig konstant. I oppgaven skal integralene
Z x
Z x
G(x) =
f (t) dt og F (x) =
g(t) dt ,
0

studeres nrmere8 .
a) Bestem pariteten til F og G. Alts
a om F og G er odde eller like.
b) Bestem G(2nL) og F (2nL), hvor n Z.
c) Vis at F ogs
a har periode 2L, hvilke betingelser kreves for at G ogs
a
skal ha periode 2L?
8 Merk

at egenskapene vi viser her minner svrt om egenskapene til sin x, og cos x og blant elementre funksjoner er disse de eneste periodisk, odde og like funksjonene.

Tillater en derimot delte funksjonsuttrykk, finnes det mange flere eksempler, disse mye brukt innen
signalbehandling og fourier-analyse.

40

5 Diverse substitusjoner

2.5

II

DIVERSE SUBSTITUSJONER

Vi betegner en substitusjon som flgende overgang


Z
Z
f [g(t)]g 0 (t) dt
f (x) dx =

og formelt sett er dette en avbildning fra et omr


ade X til et omr
ade Y via funksjonen x 7 g(t). Hva dette i praksis vil si er at vi g
ar fra et koordinatsystem x,y
til et nytt koordinatsystem x0 ,y 0 og det eneste som er konstant mellom systemene
er at arealet under f (t) (a, b) og f (g(t)) (, ) er like store.
Vi begynner med a se p
a noen f
aq
eksempler p
a nyttige susbtitusjoner. Inneholp
der integralet (x a)(b x eller xa
s
a
kan
substitusjonen x 7 a cos2 +
bx
b sin2
Eksempel 2.5.1.
Z
I=
a

dx
p

(x a)(b x)

Vi benytter den anbefalte substitusjonen x 7 a cos2 + b sin2 . Da er




(x a)(b x) = a(cos2 1) + b sin2 b(1 sin2 ) a cos2
= (a sin2 + b sin2 )(b cos2 a cos2 )
= (b a)2 sin2 cos2
Mens den deriverte kan skrives som
dx
= 2a cos sin + 2b sin cos = 2(b a) sin cos
d
Herfra m
a vi arbeide med grensene v
are. N
ar x = a f
ar vi likningen a =
a cos2 + b sin2 som medfrer (a b) sin2 = 0. Her kan vi se bort i fra det
trivielle tilfellet a = b. S
a vi m
a ha sin2 = 0 eller = n, n Z. Vi velger
n = 0 for enkelhetens skyld.
Tilsvarende for x = finner vi at = /2 + n. Denne gangen kan vi
gjette at vi m
ikke velge n fritt. Det er relativt trygt a
a velge = /2. Vi kan
rettferdiggjre dette ved a se at vi nsker at x ker fra a til b, og dersom vi velger
en annen verdi for den vre grensen vil ikke dette skje. Substitusjonen gir alts
a
Z /2
Z /2
Z b
dx
(b a) sin cos d
p
p
=2
=
2
d =
(x a)(b x)
(b a)2 sin2 cos2
a
0
0
som nsket. Noen
a er cos x og sin x
tillegskommentarer: For x [0, /2] s
2
2
positive slik at sin cos = | cos x|| sin x| = cos x sin x. Vi kan og se raskt
p
a konvergens, eneste farememomenter er x = a og x = b. Integralet oppfrer
R1

seg som 0 dx/ x nre disse punktene og konvergerer dermed. En har fra
taylorutvikling at
Z
Z
dx
p
(x a)1/2 (b a)1/2 dx = 2(x a)1/2 (b a)1/2
(x a)(b x)
som er konvergent. Helt tilsvarende omkring x = b.

II

5 Diverse substitusjoner

41

evaluere bestemte integral ved bruk av substitusjon krever mfintlighet.


A
Flgende theorem gir oss tilstrekkelige betingelser for en gyldig substitusjon
Theorem 2.5.1. Dersom funksjonen t = (x) tilfredstiller flgende
1) (x) er en kontinuerlig en-til-en funksjon definert p
a intervalet [, ] og har
en kontinuerlig derivert der.
2) Verdiene til (t) ligger i intervalet [a, b]
3) () = a og () = b.
da vil flgende formel holde
Z b

f (g(x))g 0 (x) dx

f (t) dt =
a

for alle f stykkevis kontinuerlig funksjoner derfinert p


a [a, b].
Gjennom nonen eksempler vil vi vise visse fallgruver om en ikke passer p
a
betingelsenene ovenfor. Det enkleste er alts
a benytte seg av monotone9 substitusjoner. Ellers m
a en dele opp intervallet til substitusjonen blir monoton.
Eksempel 2.5.2.
Z

I=
2

dx
4 + x2

Variabelskifte x 7 1/t vil her lede til et galt svar


Z 2
Z 1/2
Z 1/2
dx
dt
dt

I=
=
=

2
2+1
2 1+ 1
4
+
x
4t
t
2
1/2
1/2
t2

1/2

1
=
=
arctan 2t
2
4
1/2
Som er umulig siden funksjonen er strengt positivt. Dette skjer fordi substitusjonen 1/t ikke er deriverbar i origo10 . Rett svar er selvsagt
Z 2
h
 i
dx
x i2
1 h

I=
= arctan
=

=
2
4
+
x
2
2
4
4
4
2
2
En annen som kan komme er n
ar en bruker den iverse substitusjonen x = (t)
og x = (t) tar flere verdier11 .
9 En

monoton funksjon er en funksjon som er avtagende eller synkende p


a ett gitt omr
adet.
er jo selvsagt ikke helt rett. Problemet ligget i at vi m
a dele opp intervallet v
art ved origo
fr vi kan derivere. Dette handler om at x 7 1/x mapper [2, 2] til (, 1/2] [1/2, ) og
ikke [1/2, 1/2] som en kanskje skulle trodd i utgangspunktet.

10 Dette

11 Innen

kompleks analyse klaller vi slike funksjoner gjerne for branch cuts.

42

5 Diverse substitusjoner

II

Eksempel 2.5.3.
Z
I=

(x 2)2 dx

bruke substitusjonen u 7 (x 2)2 vil resultere i et galt


Ved a
resultat. Dette er
fordi den
inverse funksjonen ikke er unikt definert x = 2 t . enene grenenen
x1 = 2 t kan ikke ta verdier for x > 2, og den andre x2 = 2 t kan ikke ta
verdier for x < 2. Direkte (men feil) utregning via u 7 (x 2)2 gir

1
Z 3
Z
1 1
2 3/2
8
7
1
2
(x 2) dx =
u du =
u

=
=
2
3
3
3
3
0
4
4
penbart feil, da integranden er positiv p
f
A
a hele omr
adet. For a
a det korrekte
dele integralet i to deler
svaret er det ndvending a
Z 2
Z 3
I=
(x 2)2 dx +
(x 2)2 dx
0

S
a kan vi la x 7 2 t i frste integrealet og x 7 2 + t i det andre. Dette gir
Z 2
Z 0
Z
dt
1 4
8

I1 =
(x 2)2 =

t
=
t dt =
.
2 0
3
2 t
0
4
Z 3
Z 1
Z 1
1
1
dt
I2 =
(x 2)2 =
t dt =
t =
.
2 0
3
2 t
2
0
beregne
Derfor s
a er I = I1 + I2 = 38 + 13 = 3. Resultatet kan bekreftes ved a
integralet direkte
3

Z 3
1
8
(x 2)
2
=
+
=3
(x 2) dx =
2
3
3
0
0
En kan og betrakte12 en substitusjon som at en omformer et materialet eller
vske som en ikke kan trykkes sammen13 . En substitusjon kan da sees p
a som
an en drar materialet i en eller annen retning, merk at transformasjonen m
a
vre kontinuerlig, og kan heller ikke innfre hulli omr
adet.

En avbildning fra X til Y ikke er entydig og det kan finnes mange m


ater a
komme seg fra X til Y p
a. En kan for eksempel g
a fra X til Z ogs
a fra Z til Y ,
eller en rekke andre m
ater. Omr
adene vi vil arbeide med vil i hovedsak vre
interval p
a den reelle tallinjen slik at en kan skrive X = [a, b] og Y = [, ].
Men det er ikke noe mot at X er et interval og at Y er en sirkel for eksempel.
Ofte n
ar vi bruker substitusjon er m
alet a forenkle integranden, men hva om
forenkle grensene i stedet. Hvordan skal vi finne en substitusjon slik
vi nsker a
at grensene g
ar fra [a, b] til [, ]? Substitusjonen kan skrives som
Z b
Z
f (t) dt =
f (g(x))g 0 (x) dx ,
a

hvordan m
a da t = g(x) velges? Flgende propsisjon gir oss heldigvis svaret
12 En

alternativ synsvinkel er en liner transformasjon mellom to topologiske rom.


kalles et slikt materialet med konstant volum for inkompressibelt. N
ar det gjelder gasser,
kalles en prosses med konstant volum for en isokor prosess.

13 Formelt

II

5 Diverse substitusjoner
y

cos x

1.5

1.5

0.5

0.5

0
0.5

1.5

y0

1t2 2
1+t2 1+t2

x0

x
0

43

(a) Funksjon med areal 1.

0.5

1.5

(b) Funksjon med areal 1.

Figur 2.13: Arealet under begge funksjonene er like og t 7 tan(x/2) mapper


x, y til x0 ,y 0 .
R
Theorem 2.5.2. Integralet f (t) dt kan bli transformert til et annet integral
med grenser og via den linere transformasjonen

x+

T (x) 7

(2.31)

Da kan integralet skrives som


Z

f (t) dt =


f

x+


dx

(2.32)

anta at x(t) =
Beviset overlates til leser. Det er ikke vanskeligere enn a
Ax + B. Kravene om at x() = , x() = bestemmer konstantene A og B.
Fra dette flger
Korollar 2.5.1.
Z


f (b a)t + a dt

Z0 
a + bx
dt
= (b a)
f
1
+
x
(1
+
t)2
0

f (x) dx = (b a)

(2.33)
(2.34)

for alle a, b R.
Dette flger fra og sette inn henholdsvis = 0, = 1 og = 0, i
likning (2.32).
se likning (2.32) p
vise korollaret frst.
En noe mer intuitiv m
ate a
a er a
pent interval (a, b) kan krympes ned til enhetsintervalet
Dette viser at ethvert a
(0, 1) via en eller annen transformasjon U . Dette betyr ogs
a at en kan krympe
ta inversen av
intervalet (, ) ned p
a (0, 1) via en transformasjon V . Ved a
V , g
ar vi motsatt vei fra (0, 1) til (, ) alts
a forstrres omr
adet. Dette er
frst bruke transformasjonen U , og deretter V 1
illustrert i figur (2.14). Ved a

44

5 Diverse substitusjoner

U
0

II

U (V 1 )

Figur 2.14: Mapping fra (a, b) til (, ).


vise at U (V 1 ) er samme transformasjon som i
g
ar vi alts
a fra (a, b) til (, ). A
likning (2.31) overlates til leser.
Som vist ovenfor er ikke avbildninger unike, det finnes mange veier en kan
ta fra X til Y - I tabell (2.2) er et utvalg transformasjoner fra X til Y vist. Her
betgne omr
blir notasjonen (a, b) for a
adet en integrerer over.
Tabell 2.2: Et utvalg av sentrale substitusjoner
X

t = g(x)

(a, b)

(1, 1)

(0, a)
(0, 1)
(0, 1)
(1, 1)
(0, )
(1, )
(0, )
(0, )
(0, )
(0, )
(0, )
(0, )
(, )

(0, 1)
(a, b)
(, )
(, )
(a, b)
(0, 1)
(1, 1)
(1, 1)
(0, 1)
(0, 1)
(1, 0)
(1, 0)
(, )

b+a
2x
+
ba
ba
x/a
a + (b a)x
(1 2x)/4(x2 x)
x/(1 x2 )
(a x)b/(b x)
1/x
(x 1)/(x + 1)
arctan(x)/4 1
x/(1 + x)
tanh x
1/(1 + x2 )
exp(x)
x + 1/x

En transformasjon som er utelatt fra tabellen er hvordan en kan mappe (a, b)


til R = (, ), og motsatt. Grunnen til dette er plassmangel, og resultatet er
gitt under
Proposisjon 2.5.1. La a, b vre to reelle tall med b > a, da er
T (x) =

1 (b a)(b + a 2x)
4 x2 (a + b)x + ab

En funksjon slik at
T

X Y
Hvor X = (a, b) og Y = R.

(2.35)

II

5 Diverse substitusjoner

45

vise, men det holder a


sjekke at at x a frer til
Dette overlates til leser a
at T , x b frer til at T , og at T er kontinuerlig p
a (a, b).
For a utlede resultatet kan en for eksempel mappe (a, b) p
a (1, 1) og deretter
pent
(1, 1) p
a R. Det eksisterer heldigvis enklere transformasjoner fra et a
interval til den reelle tallinjen om en tillater seg bruk av hyperbolske funksjoner.
finne en slik transformasjon er gitt i oppgaveteksten under.
A
Merk avbildningene ovenfor kan b
ade snus (0, 1) = (1, 0) og deles opp
(0, ) = (0, 1) (1, ). Dette vil komme til nytte i flgende omskrivning
Proposisjon 2.5.2.
Z

Z
g(x) dx = S

1
g(Sx) + 2 g
x

S
x


dx

(2.36)

Hvor S > 0 er en vilk


arlig konstant.
benytte substitusjonen x 7 St dx = S dt f
Bevis. Ved a
as
Z
Z
g(x) dx = S
g(St) dt
0

Grensene blir uforandret siden S > 0. Merk at


Z
Z 0  
dt
S
,
g(Sx) dx =
g
t
t2
1
1

(2.37)

dele opp intervalet i


via substitusjonen x 7 1/t dx = dt/t2 . Ved a
(0, 1) (1, ) kan n
a integralet skrives som
 
Z 1
Z
Z 1
1
S
S
g(Sx) dx + S
f (Sx) dx = S
g(Sx) + 2 f
dx
x
x
0
1
0
Der likning (2.37) ble brukt. Dette fullfrer beviset.
Spesielt s
a vil tilfellet n
ar S = 1
Z
Z
g(x) dx =
0

1
g(x) + 2 g
x

1
x


dx ,

bli mye brukt fremmover.


Oppgaver
vise at
12. Benytt integralet fra eksempel (2.5.1) til a
Z b
dt

p
J=
=
.
ab
a t (t a)(b t)
Bestem ogs
a integralet
Z
c

hvor a, b, c, d R.

du
p
,
2
u (u c2 )(d2 u2 )

(2.38)

46

II

5 Diverse substitusjoner

13. En annen transformasjon er


a


f

a2
t + 2
t

dt
=
t


f

a2
t+
t

dt
,
t

noen ganger kjent som Wolstenholme transformasjonen. Vis at denne stemmer


14. La f (x) vre en kontinuerlig funksjon og
Z

f (x) dx =

f (x) dx eksisterer. Vis at



1
x
dx
x

Denne overgangen kalles ofte for Slobin transformasjonen.


Z

15. Beregn integralet


0

dx
a2 + x

1
x

2 , hvor a 2.

16. a) Benytt transformasjonene

U (x) =

ba
a+b
x
x+
og V (x) =
2
2
1 x2

bevise proposisjon (2.5.1).


til a
b)

Bestem funksjonene v(a, b), og u(a, b) slik at


Z

f (t) dt =
a

1
2



f arctanh v(a, b, x) u(a, b, x) dx

Denne transformasjonen har en entydig invers p


a R i motsetning til
likning (2.35) gitt ved
Z

Z
f (t) dt = Q

f (Q tanh x + P )
a

dx
cosh2 x

Bestem konstantene Q og P slik at likheten stemmer. Her er P = P (a, b)


og Q = Q(a, b) funksjoner av a og b.

II

2.5.1

5.1 Weierstrass substitusjon

WEIERSTRASS

47

SUBSTITUSJON

I denne delen skal vi se nrmere p


a en substitusjon kjent som weierstrass substitusjon14 . Teknikken er oppkalt etter den tyske matematikeren Karl Weierstrass
sette
(1815-1897) og best
ar i a
x
t 7 tan
.
2
Meningen med substiusjonen er a transformere trigonometriske integral over til
gjre dette trengs uttrykk for b
rasjonale funksjoner. Men for a
ade dx, sin x og
cos x
Theorem 2.5.3. (Weierstrass) Anta at a, b [, ] da er
Z

R(sin x, cos x) dx =
a


R

1 t2
2t
,
2
1 + t 1 + t2

2 dt
1 + t2


x

via substitusjonen t = tan 2 . Hvor R er en rasjonal funksjon, = tan(a/2),


2t
= tan(b/2) og tan(x) = 1t
2 .
Bevis. Dett beviset tar utgangpunkt i figur (2.15), hvor sidene ble valgt p
a en

1 + t2

x/2
A

Figur 2.15: Figur med t = tan(x/2).


slik m
ate at t = tan(x/2). Dessverre f
ar en bare uttrykk for sin(x/2) og cos(x/2)
fra figuren
cos

x
2

1
1 + t2

og sin

x
2

t
1 + t2

(2.39)

f
For a
a de rette verdiene benyttes dobbelformelene

14 Flere

cos(A + B) = cos A cos B sin A sin B

(2.40)

sin(A + B) = cos A sin B + sin A cos B

(2.41)

gi noen refferanser til hvorfor. Tekbker kaller denne substitusjonen for Weierstrass uten a
paramterisere
nikken var kjent lenge fr Weierstrass var fdt, blant annet fra Euler og ideen om a
r. Se http://math.stackexchange.com/questions/
enhetssirkelen har vrt kjent i flere tusen a
461527/on-the-origins-of-the-weierstrass-tangent-half-angle-substitution for flere detaljer. I de fleste verk s
a er teknikken kjent som weierstass-substitusjonen og derfor er
det navnet som vil bli benyttet her og.

48

5 Diverse substitusjoner

II

sette inn A = B = x/2 i likning (2.40) kan cos x skrives som


Ved a
2 
2
x
x 
t
1 t2
1

=
cos(x) = cos2
sin2
=
.
2
2
2
2
1 + t2
1+t
1+t
Tilsvarende s
a kan sin x skrives som



x
x
t
1
2t

cos
=2
,
sin x = 2 sin
=
2
2
2
2
1
+
t2
1+t
1+t
sette inn A = B = x/2 i likning (2.41). For a
bestemme dt deriveres
ved a
t = tan(x/2) med kvotientregelen
dt =

1 cos2 (x/2) + sin2 (x/2)


1
dx = (1 + t2 ) dx
2
cos2 (x/2)
2

Dette gir som nsket at dx = 2 dt/(1+t2 ), hvor enhetsformelen og likning (2.39)


ble benyttet i siste overgang. Til slutt har en
sin x
2t . 1 t2
2t
tan x =
=
=
cos x
1 + t2 1 + t2
1 t2
og dette fullfrer beviset.
Eksempel 2.5.4. Vis at
Z

dx
= log |tan x + csc x| + C
cos x

f
penbaring og deretter se at en kan gange med
Standardtrikset her er a
a en a
sec x + tan x
sec x + tan x
benytte seg av substitusjonen u = sec x + tan x. Dette er dog svart magi
fors
aa
og overlates til leser. Her vises heller frem Weierstrass i sin prakt
Z
Z
Z
2
dx
1 + t2
(1 + t) (t 1)
=
dt
=
dt
cos x
1 t2 1 + t2
(1 + t)(1 t)
bestemme integralet kan en n
For a
a dele opp integralet relativt enkelt




Z
Z
1+t


1
1
dx
+ C = log 1 + tan(x/2) + C
=

dt = log


cos x
1+t
t1
1t
1 tan(x/2)
ha en
Men dette likner ikke helt p
a det som skulle vises. Heldigvis hjelper det a
liten algebratrollmann i magen. Legg merke til at
1+t
1+t 1+t
2t
1 + t2
=

=
+
2
1t
1t 1+t
1t
1 t2
sette inn har en alts
Ved a
a at

Z
2t
dx
1 + t2
= log
+
2
cos x
1t
1 t2



= log |tan x + sec x| + C

som var det som skulle vises. I siste overgang ble det ikke brukt mer enn theorem (2.5.3).

II

5.1 Weierstrass substitusjon

49

Eksempel 2.5.5. Som et siste eksempel skal vi se p


a det bestemte integralet
Z 2
dx
2
=
2
+
cos
x
3
0
Merk at det ikke fungerer og bruke Weierstrass substitusjonen med en gang.
Siden tan(0) = 0 og tan(2/2) = 0. Grunnen er at for at tan(x/2) skal vre
unikt definert, m
a en begrense intervalet til x (, ]. Problemet unng
as dog
dele integralet inn i perioder eller benytte korollar (2.4.1).
ved a
Z 2
Z
Z 2
Z
dx
dx
dx
dx
=
+
=
2 + cos x
2 + cos x
0
0 2 + cos x

2 + cos x
Her ble substitusjonen t 7 2x ble benyttet p
a siste integralet. Dette forandrer
grensene (, 2) (, 0). Ved n
a og benytte seg av Weierstrass, f
as

Z 2
Z
Z 
1

2 dt
2 dt
dx
1 t2
=
=
2+
2
2
2
2 + cos x
1+t
1+t
0
3 + t

Siste integralet kan bli lst via substitusjonen t 7 3 u. S


a
Z 2
Z
dx
2
2
du
=
=
2
2
+
cos
x
1
+
u
3
3
0
Siste integralet er bare den deriverte av arctan u.
Hittil har integralene v
are vrt relativt hyggelige og pene, og i slike tilfeller
er Weierstrass svrt nyttig. Beklagigvis bryter metoden helt sammen for de aller
fleste kompliserte integrander.
3
Z 
Z
Z
Z
1 t2
1 + 2t2 + t4
dx
2 dt
3
=
=
2
dt
sec x dx =
cos3 x
1 + t2
1 + t2
(1 t)3 (1 + t)3
Integralet kan videre bli lst via delbrksoppspalting, men denne blir lang. Konklusjonen blir at Weiestrass-substitusjon blir en siste utvei for a beregne trigonometriske integral. Den omskriver trigonometriske funksjoner til rasjonale, men
inteuttrykkene en ender opp med kan vre like vanskelig eller vanskeligere a
beregne slike integral i det
grere. Som vi skal se senere kan det vre enklere a
komplekse planet.
Oppgaver
1. La oss se p
a to klassiske integral som kan lses rimelig enkelt via Weierstrass. Vis at
Z 2
d
2
=
(2.42)
2
a
+
b
sin

a b2
0
r
Z /2
d
2
ab
=
arctan
(2.43)
2
2
a + b sin
a+b
a b
0
hvor |a| |b| er reelle konstanter. Vis spesielt at
Z 2
Z /2
d
d
6
=
2
+
sin

2
+
sin
0
0

50

II

5 Diverse substitusjoner

2.5.2

EULER

SUBSTITUSJON

Studiet av rasjonalle funksjoner med radikale nevnere har vrt studert lenge
Z
A(x)
p
dx
B(x)
og har gitt oppspring til elliptiske kurver, elliptiske integraler, differensial geometri og mer. H
apet er at om A og B er pene nok s
a kan integraler skrives p
a
formen P (x)/Q(x) hvor P og Q er polynomer. Studiet av disse integralene
er ferdig i den forstand at alle slike integral kan bli lst ved hjelp av samme
oppskrift.
gjre denne omformingen p
bruke hyperbolske eller inverse
En m
ate a
a er a
benytte seg av en rekke
trigonometriske substitusjoner. En annen m
ate er a
substitusjoner kalt euler-substitusjonene. I nyere tid har disse knepene mistet
mye av sin popularitet av uviss grunn.
Eksempel 2.5.6.
Z



p
dx


= log x + x2 + a ,
2
x +a

aR

arbeide seg gjennom.


Dette er et standard integral, men det er likevell nyttig a
Vi bruker substitusjonen
p
z 7 x + x2 + a
(2.44)
Dermed s
a kan dx uttrykkes som



x
x + x2 + a
z
dz = 1 +
dx =
dx =
dx
2
2
2
x +a
x +a
x +a
Dette medfrer at
Z
I=

dx
x2 +a

dx

=
x2 + a

dz
z

sette inn kan alts


. Med a
a integralet skrives


p
dz


= log |z| = log x + x2 + a + C
z

Variabelskifte so ble brukt i dette eksempelet leder til en svrt elegang


lsning, men det ikke like klart hvordan substitusjonen kom frem. ?? kan skrives
p
a formen
p
x2 + a 7 z x ,
og det skal vise seg at dette er nyaktig en av euler substitusjonene.
Proposisjon 2.5.3. Euler substitusjonene er brukt for a
a for beregne integraler p
men
Z
p
R(x, ax2 + bx + c ) dx
ved a
fjerne kvadratroten. Anta att ax2 + bx + c ikke er ulelukkende negativt og at
a 6= 0. Dersom

II

5.2 Euler substitusjon


1) a > 0 benytt variabelskiftet
p

ax2 + bx + c 7 x a + t

51

(2.45)

2) polynomet ax2 +bx+c har to distinkte reelle rtter , da kan substitusjonen


p
ax2 + bx + c 7 (x )t
(2.46)
benyttes.
3) c > 0 kan substitusjonen
p

ax2 + bx + c 7 xt + c

(2.47)

bli brukt.
Eksempel 2.5.7.
Z
I=

dx

(1 + x) 1 + x x2

Siden c > 1 s
a kan likning (2.47) brukes
p
1 2t
1 + x x2 7 tx + 1 x =
1 + t2
Differensialene v
are blir
dx = 2

dx
t2 t 1
dt
dt
= 2
2
2
2
(1 + t )
1 + t2
1+xx

2
t1
Siden 1 + x x2 = tx + 1 = t 1+t
Dette frer og til at 1 + x =
2
Setter vi alt inn f
as
Z
Z
Z
dx
dt
1
1 + t2 2 dt

=
2
I=
=
1 + x 1 + x x2
t2 2t 1 + t2
t2 2t


Z
Z
t
(t 2) t
dt
(t 2)0
+C
=
dt =

dt = log
t(t 2)
t
t2
t2

t2 2t
1+t2

substituere tilbake for x f


Ved a
as


Z

dx
1 + x x2 1
1

= log
+C
1 + x x2 2x 1
1 + x 1 + x x2
som fullfrer eksempelet.
Abels substitusjon
kvitte seg med rottegnet i integralet er a
benytte seg av Abels
En annen m
ate a
substitusjon. Den virker p
a integraler p
a flgende former
Z
Z
dx
dx

,
2
m
2
2
m
(ax + bx + c) ax + bx + c
(x + p) kx2 + l
la den deriverte
Trikset er a
av rottegnet
bli satt som den nye variablen. Teknik

ken baserer seg p


a at ( A ) = A0 / A . La alts
a henholdsvis
p
0
p
0
t=
ax2 + bx + c ,
t=
kx2 + l

52

II

5 Diverse substitusjoner

Eksempel 2.5.8.
Z
I=

(x2

La som i teksten t = ( x2 + t )0 =
begge sider med hensyn x f
as

dx

+ 2) x2 + 1

x
x2 +1

. Slik at t x2 + 1 = x. Deriverer vi

p
0
dt p 2
x +1 +t
x2 + 1 = 1
dx
dt p 2

Siste er jo bare hvordan vi definerte t. S


a vi har
x + 1 + t2 = 1. Ved a
dx
2
trekke fra t p
a begge sider og dele kan uttrykket skrives som

dt
dx
=
1 t2
x2 + 1

Tilslutt kan x2 + 2 uttrykkes p


a flgende m
ate x2 =
Innsatt kan integralet skrives som
Z

1
dx

=
2
x + 2 x2 + 1

1 t2 dt
1
=
2
2
2t 1t
2

Her kan en igjen benytte at arctanh x =


Z

1
2

log

t2
1t2

x2 + 2 =

dt
=
1 t2 /2

1+x
1x

2t2
1t2

2
t
arctanh
2
2

. S
a

1
2 +t
2x2 + 2 + x
dx
1

= log
+C
= log
2
2
2 2
2 t
2 2
(x + 2) x + 1
2x2 + 2 x

II

6.1 Brker

2.6

53

B R K E R O G K V A D R AT R T T E R

beregne
I denne delen skal vi se nrmere p
a noen spesielle teknikker for a
brker, kvadratrtter, og kombinasjoner av disse. Tidligere har en sett p
a en
rekke generelle substitusjoner, mens her blir noen substitusjoner som er spesielt
effektive p
a kvadratrtter og brker nevt.
Merk at de kvadratrttene som stort
sett nevnes i denne delen er p
a formen ax2 + bx + c , med rasjonelle konstanter.

2.6.1

BRKER

se litt p
vre forsiktig.
La oss begynne med a
a fortegn og viktigheten av a
Eksempel 2.6.1.
0

I=
1/2

2x3

dx
3x2 + 1

faktorisere polynomet under rottegnet. Ser raskt at x = 1 er


Vi begynner med a
en lsning s
a
2x3 3x2 + 1 = (2x3 2x) (x2 1)
= 2x2 (x 1) (x 1)(x 1)
= (x 1)(2x2 x 1)
bruke andregradsformelen, eller mer faktorisering kan polynomet skrives
Ved a
sette inn f
som 2x3 3x2 + 1 = (x 1)2 (2x + 1). Ved a
as
Z 0
Z 0
dx
dx

p
I=
=
2
(x 1) (2x + 1)
1/2
1/2 |x 1| 2x + 1
For 1/2 < x < 0 s
a har vi 3/2
a er |x 1|
< x 1 < 1 og 0 < 2x + 1 < 1. S
sette inn f
negativ p
a intervallet, mens 2x + 1 beholder fortegnet sitt. Ved a
as
Z 0
1
dx

I=
1

x
2x
+1
1/2

dx
Tanken er n
a a la t 7 2x + 1 slik at x = (t2 1)/2. Derivasjon gir dt = 22x+1
.
Z
I=
0

dt
1

t2 1
2

2
3

Z
0

dt
2
1

= arctanh
1 (t/ 3 )2
3
3

bruke definisjonen
Hvor vi brukte . Vi kan skrive svaret en del penere ved a
1+x
arctanh x = 12 log 1x
. Siden

1 + 1/ 3
( 3 + 1)2
3 +1 3 +1
= log

log
= log
= log(2 + 3 )
2
1 1/ 3
3 1 3 +1
S
a kan det endelige svaret skrives som
Z 0

dx
2
1
1

= arctanh = log(2 + 3 )
3
2
3
3
3
2x 3x + 1
1/2

sett inn refferanse til


standard arctanh integral

54

II

6 Brker og kvadratrtter

uttrykke integralet som en rasjonell funksjon hvor teller


Ofte er det mulig a
ax+b
og nevner er linere, ax+b
cx+d . I disse tilfellene er flgende susbstitusjon t = cx+c
elliminere rottegnet.
eller t2 = ax+b
cx+c nyttig for a
Eksempel 2.6.2.
Z

dx

(1 x) 1 x2

I=

Frst m
a vi skrive om integralet slik at vi f
ar frem det rasjonale uttrykket v
art
Z
Z
Z
dx
dx
dx
q

q
I=
=
=
2
1+x
1+x
(1 x) 1 x
(1 x) 1x (1 x)2
(1 x)2 1x
la t2 7
Ved a

1+x
1x

s
a f
ar vi at x =

t2 1
t2 +1

dx =
Mens 1 x =
Z
I=

2
1+t2

. Derivasjon gir oss at

4t
dt
(1 + t2 )2

. Dette er alt vi trenger. Innsetning gir n


a

dx
q
=
1+x
(1 x)2 1x

Z 

1 + t2
2

2

4t
dt
=
2
2
(1 + t ) t

Z
dt = t + C

Alts
a er
Z

dx

=
(1 x) 1 x2

1+x
+C
1x

som var det som skulle vises.

finne igjen
Uttrykk p
a formen liner kvadratisk kan forenkles ved a
den deriverte.
Eksempel 2.6.3.
Z
I=

p
(x + 1) 2x2 + 3x + 1

finne koeffisienter og slik at


Tanken er n
a at vi nsker a
x+1=a

d
(2x2 + 3x + 1) + = a(4x + 3) +
dx

Dette gir = 1/4 og = 1/4 via innsetning eller falkeblikk. Integralet kan alts
a
skrives som

Z 
1
1 p 2
I=
(4x + 3) +
2x + 3x + 1 dx
4
4
Z
1 2
1 p 2
2
2/3
=
(2x + 3x + 1) +
2x + 3x + 1 dx
4 3
4

II

6.1 Brker

55

p
R
R
Siden f 0 (x) f (x) dx =
v dv = 23 f (x)3/2 + C. Dermed blir uttrykket noe
enklere og integrere og vi kan fokusere p
a siste delen. Fokuset i denne delen er
p
a selve omskrivningen og ikke lsningen av det siste itnegralet. I korte trekk
brukes z 7 4x + 3 slik at
Z q
Z p
Z p
2
2
2
2
J=
2x + 3x + 1 dx =
z 1 dx =
cosh2 y 1 sinh y dy
16
16
Hvor vi benyttet oss av substitusjonen
z
7 cosh y i siste overgang. Siden
p
cosh2 x sinh2 x = 1 s
a er cosh2 x 1 = sin2 x . Alts
a f
as
Z
Z
1
1
cosh(2y) 1
dy =
sinh y cosh y y
J = sinh2 y =
2
2
2
1
Svaret blir alts
a 12 sinh(arccosh
arccosh
 y)y
 y. Vi har videre at sinh(arccosh y) =
p
p 2
y 2 1 og arccosh y = log y + y 2 1 . Dette gir

Z p

2x2

p
2
2
2
+ 3x + 1 dx =
(4x + 3) (4x + 3) 1
arccosh(4x + 3)
32
32

p
1
2
= (4x + 3) 2x2 + 3x + 1
arccosh(4x + 3)
8
32

Det hele og fulle svaret blir alts


a
Z
p
I = (x + 1) 2x2 + 3x + 1

p
1
1
2
2
3/2
2
= (2x + 3x + 1) +
(4x + 3) 2x + 3x + 1
arccosh(4x + 3)
6
32
128
En kan ogs
a bruke metoden med uttrykk p
a formen

liner
kvadratisk

Eksempel 2.6.4.
Z
I=

x+3
dx
+ 4x 3

4x2

Igjen s
a leter vi etter a skrive om teller, ved a finne den deriverte av 4x2 + 4x 3.
Legg merke til at 4x2 + 4x 3 = (2x + 1)2 4. S
a
x+3=


d 
(2x + 1)2 4 + = 4(2x + 1) +
dx

Dette gir = 1/8 og = 5/2. Alts


a kan vi skrive
1

5
2

Z
Z
1
(4x2 + 4x 3)0 dx
5
(2x + 1)0 dx

+
8
4
(2x + 1)2 4
4x2 + 4x 3
4x2 + 4x 3
Z
Z
1
du
5
dy
+
p
=
4
4
2 u
y2 4
R

Det frste integralet er et standard integral dx/2 x = x + C og det siste


integralet kan bli lst ved en valgfri euler-substitusjon.
Z

(8x + 4) +

dx =

56

6 Brker og kvadratrtter

2.6.2

II

K V A D R AT R T T E R

Vi har sett mange eksempler p


a integraler med kvadratrtter tidligere. Her
kan en bruke euler-substitusjoner, hyperbolske eller trigonometriske substitusjo forenkle
ner ogs
a videre. Her ser vi p
a noen f
a mindre kjente teknikker for a
integraler med kvadratrtter i seg.
Eksempel 2.6.5.
Z
I=

x2

dx

+ 1 x2 1

bruke v
Her kan vi begynne a
are magiske trigononometriske identiteter. Men
gange med den konjugerte
det er langt enklere a

1
a+ b
a+ b
1

=
=

ab
a b
a b a+ b
Hvor tredje kvadratsetning ble brukt, (a b)(a + b) = a2 b2 . Bruker vi det
samme p
a integrlet v
art f
ar vi

Z
Z 2
dx
x + 1 x2 1

dx
=
(x2 + 1) (x2 1)
x2 + 1 x2 1
Z p
p
1
=
x2 + 1 x2 1 dx
2
Hvor begge integralene kan bli lst via delvis integrasjon eller en luddig substitusjon. Forsec:II-Delvis-integrasjon det frste integralet kan substitusjonen
x 7 tan x benyttes d
a f
ar en
Z p
Z p
Z
dt
J=
x2 + 1 dx =
(tan t)2 + 1
=
(sec t)3 dt
(cos t)2
bruog dette integralet vil bli studert nrmere senere15 , fremgangsm
aten blir a
ke delvis integrasjon samt integralet av sec(x) som har blitt studert tidligere,
eksempel (2.5.4). Alternativt gir delvis integrasjon
Z
Z p
p
x2
2
2

x + 1 dx = x x + 1
dx
J=
x2 + 1
R 2
R
R
Herfra legger vi merke til at xx+11
dx =
x2 + 1 dx x21+1 dx. Hvor
2 +1
det frste integralet kjenner vi igjen som J. Alts
a er


Z p
Z
p
1
2
2

J=
x + 1 dx = x x + 1 J
dx
x2 + 1
Z
p
dx

2J = x x2 + 1 +
x2 + 1
1 p
1
J = x x2 + 1 + arcsinh x + C
2
2
Hvor det siste integralet har
R blitt behandlet i seksjon 4 om hyperbolske integraler
og funksjoner. Integralet
x2 1 dx kan lses tilsvarende og overlates til leser.
15 Se

eksempel (2.8.1) fra avsnitt (2.8) om trigonometriske integraler.

II

6.2 Kvadratrtter

57

Eksempel 2.6.6. Andre integraler kan skrives om fr de integreres


Z 2
x +2
I=
dx
x2 + 1
derasjonalisere uttrykket slik at vi kan skrive integranden som
Her velger vi a

x2 + 2 x2 + 2
x2 + 1 + 1
1
1

=
=
+
2
2
2
2
2
2
x +1
x +2
(x + 1) 2 + x
1+x
(x + 1) 2 + x2
Dermed kan integralet skrives som
Z 2

Z
p
x +2
dx

2 + 2 +

+
dx
=
log
x
x
2
2
x +1
(x + 1) 2 + x2

2
hvor vi benyttet oss av eksempel (2.5.6) og euler-substitusjonen z 7
x+ x + a .
Merk at integralet kunne blitt lst like elegant ved bruk av x 7 2 sinh t. Det
siste integralet samt generaliseringer overlates til leser.
En nyttig egenskap n
ar det kommer til brker er flgende
Lemma 2.6.1.
Z
Z
Z n2
1 1 + xn2
dx
x
dx
=
=
dx
n
n
2 0
1+x
1+x
1 + xn
0
0

(2.48)

Gitt at n er et reelt tall strre enn 1.


penbart vann. Ved a
benytte seg av
Bevis. For n = 2 holder proposisjonen a
substitusjonen 1/x s
a har en
Z
0

dx
=
1 + xn

1
xn dx
=
1 + xn xn x2

xn2
dx
1 + xn

(2.49)

Disse integralene m
a da vre like store. Addisjon gir da
Z

dx
+
1 + xn

Z
0

xn2
dx =
1 + xn

1 + xn2
dx
1 + xn

dele p
a
a 2 og bruke likning (2.49) fullfrer beviset.
Proposisjon 2.6.1. Gitt at n er positiv og a, b R, da er
Z
0

a + bxn2
a+b
dx =
1 + xn
2

1 + xn2
dx
1 + xn

(2.50)

Dette er en generalisering av lemma (2.6.1), og flger nesten direkte fra


se p
lemmaet. I stedet for a
a beviset studeres heller integralene i likning (2.48)
kunne veksle mellom disse ulike formene er nyttig og senere skal vi
for n = 4. A
R
regne med16 .
se at integraler p
a formen dx/(1 + xn ) er noe enklere a
16 Dette

vil komme godt frem i delen om kompleks integrasjon

58

II

6 Brker og kvadratrtter

bestemme integralene
Eksempel 2.6.7. Vi nsker a

dx
,
1 + x4

x2 dx
1 + x4

og
0

1 + x2
dx
1 + x4

merk frst at fra lemma (2.6.1) s


a er de to frste like og vi har sammenhengen

dx
=
1 + x4

I=
0

Z
0

x2 dx
1
=
4
1+x
2

1 + x2
dx
1 + x4

notere seg er at ved a


dele p
en annen nyttig ting a
a siste integralet p
a x2 s
a er
1
I=
2

1 + 1/x2
1
dx =
x2 + 1/x2
2

Z
0

1 + 1/x2
dx
(x x1 )2 + 2

bruke substitusjonen u 7 x x1 s
Ved a
a er du = (1 + 1/x2 ) dx og grensene
blir (, ). Dette er ofte en nyttig omskrivning.
I=

1
2

du
1
=
u2 + 2
2 2

dy
=
y2 + 1
2 2

hvor substitusjonen u 7 2 y ble brukt. Oppsumert har en alts


a at
Z

1 + x2
dx =
1 + x4
2
Z
2
x
dx

dx =
=
4
1 + x4
1
+
x
2
2
0

(2.51)
(2.52)

bruke samme fremgangsm


Ved a
ate kan en vise at flgende korollar
Korollar 2.6.1.
Z

xn1 + x3n1
1

dx =
1 + x4n
n 2 2

for alle n R+ .
Oppgaver
Z

1. Bestem integralene
0

dx
og
1 + x3

x
dx .
1 + x3

dx

+ 1) x2 + 2
b) Integralet ovenfor kan generaliseres til

2. a) Bestem integralet

Z
Ia =

(x2

dx
1

=
arctan
2
2
a1
(x + 1) x + a

Bevis generaliseringen ovenfor. Hva blir I1 ?



a 1x

+C.
x2 + a

II

6.2 Kvadratrtter

59

2 + x2
uten bruk av (2.50).
1 + x4
0
b) Bevis at likning (2.48) flger fra
Z
Z
a + bxn2
a + b 1 + xn2
dx =
dx ,
1 + xn
2
1 + xn
0
0
Z

3. a) Bestem integralet

Bevis s
a det motsatte alts
a at likningen over (2.6.1) flger fra likning (2.48).
?? 15

60

7 Delvis integrasjon

2.7

II

D E LV I S I N T E G R A S J O N

I denne delen skal vi igjen beske delvis integrasjon


Z
Z
uv 0 = uv u0 v ,
med tilhrende smarte knep og metoder. Som oppfriskning tas et lite eksempel
som vil komme til nytte senere
Proposisjon 2.7.1.
Z
ex
ex sin x dx = 2
+ 2
Z
ex
ex cos x dx = 2
+ 2



sin x cos x + C


sin x + cos x + C

Bevis. Resultatet ovenfor kan vises noe enklere ved hjelp av kompleks analyse.
Men dette blir frst sett p
a litt senere. Tar frst for oss frste integral, og kaller
det for I. En delvis integrasjon gir da
Z x
1
e
cos x dx
I = ex sin x = ex sin x

bruke delvis integrasjon atter en gang blir


Der u = sin x og v = ex /. Ved a
n
a integralet


Z
ex

1
cos x +
ex sin x dx
I = ex sin x

Legg merke til at en ender opp med det originalet integralet I p


a hyre side av
skrive ut og forenkle
likningen. Ved a
I=

1 x

2
e sin x 2 ex cos x 2 I

gange begge sider med 2 og legge til 2 I p


Ved a
a begge sider f
as
(2 + 2 )I = ex sin x ex cos x
Deles begge sider p
a 2 + 2 f
ar en som nsket at
Z

ex 
sin x cos x + C
ex sin x dx = 2
+ 2
gjenns vise det andre integralet. A
helt tilsvarende regning kan benyttes for a
kjenne integralet p
a hyre side er et velkjent indianertriks, og kommer ofte til
nytte.

II

2.7.1

7.1 Delvis kanselering

D E LV I S

61

KANSELERING

Anta at en skal integrere en funksjon som p


a kan deles opp til f = g + h.
Tanken er n
a a bruke delvis integrasjon p
a h slik at integralet av g blir kanselert.

Z
Z
Z 
g+h= g+ r g =r
Det spiller selvsagt ingen rolle om det er g eller h som blir utsatt for den delvise
integrasjonen. Eneste forutsetningen at h kan skrives som et produkt av to
funksjoner u og v 0 , p
a en slik m
ate at g = u0 v. Dette blir forh
apentligvis klarere
i det neste eksempelet
Eksempel 2.7.1. La oss se p
a det ubestemte integralet av funksjonen
f (x) = (1 + 2x2 )ex

Integralet best
ar av summen av to funksjoner hvor det ikke er mulig a integrere
prve delvis integrasjon, ulike
noen av delene alene. Leser oppfordres til a
substitusjoner eller andre metoder. Resultatet vil dessverre vre nedsl
aende,
ingen av metodene biter p
a integralet. Lsningen blir heldigvis a dele integralet
Z
Z
2
2
2 x2
J = (1 + 2x )e dx = 2x2 ex + ex dx ,
og benytte delvis integrasjon p
a siste leddet med u = exp(x2 ) og v 0 = 1. Da er
0
2
u = 2x exp(x ) og v = x. Ikke minst s
a er u0 v = 2x2 exp(x2 ).


Z
Z
2
2
2
2
J = 2x2 ex dx + xex x 2xex dx = xex + C
Hvilken delvis integrasjon som fungerer er noe som kommer med erfaring. La
oss se p
a et noe vanskeligere eksempel
Lemma 2.7.1.
Z

y
x2 y 2
dy = 2
2
2
2
(x + y )
x + y2

(2.53)

dele integralet opp i to deler


Bevis. La oss begynne a
Z 2
Z
Z
x + y 2 2y 2
1
2y 2
I=
dy
=
dy
+
dy
(x2 + y 2 )2
x2 + y 2
(x2 + y 2 )2


Z
Z
1
d
1
=
dy + y
dy
2
2
2
x +y
dy x + y 2
En kan n
a rimelig enkelt benytte seg av delvis integrasjon p
a siste leddet. Her
er u = y og v = 1/(x2 + y 2 ).


Z
Z
1
y
1
y
I=
dy +

dy = 2
+ C.
x2 + y 2
x2 + y 2
x2 + y 2
x + y2
se omskrivningen av integralet kan en se at
For a
Z
1
2x
dx = 2
+C
(x2 + a2 )2
x + a2

62

II

7 Delvis integrasjon

Via eksempelvis u = x2 + a2 , og at 1/u2 = (1/u)0 . Slik at


Z
Z
1
2y
d
1
dy
=
dy = 2
2
2
2
2
2
(x + y )
dy x + y
x + y2
og dette fullfrer beviset.
Kanseleringen var ikke strengt talt ndvendig, siden 1/(x2 + y 2 ) ikke er spesielt
vanskelig a integrere, men poenget er at den delvise kanseleringen gjorde bereg bruke denne teknikken er aldri eneste utvei, teknikken
ningene noe enklere. A
bruke produktregelen og kjerneregelen baklengs. Som
er helt tilsvarende som a
eksempel kunne en lst oppgavene ovenfor p
a flgende m
ate
2

f (x) = (1 + 2x2 )ex = x0 ex + x(ex )0 = (xex )0


Der produktregelen ble benyttet i siste overgang. Tilsvarende
2

x y
=
(x2 + y 2 )2

(x2 + y 2 )

d
d


yy
(x2 + y 2 )
d
y
dy
dy
=
.
(x2 + y 2 )2
dy
x2 + y 2

legge
Har man ikke langt mosegrodd skjegg kan disse tingene vre vanskelig a
merke til. Det er derfor delvis kanselering i mange tilfeller er vell s
a nyttig.
Oppgaver
Z
1. I = ex sin x + ex cos x dx

2. Gitt at17 =

ex dx beregn flgende integral18

x2 ex dx .

3. Bestem konstanten c og k slik at


Z

I = (x + c) ex x dx = kx ex x + C .

Z
4. I =

xex
dx
(1 + x)2

Z
5. I =

17 Integralet

log(log x) +

2
1

dx
log x
(log x)2

betegnes ofte som det gaussiske integralet og vil bli vist p


a ulike m
ater i Del III.
2

betegnes ofte som forventingsverdien til den gaussiske funksjonen ex og et hint for a
lse det er selvsagt delvis integrasjon. Men nok fotnoter n
a.

18 Integralet

II

7.2 Eksponentialfunksjonen

2.7.2

63

EKSPONENTIALFUNKSJONEN

Anta at du er blitt tatt til fange av en ond verdensleder og f


ar ikke dra hjem
fr du har bestemt flgende integral
Z
(x4 6x2 + 8x 3)ex dx
(2.54)
Er du noenlunde st p
a de grunnleggende integrasjonsteknikkene begynner du
vel allerede n
a og bite negler og skjelve. For den normale fremgangsm
aten er
delvis integrasjon, hvor i hver delvise integrasjon derives polynomdelen slik at
den tilslutt forenkles til en konstant. Metoden fungerer, men kan og fre til
svrt mange delvise integrasjoner og er svrt langtrekkelig. I oppgaven ovenfor
beregne integralet. Etter
trengs det eksempelvis 4 delvise integrasjoner for a
omstendig og monoton regning, kan svaret skrives som
Z
(x4 6x2 + 8x 3)ex dx = (x 1)4 ex + C
Legg merke til at polynomet p
a venstre og hyre side er av lik grad. Dette gjr
gjre flgende antakelse
det rimelig a
Proposisjon 2.7.2. Integralet av p(x)ex kan alltid skrives p
a formen
Z
p(x)ex dx = q(x)ex ,

(2.55)

der p og q har samme grad.


arlig polynom.
Bevis. La oss starte med funksjonen f (x) = q(x)ex , hvor p er et vilk
Derivasjon gir oss da at


f 0 (x) = q 0 (x)ex + q(x)ex = q(x) + q 0 (x) ex = p(x)ex
Der p(x) = q(x) + q 0 (x). Legg merke til at p og q begge er polynomer av samme
integrere p(x) har en n
grad. Ved a
a at
Z
Z
p(x)ex dx = f 0 (x) dx = q(x)ex + C
Hvor p og q selvsagt har samme grad.
La oss benytte oss av proposisjonen p
a et litt enklere eksempel
Eksempel 2.7.2.
Z
I=

(x2 1)ex dx

Polynomet av grad 2, og det er derfor m


a ogs
a svaret inneholde et polynom av
grad to. Vi gjr derfor flgende ansatz
Z
(x2 1)ex dx = (ax2 + bx + c)ex + C

64

7 Delvis integrasjon

II

derivere begge sider av likningen f


Ved a
as


(x2 1)ex = (ax2 + bx + c) + (ax2 + bx + c)0 ex

= ax2 + [b + 2a]x + [b + c] ex
Ved n
a og anta at hyre og venstre side er like m
a alle koeffisientene vre like.
Dette gir flgende likninger
1=a
0 = b + 2a
1 = b + c
Frste likning gir at a = 1. Neste likning gir at b = 2a = 2, og siste likning
gir at c = b 1 = 1. Dermed s
a er
Z
(x2 1)ex dx = (x2 2x + 1)ex = (x 1)2 ex + C
lse n-delvise integrasjoner med a
lse n + 1 likningsett.
En bytter alts
a ut a
lse disse p
Siden det finnes mange svrt raske og nyaktige m
ater a
a er det
ofte a foretrekke. Ved hjelp av gauss-elliminiasjon kan likninsettet ovenfor lses
som flger

1 0 0
1
1 0 0
1
1 0 0
1
2 1 0
0 0 1 0 2 0 1 0 2
0 1 1 1
0 1 1 1
0 0 1
1
hvor en kan lese mer om dette i enhver liner algebrabok.
Oppgaver
Z
1.
ex (x4 6x2 + 8x 3) dx
Z
2.

eax+b (x2 1) dx

3. La oss se p
a en svak generalisering av oppgaven. Ved a sette inn generelle
uttrykk for polynomene kan likning (2.55) skrives som
!
Z
n
n
X
X
x
k
x
k
e
ak x dx = e
ck x
+C,
k=0

k=0

hvor ak er kjente konstanter og ak 6= 0 for alle k og tilsvarende for ck .


uttrykke koeffisientene cn ved hjelp av an . Vis at ved a

M
alet blir n
aa
flge samme fremgangsm
ate som i eksempel (2.7.2) ender en opp med
n
X
k=0

ak xk = cn xn +

n1
X
k=0

ck1 + kck .

II

7.3 Tabell og reduksjonsformler

65

Dette er et likningsett som kan lses, vis at koeffisientene cn kan defineres


rekursivt som
cnk1 = ank (n k)cnk

k = 0, 1, , n 1 .

Hva er initialbetingelsen? Test ut iterasjonen p


a likning (2.54). Dersom ex
ax+b
i stedet hadde vrt p
a formen e
er det mulig a n
a definere cn rekursivt,
hva blir i s
afall rekursjonen?
Z
4.
ex xn dx ,
nN
0

2.7.3

TA B E L L

OG REDUKSJONSFORMLER

slippe a
gjennomfre delvis integraI forrige del ble det sett p
a ulike m
ater a
sjon p
a. Eksempelvis takket vre at ex men hva med integraler p
a formen
Z
(x3 1) cos x dx
(2.56)
0

Her kan en ikke lengre anta hva svaret skal bli da cos x og sin x deriverte har en
periode p
a fire og ikke en. Det en st
ar igjen med er alts
a vanlig delvis integrasjon.
Metoden vi skal bruke her ble visstnok frst tatt i bruk p
a 1960 tallet [? ] og
er ikke spesielt kjent. Noe av grunnen nok at metoden ikke revolusjonerer delvis
integrasjon; men heller automatiserer arbeidet.
La oss integrere funksjonen f g, og lar f (k) betegne den kte deriverte og g(k)
bruke delvis integrasjon f
betegne den kte integrerte. Ved a
ar en da
Z
Z
f g = f g1 f 1 g1
Z
= f g1 f 1 g2 + f 1 g2
=
= f g1 f 1 g2 + f 2 g3 + (1)k+1 f k gk+1 +
Dersom det eksister en n slik at f n = 0 eller gn = 0 stopper prossesen etter
n-delvise integrasjoner. Vi illustrerer dette med et eksempel.
Eksempel 2.7.3. Vi tar utgangspunkt i likning (2.56) og nsker frst a beregne
det ubestemte integralet
Z
(x3 1) cos x dx
vi velger n
a u = x3 , og v = cos x. Grunnen er enkel, etter fire derivasjoner blir
u null, mens v er periodisk og blir aldri null. De deriverte og integrerte er gitt i
tabell (2.3) Ved a gange sammen uk med vk+1 som vist med pilene i tabellen
kan integralet skrives som
Z
(x3 1) cos x dx = (x3 1) sin x 3x2 ( cos x) + 6x( sin x) 6 cos x
= (x3 1) sin x + 3x2 cos x 6x sin x 6 cos x
som nsket.

66

II

7 Delvis integrasjon
Tabell 2.3: Viser de deriverte til u = x3 1, og integrerte til v = cos x.
u

x3 1
3x2
6x
6
0

cos x
sin x
cos x
sin x
cos x

&
&
&
&

Dersom vi videre trenger a bruke delvis tre eller flere ganger kommer vi alltid
skrive opp de deriverte som i tabellen ovenfor. Denne teknikken kan og
til a
utlede taylorrekka til ulike funksjoner. Under viser vi dette for ex .
brukes til a
Eksempel 2.7.4. Vi studererer flgende funksjon
Z x
I=
et dt
0
x

P
a den ene siden vet vi at integralet er likt I = [et ]0 = ex 1, siden et per

definisjon er sin egen derivert. Men vi kan og bruke delvis integrasjon for a
bestemme uttrykket la v = 1 og u = et , da f
ar vi tabell (2.4) Via sammenlikning
Tabell 2.4: Viser de deriverte til u = x3 1, og integrerte til v = cos x.
u

v
t

e
et
et
et
et
bruke at
og a

&
&
&
&

1
t
t2 /2
t3 /6
t4 /24

Rx

tn dx = xn /(n + 1), s
a har vi
Z x
I=
et dt
x
0
1 3 t
1 4 t
1 2 t
x
t
t e +
e 1 = te t e + t e
2
6
24

 0
1
1
1
= ex x x2 + x3
x4 +
2
6
24
0

Deler vi begge sider p


a ex , eller ganger med ex f
ar vi
1 2
x +
2
1
= 1 x + x2
2

1 ex =
ex

1 3
x
6
1 3
x +
6

1 4
x +
24
1 4
x +
24

sette inn u = x, f
som gir oss rekkeutviklingen for ex , ved a
ar en tilsvarende
utviklingen for ex .

II

7.3 Tabell og reduksjonsformler

67

For flere kildehenvisninger og eksempler anbefales [? ] p


a det varmeste. La

oss ta en titt p
a en siste klasse av integraler, hvor tabell-integrasjon viser seg a
vre nyttig.
Eksempel 2.7.5. La oss betrakte flgende integral
Z
x3
dx
K=
(1 + x)5
bruke substitusjonenen u 7 x + 1, slik at
Den normale fremgangsm
aten er a
Z
Z
Z 3
x3
(u 1)3
u 3u2 + 3u 1
K=
dx
=
du
=
du
5
5
(1 + x)
u
u5
1 + 4u 6u2 + 4u3
1 (2x + 1)(2x2 + 2x + 1)
=
=
4
u
4
(x + 1)4
Hvor igjen integrasjonskonstanten ble droppet av plassmangel. Bruker vi heller
tabell-integrasjon kan K skrives p
a en mer givende form (suggestive form). Ved
Tabell 2.5: Integrasjon av f (x) = x3 /(1 + x)5
u

x3
3x2
6x
6
0

&
&
&
&

14
1
12
1
24
1
24

(1 + x)5
(1 + x)4
(1 + x)3
(1 + x)2
(1 + x)1

bruke verdiene fra tabell (2.5) kan K og skrives som


a
1
x3
3
x2
6
x
6
1

4 (1 + x)4
12 (1 + x)3
24 (1 + x)2
24 1 + x
"

 
2 
3 #
1 1
x
x
x
=
1+
+
+
4 1+x
1+x
1+x
1+x

K=

Dersom en har ett godt falkeblikk19 kan en se at


"
#
1+3
Z
1
x
x3
dx =
1 .
(1 + x)5
1+3
1+x

(2.57)

(2.59)

flge tankerekken ovenfor kan vi generalisere til flgende eksempel


Ved a
19 Legg

1+

merke til at

x
1+x

Pn

k=0

x
1+x

xn+1 1
x1

2

. Dette medfrer at vi kan skrive uttrykket i parentesen som

x
1+x

3

3 
k
X
x

1+x


= (1 + x) 1

x
1+x

3+1 

k=0

(2.58)
sette inn likning (2.58) inn i (2.57) f
Ved a
ar vi som nsket likning (2.59).

68

7 Delvis integrasjon

II

Eksempel 2.7.6.
Z
I=

xn
dx
(1 + x)n+2

Vi vil beregne integralet ved akkuratt samme fremmgangsm


ate som fr. Merk at
beregne dette og liknende integral.
senere vil vi se p
a andre m
ater a
1
n
n(n 1
xn
xn1
xn2

n + 1 (1 + x)n+1
n(n + 1) (1 + x)n
(n 1)n(n + 1) (1 + x)n1
n3
n(n 1)(n 2)
x


(n 2)(n 1)n(n + 1) (1 + x)n2

I =

Legg merke til at vi bare har et endelig antall ledd (n + 1, faktisk) siden tilslutt
s
a vil xn deriveres til null. Integralet kan dermed skrives som
"

1 
2

n #
1
1
x
x
x
I=
1+
+
+ +
(n + 1) (1 + x)
1+x
1+x
1+x
Som faktisk ble ganske pent. Siden utttrykket inneholder en geometrisk rekke,
kan vi forenkle summen til
"
#
n+1
Z
xn
1
x
I=
dx =
1
(1 + x)n+2
n+1
1+x
som er ganske stilig.

II

8 Trigonometrske funksjoner

2.8

69

TRIGONOMETRSKE FUNKSJONER

I denne delen vil det bli sett p


a en rekke spenstige trigonometriske integral,
men frst tas noen generelle r
ad20 . Anta at integralet kan skrives som
Z
R(sin x, cos x) dx
hvor R er en rasjonell funksjon. La oss ta en titt p
a noen spesialtilfeller
1) Gitt at R(sin x, cos x) = R(cos x) sin x dx, da foretrekkes substitusjonen
u 7 cos x.
2) Tilsvarende om R(sin x, cos x) = R(sin x) cos x dx da foretrekkes substitusjonen u 7 sin x.
3) Om R = R(tan x), anbefales tan x y.
4) Dersom R = R(sin2 x, cos2 x) s
a kan igjen substitusjonen tan x y benyttes, da er


Z
Z
1
t2
dt
R(cos2 x, sin2 x) dx = R
,
1 + t2 1 + t2
1 + t2
Fungerer intet av det ovenfor, m
a en nok krype til korset og benytte seg av den
universelle Weierstrass substitusjonen fra avsnitt (2.5.1). La oss se p
a unntaket
som bekrefter regelen
beregne flgende integral
Eksempel 2.8.1. En har tidligere prvd a
Z
dx
cos3 x
ved a bruke weiestrass, dette leder dog til en meget urovekkende delbrksoppspalting.
Et alternativ er a benytte delvis integrasjon med u = sec x du = sec x tan x dx,
og v 0 = sec2 x dx v = tan x.
Z
Z
sec2 x dx = sec x tan x sec x tan2 dx
Z
Z
= sec x tan x + sec x dx sec3 x dx
Hvor det i siste overgang ble brukt at tan2 x = sec2 x1. Ved a legge til integralet
av sec3 x p
a begge sider s
a er
Z
Z
3
2 sec x dx = sec x tan x + sec x dx
Det siste integralet har blitt beregnet i eksempel (2.5.4) og dermed s
a er
Z
1
1
sec3 dx =
sec x tan x + log |tan x + csc x| + C
2
2
som var det som skulle vises.
20 Deler

av
denne
introduksjonen
er
basert
IntegrationOfRationalFunctionOfSineAndCosine

p
a

http://planetmath.org/

70

II

8 Trigonometrske funksjoner

Dette var langt enklere enn a integrere den rasjonale funksjonen som oppst
ar
bruke weiestrass-substisjonen. La oss se p
ved a
a to integraler til som kan lses
ved ulike metoder
Herfra tas et par nyttige egenskaper til trigonometriske funksjoner
Proposisjon 2.8.1.
Z
Z

xR(sin x, cos2 x) dx =
R(sin x, cos2 x) dx ,
2
0
0
hvor R er en rasjonell funksjon.
Et elementrt bevis er vist i oppgave (5), og det anbefales p
a det sterkeste a
vise denne identiteten. Under gis det kort og stt bevis som ikke er like
prve a
intuitivt
vise identiteten for
Bevis. Det frste vi kan legge merke til er at det holder a
f (sin x) siden
R(sin x, cos2 x) = R(sin x, 1 sin2 x) = f (sin x)
La oss s
a definere flgende funksjon


f (sin x) .
g(x) := x
2
En kan n
a enkelt se at g( x) = g(x), siden
R sin( x) = sin x. Funksjonen
putte inn
g er alts
a symmetrisk omkring og vi har 0 g(x) dx = 021 . Ved a
definisjonen av g f
as
Z 

x
f (sin x) dx = 0 dx ,
2
0
R
Beviset fullfres ved a dele opp integralet og legge til 2 0 f (sin x) dx p
a begge
sider.
Et liknende integral er
Proposisjon 2.8.2. La p og q vre to relle tall, da er
Z

Z
f (p cos x + q sin x) dx =

Z
f ( cos x) dx = 2

hvor =

f ( cos x) dx
0

p2 + q 2 .

Bevis. Venstre siden kan skrives som


p
p cos x + q sin x = p2 + q 2

p
cos x + p
sin x = cos(x a)
p2 + q 2
p2 + q 2
R
21 Dette flger direkte fra korollar (2.3.2) med a = /2 eller ved a
la I =
g(x) dx. Via sub0
R
R0
stitusjonen x 7 x s
a er I =
g(x) dx =
g(u) du = I . Dette medfrer at
0

2I = I + (I) = 0 s
a I = 0. Alt dette flger fra at g( x) = g(x).

II

8 Trigonometrske funksjoner

71

Hvor sumpformelen cos(a b) = cos a cos b + sin a sin b ble benyttet med a =
arccos(p/ p2 q 2 ) = arccos p/. Integralet kan n
a skrives om som flger
Z 2a
Z 2

f ( cos x) dx
f cos(x a) dx =
a

2a

g(x) dx

g(x) dx +

g(x) dx +

hvor forkortelsen g(x) = f ( cos x) ble innfrt. De to siste integralene kan


skrives
Z 2
Z 2a
Z 2
f ( cos x) dx +
f ( cos x) dx =
f ( cos x) dx

2a

Hvor en enten bruker at cos x har en periode p


a 2, eller subsitusjonen x = u2
p
a frste integralet. Dette har en og sett fr, for eksempel i likning (2.21). Ved a
bruke dette har en n
a at
Z 2
Z
Z 2

f cos(x a) dx =
f ( cos x) dx +
f ( cos x) dx
0

0
2

Z
f ( cos x) dx = 2

=
0

f ( cos x) dx
0

p
spare plass.
hvor igjen forkortelsen = p2 + q 2 ble brukt for a
se den siste overgangen p
Det er flere m
ate a
a, den har for eksempel blitt
studert i detalj i oppgave (8). Dette fullfrer beviset.
N
a tas en artig trigonometrisk identitet som vil bli mye brukt fremmover.
Lemma 2.8.1.

arctan x + arctan

1
x


=

vise at f (x) = /2 er a
se at den deriverte er null.
Bevis. En m
ate a


d
1
1
1
1
1
f (x) =
+

.
dx
1 + x2
1 + (1/x)2
x2
1 + x2
1 + x2
Dermed s
a m
a f vre lik en konstant s
a lenge x 6= 0. Dette kan en se for
integrere f 0 (x) = 0. For a
bestemme denne konstanten s
eksempel ved a
a er
f (1) = 2 arctan(1). Siden tan(/4) = 1 s
a er arctan 1 = /4 da arctan y er
inversen av tan y. Dermed s
a er f (x) = f (1) = /2, siden f (x) er konstant.
betrakte flgende figur
Alternativt kan og identiteten og vises ved a

som er et stt lite bevis.

tan = x/1 = arctan x


1
tan = 1/x = arctan
x

++
=
2

+ =
2
1

arctan x + arctan
=
x
2

72

Korollar 2.8.1. La a, b R da er

 a+b 

arctan

1 ab
 a+b 
arctan a + arctan b =

arctan
+

1 ab

/2

sett inn refferanse i


fotnote.

II

8 Trigonometrske funksjoner

hvis

ab < 1

hvis

ab > 1

ellers

a kort inkluderes det her. Beviset har og vrt gitt p


a
Bevis. Siden beviset er s
eksamen p
a videreg
aende . Tilfellet ab = 1 har allerede blitt drftet i lemma (2.8.1). Sider ab = 1 impliserer a = 1/b, s
a er arctan 1/b + arctan b = /2.
Fra trigonometrien har vi
sin(x + y) = sin x sin y + cos x sin y
cos(x + y) = cos x cos y sin a sin b
bruke disse uttrykkene kan tan(x + b) skrives p
Ved a
a formen
(sin x cos y + cos x sin y)/(cos x cos y)
sin(x + y)
=
cos(x + y)
(cos x cos y sin x sin y)/(cos x cos y)
tan x + tan y
=
.
1 tan x tan y

tan(x + y) =

Setter vi n
a inn a = tan x og b = tan y f
ar vi
tan(arctan a + arctan b) =

a+b
,
1 ab

ta arctan p
og beviset fullfres ved a
a begge sider av likningen. Anta at ab > 1,
bruke lemma (2.8.1) to ganger har vi
da vil alltid a1 1b < 1. Ved a
1
1
+ arctan
a
b
 a+b 
 a+b 
= arctan
= arctan
+
ab 1
1 ab

arctan a + arctan b = arctan

vise. Dette fullfrer beviset.


som var det vi nsket a
Et annet viktig integral er flgende
Proposisjon 2.8.3.
Z

/2

log(sin x) dx = log 2

log(sin x) dx = 2
0

vise dette tas frst et lite hjelperesultat


For a
Lemma 2.8.2.
Z

/2

/2

log(sin x) dx

log(sin 2x) dx =
0

II

8 Trigonometrske funksjoner

73

Bevis. Dette lemmaet flger direkte fra likning (2.23) i proposisjon (2.4.2), se
oppgave (7) for et bevis. Vi viser likevell resultatet uten bruk av proposisjonen.
Via substitusjonen u 7 2x s
a er
/2

log(sin 2x) dx =
0

1
2

log(sin u) du
0

sette
Hyresiden av likniningen kan n
a skrives om via proposisjon (2.4.5) ved a
m = 0, n = 1 og f (x) = log(x) i likning (2.30). Da er
! Z
Z /2
Z /2
/2
1
log(sin 2x) dx =
2
log(sin x) dx =
log(sin x) dx
2
0
0
0
som nsket. Alternativt kan en og snitte det integralet p
a midten, og bruke
substitusjonen u 7 x. Da kan integralet skrives som

/2

log(sin u) du =
0

f (u) du +

f (u) du
/2

/2

/2

f ( u) du = 2

f (u) du +

=
0

/2

f (u) du .
0

Hvor f (u) = log(sin u), og substitusjonen u 7 x ble benyttet i andre over bytte
gang. Heldigvis s
a er f ( u) = f (u), siden sin x er periodisk. Ved a
tilbake til x som integrasjonsvariabel har en alts
a at
" Z
# Z
Z /2
/2
/2
1
log(sin x) dx =
log(sin x) dx
2
log(sin 2x) dx =
2
0
0
0
som var det som skulle vises.
vise proposisjon (2.8.3).
En er n
a klar til a
Bevis. Fra proposisjon (2.3.1) eller via substitusjonen u 7 /2 x har en
Z

/2

/2

log(sin x) dx =
0


log sin(/2 x) dx =

/2

log(cos x) dx

legge sammen disse integralene f


Ved a
ar en at
/2

Z
log(cos x) + log(cos x) dx =

/2

log(1/2 sin 2x) dx


0

Dermed kan integralet skrives som


/2

/2

/2

Z
log(sin 2x) dx

log(sin x) dx =

log(2) dx
0

benytte lemma (2.8.2) s


Siste integralet beregnes relativt enkelt og ved a
a er
Z

/2

/2

log(sin x) dx

log(sin x) dx =

log(2) dx
2

74

II

8 Trigonometrske funksjoner

trekke fra det opprinnelige integralet p


Beviset fullfres ved a
a begge sider. For
vise den siste likheten kan en skrive
a
Z
Z /2
Z /2
log(sin x) dx = 2
log(sin 2u) du = 2
log(sin x) dx = log 2
0

Hvor substitusjonen 2u 7 x og lemma (2.8.2) ble benyttet igjen.


Avslutningsvis tas et integral som vil dukke opp flere ganger senere
Eksempel 2.8.2. Gitt at er en vinkel slik at (, ) vis at
Z 1
dx

I() =
=
.
2
2 sin
0 x + 2x cos + 1
skrive om nevneren
Begynner med a
x2 + 2x cos + 1 = x2 + 2x cos + cos2 + sin2
= (x + cos )2 + sin2
Integralet kan n
a skrives som
Z 1
Z 1
dx
dx
=
,
2
2 + 2x cos + 1
2
x
0 (x + cos ) + sin
0
Dersom = 0 s
a blir integralet

0
Z 1
1
1
dx
=
=
2 + 02
(x
+
cos
0)
x
+
1
2
0
1
Hyresiden blir tilsvarende
lim

1
=
2 sin
2


lim

sin

1
=

1
2

Der det ble brukt at sin x x n


ar x 0. Anta n
a at 6= 0, vi kan da fritt benytte
substitusjonen22 u sin 7 x + cos og du sin = dx. Dette gir


1
Z 1
Z 1
sin() du
1
du
1
x + cos
=
=
arctan
2
2
2
sin 0 1 + u2
sin
sin
0 u sin + sin
0
sette inn grensene blir integralet p
Ved a
a formen



 cos 
1
1 + cos
I() =
arctan
arctan
sin
sin
sin
Dersom differansen av de to siste leddene blir /2 kan uttrykket skrives som
a
n
/2 sin , som nsket. Krukset blir a
a benytte addisjonsformelen fra korollar (2.8.1)23 ,


xy
arctan x arctan y = arctan
.
1 + xy
22 Hvorfor
23 Hvorfor

kan vi ikke bruke substitusjonen n


ar = 0?
xy
brukes arctan 1+xy
konsekvent her og ikke arctan

xy
1+xy

+ ?

II

8 Trigonometrske funksjoner

75

kun se p
Ved a
a brken (x y)/(1 + xy) s
a er x y = 1/ sin og


xy
1 + cos cos
1 .
sin

1+
=
=
= tan
1 + xy
sin
sin
sin
1 + cos
2
Der det ble benyttet i andre overgang at sin2 + cos2 = 1. Siste overgang kan
en se fra halv-vinkel formlene for sinus og cosinus sin = 2 sin(/2) cos(/2)
og 1 + cos() = 2 cos2 (/2). Integralet forenkles dermed til
Z


dx

1

=
=
arctan
tan
x2 + 2x cos + 1
sin
2
2 sin

som nsket.
Oppgaver

x cos2 x sin x dx

1. Bestem integralet I =
0

2. Vis at flgende likhet holder


Z

f x +x

 arctan x

dx =
x
2

Z
0


1
f xn + xn dx
x

for alle funksjoner f .


3. Vis at
1

Z
J=
0

holder der [0, 2].


Z
4. Bestem integralet

1 x2

dx =
1 x2 (sin )2
4 cos2 /2

arctan(x) dx
n
ar a som vanlig er en vinkel i
x2 2x sin(a) + 1

a (, ).
Z
5. Bestem integralet I =
0

x sin x
dx
1 + cos2 x

76

II

9 Logaritmiske funksjoner

2.9

LOGARITMISKE FUNKSJONER

I forrige del ble det sett p


a integral p
a formen
Z
p(x) log x dx
der lsningen var relativt enkel. Benytt delvis integrasjon og sett u = log x og
v = P (x), der P 0 (x) = p(x). I denne delen blir studiet av integraler p
a formen
Z

R(x) log dx
a

begynt p
a. Her er R(x) er en rasjonal funksjon p
a formen P (x)/Q(x) med P og
Q som polynomer. La oss frst ta et lite hjelperesultat
Lemma 2.9.1.

Z
0

log x
dx = 0
1 + x2

dele integralet. Legg frst merke til at


Bevis. Fremgangsm
aten blir a
Z
1

log x
dx =
1 + x2

log 1/x
dx
=
1 + (1/u)2 u2

Z
0

log 1/u
du
1 + u2

(2.60)

dele integralet i to f
Via substitusjonen x 7 1/u. S
a dx = 1/u2 du. Ved a
as n
a
Z
0

log x
dx =
1 + x2

Z
=
0

log x
dx +
1 + x2

log x
dx +
1 + x2

log x
dx
1 + x2
log 1/x
dx = 0
1 + x2

I andre overgang ble likning (2.60) ble benyttet og log 1/x = log x.
En generalisering av dette lemmaet kommer i en senere seksjonen for n
a kan
vise flgende
resultatet brukes til a
Proposisjon 2.9.1.
Z
0

log ax

dx =
log(ab)
x2 + b2
2b

(2.61)

bruke substitusjonen bw = x slik at dw = dx/b. Grensene


Bevis. Velger frst a
blir det samme
Z
Z
Z
log ax
log abw
1 log abw
dx = b
dw =
dw
(2.62)
x2 + b2
(wb)2 + b2
b 0
u2 + 1
0
0
Vi kan dele integralet i to siden log abw = log ab + log w, slik at
Z
Z
Z
log ax
log ab dw
1 log w
dx
=
+
dw
x 2 + b2
b
1 + w2
b 0 1 + w2
0
0

II

9 Logaritmiske funksjoner

77

Fra lemma (2.9.1) er det siste integralet null, mens det frste er likt
Z
i
log ab h
log ab dw

arctan()

arctan(0)
=
log(ab)
2
b
1+w
b
2b
0
og en er ferdige. Her brukte en at arctan x n
ar x og arctan 0 =
bruke lemmaet. Ved a
heller bruke
0. Vi kan og vise proposisjonen uten a
substitusjonen w 7 1/y i likning (2.62) f
as
Z
Z
Z
1 0 log ab/y dy
1 log ab/y
1 log abw
dw =
=
dy (2.63)
b 0 1 + w2
b 1 + (1/y)2 y 2
b 0
1 + y2
ta gjennomsnittet av likning (2.61) og (2.63) f
Ved a
ar en
Z
Z
Z
log ax
1
log ay + log ab/y
log ab dy
dx
=
dy
=
x2 + b2
2b 0
1 + y2
2
1 + y2
0
0
som vist fr. Her ble det bare brukt at log A + log B = log AB.
La oss videre se p
a et integral som dukker opp i [? ].
Eksempel 2.9.1.
Z b
a

1 log(ab)
log x
dx =
log
(x + a)(x + b)
2 ba

(a + b)2
4ab

Integralet kan dog vises relativt enkelt via substitusjonen x 7 ab/t. Da er


Z
b

log ab/t
ab
dt =
ab(b/t + 1)(a/t + 1) t2

Z
a

log ab log t
dt
(t + a)(t + b)

ta gjennomsnittet av integralene f
Slik at ved a
as
Z b
Z b
log x
1
log x + log ab log x
dx =
dx
2 a
(x + a)(x + b)
a (x + a)(x + b)
Merk den fine kanseleringen som skjer med logaritmen i telleren. Det gjennst
aende integralet kan beregnes via delbrksoppspalting


1
1 (b + x) (x + a)
1
1
1
=
=

(x + b)(x + a)
b a (x + b)(x + a)
ba x+a
x+b
(2.64)
integrere begge sider av (2.64) fra a til b f
Ved a
ar en at
Z
a


b


dx
1
x+a
1
a+b
2a
=
log
=
log
log
(x + b)(x + a)
ba
x+b a
ba
2b
a+b

Integralet kan dermed skrives som




Z b
Z
1 b
log ab dx
1 log(ab)
(a + b)2
log x
dx =
=
log
2 a (x + a)(x + b)
2 ba
4ab
a (x + a)(x + b)
vise.
som var det en nsket a

78

II

9 Logaritmiske funksjoner

Eksempel 2.9.2. La oss se p


a et litt annet integral med en enda rarere substitusjon.
Z
0

log(1 + x)
dx
x2 x + 1

stange hodet mott dette integralet helt til en ser den magiske
Det er mulig a
substiusjonen som lser alt. Vi begynner med a anta at en substitusjon p
a formen
x 7 1 +

K
u+1

vil fungere. Dette vil forenkle logaritmen i nevneren siden




K
log (1 + x) = log 1 1 +
= log K log(u + 1)
u+1
Konstantent K m
a bestemmes slik at u(0) = 2 og u(2) = 0 (hvorfor?). Ved a sette
inn ser en at magisk nok vil K = 3 fungere. Legg merke til at dx = 3 du/(u+1)2
og at
x(u)2 x(u) + 1 = (u2 u + 1)

3
(u + 1)

Integralet blir dermed


Z
0

log(1 + x)
dx =
x2 x + 1

Z
2



Z 2
log 3/(1 + u)
log 3 log(1 + u)
du
=
du
u2 u + 1
u2 u + 1
0

ta gjennomsnittet av integralene, f
Ved a
ar en som fr at
1
2

Z
0

log 3 log(u + 1) + log(u + 1)


du =
u2 u + 1

Z
0

log 3 du
u2 u + 1

fullfre kvadratet
Hvor beregningen av integralet eksempelvis kan gjres ved a

Z 2
Z /3

2 3 sec2 y dy
2 
4 du

=
+
=
2
2
3
6
3
/6 3 tan y + 3
0 (2u 1) + 3

I andre overgangen ble substitusjonen 2u1 = 3 tan y du = 3 /2 sec2 y dx


brukt. I tillegg s
a er 1 + tan2 x = sec2 x som vanlig. Ved a sette inn og rydde opp
f
as alts
a

Z 2
Z 2

log(1 + x)
log 3 du

dx
=
= log 3
2
2
3
0 x x+1
0 u u+1
Disse to integralene er begge eksempler p
a integral som kan bli lst via den
samme substitusjonen. La oss generalisere dette, men frst et lite lemma
Lemma 2.9.2. La P (x) vre et polynom da tilfredstiller P

 2
(d u + dc)2
K
c

= P (u)
P
d2 u + cd
d
Kd2

II

9 Logaritmiske funksjoner

79

hvor K = (c + ad)(db + c) 6= 0, hvis og bare hvis P er p


a formen
P (x) = Ax2 +

Ac2 + Cd2 AK
x+C,
dc

hvor d 6= 0. Dersom c = 0 s
a tilfredstilles funksjonallikningen kun dersom P kan
skrives p
a formen
P (x) =

A 2
x + Cx + A ,
ab

(2.65)

Hvor A og C velges fritt, og a, b 6= 0.


vise dette overlates som en oppgave til leser. Fremgangsm
sette
A
aten blir a
inn et generelt polynom i likningen, og se at hyre side vil ikke vre et polynom
om ikke graden er to. Dette frer til flgende generalisering
Theorem 2.9.1. Gitt at P (x) = Ax2 + Bx + C hvor
B=

AK Ac2 Cd2
,
dc

(2.66)

med K = (ad + c)(db + c). Da holder flgende likhet


b

log(dx + c)
1
dx =
P (x)
2

log K
dx
P (x)

for alle A, B. Dersom c = 0 da er


Z
a

log(dx)
1
dx =
P (x)
2

Z
a

log K
dx
P (x)

med K = dab, for alle polynomer p


a formen P (x) =
C kan velges fritt.

A
ab

x2 + Cx + A. Hvor A og

Dette kan vises direkte ved a benytte seg av substitusjonen u 7 c + K/(x +


vise at graden av P maksimalt kan vre to overlates til
c). Men det i tilleg til a
sette inn et generelt andregradspolynom inn i likning (2.66) kan en
leser. Ved a
vise flgende
Fra dette korollaret faller ?? direkte ut ved a sette A = ab og C = (a + b) inn
i likning (2.65)

80

II

10 Ulike tips og knep

2.10
2.10.1

ULIKE TIPS OG KNEP


REKURSJONER

OG FUNKSJONSFLGER

I avsnitt (2.7.3) ble delvis integrasjon gjort enklere ved a skrive om integralet
som en funksjonsflge. I denne delen skal vi se nrmere p
a slike flger, og
hvordan de kan leses direkte uten bruk av delvis integrasjon.
Eksempel 2.10.1.
Z
In =

cos nx dx = 0 ,

n Z/{0}

(2.67)

Dette eksempelet overlates til leser. Men viser en svrt enkel rekursjon In =
In1 .
Proposisjon 2.10.1.
Z
In =
0

/2

2
2
2(1)n

+
sin nx
3
5
2n + 1
dx =

sin x

dersom

n = 2m

dersom

n = 2m 1

gitt at m N.
Bevis. Vi begynner med den trigonometriske identiten
sin(m + 2)x sin mx = 2 sin x cos(m + 2)x ,
bruke dobbel-vinkel identitene. Ved a
dele likheten p
som vises ved a
a sin x f
as
sin mx
sin(m + 2)x

= 2 cos(m + 1)x .
sin x
sin x
dele p
Vi m
a herfra anta at x 6= 0 slik at vi unng
ar a
a null. Tilfellet hvor m = 0
integrere begge sider av likheten f
betraktes derfor for seg selv senere. Ved a
as
Z /2
Z /2
Z /2
sin(m + 2)x
sin mx
dx
dx = 2
2 cos(m + 1)x dx , (2.68)
sin x
sin x
0
0
0
definere
som likner en del p
a en rekursjonslikning. Ved a
Z /2
sin mx
Im =
dx
sin x
0
har vi alts
a vist at
Im+2 Im =

sin (m + 1)
m+1
2

(2.69)

hvor vi bare har brukt definisjonen av Im , og beregnet integralet p


a hyre side
av likning (2.68). Tanken er n
a at studerer tilfellet hvor m er odde og like hver
for seg. Dersom m er odde kan en skrive m = 2n 1. Rekursjonen gir da
I2n+1 I2n1 =

sin (2n) = 0
2n
2

II

10.1 Rekursjoner og funksjonsflger

81

Gitt at n N s
a er alts
a I2n+1 = I2n1 . Rekursjonen sier alts
a at dersom m er
odde er alle odde potenser like. For eksempel for m = 5 s
a f
as I5 = I4+1 =
fullfre utregningen m
I41 = I2+1 = I21 = I1 . For a
a en alts
a bestemme I1
Z

/2

I1 =
0

sin 1 x
dx =
sin x

/2

dx =
0

.
2

Alts
a har en at n
ar m = 2n 1 hvor n N s
a er
Z /2
Z /2
sin(2n + 1)x
sin(2n 1)x

dx =
dx =
sin x
sin x
2
0
0
som nsket. La n
a m vre like i likning (2.69), da er m p
a formen 2n, n N
I2n+2 I2n =

2
sin (2n + 1) =
(1)n .
2n + 1
2
2n + 1

vise siste overgang kan en for eksempel bruke dobbel-vinkel identitene


For a
sin

(2n + 1) = sin n cos


+ cos n sin
= cos n = (1)n
2
2
2

Rekursjonen sier alts


a at hver gang vi minker potensen med 2, s
a legger vi til en
faktor 2(1)n /(2n + 1)
2(1)n
+ I2n2
2n + 1
2(1)n
2(1)n2
=
+
+ I2n4
2n + 1
2(2n 2) + 1

I2n =

n1

X (1)n
2(1)n
2(1)1
+ +
+ I0 = 2
2n + 1
2+1
2n + 1
k=0

siden I0 = 0. Dette fullfrer tilfellet hvor m er like, og ogs


a beviset.
Lemma 2.10.1.
/2

Z
Im =
0

sin 2mx
cos x dx =

sin x

0
m
|m| 2

dersom

m=0

dersom

m Z/{0}

gitt at m N.
Bevis. Det er klart fra definisjonen at I0 = 0 siden sin 0 = 0. Siden sin(x) =
sin x flger det at Im = Im slik at det holder a betrakte Im for n N. Igjen
s
a betrakter vi differansen Im+1 Im for n 0.
Z

/2

Im+1 Im =
0

sin(2n + 2)x sin 2nx


cos x dx
sin x

/2

2 cos(2n + 1)x cos x dx

=
0

/2

cos(2n + 2)x cos 2nx dx

=
0

82

10 Ulike tips og knep

II

bekrefte
herfra kan vi bruke et resultat som vi lar det vre opp til leser a

Z /2
0
dersom m Z\{0}
cos 2mx =
/2
dersom m = 0
0
Dermed s
a er Im+1 = Im , s
afremt m 6= 0. Vi har at
Z /2
Z /2
sin 2x

I1 =
cos x dx = 2
cos2 x dx =
sin
x
2
0
0
R /2
Der dobbel-vinklel identiten ble brukt sin 2x = 2 cos x sin x og at 0 cos2 x dx =
R /4
cos2 x + sin2 x dx. Fra dette flger det at Im = /2, n
ar m 6= 0 og Im = 0
0
som nsket.
Proposisjon 2.10.2.
2
Z
Z 
1 cos mx
sin mx
dx =
`2 (m) =
dx = |m|
sin x
1 cos x
0
0

(2.70)

Hvor m Z.
Bevis. Frste likhet flger fra at sin2 x + cos2 x = 1 s
a (sin mx)2 = 1 cos mx,
2
betrakte
og (sin x) = 1 cos x. Vi har at cos(m) = cos m, s
a det holder a
m = 0, 1, . . . For m = 0 har vi
Z
Z
1 cos 0x
11
I0 =
dx =
dx = 0
1

cos
x
1

cos x
0
0
drfte tilfellet m N. For a
bevise
som stemmer. S
a herfra gjennst
ar det bare a
vise at (In+1 + In1 )/2 = In .
dette ser vi p
a en litt annen rekursjon, vi nsker a
Sumformelen for cosinus er
cos(A B) = cos A cos B sin A sin B
bruke likningen har vi at
Ved a

cos(n + 1)x + cos(n 1)x = cos nx cos x sin nx sin x
(2.71)

+ cos nx cos x + sin nx sin x = 2 cos nx cos x
Som vi skal se forenkler dette utregningene v
are noe
Z
1
1 cos(n 1)x
In+1 + In1
1 cos(n + 1)x
=
+
dx
2
2 0
1 cos x
1 cos x


Z
1 2 cos(n + 1)x + cos(n 1)x
=
dx
2 0
1 cos x
bruke likning (2.71) f
Ved a
ar vi n
a at
Z
1 cos nx cos x
=
dx
1 cos x
Z0
(1 cos nx) + (1 cos x) cos nx
=
dx
1 cos x
Z0
1 cos nx
=
dx = In
1 cos x
0

II

10.1 Rekursjoner og funksjonsflger

vise. Siste overgang flger fra at


siden som var det vi nsket a
1
[sin
n

0]
=
0
for
alle
n

N. N
a har vi vist at
n
In =

83
R
0

cos nx dx =

In+1 + In1
2In = In+1 + In1 In In1 = In+1 In
2

Dette beskriver en aritmetrisk progresjon, da differansen mellom to p


aflgende
ledd er konstant. Vi har alts
a In = I0 + (n 0)d = nd, hvor d er differansen
mellom to p
aflgende ledd og I0 = 0. Differansen mellom to p
afglende ledd er
Z
1 cos x
d = I1 I0 = I1 =
dx =
0 1 cos x
og dette fullfrer beviset.
En triviell triviell generalisering flger fra theorem (2.4.1).
Z

`2 (m) =
m

sin kx
sin x

2

dx =
m

1 cos kx
dx = |knm| ,
1 cos x

(2.72)

hvor k, n, m Z. Men dette ser bare mer rotete ut. Fra resultatene ovenfor kan
sprre seg om
det vre fristende a
Z

`k (m) =
0

sin mx
sin x

k

har en pen lukket form, eller en lukket form i det hele tatt. Vi legger merke til
at integralet er null n
ar potensen er odde og integranden jevn
`1,3,5, ,k (2m) = 0 ,
hvor k 2N 1 og m Z. Via tilsvarende regning som ovenfor kan en vise at


1 + 3k 2
4


`4 (m) = m 1 + 2m2
3


`5 (k) =
27 + 50k 2 + 115k 4
192


`6 (m) =
m 4 + 5m2 + 11m4 .
20
`3 (k) =

vise den generelle formen


Hvor igjen n 2N 1 Derimot a
Z 
`m (n) =
0

sin nx
sin x

bm(11/n)/2c

m
dx =

X
k=0

(1)

  m

m
2 (n + 1) kn 1
k
m1

er utenfor denne bokens hensik,t for et bevis se [5] . Her betegner bxc gulvfunksjonen (floor function) som vi vil f
a et gjennsyn med i del III. Kort fortalt rundet
denne funksjonen ethvert tall ned til det nrmeste heltallet b3.7c = 3, b0.5c = 0
ogs
a videre.

http://math.stackexchange.com/
we-simplify-int-0-pileft-frac-sin-nx-sin-xrightmdx

84

II

10 Ulike tips og knep

2.10.2

NYTTIG

FUNKSJONALLIKNING

Lemma 2.10.2. Gitt at R(x) er en funksjon slik at


 
1
1
R
= R(x)
x x2
og f (x) en vilk
arlig funksjon, da holder flgende
Z
Z 1

R(x)f (x) dx =
R(x) f (x) + f (1/x) dx
0

Bevis. Lemmaet flger direkte fra proposisjon (2.5.2) eller likning (2.38) med
g(x) = R(x)f (x) og S = 1.
Z
Z 1
R(x)f (x) dx =
R(x)f (x) + f (1/x)R(1/x)/x2 dx
0

0
1


R(x) f (x) + f (1/x) dx

=
0

som nsket. Vi kan og flge samme fremgangsm


ate som i proposisjonen og dele
opp integralet
Z
Z 1
Z
R(x)f (x) dx =
R(x)f (x) dx +
R(x)f (x) dx
0

1
1

Z
R(x)f (x) dx

=
0

1
1

Z
R(x)f (x) dx +

R(y)f (1/y) dy

R(1/y)
f (1/y) dy
y2

0
1


R(x) f (x) + f (1/x) dx

og i nest siste overgng y 7 1/x benyttet og at R(1/y)/y 2 = R(y). Som vist


fr.
Fra dette flger flgende vidunderlige theorem.
Theorem 2.10.1. Gitt at R(x) er en rasjonell funksjon som tilfredstiller funksjonallikningen
 
1
1
R
= R(x) ,
(2.73)
x x2
da holder flgende
Z

Z
R(x) dx

R(x)

= 2

(2.74)

R(x) log x dx
Z0

(2.75)

= 0

1
R(x) dx
(2.76)
2 0
Z0
Z

R(x) arctan x dx =
R(x) dx
(2.77)
4 0
0
R
Hvor r R. Likning (2.75) er gyldig viss og bare viss 0 R(x) dx konvergerer.
R(x)
dx
xr + 1

II

10.2 Nyttig funksjonallikning

85

vise likning (2.76) og (2.77), hvor de to frste likningene blir


Bevis. Velger a
benytte
overlatt til leser. Merk at alle likningene vises p
a samme m
ate ved a
bruke lemmaet med f (x) = 1/(xb + 1) f
lemma (2.10.2). Ved a
as


Z
Z 1
R(x)
1
1
xb
R(x)
dx
dx
=
+

xb + 1
xb + 1
(1/x)b + 1 xb
0
0


Z 1
1
xb
R(x)
dx
=
+
xb + 1
1 + xb
0
Z
Z 1
1
R(x) dx .
=
R(x) dx =
2 0
0
Beviset for siste likning gjres nesten tilsvarende igjen ved bruk av lemma (2.10.2)
n
a med f (x) = arctan x.


Z
Z 1
1
R(x) arctan x dx =
R(x) arctan x + arctan
dx
x
0
0
Z
Z

1
R(x) dx =
R(x) dx .
=
2 0
4 0
Der likning (2.74) og arctan x + arctan 1/x = /2 ble benyttet fra ??.
Korollar 2.10.1. Dersom R(x) tilfredstiller R(x) = R(1/x)/x2 s
a tilfredstiller
ogs
a r(x) = (log x)2 R(x) funksjonallikningen. Spesielt s
a er
Z
(log x)2n1 R(x) dx = 0 .
0

For alle n N, gitt at

R
0

R(x) dx konvergerer.

Bevis. Siden (log 1/x)2 = ( log x)2 = (log x)2 flger det direkte at

r

1
x

1
=
x2


2  
1
1
1
log
R
= (log x)2 R(x) = r(x)
x
x x2

vise siste del av korollaret. Grunntilfellet n = 1,


Vi
R bruker induksjon for a
R(x)
log
x
dx
=
0
flger
fra likning (2.75). Vi antar s
a at det stemmer for
0
Z
(log x)2k1 R(x) dx = 0 .
0

Alts
a at det eksisterer en eller annen funksjon som tilfredstiller R(x) = R(1/x)/x2
vise at dette medfrer at det stemslik at integralet ovenfor er null. Vi nsker a
skrive om har vi
mer for 2(k + 1) 1 = 2k + 1. Ved a
Z
Z


(log x)2k+1 R(x) dx =
(log x)2k1 (log x)2 R(x) dx
0
Z0
=
(log x)2k1 r(x) dx
0

Fra frste del av korollaret s


a er r(x) = (log x)2 R(x) en rasjonell funksjon som
2
tilfredstiller r(1/x)/x og resten flger ved induksjon.

86

II

10 Ulike tips og knep

tygge igjennom en betraktelig andel av


Theorem (2.10.1) kan benyttes til a
vanskelige integral. For eksempel flger lemma (2.9.1) direkte fra (2.75).
Eksempel 2.10.2. La oss vende tilbake til en gammel klassiker
/2

Z
0

=
b
(1 + (tan )
4

(2.78)

bruke substitusjonen x 7 tan f


hvor b R. Ved a
as
Z
0

/2

d
=
(1 + (tan )b

Z
0

dx
1
=
2
b
(1 + x )(1 + x )
2

Z
0

dx

=
2
1+x
4

I andre overgang ble likning (2.76) ble benyttet og R(x) = 1/(1+x2 ) tilfredstiller
selvsagt R(1/x)/x2 = R(x).
Et utvalg av funksjoner som tilfredstiller likningen er eksempelvis
R(x)

1
1
x
1
1
1+ 2
2

x
x
(1 + x2 )2
x +x+1
1 + x2

Sprsm
alet blir da, hvordan en kan finne flere funksjoner som tifredstiller likningen? En kan for eksempel legge merke til at
R(x) =

h(x) + h(1/x)
,
x

tilfredstiller likningen hvor h er en rasjonell funksjon. Tilsvarende s


a funger

g h(x) + h(1/x)
R(x) =
x
ogs
a hvor igjen g og h er rasjonelle funksjoner. En annen m
ate a konstruere slike
studere indeksene til polynomene, ved a
skrive ut
polynom p
a er a
x2
x2
=
(1 + x2 )2
x4 + 2x2 + 1
Her er koeffisientene til telleren (1), mens i nevneren s
a er koeffisientene (1, 2, 1).
Alts
a er indeksene det samme om de leses fra hyre eller venstre. En definisjon
av polynom med denne egenskapen er flgende
Definisjon 2.10.1. Et polynom q(x) er symmetrisk24 med indeks k dersom
p(x) = xk p(1/x)
Eksempelvis er indeksen til q(x) = x3 + x lik 4. Flgende proposisjon25 da vises.
24 Slike

polynom betegnes gjerne som palindom ogs


a. Det kan og opplyses om at alle av like
grad polynomer som er symmetrisk har komplekse rtter som ligger p
a enhetssirkelen. En
kan lese mer om dette her http://www.math.uconn.edu/kconrad/blurbs/galoistheory/
numbersoncircle.pdf.
25 Takk til flounderer for denne observasjonen. Beviset kan leses her.

II

10.3 Integral par

87

Proposisjon 2.10.3. La p(x) og q(x) vre to polynomer, hvor p(x) er symmetrisk


med indeks k 2 og q(x) er symmetrisk med indeks k. La R(x) vre definert
p(x)
q(x)

R(x) =

Da tilfredstiller R funksjonallikningen
 
1
1
= R(x)
R
x x2
for alle x.
Dette gir oss for eksempel direkte at
R(x) =

x4 + 3x3 + 3x2 + x
x7 x4 x3 +

tilfredstiller likningen.
Oppgaver
1. Vis at flgende funksjon

R(x) =

x2
x4 + 2ax2 + 1

r 

1
1+ 2
x

tilfredstiller differensiallikningen R(1/x)/x2 = R(x). Her er r > 1 og


a R.
2. Vis at
Z

dx
=3
1 + x2

Z
3. Bestem integralet
0

dx
3
=
2
1+x
2

Z
0

dx
1 + x2

xa xb
dx
, hvor a, b R
a
b
(1 + x )(1 + x ) 1 + x2

4. Vis at
Z
I=
0

(log x)2

1 + x2
dx = 2
1 + x4

Z
0

(x log x)2
dx = 2
1 + x4

Z
0

(log x)2
dx
1 + x4

88

2.10.3

10 Ulike tips og knep

INTEGRAL

II

PA R

I denne delen skal vi se nrmere p


a et av favorittintegralene til forfatter.
forst
Teknikken som benyttes er relativt enkel a
a; anta en har to integral
Z
Z
J = f (x) dx
og
K = g(x) dx
som hver for seg er vanskelig a beregne. M
alet blir n
a a konstruere et likningsett
av to uavhengige linerkombinasjoner av J og L
u1 (x) = a11 J + a12 L
u2 (x) = a21 J + a22 K
lse enn J og K hver for seg.
p
a en slik m
ate at likningsettet er enklere a
beregne er
Eksempel 2.10.3. Integralene som nskes a
Z
Z
J = cos log x dx
og
K = sin log x dx
som en har sett fr, kan delvis kanselering vre gull verdt. Dette gis en sjangse
og en beregner J + K
Z
J + K = cos log x + sin log x dx
Det brukes n
a delvis integrasjon p
a sin log x med u = sin log x og v 0 = 1, s
a
0
v = x og u = 1/x cos log x. Innsatt f
as da


Z
Z
cos log x
cos log x dx + x sin log x x
dx = sin log x + C
x
Tilsvarende for tilfellet for J K kan integralet skrives som
Z
J K = cos log x sin log x dx
ved samme fremmgangsm
ate som fr benyttes delvis integrasjon p
a cos log x.
Hvor u = cos log x, u0 = (1/x) sin log x og v 0 = 1, v = x.


Z
Z
sin log x
sin log x dx + x cos log x x
dx = cos log x + C
x
bruke disse to opplysningene har en alts
Ved a
a likningssettet
x sin log x = J + K
x cos log x = J K
ta gjennomsnittet av likningene f
Ved a
ar en
J=

x sin log x + x cos log x


2

(I)
(II)

II

10.3 Integral par

89

ta (I II)/2 har en ogs


og ved a
a
x sin log x x cos log x
2

K=
Dermed har en at
Z

x
sin log x +
2
Z
x
sin log x
sin log x dx =
2
cos log x dx =

x
cos log x + C
2
x
cos log x + D
2

beregne.
som var det en nsket a
beregne integralet p

Merk at det finnes mange alternative m


ater a
a. Ved a
sette u = log x f
as
Z
Z
J = ex sin x dx
og
K = ex cos x dx .
Herfra kan en enten benytte delvis integrasjon p
a gammlem
aten p
a hvert integral. En kan benytte teknikken med integral-par p
a J og K. Ellers kan en og
skrive om J og K til kompleks form. Merk at alle disse metodene krever noe
mer innsats enn metoden ovenfor
Dessverre finnes det ikke mange integraler denne metoden er nyttig for, og
derfor ender den opp som mer et stillig triks enn en nyttig teknikk.
Oppgaver
1. Bestem integralene
Z
I+ =

sin2 x dx

Z
og

I =

cos2 x dx

2. Bestem integralet
Z
I=

sin x
dx
cos x + sin x

finne et passelig J og benytte deg av et velkjent indianertriks. Via


Ved a
samme fremgangsm
ate bestem ogs
a integralet
/2

Z
I=
0

a cos x b sin x
dx .
b sin x + a cos x

3. Bestem integralet
Z
I=
.

3ex + 5 sin x + 10 cos x


dx
ex + 4 sin x + 3 cos x

90

II

11 Oppgavesammling II

2.11

O P P G AV E S A M M L I N G I I

2.11.1
Z
1.
Z
2.
Z

INTEGRAL
1
dx
x x2

cos x sin x
dx
cos x + sin x

Z
15.
Z

dx
1
+
sin x
0
Z
x+1
4.
dx
x7/2
Z
1
5.
dx
x3 + x7
Z e
6.
sin(log x) dx

x cos4 (x) dx

17.
1

ln x
x

2

18.

19.
0

20.

dx
1 + esin x

/2

4a3
dx
a4

x4

0
e

33.
1

1 + x4
1 + x6

Z
21.

34.

Z
1
ln x e + ex dx 22.
x

/2

8.

9.
0

Z
10.

x
dx
1 + cos x

Z
11.
12.
1

Z
13.

dx
x+x x

25.

dx
p

x 1 + n x

26.

log x 1
dx
x2 (log x)2
2x 3x
dx
4x

dx
Z 5

2
x +x
0 x + x 2x + 2 35.
dx
x8 + 1
Z 1 2014
x
dx
Z
x
x2
1 1 + e
36.
dx
2
Z
(x2 a2 ) + x2
0
sin x sin 2x sin 3x dx
0
Z
4x5 1

Z 4
37.
dx
7
5
x dx
(x + x + 1)2
0

7
7x + 7x
3
r
Z
Z
x1
x
dx
38.

dx
2 + 2x + 10
x
+
1
x+1
x
1

Z
sinh x + cosh x
dx
27.
cosh x sinh x

Z p
14.

24.

dx

p
Z

sin x
dx 23.
cos x + sin x

(x 1) dx
2x 1 log(2x 1)

9x

dx
4 + 2x

32.

Z
1 + cos x dx

31.
Z

ln(2x)
dx
x2 + 9

dx

(x + 2)2
dx
(x + 7)5

30.

Z 

Z
7.

29.

x2 (log x 1)
dx
x4 (log x)4

16.

3.

dx
x7 x

/2

Z
dx
tan x dx
a cos2 x + b sin2 x39.

Z r
x + x2 + 1
x+1

dx 28.
dx
x2 + 1
x3

40.

x sin x arctan ex
dx
1 + cos2 x

II

11.2 Oppgaver
Z

/2

1
1
41.
+
dx
log tan x
1 tan x
0
Z log 2 2x
e 2 + e2x
dx
42.
ex + ex
0
Z /2
sin2(sin x) + cos2(cos x) dx
43.
0

Z
44.

/2

45.

Z
0

8x3 cos4 x sin2 x


dx
2 3x + 3x2

50.

2 sin x + x cos x dx

47.

49.

51.

sin x
ex sin x cos x

csc x sin x dx

46.

48.

3/7

91

x3

y dy
(1 + y 2 ) (y 4 + y 2 + 1)

Z log 3

x2 + 1 x2 1

dx
x4 1
p
p
3
7
1 x7 1 x3 dx

52.
log 2

/4

/4

2.11.2

x sin x2 dx
sin x2 + sin(log 6 x2 )

x7 3x5 + 7x3 x + 1
dx
cos2 x

53.

1 x2
dx
2x4 2x2 + 1

O P P G AV E R

1. Vis at integralene
Z

I1 =
0

x2 y 2
dy dx og
(x2 + y 2 )2

I2 =
0

x2 y 2
dx dy
(x2 + y 2 )2

er ulike.
a integralet i frste oppgave, samt
2. I denne oppgaven skal vi se nrmere p
noen liknende integral
a) La a, y R, slik at a > 0, vis at
Z

x2 y 2
dx = 0
(y 2 + x2 )2

b) Vis videre at vi har


Z
0

a log a
1
x2 y 2
log x dx = 2
arctan
2
2
2
2
(y + x )
y +a
y

a
y


(2.79)

og drft tilfellene a = 0, a = 1 og a . Det kan vre fordelaktig a


bestemme
bruke resiltatet fra lemma (2.7.1). Bruk likning (2.79) til a
Z
a

x2 y 2
log x dx
(y 2 + x2 )2

92

11 Oppgavesammling II

II

vise nytten til


c) Det p
aflgende integralet blir ofte trukket frem for a
kompleks analyse. Men som vi skal se kan det og beregnes mer elementrt.
Bestem
Z
log x
I1 =
dx
(2.80)
(1 + x2 )2
0
Hvor en kan f
a bruk for en eller flere tidligere deloppgaver.
3. Beregn dobbeltintegralet

x+ x2 +a2

Z Z
K=
1

dt
dx ,
t

hvor a R. Og vis at lsningen kan skrives som


x
p
+ x log a x2 + a2 + C ,
K = x arcsinh
a
bruke definisjonen sinh = [exp(x) + exp(x)] /2.
bla ved a
4. Drft integralet

r2

sin d
,
+ R2 2Rr cos

for tilfellene n
ar r > R > 0 og R > r > 0.
5. Bestem flgende integral
Z

x2x

+1



(2x2 +1)
+ log x2x
dx

6. En definisjon av naturlige logaritmen er gitt som arealet under 1/t fra t = 0


til t = x.
Z x
1
log x :=
dt .
t
1
a) Vis ved a bruke definisjonen ovenfor at logaritmefunksjonen har flgende
egenskaper:
log 1 = 0
d
dx
log x = x1
log ar = r log a
log ab = log a + log b
b) Som en ekstra utfordring vis at
log(1 + x ) x ,
med likhet hvis og bare hvis = 1.
7.

x 0, 1

II

11.2 Oppgaver

93

a) La R vre ett reelt tall. Bestem




Z
1 + x
dx
log
I() =
.
2
log x(1 + x2 )
0

b) Vis at

1 + x4+

log

1 + x2+
1 + x2

15

dx

=
log x
4

2+ 6


q

3 5

Hvor en kan f
a bruk for frste deloppgave.
c)

Konvergerer flgende integral?


Z
log (1 + x ) dx
1 + x2
log x
0

8. Gitt at a og b er to reelle tall slik at a2 + b2 = 52 og


Z

2ab
x

dx
3
x
ab

tan =
a

Vis at cos 2 kan skrives p


a formen 8 2 + 8 1, hvor = ka, k R.
Finn ogs
a cos uttrykt ved a.
9. Vis at t m
a vre periodisk for at
Z

ex sin x dx =

ex cos x +

1
dx
t

10. Vis at
Z
0

cos2 x dx
=
1 cos x sin x

Z
0

sin2 x dx
1 cos x sin x

og bestem verdien av integralet.


11. Ls integralet
1

1 x2

dx
2
4
0 (1 + x ) 1 + x

med og uten bruk av substitusjonen 1 + x4 = (1 + x2 ) cos .


Z

12. (American Mathematical Monthly # 11457)


Vinkelen (x) er definert utifra flgende figur

94

II

11 Oppgavesammling II

(x)
a x

Vis at den gjennomsnittlige verdien for er gitt som


1
ba

(x) dx =
a

ba

4 b+a

der 0 < a < b. Virker utrykket logisk for grensetilfellene a 0 og b ?


sette
Argumenter. Hint: 1) I den delvise integrasjonen er det fordelaktig a
u = (x) og v = x ab/(a + b).
13. Gitt funksjonen
f (x) = arccos x + arcsin x. Vis at f 0 (x) = 0 og bestem
Z
f (x) dx hvor D er hele definisjonsmengden til f (x). (Alts
integralet,
a
D

alle lovlige x-verdier.)


14. Bestem integralet
Z

/2

I=


log

(1 + sin x)1+cos x
1 + cos x


dx ,

og vis at svaret er positivt.


15. Vis at
Z
0

dx
(1 +

sin x ) (1

+ cos x )

, > 0

er uavhengig av , og bestem integralet. Hva er problemet n


ar , 0?
16.
a)

Vis at
Z 1
 Z 1

Z 1 4
x +1
dx
2
=
Ax
+
Bx
+
C
dx
,
6
2
0 x +1
0
0 Ax + Bx + C

og bestem dermed koeffisientene A,B og C. Hva blir integralet?


bestemme integralene
b) Bruk a) til a
Z
Z
x4
dx
I0 =
dx og I4 =
6
1
+
x
1
+
x6
0
0
ha klart a).
Merk det er ikke ndvendig a

II

11.2 Oppgaver

95

17. Bestem flgende integral

Z
lim

sin t
dt
1 + cos2 (tx)

18. La f og g vre to integrerbare funksjoner p


a [8, 8] og la
Z 4
Z 4
I=
4f (x) 8g(x) dx +
8g(x) 4f (x) dx .
8

Vis at I = 2 , n
ar det oppgis at f er en likefunksjon, g er en oddefunksjon
og gjennomsnittsverdien av f og g p
a [0, 8] er 1.
19. La f vre en strengt voksende og kontinuerlig deriveerbar funksjon p
a
Z 5
[1, 5] slik at: f (1) = 0 og f (5) = 10. Anta at
f (x) dx = 7, bestem
1

integralet

10

f 1 (x) dx

der f

betegner inversen til f .

R /2
(sin x) 2 +1 dx
0
20. Bestem R /2

(sin x) 2 1 dx
0
Z
21. Vis at
a

2x dx
p

(x2

a2 )(b2 x2 )
Z

22. Bruk flgende integral


0

er uavhengig av a og b. Hvor a, b R.

22
x4 (1 x)4
vise at
dx til a
> . Vis s
a at
2
1+x
7

1
22
1
22

<<

7
630
7
1260
for eksempel benytte seg av at 1 < 1 + x2 < 2 n
ved a
ar x (0, 1).
23. Bestem flgende integral
Z
I=
0

x arctan x
2

(1 + x )

dx

Hvor er det gylne snittet, gitt som 2 = 4 sin(3/10) = 1 + 5 . Vis og

at flgende ulikhet holder 1/3 < I < 1/2. Hvor det kan vre fornuftig a
benytte seg av tilnrmingene fra forrige opgave.
24. Bestem monster-integralet
Z t
0

2
 x 1
(1 x2 ) ln(1 + x2 ) + (1 + x2 ) (1 x2 ) ln(1 x2 )
xe x2 + 1 dx
(1 x4 )(1 + x2 )

hvor t (0, 1).

96

II

11 Oppgavesammling II

25. Beregn integralet


ZZ

arctan exy dy dx

hvor D :
susen


(x, y) R : x + y 2 4x . Hint: En god tegning gjr ofte
2

4/3

Z
26. Vi lar D =

2x2 + x + 1
dx. Vis at eD kan skrives p
a formen m2
+ x2 + x + 1

x3

3/4

hvor m Q
27. En har tidligere sett at


a
cos(ax) + 1
n
ar
f (x) =
2
0
ellers

x
a
a

Er en sannsynlighetsfordeling. Alts
a at integralet over R er lik 1. I denne
oppgaven blir det sett nrmere p
a den moment-genererende funksjonen til
fordelingen f . Den er gitt som
Z
Mf (t) =
ext f (x) dx

for en kontinuerlig sannsynlighetsfordeling og kommer igjen i mange sammenhenger. Vis at den momentgenererende funksjonen til f kan skrives
som
Mf (t) =

1 sinh()
(1 + 2 )

Der = t/a. Vis videre at


0
lim Mf (t) = E(x) og
t0

lim Mf (t)

t0

00

= Var(x)

Hvor
Z

Z
xf (x) dx og

E(x) =

Var(x) =

2
x E(x) f (x) dx .

Stemmer resultatet generelt? Argumenter.


28. Vis at26
Z
0

/3

1/2

cos d =
( 3 cos sin ) sin
8

29. Vis at integralene er like


Z
Z
In =
x(cos x)2n dx =
0

3
.
2

x(sin x)2n dx

og bestem verdien av integralene gitt at n N er et naturlig tall.


26 Beviset

tilegnes Rob Johnson og kan leses her.

II

11.2 Oppgaver

97

30. Vis at dersom a N er et naturlig tall s


a er
Z a
dx

=
2
2
4
0 x+ a x

31. I denne oppgaven skal vi se p


a en rekke relaterte integral. Vi begynner i
det sm
a. Vi definerer funksjonene K og K som
Z

/2

K(m, n) =
0

sinm x dx
og K (m, n) =
sinn x + cosn x

/2

Z
0

cosm x dx
sinn x + cosn x

Hvor n, m N.
a) Bestem uttykk for K(1, 3) og K (1, 3).
b) Vis at K(n, m) = K (n, m) for alle n, m R. Hva er K(n, n)?
c)

Bestem flgende integral


Z

/2

dx
(sin x)2014
(sin x)2014 + (cos x)2014 1 + x

I=
/2

Hvor er en reell konstant.


32. Vis de fire likhetene
Z na
log(x a)
dx
x2 + a2
ma

a/m

a/n

m

log 2a2
arctan n arctan m
2a

log(x + a)
dx
x2 + a2

m

log 2a2
arctan 1/m arctan 1/n ,
2a

holder for alle reelle positve tall a, n, m som tilfredstiller nm = n + m + 1.


33. Gitt at 4c b2 = 2 /100 vis at
Z

a/2

b/2

x2

dx
= 5,
+ bx + c

hvor a/2 = ( 10b)/20.


a ubestemt integralene som studeres i denne
34. Dette blir et av de ytters f
delen. La
rq
q

f (x) =
x + 2 2x 4 + x 2 2x 4
Z
og bestem integralene

Z
f (x) dx og

35.

f (x) dx.

98

II

11 Oppgavesammling II
a)

Bestem flgende ubestemte integral

Z
.
1
x +1

dx
I=
2
x x +x+ x
x x

b)

Finn konstanter a og b slik at

Z b
.
x
1

= 0,
2 x
x
x
+
x
+
x
x
a

og b > a. Merk at integralene i a) og b) er h


arfint forskjellige.
36. Vis at integralet I kan skrives som
Z 
I=

arctan x
x arctan x

2
dx =

1
+C
tan( tan )

bruke
hvor er en funksjon som avhengiger av x. Det kan vre nyttig a
tan(a b) =

tan a tan b
1 + tan a tan b

(2.81)

som er sumformelen for tangens.


37. Dersom f (0) = 1 , f (2) = 3 og f 0 (2) = 5, bestem
Z
I=

xf 00 (2x) dx

/2

38. Integralet

sin(2x)3 cos(3x)2 dx kan skrives som

/6

 a b
b

. Bestem ab + ba + 1

39. La f og g vre to funksjoner som har kontinuerlige frste og andrederiverte p


a [5, 7]. Anta videre at funksjonene er integrerbare p
a intervalet.
Beregn
Z 7
I :=
f (x)g 00 (x) dx
5

n
ar det oppgis flgende:
(i)
f (x), f 00 (x) > 0 for alle x [5, 7].
(ii)
Funksjonen f /g har kritiske punkt i 5 og 7.
(iii)
For alle x [5, 7] s
a er g 00 (x) = 1/f 00 (x).
00
(iv)
g(x) = 2g (x) f (x)[g 00 (x)]2 for alle x [5, 7].
40. Bestem a og b slik at
Z
I=

b

2
(sin arccos x
dx

oppn
ar sin maksimale verdi. Hvor stor kan I bli?

II

11.2 Oppgaver

99

41. Vis at dersom f (x) = cos(log x) og g(x) = sin(log x) + cos(log x) s


a er
1

g(x)2 dx

f (x) dx =
0

Z 3
2+ 1y

dy = 2
(y 2)3 log y dy
2 1y
0
1
Z

Z
Z x
2
8t
dt
43. Bestem integralet eA
4xe2x dx dx, der A =
2
0 4t + 1
Z

42. Vis at J =

Z
44. Vis at
0

(1 y)

x29
(5x2

+ 49)

17

dx =

492

14!
.
515 16!

beregne
45. La oss ta enda en liten digresjon til numerisk integrasjon. For a
et integral kan en dele opp intervalet i n deler og konstruere et trapes p
a
hver del. Dette gir flgende formel
n1

X

h
T (n) =
f (a) + f (b) + h
f (xk )
2
k=1

a og b er henholdsvis nedre og vre grense, n er antall interval, h =


beregne
(b a)/n, og xk = a + hk. La oss bruke trapesmetoden til a
integralet
Z
A=

f (x) dx
a

gitt at integralet er begrenset, og f er en kontinuerlig funksjon. I tillegg


opplyses det om at
f (x) + f (a + b x) ,
er konstant for alle x i intervalet. Vis at
1)

lim T (`)

T (1)

T (`)

T (1) ` N

2)
Z
3)

f (x) dx =

T (1)

Dette fungerer ikke bare for trapesmetoden, men enhver metode som bruker lik steglengde for hele intervalet. Eksempelvis midtpunktmetoden,
Riemansummer, Simpsons metode og ogs
a hyrer ordens metoder. Derimot fungerer det ikke for Romberg integrasjon, eller Gauss-kvadraturer.

46. Anta at en har en beholder som er fyllt med en liten mengde vann. Vi
ker s
a temperaturen i beholderen. M
alet er studere hvordan det molare
metningstrykket (heretter kalt damptrykket) varierer som en funksjon av

100

11 Oppgavesammling II

II

gjre dette lager en en enkel model ved


en rekke kjente parameter. For a
hjelp av Clausius-Clapeyrons ligning
dp
S
=
dt
V
Denne beskriver hvor raskt damptrykket forandrer seg, alts
a dp/ dt Her er
S forandring i entropien27 og gitt som S = nf /t hvor f er smeltevarme (n
ar vann g
ar over til damp), n er antall mol og T er temperaturen.
Videre s
a er V forandringen i volumet og gitt som V = Vg Ve . Hvor
Vg er gassvolumet og Ve er vskevolumet. Tilnrmelsen V h Vg kan

benyttes da det antas en har mye mer gass enn vske. Gassen antas a
flge ideel gasslov pV = nRT , hvor n er mol, R er gasskonstanten og T er
temperaturen.
a)

Vis at Clausius-Clapeyrons ligning kan omskrives til



d 
Lf
log p = 2
dT
T R

anta at smeltetemperaturen Lf
Det a
ikke avhengig av tempereturen er gans
ke t
apelig. I stedet s
a er det rimelig a
anta at Lf avtar linert med temperaturen for lave temperaturer. 280 K < T <
440 K.
Lf (T ) = T .
Bruk vannets trippelpunkt som reffereanse P0 = 611 Pa ved T0 = 273.16 K og
vise at
uttrykket for smeltevarmen til a
p(T ) kan skrives som
p(t) = K T exp{Z/T }
og kartlegg dermed konstantene Z, K
og .
47. I denne oppgaven skal en studere nrmere konstanten28 , og noen av
dens egenskaper. Omega konstanten er definert slik at den oppfyller
e = 1 .
a) Vis at konstanten er unikt definert. Vis alts
a at likningen xex = 1
har nyaktig en lsning
I resten av oppgaven skal en se nrmere p
a funksjonene f (x) = x og
g(x) = x2 log x.

27 Entropi

gir et m
al p
a hvor mye uorden eller kaos som er i et system
funksjonen betegnes og ofte som LambertW(1), og en lengre artikkel finnes p
a wikipedia
http://en.wikipedia.org/wiki/Omega_constant.

28 Omega

II

11.2 Oppgaver

101

b) Vis at skjringspunktene mellom f og g er henholdsvis x0 = 0 og


x1 = e .
c) Vis at arealet avgrenset av f , g, x0 og x1 kan uttrykkes via konstanten som
A=

1
1 2
1
1
+
e + e3 =
6
9
62
93

benytte seg av definisjonen til .


Hvor det kan bli nyttig a
d) Omega konstanten kan og defineres rekursivt via
n+1 =

1 + n
1 + en

ansl
med startverdi 0 . Bruk startverdien 0 = 0 og to iterasjoner til a
a
integralet. En bedre startverdi er 0 = log 2, bruk denne og at e 8/3 til
ansl
a
a integralet. Benytt n
a en iterasjon.
48. Gitt at a = 1 og b2 = 1 1/2 log 1/2, der b > 0. Vis at
Z

xf (x) dx = A log(2) (1 + log 2) + B

I :=
a

og kartlegg dermed konstantene A og B. Her er f er implisitt gitt som


x2 + f ef = 1 ,

f > 1 .

benytte delvis integrasjon for a


oppn
Hint: Det kan lnne seg a
a x2 for
bruke substitusjon.
deretter a
49. I denne oppgaven skal en s
a vidt vre innom kvantefysikk. Merk at ingen
forkunnskaper utover integrasjon er ndvendig. Oppgaven dreier seg i
praksis om Carbon-14 datering, men som sagt fokuserer en her p
a det
matematiske. N
ar et C14 atom henfaller, (brytes ned) skiller det ut blant
annet ut en partikkel, (alfa-str
aling). Denne partikkelen har da et
potensial
V (r) =

2Ze2
40 r

Radien til partikkelen er gitt som r1 , mens str


alingen fra partikkelen n
ar
til til radien r2  r1 , Energien til partikkelen en da vre
E = V (r2 ) =

2Ze2
40 r2

og tilslutt s
a er hvor mye str
aling som blir avgitt fra partikkelen gitt som


Z q


2 r2

T
exp
2m
V
(r)

E
dr
=
} r1

102

II

11 Oppgavesammling II
a)

Vis at en kan skrive V (r) = Er2 /r og at log T kan uttrykkes som


Z 1 r
2
1

log T =
2mE r2
1 dx
}
x
r2 /r1

underske hvordan log T ker n


Videre s
a nsker en a
ar E avtar.
b) Forklar hvordan en kan skrive om integralet til
Z 1r
Z r1 /r2 r
Z 1 r
1
1
1
1 dx =
1 dx
1 dx ,
I=
x
x
x
0
0
r1 /r2
og, beregn integralet eksakt.
c) For lettere regning kan en benytte seg av en frekk tilnrming. Det
frste integralet er lik /2, og for det andre integralet har en at x <
a x1 1 x1 .
r1 /r2  1 slik at faktoren 1 bidrar tilnrmet ingenting, s
Bestem n
a et tilnrmet uttrykk for I og log T .
d)

Tilslutt leker en oss litt med algebra. Vis at




p
Z
log T
= 2 K1 K2 Zr1
E

hvor verdiene for K1 og K2 er henholdsvis


e2
K1
2mc2
40 }c

s
og

K2 4

e2
40 }c

mc2
}c

III

104

3.1

1 Introduksjon III

III

INTRODUKSJON III

I frste del dyppet du forsiktig trne ned i innsjen av integrasjonsteknikker.


Mens i andre del lrte du, og forh
apentligvis mestret en rekke teknikker for
holde deg flytende. Men jeg h
holde pusten for n
a
aper du er god til a
a skal
vi dykke dypt, dypt ned i mrket av integrasjonsmetoder. Denne delen vil ha
fokus p
a to hovedomr
ader, spesielle funksjoner og spesielle teknikker. Videre
vil du finne flere eksempler p
a hvor disse to omr
adene virker i skjnn harmoni
lse vanskelige integral. Vi vil studere rekkeutvikling, kompleks anaylyse,
for a
integraltegnet, omskrivninger til dobbel integraler og mye mer.

III

2.1 EulerMascheroni konstanten

3.2

105

V I K T I G E K O N S TA N T E R

I denne seksjonen blir et knippe viktige konstanter tatt opp. Det er viktig a
huske p
a at det a oppgi svaret som (3) eller (1/4) ikke er noe forskjellig fra et
svar som eller e. Forskjellen kommer i hva en legger i et elementrt uttrykk.
Herfra blir et par konstanter nevnt, som forfatter mener er elementre.

3.2.1

EULERMASCHERONI

K O N S TA N T E N

= 0.577 215 664 901 532 860 606 512 090 082 402 431 042 159 335 939 92
Definisjon 3.2.1. Eulers konstant ble frst introdusert av Leonard Euler (1707
1783) i 1734 som


1
1
1
:= lim 1 +
+
+ ... +
log(n)
(3.1)
n
2
3
n
Grunnen til at konstanten ofte omtales som EulerMascheroni er at selv om
det var Euler som frst studerte grensen var det Lorenzo Mascheroni (1750
1800) som fra 1790 skrev flere utdypende papir om konstanten, og dermed
grunnlaget for videre studier.
Nyaktig n
ar notasjonen , ble kom i bruk er uvist, men det som er sikkert
er at navnet er valgt grunnet konstantens nre relasjon til Gammafunksjonen
h
Y
1
z  z/n i
= zez
e
1+
(z)
n
n=1

(3.2)

hvor beviset kommer vi tilbake til senere. Formelen gir oss og med en gang
relasjonen 0 (1) = 0 (1) = .
Det er fortsatt ukjent om er et algebraisk eller trancendentalt. Faktisk vet
en ikke engang om er irrasjonelt, selv om det er sterke antakelser om det.
vre nrt tilknyttet gammafunksjonen spiller og funksjonen en
Foruten om a
viktig rolle innen Analyse (Bessel funksjoner, eksponensial-integral, summer,
osv), og dukker og hyppig opp i Tallteori.
Det er liten tvil om at selv om konstanten ikke er like kjent som eller e s
a
er den en av de aller viktigste matematiske konstantene vi har i dag.
Integral-representasjoner av gamma
Z
Z
2
x
=
e log x dx = 4
xex log x dx
0
0
 
Z 1
Z 1
1
=
log log
dx =
Hx dx
x
0
0
Z
Z 1
1
1
1
1
=

dx
=
+
dx
x1
x
e
xe
log
x
1

x
0
0


Z
1
1
x
=

e
dx k > 0
x 1 + xk
0
vise flgende
Som eksempel velger vi her a

106

III

2 Viktige konstanter

Eksempel 3.2.1.
Z
=


log log

1
x

ex log x dx

dx =
0

bevise flgende trengs definisjonen av (x) som dessverre kommer


Bevis. For a
frst senere og er gitt som
Z

tx1 et dt

(x) =
0

derivere likningen f
Merker oss at ved a
as
d
(x) =
dx

0 (1) =

tx1 et log t dt

et log t dt

(3.3)

Dersom vi n
a benytter oss av substitusjonen x 7 log(1/t) i likning (3.3) f
as
0 (1) =


log log

1
x


(3.4)

dx

vise at 0 (1) = kan gjres ved a


studere se p
A
a logartimen av likning (3.2),
ogs
a derivere. Men for yeblikket overlates dette til leser. Vi har alts
a
= 0 (1) =


log log

1
x

Z
dx =

ex log x dx

vise.
som var det vi nsket a
Vi kommer tilbake til 0 (1) n
ar vi diskuterer digamma-funksjonen.

3.2.2

C ATA L A N S K O N S TA N T
G = 0.915 965 594 177 219 015 054 603 514 932 384 110 774

Catalans konstant er navngitt etter Eug`ene Charles Catalan (1814-1894) og er


definert som
G :=

1
1
1
1
2 + 2 2 +
12
3
5
7

Konstanten dukker hyppig opp i problemer innne kombinatorikk, og inneholder


mange fine identiter. Det er ikke kjent hvorvidt G er irrasjonell, eller trancendental og noen ganger betegnes og konstanten som Catalan.

III

2.3 GlaisherKinkelin konstanten

107

Noen viktige identiter

(1)n
(2n + 1)2
n=0
Z 1
Z
arctan t
=
dt =
arctan et dt
t
0
0
Z /4
Z /4
=
log cot t dt =
log tan t dt

G = (2) =

0
/4

/4

log(sin 2t) dt =
0

=
0

Z
=
0

Z
=
0

log(cos 2t) dt
0

dx dy
1
+ x2 y 2
0
Z
log t
log t
dt =
1 + t2
1
+ t2
1

/4

t dt
sin t cos t

hvor er Dirichlet beta funksjonen.


Eksempel 3.2.2. I Del II ble flgende resultat bevist
Z
a

log ax
=
log
x 2 + b2
2b
b
0
bruke at
ved a
Z
0

log x
=0
x2 + 1

legge merke til at arealet fra 0 til 1 var like stort som fra 1 til
Dette s
a vi ved a
. Noe merkelig var at verdien av disse integralene aldri ble vist! Fra tabellen
over ser vi at integralene er lik G, og dette viser vi n
a med rekkeutvikling.
Bevis.

3.2.3

GLAISHERKINKELIN

K O N S TA N T E N

108

III

3 Spesialfunksjoner

3.3

SPESIALFUNKSJONER

3.3.1

G U LV

O G TA K - F U N K S J O N E N E

mappe
I matematikk og Informattik brukes gulv (floor) og tak (ceiling) til a
et reelt tall til det strste tidligere heltallet eller minste p
aflgende heltall.
Definisjon 3.3.1. La m Z og x R. Da er
bxc = m
dxe = m + 1

dersom m x m + 1
dersom m x m + 1

Definisjonen ovenfor kan ogs


a skrives som bxc = max{m Z | m x} og
bxc = min{n Z | m x}. Definisjonen ovenfor vil derimot vre noe mer
anvenbart I tilleg har vi fraktal delen {x}.
Definisjon 3.3.2. La x R. Da er fraktaldelen av x gitt som
{x} = x bxc
for alle x i 0 {x} 1.
x
1
0.5

bxc

dxe

1
0
3

1
1
4

{x}
0
0.5
0.141592 . . .

Eksempel 3.3.1.
I forhold til integrasjon er gulv og tak-funksjonene ofte brukt som en enkel
bytte mellom integraler og summer. Som et trivielt eksempel har en
m
ate a
Z n
n Z k+1
n
X
X
n(n + 1)
dxe dx =
k dx =
k=
2
0
k
k=0

k=0

hvor n N. Alts
a kan vi skrive summen av de n frste heltallene som et integral.
Proposisjon 3.3.1. La x R da er
Z x
bxcbx 1c
btc dt =
+ bxc{x}
(3.5)
2
Z 0x
1
bxcbx 1c
{t} dt = x2
bxc{x}
(3.6)
2
2
0
Rx
Rx
Bevis. Her trenger vi bare vise frste likning siden 0 {t} dt = 0 t btc dt =
Rt
1 2
a p = bxc, da er
2 x 0 btc dt. La n
Z

[t]dt =
0

p1 Z
X
k=0

p1
X
k=0

k+1

Z
[t] dt +

[t] dt =
p

k + p(x p) =

p1 Z
X
k=0

k+1

Z
k dt +

p(p 1)
+ p(x p)
2

p dt
p

III

3.1 Gulv og tak-funksjonene

109

Som var det vi ville vise siden bxc n = bx nc, n N og x p = x bxc =


{x}.
Proposisjon 3.3.2. La x R+ da er
Z x 
1
1
dt = 1 + log x + H
t

(3.7)

Hvor = b1/xc + 1 og Hn er summen av de n frste leddene i den harmoniske


rekken.
Korollar 3.3.1. La x 1 da er
Z x
0

1
t


dt = 1 + log x

(3.8)

og spesielt s
a er
Z 1
0

1
t


dt = 1

(3.9)

for x = 1.
vise dette overlates derfor til leser. For a
vise
Bevis. Korollaret flger direkte. A
proposisjonen setter vi y 7 1/t og = b1/xc.

Z
Z x 
Z
Z k+1
X
{y}
xk
1
y ( 1)

+
dt =
dy
=
dy
dy
2
2
t
y
x
y2
1/x
k
0
1/x
k=

(3.10)
Integralet kan skrives som


Z
y+1
1
=

x
+ log + log(x)
y2

1/x
skrive ut summen f
Ved a
ar vi en teleskoperende sum
Z k+1

X
X
xk
1
dy
=
log(k + 1) log(k)
= H log gamma
2
x
1
+
k
k
k=

k=

sette sammen de to forrige likningene f


ved a
ar vi alts
a




Z x
1
1
1
dt = ( 1)
x + log(x) + H == 1 + log x + H
t

0
som var det som skulle vises.
Theorem 3.3.1. La f vre en stykkevis kontinuerlig funksjon for x (0, 1) slik at
R1
|f (x)|/x dx < er endelig. Da har en
0
Z 1  
Z 1
1
dx
f (x)
f
=
dx
x 1x
x
0
0
hvor {x} fortsatt betegner fraktal delen av x.

110

III

3 Spesialfunksjoner

Bevis. Dette beviset tillegnes Olivier Oloa og kan leses i sin opprinnelige form
skrive om integralet til en sum
her. Som fr er frste steg a
Z 1  
Z 1/k
X
1
dx
dx
f
=
f ({1/x})
x
1

x
1
x
0
k=1 1/(1+k)
Z

k+1
X
du
=
f {u}
u(u
1)
k=1 k
Z k+1
X
du
=
f (u k)
u(u 1)
k=1 k
Z

X 1
dv
=
f (v)
(v
+
k)(v
+ k 1)
k=1 0
Z 1

X
1
f (v)
=
dv
(v + k)(v + k 1)
0
k=1
Z 1
dv
=
f (v)
v
0
Hvor bytte av summasjonstegnet og integraltegnet flger fra dominererende
konverges theoremet. La v (0, 1), og la N , en har da
N
X
k=1

X
f (v)
= f (v)
(v + k)(v + k 1)

k=1

1
1

v+k1
v+k


=

f (v)
f (v)

v
v+N

se
Som g
ar mot f (v)/v n
ar N . Mer at 1 = (v + k) (v + k 1) for a
teste at summen virkelig konvergerer har vi
delbrksoppspaltingen. For a



N
f (v)

X
f (v)
|f (v)|
|f (v)|
|f (v)|
1

=
f (v)

(v + k)(v + k 1) v
v+N
v
v+N
v
k=1

R1
som flger fra antakagelsen om at 0 |f (x)|/x dx var endelig. Siden absollutt
konvergens impliserer konvergens fullfrer dette beviset.
f
B
ade i beviset ovenfor og i theorem 4.22 ble f {x} = f ({x}) brukt, for a
a litt
penere notasjon.
Proposisjon 3.3.3. La a vre et reelt tall, da holder
Z 1s
{1/x}
dx
2 arcsin a
=
2 {1/x} 1 x
1

a
a
0
for alle a (0, 1].
Proposisjon 3.3.4.
Z
0

for all n N\{1}.

s
n

{1/x}
dx

=
1 {1/x} 1 x
sin(/n)

III

3.2 Gammafunksjonen

111

Korollar 3.3.2.
1

s
n

{1/x}
dx
=
1 {1/x} 1 x

Bevis. Dette integralet flger fra ?? med a = 1 eller proposisjon (3.3.4) med
n = 2.

3.3.2

GAMMAFUNKSJONEN

I denne delen skal vi se nrmere p


a -funksjonen og noen av dens egenskaper. For et historisk perspektiv anbefalles artikkelen [? ] p
a det sterkeste, og
for en mer utfyllende introduksjon anbefales [? ] denne delen er hovedsaklig
basert p
a disse kildene.
Det finnes mange funksjoner innen matematikk som bare er definert for hele
tall1 . Eksempelvis s
a kan summmen av de n frste naturlige tallene skrives som
T : N 7 N

T (n) =

n
X

= 1 + 2 + + (n 1) + n

k=1

ved hjelp av enkel algebra kan summen skrives p


a lukket form som
n(n + 1)
2

T (n) =

(3.11)

N
a er formelen bare gyldig for naturlige tall. Men likevel gir formelen en mulig
definisjon av hva det vil si og legge sammen de 5 21 frste naturlige tallene. En
legge sammen de n frste tallene, vil vre a
gange
naturlig fortsettelse til det a
sammen de n frste tallene
T : N 7 N

T (n) =

n
Y

k = 1 2 (n 1) n = n!

k=1

Det ble lenge lett etter et lukket uttrykk for produktet slik som likning (3.11) gir
en naturlig utvidelse for addisjon av de naturlige tallene. Motivasjonen er alts
a
gi mening til n! n
a
ar n ikke er et heltall.
konstruere en slik funksjon
Leonard Euler viste heldigvis at det var mulig a
som flger
Definisjon 3.3.3. La s = + it, hvor , t R. Vi definerer -funksjonen for
> 0 som
Z
(s) :=
ts1 et dt
0

Dette integralet betegnes ogs


a noen ganger som Eulers frste integral, eller
vise at denne funksjonen kondet Euleriske integralet (The Eulerian integral). A
vergerer dersom > 0 er vist i lemma (C.1.4) i Appendix C. Et sterkere resultat
er a vise at -funksjonen er analytisk for > 0, dette er og vist i appendiks C (se
vise at denne funksjonen virkelig er
lemma 34.15) i For n
a begrenser vi oss til a
vise at
en utvidelse av fakultetsfunksjonen. Vi nsker alts
aa
1 Slike

funksjoner kalles gjerne for aritmetiske- eller tallteoretiske-funksjoner.

112

III

3 Spesialfunksjoner

Proposisjon 3.3.5. -funksjonen tilfredstiller


(x + 1) = x(x)

xR

(3.12)

og dersom n N s
a er (n + 1) = n! med (1) = 1.
bruke delvis integrasjon s
Bevis. Ved a
a kan vi skrive
Z
Z

0
(x + 1) =
tx et dt = tx et
xtx1 et dt = x(x)
0

anta
Hvor et vokser raskere enn tx , x > 0 slik at tx et 0 n
ar t . Ved a
at n N f
as
(n + 1) = n(n) = n(n 1)(n 1)
= n (n 1) 2 1 (1)
= n! (1)
vise at (1) =
og det er trivielt a

R
0

et dt = 1.

finne funksjoner slik at f (1) = 1 og at f (x + 1) = xf (x) er ikke spesielt


A
vanskelig og det finnes hele funksjonsgrupper som tilfredstiller disse kravene.
Eksempelvis har vi
!
1x
d
1
2 
log
F (x) =
(1 x) dx
2x
2
eller
"
f (x) = eg(z) z


Y
n=1

z  z/m
1+
e
m

#1

Hvor i siste definisjon s


a er g alle funksjoner som tilfedstiller g(z + 1) g(z) =
+ 2ki k Z. Som en artig kuriositet f
as gammafunksjonen ved a velge den
enkleste funksjonen g som g(z) = z. Enda mer t
apelige eksempler inneholder
eksempelvis

1/x
0<x1

1
1<x2

1
2
<x3
G(x) =

(x

1)(x

2)
3
<
x4

..

.
vre akkuratt lik defiS
a hvorfor velges utvidelsen av fakultetfunksjonen til a
nisjon (3.3.3)? Svaret p
a sprsm
alet f
ar en kanskje fra at gammafunksjonen
er logaritmisk konveks. At en funksjon er konveks betyr dette at funksjonen er
synkende for alle verdier. Eventuelt dra et linjestykke mellom to vilk
arlige punkt
p
a funksjonen, da vil alle punkt p
a linjestykket ligge over funksjonen. Dette kan
formelt skrives som

III

3.2 Gammafunksjonen

113

y
4
3
2
1

x
3

Figur 3.1: x2 er konveks da alle punkter p


a den rd linjen ligger over funksjonen.

Definisjon 3.3.4. En funksjon f : (a, b) R er kalt konveks hvis for ethvert


par x, y (a, b) s
a gjelder ulikheten

f x + (1 )y f (x) + (1 )f (y)
for alle (0, 1).
Merk at kravet om logaritmisk konveksitet er langt kraftigere en kravet om
at en funksjon er konveks. At en funksjon er logaritmisk koveks betyr at den
vokser svrt raskt p
a omr
adet. For eksempel er x2 konveks mens log(x2 ) =
x log x er ikke konveks. Ingen polynomer vokser raskt nok, og et eksempel som
2
er logaritmisk konveks er ex . Gammafunksjonen vokser alts
a like raskt eller
2
raskere enn ex .
gi litt trening i a
bruke -funksjonen velger vi a
bevise at den er logaFor a
ritmisk konveks. Til det trengs flgende ulikhet
Lemma 3.3.1. (H
olders ulikhet). La p,q vre to positive reelle tall slik at
1 . Da for alle integrerbare funksjoner f, g : [a, b] R, s
a har vi
Z

b



f (x)g(x) dx

a

!1/p

b
p

|f | dx
a

1
p

+ 1q =

!1/q

b
q

|g| dx
a

Et bevis for ulikheten finnes i [? , thm. 3.5] eller [? , Lem. 4.4], men theoremet holder og under mer generelle forutsetninger2 . For flere slike integralulikheter anbefales [? , p. 206-211].
Proposisjon 3.3.6. -funksjonen : (0, ) R er logaritmisk konveks.
2 Vi

har kf gk1 kf kp kgkq , hvor 1/p + 1/q = 1 se [? ] eller igjen [? , thm. 3.8] for et bevis. Her
betegner kgkp normen til g i Lp vektorromet.

114

III

3 Spesialfunksjoner
1
p

Bevis. La p (1, ) og


x
y
+
p
q

1
q

= 1. Herfra ser vi p
a

t(x/p+y/q1) et dt

=
0

tx/p ty/q t1/p t1/q et/p et/q dt siden

=
Z0

tx1 et

1/p

Z

x1 t

ty1 et

1/q

1/p Z
dt

1
1
+
=1
p
q

dt
1/p

y1 t

dt

= (x)1/p (y)1/q .
Dersom vi n
a lar = 1/p og dermed 1 = 1/q s
a har en at (0, 1) og

x + (1 )y (x) (y)1
h
h
i
i
log x + (1 )y log (x) (y)1
= log (x) + (1 ) log (y) .
for enhver x, y (0, ). At log (x) er konveks flger dermed direkte fra definisjon (3.3.4).
Hva dette i praksis vil si for -funksjonen er at vi kan etablere en rekke
ulikheter, og egenskaper til gammafunksjonen da den oppfrer seg relativt sett
pent. Kanskje det viktigste egenskapen er at gammafunksjonen er den eneste
funksjonen som har disse egenskapene. Alts
a
Theorem 3.3.2. (BohrMollerup) Gitt en funksjon f : (0, ) (0, ) som
tilfredstiller
1) f (1) = 1
2) f (x + 1) = xf (x)
3) log f er konveks.
Da flger det at f (x) = (x) x (0, ).
Vi har allerede vist at (s) tilfredstiller kravene ovenfor, men a vise at envher
funksjon som tilfredstiller kravene er identisk lik krever mer spissfindighet. Beviset er relativt elementrt men noe langt, og bevises derfor heller i avsnitt (C.3).
Igjennom beviset viser en at -funksjonen kan skrives p
a flgende form
Proposisjon 3.3.7. La s = it + , da er
(s) = lim

n! ns
s(s + 1) (s + n)

(3.13)

forutsatt at > 0
Denne proposisjonen er definert for alle s 6= 0, 1, 2, . . ., og er identisk lik
(s), n
ar > 0. Siden funksjonen er definert i det negative planet beskriver vi
likning (3.13) som en anlytisk utvidelse av (s). Senere skal vi se at likning (3.13)

III

3.2 Gammafunksjonen

115

er en meromorf funksjon i det komplekse planet. Grovt sagt at polene punktene


hvor funksjonen er udefinert er enkle.
Her viser vi proposjonen p
a en mer moderne m
ate, og tar utgangspunkt i
flgende lemma
Lemma 3.3.2. (Produkt representasjonen av -funksjonen)
Z


1

(s) = lim

t
n

n

ts1 dt

dersom s = it + , hvor > 0 og t R.


Som er vist p
a side (223). Selv uten bevis virker lemmaet intuitivt siden vi har
ex = limx (1 x/n)n .

ta utgangspunkt I lemmaet ovenfor har vi


Bevis. Ved a
Z
0


n
Z 1
t
s1
s
1
t
dt = n
(1 u)us1 dt
n
0
!
1
 s
Z
n 1
u (1 u)n
s
n1 s
+
=n
(1 u)
u du
s
s 0
0
 Z 1

n
= ns
(1 u)n1 us du
s 0
=
n(n 1) 1
s(s + 1) (s 1 + n)
n!
ns
=
s(s + 1) (s + n)

= ns

u(s1)+n du

ta grensen n
og a
ar n p
a begge sider av likheten fullfrer beviset.

bruke likning (3.13) kan flgende theorem utledes


Ved a
Theorem 3.3.3. Weiserstrass produktet er definert som

 z
Y
1
z
:= zez
1
exp
(z)
n
n
n=1

(3.14)

for alle z C forutenom z N.( Alts


a polene 1, 2, . . . til gammafunksjonen.)
Dette er og en analytisk utvidelese av (s) og er kjent som et av Weierstrass
produktene. Under flger et kort bevis

116

III

3 Spesialfunksjoner

Bevis. Vi tar utgangspunkt i likning (3.13) og ser p


a inversen.


1
z(z + 1) (z + n)
= lim nz
n
(z)
1 2 (n 1) n

n 
Y
z+k
= z lim nz
n
k
k=1

= z lim e

z log n

n
Y

z/k

k=1

n 
Y
z  z/k
e
1+
k

k=1

= z lim ez log n ez(1+1/2+1/3+...)


n

1+

k=1

= z lim ez(Hn log(n)


n

= zez

n 
Y

n 
Y
k=1


Y
n=1

1+

z  z/k
e
k

z  z/k
e
k

z  z/n
1+
e
n

Hvor Hn er de n frste tallene i den harmoniske serien.


det andre Weiestrassproduktet definerer sin x og er gitt som


Y
z2
1 2
sin z = z
n
n=1

(3.15)

Som kan bevises formelt via Weierstrass faktoriserings teorem ellers kan en anta at
gange sammen
sin z kan skrives som et produkt av alle sine nullpunkter, ved a
rttene p
a en smart m
ate f
as teoremet. For n
a tar vi det for gitt at likning (3.15)
vise flgende teorem.
stemmer, og nsker heller a
Theorem 3.3.4. (Eulers refleksjons formel) For alle s (0, 1), s
a holder identeiteten

(s)(1 s) =
(3.16)
sin(s)
vise denne sammenhengen uten Weierstrass produktene er noe trasig, men
A
kan gjres ved a skrive om hyresiden til et integral som lses via kompleks ana vise at (x) = (x)(1 x) oppfyller en bestemt differensiallikning
lyse eller a
hvor lsningen gir oss resultatet.
Bevis. Ved a ta utgangspunkt i likning (3.13) s
a har vi likning (3.14) s
a har vi at
(1 z)(z) = z(z)(z) og
n!
n!
ns
ns
n s(s + 1) (s + n)
s(s + 1) (s + n)
1


 =
Qn
= lim
s
s
n s2
sin
s
1 n
k=1 1 + n

(s)(s) = lim

gange
Her ble likning (3.14) brukt i siste overgang. Beviset fullfres n
a ved a
med s p
a begge sider og bruke s(s) = (1 s). Som flger direkte fra
proposisjon (3.3.5).

III

3.2 Gammafunksjonen

117

bruke resultatet ovenfor kan vi eksempelvis utlede flgende


Ved a
Lemma 3.3.3.


1
2

2
eu du =

sette z = 1/2 inn i likning (3.16) f


Bevis. Ved a
as


1
2

 2
 

1
1

1
=
=
2
2
sin 2

bruke selve definisjonen av gammaHerfra ser vi at (1/2) = . Videre ved a


funksjonen s
a har vi at
  Z
Z
Z
2
1
1 u2
(1/21) t
e
du =
eu du

=
t
e dt =
2
2
0

0
kombinere dette resultatet med uttrykket for (1/2)
Via substitusjonen t 7 u2 . A
fullfrer beviset.
Som er et svrt viktig resultat innen statistikk og sannsynlighetsregning.
Proposisjon 3.3.8. (Dobbel Identiten) For alle s slik at s, s + 1/2 6= 0 s
a er


1
= 212s 1/2 (2s)
(3.17)
(s) s +
2
Vi skal senere se p
a et enklere bevis for denne identiten ved bruk av betafunksjonen, men inntil da blir beviset noe tungt
bruke et nyttig uttrykk for (2s), vi kan skrive
Bevis. Teknikken blir a
n!ns
n!ns/2
s(s + 1) (s + n) (s + 12 ) (s +

2s(2s + 1) (2s + 2n)
(2n)!(2n)2s


1
(n!)2 n1/2 22n+1
lim
2s n (2n)!(s + n + 21 )


1
(n!)2 22n+1
lim
1
2s n (2n)!n1/2 (1 + ns + 2n
)


2 2n+1
1
(n!) 2
lim
2s n (2n)!n1/2

(s)(s + 1/2)
= lim
n
(2s)

=
=
=

1
2

+ n)

vise identiteten m
For a
a vi alts
a bestemme den siste grensen. For enkelhetensskyld defineres


(n!)2 22n+1
C = lim
n
(2n)!n1/2

118

III

3 Spesialfunksjoner

Uttrykket m
a holde for alle s, det m
a eksempelvis holde for s = 1/2. Innsetning
gir da
C=

(s)(s + 1/2) 2s
(1/2)(1)
2 =
2=2
(2s)
(1)

sette inn uttrykket


hvor (1) = 1, og (1/2) = fra lemma (3.3.3) ble brukt. A
gange med (2s) fullfrer beviset.
for C, og a
Til slutt regner vi ut to siste integral herfra blir resten av resultatene knyttet
vise flgende
til gammafunksjonen tatt i oppgavedelen. Vi nsker a
Vi har tidligere sett at de logaritmiske egenskapene til gammafunksjonen er
svrt viktige. Bla gir har vi et resultat som sier at
Theorem 3.3.5. (Stirrlings approximasjon) Gitt flgende grense
lim

n!
2n

n
e

n = 1

s
a er
n!

 n n
e

2n

for store n.

Stirrlings approximasjon eller formel, kan og skrives som, log(n!) 2n +


n log n n . Resultatet kan selvsagt generaliseres for gammafunksjonen. Et forenklet bevis for overnevnte finnes i oppgavesammlingen under. Vi velger heller
prve a
vise flgende
a
Proposisjon 3.3.9.
1

log (x) = log 2

skrive flgende integral


Bevis. Beviset begynner med a
Z

log (x)(1 x) dx

(3.18)

P
a den ene siden kan integralet deles opp, slik at
Z

Z
log (x)(1 x) dx =

log (1 x) dx

log (x) dx +

0
1

Z
log (x) dx

=
0

log (y) dy
1

log (x) dx .

=2
0

III

3.2 Gammafunksjonen

119

Alternativt kan integralet og skrives om ved a bruke Eulers refleksjonsformel, s


a
Z
0

log (x)(1 x) dx = log


dx
sin(x)
Z 1
Z 1
=
log
log sin(x) dx
Z

= log + log(2) dx
se det siste integralet kan substitusjonen y 7 x bli brukt s
For a
a da f
as
1

log sin(x) dx =
0

log(sin y) dy
0

som er et standard integral som har blitt beregnet fr, se proposisjon (2.8.3).
Ved sammenlikning har en n
a at
Z

Z
log (x)(1 x) dx =

log (x)(1 x)

log (x) dx = log + log(2)

2
0

log (x) dx = log 2

vise.
som var det vi nsket a

Oppgaver
Z

1. Vis at (n) =

ee

+nx

dx , hvor n N.

2. I denne oppgaven skal vi kartlegge en tilnrmelse til (x), ogs


a bedre
kjent som Stirrlings formel. Vi utvikler bare rekken til andre ledd, da
videre utvikling krever mer arbeid. Bestem et lukket uttrykk for
S=

N
X

log i ,

i=1

og videre vis at S tilnrmet er en Riemansum til integralet


Z n
SC
log x dx .
1

bekrefte at
Bruk uttrykket for S og integralet til a

N! C
for store N , der C er konstant.

N
e

N

120

III

3 Spesialfunksjoner

bli mer
3. I denne oppgaven er ikke integrasjon i fokus. Men heller det a
bruke (x) funksjonen. Uansett er det sterkt anbefalt a
prve
st i a
seg p
a flgende oppgaver. Vi tar atter en gang og dypper fingrene ned
lre matematikken og ikke
i kvantevannet, merk som vanlig at m
alet a
fysikken, men litt forklaring trengs likevell. Et kvantifisert magnetisk moment har 2 kvantetilstander som betegnes med verdiene s = 1. Avhengig
av om det magnetiske momentet peker med eller mot p
asatt magnetfelt.
Tenkt p
a en magnet med + og . Vi betrakter et stort antall N slike magnetiske moment. Hvor N+ har verdien s = +1 og N = 1. Videre s
a
er N+ + N = N . Tilslutt s
a er det totale magnetiske momentet gitt som
m = (N+ N )/N . M
alet med blir a bestemme Entropien S (uorden eller
kvantevillskapen) til systemet. Stefan-Boltzans lov at entropien i et slikt
magnetisk system er gitt som


N!
S = k log
N !N+ !
a) Bruk uttrykket for Entropen og Stirllings formel (se forrige oppgave)
vise at entropien S kan skrives som
til a


S = N k A B(1 + m) log(1 + m) C(1 m) log(1 m)
og kartlegg dermed konstantene A, B og C.
4. Bestem flgende integral
Z
I=
0

eax ebx
x+1

der a,b positive konstanter, og < 1. Integralet refferes noen ganger til
som et generalisert frullani integral, der det klassiske frullani integralet f
as
studere grensetilfellet 1, som vi skal studere senere.
ved a
5. Definer flgende funksjoner
Z 1
f (x) :=
log (x + t) dx
0

Z
og

g(x) :=

log x dx ,
0

vis differansen mellom f og g er hyst en konstant, og kartlegg konstan bestemme integralet


ten3 . Bruk dette til a
Z 1
log (x + t) dx .
0

derivere hyresiden kan vre noe keitete, det anbefales derfor a


bruke
A
benytte seg av
en luddig substitusjon for deretter a
Z h(t)

d
d
f (x) dx = f (h(t))
h(t) f (g(t))
g(t)
(3.19)
t g(t)
dx
dx
3 Merk

denne metoden bare er derivering under integraltegnet i forkledning. Denne metoden skal
granskes nrmere senere.

III

3.3 Betafunksjonen

121

den fundamentale kalkulussetningen i skjnn harmoni med produktregelen. Flgende integral




Z u+1
(x)
log
dx = u log u u
(3.20)
2
u
bestemme integralet relativt enkelt.
kan og benyttes til a
Likning (3.20) betegnes ofte som Raabes formel [? ]. Bevis Raabes formel uten
bruk av derivasjon.

3.3.3

B E TA F U N K S J O N E N

Integralet som vi studerte i forrige seksjon er ogs


a kjent som Eulers andre
integral og i denne seksjonen skal vi se nrmere p
a Eulers frste integral, kanskje
bedre kjent som betafunksjonen og er definert som flger
Definisjon 3.3.5. For alle x, y > 0 s
a defineres
Z 1
B(x, y) =
tx1 (1 t)y1 dt

(3.21)

Tilsvarende som for gammaintegralet er det viktig a vite at integralet virkelig


er konvergent for alle x, y > 0. Eksempelvis om x < 1 s
a vil integranden g
a mot
uendelig n
ar vi nrmer oss origo. Tilsvarende s
a vil integranden bl
ase opp for
t = 1 n
ar y < 1.
beta- og gammafunksjonen er svrt nrt knyttet til hverandre og vi har
flgende resultat.
Theorem 3.3.6. La B(x, y) vre definert som
Z 1
B(x, y) =
tx1 (1 t)y1 dt
0

Da har vi for alle x, y hvor Re(x), Re(y) > 0 relasjonene


B(x, y) =

(x)(y)
(x + y)
Z

=
0

Z
=2

tx1
dt
(1 + t)x+y

(3.22)
(3.23)

/2

(sin )2x1 (cos )2y1 d

(3.24)

Beviset for likning (3.22) overlates til leser se oppgave (3), da igjen gir
komfortabel med beta og det god trening i b
ade bevisfring, algebra, og a
funksjonens egenskaper. Senere skal vi bevise relasjonen b
ade med laplace vise liktransformasjoner og kompleks analyse. Men for n
a velger vi heller a
ning (3.23) og (3.24).
Bevis. Tar utganspunkt i definisjonen
Z 1
B(a, b) =
ta1 (1 t)b1 dt
0

(3.25)

122

III

3 Spesialfunksjoner

Vi benyttes oss av subtitusjonen x 7 t/(1 t) som gir t [0, ), slik at


a1 
b1
Z 
Z
x
xa1
1
dx
B(a, b) =
=
dx
1+x
1+x
(x 1)2
(1 + x)a+b
0
0
vise likning (3.24) bruker vi substitusjonen
dette beviser likning (3.23). For a
t 7 (sin )2 p
a likning (3.25) og f
ar flgende uttrykk
Z 1
B(x, y) =
tx1 (1 t)y1 dt
0
/2

sin2

x1

1 sin2

y1

2 cos sin d

/2

=2

(sin )2x1 (cos )2y1 d

som fullfrer beviset.


vise at
Eksempel 3.3.2. Vi nsker a


Z 1
1
u
du
/
M
()
:=
=
,

1+u
1+u
sin ()
0

(3.26)

hvor , > 1, er reelle konstanter og = /.


Bevis. Vi betegner integralet som I og benytter oss av substitusjonen
x 7 1 + u u = (x 1)1/ slik at
Z 1
Z
u
1
(x 1)(1)/
du
=
(x 1)(1/)1 dx
I=

1
+
u

x
0
1
Settes n
a x 7 1/v f
as


Z 1
1

1
v / (1 v)1+/ dv = B 1 ,
0


Ved a n
a benytte seg av definisjonen til betafunksjonen, og Eulers refleksjonsformel f
ar vi
Z 1
1 (1 /) (/)
/
u
du
=
=

1
+
u

(1)
sin
()
0
vise.
som var det vi nsket a
Resultatet kan og vises ved a ta utgangspunkt i likning (3.23), via substitusjonen
t 7 x . Som gir
Z
Z
ta1
ta1
B(a, b) =
dt
=

dt
(1 + t)a+b
(1 + x )a+b
0
0
dele likningen p
og resultatet flger ved a
a og sette b + a = 1, a = s
a


Z 1
x
1

/
dx = B
,1
=

1
+
x

sin
(/)
0
tilsvarende som fr4 .
4 Merk

at betafunksjonen er en symmetrisk funksjon, B(x, y) = B(y, x).

III

3.3 Betafunksjonen

123

Korollar 3.3.3. Gitt at n R s


a er
Z
Z
Z n2
1 1 + xn2
dx
x
/n
dx
=
=
dx =
n
n
2 0
1 + xn
1
+
x
1
+
x
sin
/n
0
0

(3.27)

Beviset uttelates, men flger direkte fra likning (3.23) og lemma (2.6.1). En
svak generalisering av likning (3.26) er flgende
Z

b2 + u1
du = a2(1+1/)
a2 + u

b2
a2(1)/
+
sin(/)
sin(/)


(3.28)

hvor = /, men dette overlates til leser. Dette resultatet gir blant annet

Z
0

1 + u1

du =
1 + u

1
1
+
sin(/)
(sin )


(3.29)

sette a = b = 1 i (3.28). Leser kan selv sjekke at likning (3.29) reduseres


ved a
til (3.27) i tillfellet hvor 1 = 1. Integraler p
a denne formen har blitt
studert i forrige seksjon, og vil igjen bli studert i delen med kompleks analyse.
Oppgaver
1. Bevis korollar (3.3.3)
1
2

Z
0

1 + xn2
dx =
1 + xn

dx
=
1 + xn

xn2
/n
dx =
,
n
1+x
sin /n

kun bruke likning (3.26). Unng


bruke lemma (2.6.1).
ved a
a alts
aa
Z

2. Bestem flgende integral

enx
dx n
ar n [0, 1].
1 + ex

3. Vi skal i denne oppgaven bevise relasjonen mellom beta- og gammafunksjonen


B(x, y) :=

(x)(y)
(x + y)

og til dette benyttes flgende funksjon


(x) :=

B(x, y)(x + y)
.
(y)

Vis at (x) oppfyller flgende egenskaper

1)

(1) = 1

2)

(x + 1) = x (x)

124

III

3 Spesialfunksjoner
3)

log (x) er konveks

fullfre beviset.
og benytt Bohr-Mullerup teoremet til a

bevise av multiplikasjons theoremet bedre


4. I denne oppgaven nsker vi a
kjent som Legendres dobbel formel (Legendres duplication formula).



1
(z) z +
= 212z z (2z) .
2
Vis frst at


1
B(a, a) = 212a B a,
2
bevise Legendres dobbel formel. Vis og at identiteten
og bruk dette til a
kan skrives som


(2n)!
1
= 2n
.
(3.30)
n+
2
2
n!
dersom n N.
5. Wallis integralene er nesten like kjent som Wallis produktet og er definert
som
Z
Wn =
0

/2

(sin x)n dx =

/2

(sin x)n dx .

Vis at integralene er like og bestem verdien til til Wn for odde og like n.
Du kan fritt benytte deg av Legendres form fra forrige oppgave.

III

3.4 Digamma-funksjonen

3.3.4

125

DIGAMMA-FUNKSJONEN

I denne delen skal vi se nrmere p


a digammafunksjonen (psi)
Definisjon 3.3.6. Digammafunksjonen er definert som flger
(z) =

0 (z)
d
=
log (z)
(z)
dz

ogs
a kjent som den logaritmisk-deriverte av gammafunksjonen. Noen ganger
betegnes ogs
a funksjonen som 0 , som vi skal senere. For n
a beholdes den
originale notasjonen for enkelhetens skyld. Et viktig resultat er flgende
(1 z) (z) = cot(z)

(3.31)

Bevis. Resultatet ligner veldig p


a Eulers refleksjonsformel. S
a en naturlig start
derivere formelen, vi har da
er a
d
d

(x)(1 x) =
dx
dx sin(x)
1
cos(z)
0 (x)(1 x) (x)0 (1 x) = 2
sin(z) sin(z)
1
(z)(z)(1 z) (z)(1 z)(1 z) = 2
cot(z)
sin(z)





(z)(1 z) (1 z) (z) =
cot(z)
sin(z)
(1 z) (z) = cot(z)
vise. Her ble det brukt at 0 (x) = (x)(x), Eulers
som var det vi nsket a
refleksjonsformel, og noen velkjente trigonometriske identiter.
Enda en viktig identitet er flgende
(1 + z) (z) =

1
z

(3.32)

Bevis. Vi starter med flgende likning


(1 + z)
= z.
(z)

(3.33)

Dette flger selvsagt direkte fra at per definisjon s


a er (z + 1) = z(z). Derivasjon av begge sider gir

(1 + z)
(1 + z) (z) = 1
(z)
bruke likning (3.33) ogs
og beviset fullfres ved a
a dele begge sider p
a z. Her
ble det ikke brukt noe mer enn kvotientregelen og definisjonen av digammafunksjonen.
Vi tar et raskt lite eksempel for a vise hvor kraftig disse enkle relasjonene kan
vre

126

III

3 Spesialfunksjoner

Eksempel 3.3.3. Vis at


Z
0

log x
(1 +

2
x2 )

dx =

definere flgende integral


Bevis. Vi begynner med a
Z
xa
I(a) :=
2 dx
(1 + x2 )
0
bestemme er I 0 (0). Hvorfor vi
Der vi legger merke til at integralet vi nsker a
bytte om integral og derivasjonstegnet kommer senere, men for n
har lov til a
a
holder det a si at dette er lovlig da funksjonen er begrenset. Utregningen av I(a)
g
ar som flger
Z
xa
I(a) =
2 dx
(1 + x2 )
0


Z
1
a+1
t
2
=
dx
t=x ,=
2 0 (1 + t)2
2


1
a+1
a+1
= B
,2
2
2
2

 

a+1
a+1
1
2
=
2
2
2
Dersom vi n
a deriverer I(a) med hensyn p
a a f
as

 
 



1
a+1
a+1
a+1
a+1
I 0 (a) =
2

2
4
2
2
2
2
settes n
a a = 0 f
as
     
 
1
1
3
1
3
1 

0
I (0) =

=
(2) =
4
2
2
2
2
4 2
4
og vi er ferdige.
Under blir noen av detaljene i beviset gjennomg
att


1
2

 

 2
1
1
1
1 2

1+
=
=
=
,
2
2
2
2
2

hvor (x + 1) = x(x) ble benyttet. Tilslutt s


a er
 
 

 
 
1
3
1
1
1

= 1+

= 1 = 2 ,
2
2
2
2
2
hvor likning (3.32) benyttet.
La oss vise en annen representasjon av (x), nemlig

(z) =

X 1
1
1
+

z
n
n
+
z
n=1

(3.34)

III

3.4 Digamma-funksjonen

127

Bevis. Snur vi oppmerksomheten v


ar mot likning (3.14) og tar logaritmen f
ar vi

log

log

1
(z)

1
(z)

= log ze


Y
n=1

!
 z
z
1
exp
n
n

= log exp{z} + log z +

nzo

z X

log exp
log 1 +
n
n
n=1
n=1

Brukes n
a at log 1/A = log A og begge sider ganges med 1 kan vi skrive
uttrykket p
a en enkel deriverbar form
!
h

X
z i
z
z log z +
log 1 +
n
n
n=1



X
0 (z)
1
1
1
= (z) =
+

(z)
z
n
1+z
n=1
d
d
(log (z)) =
dx
dz

som var det som skulle vises.


Overgangen fra sum til logaritme kan virke noe underlig, vi ser p
a produktet
an /bn og tar logaritmen

Y
an
K = log
b
n=1 n


= log

a1 a2 . . .
b1 b2 . . .

= (log a1 + log a2 + . . .) (log b1 + log b2 + . . .)


!
!

X
X
=
log an
log bn
n=1

n=1

hvor tilsvarende utregning kan bli brukt p


a produktet i likning (3.2).
Fra denne likningen kan vi enkelt finne et par konkrete verdier for digammafunksjonen.
(1) = 0 (1) =
 
1

= 2 log 2
2
 
1

= log 3
3
2
2 3
 
1

=
3 log 2
4
2
Disse kan utledes ved a studere likning (3.34) nyere. Frste verdi faller direkte
ut da

X
1
1
(1) = 1 +

=
n
n
+
1
n=1

128

3 Spesialfunksjoner

III

Siden rekken er teleskoperende s


a har en at
k
X
1
1

n
n+1
n=1

 


 

1
1
1
1
1
1
1
= 1
+

+ +

2
2
3
k1
k
k
k+1
1
=1
.
k+1
Slik at n
ar k s
a vil S 1. Bruksomr
adene til digammafunksjonen og
ulike integralrepresentasjoner vil komme i form av sm
a drypp fremmover n
ar
skrive opp
vi utforsker ulike integrasjonsteknikker. For n
a begrenser vi oss til a
noen grunnleggende identiter.

Sk :=

Noen viktige identiter

X
1
1

k
z+k
k=1
Z 1
1 tz
(z + 1) = +
dt
1t
0
Z t
e
ezt
(z) =

dt
t
1 et
0

X
1
0 (z) =
(n + z)2
n=0

(z + 1) = +

0 (1) =

2
6

Oppgaver
1. Vis flgende

log B(x, y) = (x) (x + y)


x
2. Vis flgende


1
2


= 2 log 2

beregne integralet
og bruk resultatet til a
Z 1
t 1

I=
dt .
t (1 t)
0
3. Bestemm integralet
Z
I(m) =

(x) sin 2x dx
0

hvor Z. Hva skjer n


ar ? Strider dette mot Rieman-Lesgue
lemmaet, hvorfor/hvorfor ikke?

III

3.6 Riemann zeta funksjonen

3.3.5

129

P O LY G A M M A - F U N K S J O N E N

Denne seksjonen blir noe kort, da polygamma-funksjonen ikke er like nyttig


som digamma funksjonen. Funksjonen er defiert som flger
dm
dm+1
(x)
=
log (z) .
dz m
dz m+1
alts
a er 0 (z) = (z) fra forrige seksjon. De fleste identitetene som holdt for
digammafunksjonen holder og for polygammafunksjonen, eksempelvis
m (z) =

m (z + 1) m (z) = (1)m

m!
z m1

som kan vises via induksjon da (z + 1) (z) = 1/x, som vi har vist fr.For
trigammafunksjonen 1 f
ar vi eksempelvis
1
z2
Videre s
a gjelder fortsatt refleksjonsformelen slik at
1 (z + 1) 1 (z) =

(1)m m (1 z) m (z) =

dm
cot(z)
dz m

hvor vi f
ar igjen eksempelvis har at for 1 s
a er

2

1 (1 z) + 1 (z) =
sin(z)
Dersom vi ser bort i fra (x) s
a kan vi for alle m > 0 skrive polygammafunksjonen som flgende sum
m (z) = (1)m+1 m!

X
k=0

1
= (1)m+1 m! (m + 1, z)
(z + k)m+1

Der (s, z) er Hurwitz zeta funksjonen som vi snart skal se p


a.

3.3.6

RIEMANN

Z E TA F U N K S J O N E N

Riemann zeta funksjonen er en av de viktigste og mest fascienerende funksjonene innen moderne matematikk. M
aten funksjonen oppst
ar p
a er veldig
naturlig da den dreier seg om potenserekker av naturlige tall

X
1
,
n2
n=1

X
1
,
n3
n=1

X
1
, usw.
n4
n=1

Definisjon 3.3.7. Riemann Zeta funksjonen er definert som


(s) =

X
1
ns
n=1

for Re(s) > 1.

Her er s et komplekst tall med real del strre enn 1. Men vi skal bare se
p
a tilfellet der s er reell. Som vanlig var det Euler som frst betraktet funksjo
nen, men det var Bernhard Riemann (1826 1866) som inns
a viktigheten av a
bektrakte s som kompleks.
Det var Euler som kartla verdien av (2n) hvor n er et naturlig tall. Men fr
vi ser p
a det trengs en kort introduksjon av Bernoulli tallene.

130

III

3 Spesialfunksjoner

bestemme
Bernoulli tallene Jacob Bernoulli (1654 1705) var opptatt med a
en formel for
Sk (n) = 1k + 2k + + (n 1)k
finne et
hvor k N og n N\{1}. Ved lave verdier er det relativt enkelt a
uttrykk for summen
n(n 1)
2

S2 (n) =

som kjent fra fr. Utrolig nok fant bernoulli ut at



k 
1 X k+1
Sk (n) =
Bj nk+1j .
1 + k j=0
j
der Bj st
ar for det jte Bernoulli tallet. Tallene kan bli beskrevet rekursivt som

k 
X
k+1
Bj = 0 , k 1
j
j=0
med initalverdi B0 = 1. P
a magisk vis oppst
ar og Bernoulli tallene i taylorutviklingen av visse funsjoner. Eksempelvis

X
z
1 z
1 z6
1
z4
zn
B
=
1

=
n
ez 1
1! 2
2!
6
4! 30
n!
n=0

i tabbel 1 er de frste bernoulli-tallene oppfrt.


n

10

11

12

Bn

12

1
6

1
30

1
42

1
30

5
66

691
2730

Proposisjon 3.3.10. La (z) betegne Riemann zeta funksjonen da er


Z

(z)(z) =
0

tz1
dt ,
et 1

hvor Re(z) > 1.


Bevis. Vi har fra den geometriske rekken

rk =

k=0

1
1x

|x| < 1

sette r = ez f
Ved a
as

X
k=0

ekz =

1
ez
= z
z
1e
e 1

III

3.8 Polylogaritmen

131

bruke dette resultatet kan integralet n


Ved a
a beregnes
Z

X
1
tz1 et dt
(z)(z) =
kz 0
k=1
z1
Z 
X
dt
t
=
et
k
k
0
k=1

bruke substitusjonen u = t/n f


Ved a
ar vi n
a
=
=

Z
X

(u)

k=1 0
Z
X
k=0

z1 ku

uz1 eu eku du

z1 u

!
e

ku

du

k=0

uz1 eu

du

=
0

1
1 eu


du

uz1
du
eu 1

vise.
som var det vi nsket a

3.3.7

HURWITZ

3.3.8

P O LY L O G A R I T M E N

Z E TA F U N C T I O N

Som vi har sett tidligere s


a er Rieman-zet funksjonen svrt viktig innen
matematikk. I denne delen studerer vi en lignende funksjon. Polylogaritmen er
ogs
a kjent som Jonqui`eres function og betegnes som Lis (z), hvor s er graden.
Li2 (z) betegnes som dilogaritmen, Li3 (z) som trilogaritmen usw. Per definisjon
s
a er
Definisjon 3.3.8. Polylogaritmen er definert som flger
Li( z) =

X
zk
z2
z3
=
z
+
+
ks
2s
3s

k=1

Fra definisjonen ser vi at


Lin (1) =

X
k

1
= (n)
kn

P
hvor notasjonen k er kortformen av k=1 . Dette viser sammenhengen mellom
polylogaritmen og zetafunksjonen. For dilogaritmen n = 2 har en
P

Li2 (1) = (2) =

2
6

132

3 Spesialfunksjoner

III

som vi kommer mer tilbake til senere. Legg merke til at for n = 1 s
a har en
Li1 (z) =

X zk
= log(1 z)
k
k

som vi gjennkjente som maclaurinrekken til log(1 z) mer generelt s


a har vi
flgende theorem.
Theorem 3.3.7. for n N s
a kan polylogaritmen defineres rekursivt som
Z z
Lin (t)
Lin+1 (z) =
dt .
t
0
Bevis. Vi tar utgangspunkt i hyresiden og bruker definisjon (3.3.8) s
a
!
Z z
Z z
Lin (t)
1 X tk
dt
dt =
t
kn
0
0 t
k
X 1 Z z
=
tk1 dt
kn 0
k
X 1  tk z
=
kn k 0
k

X
k

zk
k n+1

= Lin+1 (z)

Derivasjon av theorem (3.3.7) gir n


a
z

Lin+1 (z) = Lin (z)


z

som er en nyttig identitet.

3.3.9

DILOGARITMEN

Av alle polylogaritmene er nok kanskje Li2 (z) den mest nyttige, noen ganger
er funksjonen og kjent som Spences funksjon etter den skotske matematikkeren
William Guthrie Spence (18461926), som gjorde en systematisk studie av funksjonen p
a begynnelsen av 1900-tallet. Vi tar utgangspunkt i flgende definisjon
for dilogarithmen
Definisjon 3.3.9. Dilogaritmen er definert som flger
Z z
X zk
log(1 t)
Li2 (z) =
=
dt
2
k
t
0
k

for alle z < 1.

III

3.9 Dilogaritmen

133

Den frste identiteten vi skal vise er tillegn Abel, og refferes noen ganger
som Pentagon Identiteten.
Theorem 3.3.8. (Abels identitet) Gitt at x, y 6= [1, ) da er
log(1 x) log(1 y) = Li2 (u) + Li2 (v) Li2 (uv) Li2 (x) Li2 (y)

(3.35)

hvor u = x/(1 y) og v = x/(1 y).


frst derivere hyre og venstre side av identiteten og
Beviset g
ar ut p
aa
vise at disse er like. Integrasjon gir da at hyre og venstre side hyst kan vre
vise at konstanten er null.
forskjellige ved en konstant. Beviset fullfres ved a
fullfre omskrivningen av likning (3.35) f
Ved a
as




u(1 v)
v(1 u)
Li2 (u) + Li2 (v) = Li2 (uv) + Li2
+ Li2
1 uv
1 uv




1v
1u
log
+ log
1 uv
1 uv

(3.36)

som ofte reffereres til som pentagon identiteten. Ved hjelp av Abels theorem og
vanlig derivasjon kan en vise at dilogaritmen tilfredstiller en rekke funksjonallikninger
Proposisjon 3.3.11. text
2
Li2 (z) + Li2 (1 z) =
log z log(1 z) (Refleksjonsformelen)
6
 
1
2
1
log2 (z) (Inversjonsformelen)
Li2 (z) + Li2
=
z
6
2


1
z
Li2 (z) + Li2
= log2 (z + 1) (Landens Identitet)
1+z
2
1
Li2 (z) + Li2 (z) =
Li2 (z 2 )
(Kvadrat identiteten)
2

(3.37)
(3.38)
(3.39)
(3.40)

vise frste og andre identitet. Setter vi x = z og y = 1 z i


Bevis. Velger her a
likning (3.35) f
as
log(1 z) log(z) = Li2 (1) Li2 (z) Li2 (1 z)
Li2 (z) Li2 (1 z) =

2
log z log(1 z)
6

der det ble benyttet at Li2 (1) = (2) = 2 /6, dette viser Eulers refleksjonsformel.

vise Inversjonsformelen. Ved a


derivere Li2 (1/z) f
Bevis. nsker s
aa
as


d
1
1
Li2
= 2
dz
z
z

log 1 + z1

1/z

!
=

log(1 + z) log z
z

134

3 Spesialfunksjoner

III

Integrasjon med hensyn p


a z gir n
a at

 Z z
1
log(1 + t)
log z
Li2
=

dt
z
t
z
0
Z z
log(1 t)
1
=
dt log2 (z) + C
t
2
0
1
= Li2 (z) log2 (z) + C
2
bestemme konstanten settes z = 1 s
for a
a C = 2 Li2 (1). Men vi har at
Li2 (1) =

X
2
(1)n
=

n2
12
n=1

setter vi n
a inn dette og lar z = z f
ar vi
 
1
2
1
Li2 (z) + Li2
=
log2 (z) ,
z
6
2
som nsket.
Ved samme metode kan Landens Identitet bestemmes, ta utgangspunkt i
vise dette og kvadrat identiten overlates til
Li2 [z/(z 1)] og derivere, men a
leser.
Selv om digammafunksjonen tilfredstiller en rekke funksjonallikninger er det
dessverre bare en h
andfull verdier som kan regnes ut eksplisitt. Disse er

1 5
0,
1 ,
1
2

hvor vi gjennkjenner sistnevnte som det gylne snittet. Merk at = (1 + 5 )/2


1.6180 . . . > 1 slik at Li2 () divergerer. Derimot s
a konvergerer den konjugerte
= 1 som vi skal se i det p
aflgende eksempelet.
Eksempel 3.3.4. Vis at




5 1
2
5 1
2
Li2
=
log
2
10
2




2

5 1
3 5
2
Li2
=
log
2
15
2





2
1 5

1
5 1
Li2
=
+ log2
2
15
2
2
Bevis. La oss for enkelhetensskyld definere

1 5
=
,
2
da kan vi enten se eller regne ut at er en lsning av x2
+ x = 1. Bedre kjent
som den konjugerte av det gylne snittet, heldigvis s
a er 5 > 1 s
a < 1, og
integralene konvergerer. Videre s
a er




1 5
3 5
= og
=1
2
2

III

3.11 Elliptiske Integral

135

sette x = y = 1 inn i Abels Identitet (3.35) f


Ved a
as n
a
2 Li2 () 3 Li2 (1 ) = log2

(3.41)

der det ble benyttet at 2 = 1 . Via refleksjonsformelen fra likning (3.37)


med z = f
as
Li2 () + Li(1 ) =

2
2
log log(1 ) =
2 log2
6
6

(3.42)

bruke kvadrat identiteten fra likning (3.40) f


Og sist men ikke minst ved a
as
Li2 () + Li2 ()

1
Li2 (1 )
2

(3.43)

betegne
hvor det ble brukt igjen at 2 = 1 . Ved a
A = Li2 () ,

B = Li2 (1 ) ,

C = Li2 ()

gir likningene (3.41) til (3.43) flgende likningssett


2A 3B = log2
A +
A +

2
2 log2
6
1
B
C =
2

B =

lser en dette settet for A, B, C f


as


2
5 1
A = Li2
=
log2
2
10


3 5
2
B = Li2
=
log2
2
15


2
1
1 5
C = Li2
=
+ log2
2
15
2
vise.
som var det vi nsket a
1. Vis Abels identitet via derivasjon
log(1 x) log(1 y) = Li2 (u) + Li2 (v) Li2 (uv) Li2 (x) Li2 (y)
hvor u = x/(1 y) og v = x/(1 y).
2. Vis Kvadrat identiten og Landens identitet

3.3.10

DILOGARITMEN

136

III

3 Spesialfunksjoner

3.3.11

ELLIPTISKE INTEGRAL

P
a begynnelsen av 1900-tallet var studiet av kurver og buelengden av disse
et av de sentrale omr
adene som ble studert. Sinus og cosinus kunne brukes til
beregne buelengden av sirkler, og buelengden av hyperbolske kurvene kunne
a
beregnes tilsvarende med de hyperbolske trigonometriske funksjonene (sinh og
beregne buelengden av ellitptiske
cosh). elliptiske funksjonene ble innfrt for a
kurver, og fullfrte dermed treenigheten. Studiet av disse funksjonene var lenge
svrt sentralt og ble studert allerede av Gauss, som vi skal se senere.
Innen moderne matematikk defineres et elliptisk integral som alle funksjoner
f som kan skrives p
a formen
Z x  p

f (x) :=
R t, P (t) dt
c

hvor R er en rasjonell funksjon, og P er et polynom av enten grad 3 eller 4, og


c er konstant. Her skal vi bare studere komplette elliptiske integral og disse
skrives som f (/2) med c =. Det er som regel tre integral som forbinnes med
elliptiske funksjoner, disse er
Definisjon 3.3.10. Det frste komplette elliptiske integralet er definert som
/2

Z
K(k) :=
0

dt
p

k2

sin (t)

=
0

dt
p

(1

t2 )(1

k 2 t2 )

Definisjon 3.3.11. Det andre komplette elliptiske integralet er definert som


Z /2 p
Z 1
1 k 2 t2
2
2

E(k) :=
1 k sin t dt =
dt
1 t2
0
0
Definisjon 3.3.12. Det tredje komplette elliptiske integralet er definert som
/2

Z 1
dt
dx
p
p
K(k) :=
=
2
2
2
2
2
2 2
2
(1 + n sin t) 1 k sin (t)
0
0 (1 n x ) (1 x )(1 k x )

Hvor k kalles modulusen og k 0 = 1 k 2 kalles den komplimentre modulusen. Slik at k 2 + (k 0 )2 = 1.


Generelt sett s
a kan ikke de elliptiske integralene utregnes eksplisitt i form
av mer kjente funksjoner, men la oss vise et lite knippe
Z

Eksempel 3.3.5.


1
1 =
2
4 2

1
4

sette inn og bruke substitusjonen y 7 x4 f


Bevis. Ved a
as


1 =

Z
dx
1
(1/4)(1/2)
=
y 3/4 (1 y)1/2 dy =
4
4
4(3/4)
1

x
0

bruke eulers-refleksjonsformel med


herfra s
a er (1/4)(3/4) = 2 ,
ved a
a = 1/4. Tilsvarende s
a er (1/2) = og dette fullfrer beviset.
K

III

3.11 Elliptiske Integral

137

Vi kan vise et par tilsvarende former for de elliptiske integralene, men frst
trengs flgende lemma.
Lemma 3.3.4. La f vre en funksjon med odde periode a. Da er
Z

f (x)

dx =
x
a

a/2

f (x) cot

 x 
a

dx

(3.44)

dele integralet inn i perioder p


Bevis. Vi begynner beviset med a
a a alts
a
Z Z a Z 2a Z 3a
=
+
+
+
0

2a

gjre dette kan integralet skrives som


ved a
Z
0

X
f (x)
dx =
x

k=0

a(k+1)

ak

X
f (x)
dx =
x

k=0

Z
0

f (u + ak)
du .
u + ak

Der substitusjonen x 7 u + ak ble brukt i siste overgang. Siden f har en periode


p
a a s
a er f (x + ak) = f (x) for alle k Z. Vi deler atter en gang opp integralet
slik at

Z a
Z a
hZ a

X
X
f (x) dx i
1
1
f (x) dx
+
=
f (x)

dx
x + ak
x + ka
(k + 1)a x
a/2 x + ak
0
0
k=0

k=0

(3.45)
Der substitusjonen x 7 u a/2 ble brukt i siste overgang. Ved litt smart algebramagi s
a har vi at summen kan skrives som

X
k=0

X
X
1
1
1
1

=
+
x + ka
(k + 1)a x
x + ka
x ka
=

k=0

k=1

1
x + ka

k=

1
k + x/a
k=
 x 

=
cot
a
a

1
a

(3.46)

Der det ble benyttet at


cot(x) = lim

N
X
n=N

X
1
1
2x
=
+
2 n2
x+n
x
x
n=1

Ved n
a a sette likning (3.46) inn i likning (3.45) og sette konstantleddet utenfor
f
as
Z
Z
Z
 x 
 x 
f (x)

a/2
dx =
f (x) cot
dx =
f (x) cot
dx
x
a
a
a 0
a
0
0
som var det som skulle vises.

138

III

3 Spesialfunksjoner

Korollar 3.3.4. La f vre en like-funksjon med periode a. Da er


Z

 x 
f (x)

dx =
sin
x
a
a

a/2

f (x) dx
0

og spesielt s
a er
Z
0

f (x)
sin x dx =
x

/2

f (x) dx
0

benytte oss av lemma (3.3.4) p


Bevis. Ved a
a funksjonen f (x) = sin(x/a) som
er odde med periode 2a. Videre s
a gir halv-vinkel formlene v
are
tan

x
sin x
=
2
1 + cosx

sette inn f
ved a
ar vi n
a
Z
Z
 x 
 x 
 x 
a/2
f (x)
sin
dx =
f (x) sin
cot
dx
x
a
a 0
a
a
0
Z
h
 x i
a/2
=
f (x) 1 + cos
dx
a 0
a
Z
a/2
=
f (x)
a 0
N
a siden cos(x/a) er en odde funksjon omkring a/4 s
a er integralet null.
SOMETHING DARK SIDE
Som en liten digresjon avslutningsvis tar vi og ser p
a to integral som likner
p
a Elliptiske integral, men kan bestemmes eksplisitt.
Eksempel 3.3.6. For alle b (0, 1) s
a er
Z

/2

Z
0

/2

cos t dt
arcsin k
p
=
2
k
1 k 2 sin t


sin t dt
1
1+b
p
=
log
2b
1b
1 k 2 sin2 t

Bevis. Vi kaller integralene henholdsvis for F1 (k) og F2 (k). Via substitusjonen


x 7 k sin t s
a er
Z /2
Z
1 b
cos t dt
dx
arcsin k
p

F1 (b) =
=
=
2
2
2
b
k
1

x
0
0
1 k sin t
der substitusjonen
x 7 sin y fullfrer beviset. La oss lse F2 (b) noe liknende, ved

a la k = 1 k 2 , da er k 2 +k2 . Videre s
a er 1k 2 sin2 t = 1(1k2 ) sin2 t =
2
2
2
2
2
k +k cos t. Siden 1 sin x = cos x fra pytagoras. N
a er
Z
F2 (b) =
0

/2

sin t dt
1
=
k
b 2 +k 2 cos2 t

Z
0

/2

dx
1
=
arcsinh
k
k 2 +x2

k
k

III

3.11 Elliptiske Integral

139

Der substitusjonen
a x 7 k y. Per definisjon s
a er
x = k sin

 t ble brukt, ogs
arcsinh = log 1 + 1 + u2 , via litt algebra s
a er n
a 1 + k 2 /k2 = 1/b2 slik at
F2 (b) =

1
1
1+k
1
1+k
(1 k 2 )1/2
=
log
=
log
log
b
k
b
b
(1 k 2 )1/2
(1 k 2 )1/2

bruke at a log b = log ba fullfrer n


det a
a beviset.

140

3.4

4 Transformasjoner

III

TRANSFORMASJONER

En integral transformasjon kan bli sett p


a som en type operator som virker
p
a en funksjon f . Transformasjonen eller avbildningen skrives gjerne som
Z
F (x) =

K(x, t)f (t) dt


a

hvor K(x, t) betegner kernelen. En av de viktigste egenskapene med integral


transformasjonen er at det er en liner operator5 .
Definisjon 3.4.1. En liner transformasjon mellom to vektorrom V og W er en
avbildning T : V 7 W slik at
T (v1 + v2 ) = T (v1 ) + T (v2 ) ,
holder for alle vektorer v1 og v2 i V og for enhver skalar R.
Setter vi den formelle notasjonen til side er motivasjonen for slike tranforns forst
fjormasjoner lett a
a. Det eksisterer mange klasser av problemer som er
vanskelig, eller umulig a lse algebraisk - i deres opprinnelige form. En integraltranformasjoner avbilder en likning fra dets opprinnelige omr
adet til et annet
behandle. Deromr
adet. Likningen i det nye omr
adet kan vre langt enklere a
etter kan en mappe funksjonen tilbake til det opprinnelige omr
adet ved a bruke
den inverse av integral-tranformasjonen. Dette gjelder integraler og. For en
lengre liste over ulike integral-transformasjoner anbefales [? ].
Direkte relatert til integral transformasjoner er konvolusjoner. Dersom F og G
er integral transformasjonene til f og g, s
a er konvolusjonen f g lik funksjonen
som har transformasjonen F G. Konvolusjonen uttrykkes ofte som et integral
med hensyn p
a f og g, men ikke ndvendigvis kernelen K(x, t).

3.4.1

LAPLACE

TRANSFORMASJONEN

Laplace transformasjonen er en av de mest brukte integral-transformasjonene


innen matematikk og har flere bruksom
ader innen fysikk og ingenirvitenskap.
beregne differensiallikninger, funksjonallikninger, og visse
Den blir brukt til a
integral. For et historisk perspektiv anbefales artikklene [? ? ] fra Michael A. B.
Deakin varmt.
Definisjon 3.4.2. La F (t) vre en funksjon av t. Laplace Transformasjonen av
F er da definert som L
Z

L f (t) = F (s) =
est f (t) dt
0

vise at vi tar laplace transformasjonen


Noen ganger skrives ogs
a Lf (s) for a
av funksjonen f . Ellers brukes og den tosidide laplace-transormasjonen
Z

L 2 F (t) =
est F (t) dt

5 Kjrt

barn har mange navn. Alternative navn er en liner transformasjon, liner mapping, liner
funksjon osv.

III

4.1 Laplace transformasjonen

141

Den inverse L -transformasjonen betegnes som L 1 (f (t) eller Lf1 (s), og kan
skrives som et kontur-integral
Z +iT
1
lim
est F (s) ds ,
f (t) = L1 {F }(t) = L1
{F
(s)}(t)
=
s
2i T iT
bruke tabell (3.1) til a
bestemme inversene.
men det er ofte langt enklere a
Proposisjon 3.4.1. Transformasjonen L er en liner transformasjon
Dette flger fra at selve integralet danner en liner-transformasjon K(f ) =
Bevis.
R
f , men dette glemmer vi for n
a. Direkte innsetning gir
Z

L f (t) + g(t) =
(f (t) + g(t))est f (t) dt
0
Z
Z
=
f (t)est dt +
g(t)est dt
0
0
Z
Z
st
=
f (t)e
dt +
g(t)est dt
0
0


= L f (t) +L f (t)
som nsket.
Vi har allerede sett p
a laplace-transformasjonen av F (t, x) = ts1 , da dette
nesten er -funksjonen. En rekke slike integral er vist i tabell (3.1). merk at
Tabell 3.1: Et utvalg L -transformasjoner av elementre funksjoner.
f (t)

Lf (s)

Betingelser

1
ta
eat
cos t
sin t
cosh t
sinh t
(t a) = eas

1/s
(a + 1)/sa+1

Re(a) > 1

1
sa
s
s2 + 2

s2 + 2
s
s2 2

s2 2

R
s > |Im()|
s > |Re()|
s > |Im()|

1/s om
ecs /s om

H[c](t)

c0
c>0

transformasjonen av de trigonometriske integralene er vist i lemma (3.8.2) og


utlede a
utelates
proposisjon (2.7.1). Uttrykkene i tabellen er ikke vanskelig a
derfor. I tabellen ble to nye funksjonen innfrt Heavyside funksjonen Hc og
Dirac delta funksjonen , disse er definert som flger
Definisjon 3.4.3. Dirac delta funksjonen er definert som

+ dersom x = 0
(x) =
0
ellers
og tilfredstiller
Z

(x) dx = 1

142

III

4 Transformasjoner

Definisjon 3.4.4. Heavyside funksjonen er definert som



0 dersom n < 0
H[n] =
1 dersom n 0
studere egenskapene Laplace Transformasjonen har,
Mer spennende er det a
enn spesfifikke verdier. tabell (3.2) viser et utvalg egenskaper laplace transformasjonen har. Vi skal se nrmere p
a et par av disse egenskapene.
Tabell 3.2: Noen av de mest kjente egenskapene til L -transformasjonen.
tdomenet

sdomenet

af (t) + bf (t)
tf (t)
tn f (t)
f 0 (t)
f 00 (t)
f (n)
f (t)/t
Rt
f ( ) d
0
f (a)t
eat f (t)
F (t a)H(t a)

aF (s) + bG(s)
F 0 (s)
(1)n F (n) (s)
sF (s) f (0)
s2 F (s) sf
f 0 (0)
P(0)
n
n
sR F (s) k=1 sk1 f (nk) (0)

F () d
s
F (s)/s 
s
1
|a| F a
F (s a)
eas f (s)

Proposisjon 3.4.2.
G(s) = L

f (t)
t

F () d

=
s


Hvor L f (t) = F ().
a
Bevis. Vi begynner med a sette inn definisjonen av Laplace-transformasjonen p
hyre side

Z
Z Z
Z Z
F () d =
f (t)et dt d =
f (t)et ddt
s

I siste overgang byttet vi om integrasjonsrekkeflgen. Dette flger fra fubinis sats


siden vi krever at |f (t)et | skal vre begrenset. Siden f (t) ikke er avhengig av
kan vi sette ledddet utenfor integrasjonen


Z
Z
Z
Z
est
f (t)
F () d =
f (t)
et ddt =
f (t)
dt = L
t
t
s
0
s
0
som var det som skulle vises.
Korollar 3.4.1.
Z
0

f (t)
=
t

F () d
0

sette s = 0 i proposisjon (3.4.2).


Korolaret faller ut direkte ved a

III

4.1 Laplace transformasjonen

143

Eksempel 3.4.1. Fra korolaret flger Dirichlet integralet nesten direkte.


Z
sin
=

Siden integranden er symmetrisk trenger vi bare se p


a omr
adet > 0. Ved a
bruke korollar (3.4.1) med f (t) = sin t har vi
Z
Z
Z
f ()
1
ds
d =
L (sin t) ds =
2

s + 12
0
0
0
Hvor laplace transformasjonen av sin x ble tatt fra tabell (3.1) mens utledningen
finnes i lemma (3.8.2) og proposisjon (2.7.1). Alts
a har vi


Z
Z
sin
1
1
1
ds =
=
arctan s
=

2 0 s2 + 12
2

0
Eksempel 3.4.2. Bestem den inverse L -transformasjonen av funksjonene
F (s) =

1
s(1 + s)(1 + s2 )

via standard delbrksoppspalting kan F skrives som


F (s) =

1 1
1
1 1+s
+

2 1+s
s
2 1 + s2

R
Vi kan n
a sammenlikne med tabellen og ser at L (1) = 1/s, siden 0 esx dx =
1/s. Dette betyr at L 1 (1/s) = 1. Videre kan det siste uttrykket deles opp i to
deler






1
s
1+s
1
1
1
=L
+L
= cos t + sin t
L
1 + s2
1 + s2
1 + s2
Som flger fra at b
ade L og L -transformasjonen linere transformasjoner.
For det siste leddet kan vi bruke faseforskyvnings egenskapene L (eat f (t)) =
F (s a). Vi har at F (s) = 1/s, og F (s + 1) = 1/(1 + s) slik at


 
1
1
1
s
1
L
=e L
= es
1+s
s
oppsumert har en alts
a

et
cos t + sin t
L 1 F (s) = 1

2
2
som var det som skulle bestemmes.
forenkle eller lse differenHovedsaklig blir L -transformasjoner brukt til a
evaluere visse integral. Under er et
siallikninger. Men de kan og brukes for a
beregne L -transformasjonen av integralet, enn
eksempel hvor det er enklere a
integralet i seg selv.
Eksempel 3.4.3.
Z
0

cos xt

dt = e |x|
1 + t2
2

144

III

4 Transformasjoner

ta den
Bevis. Vi kaller integralet for I og beregner i stedet frst L (I), for s
aa
inversee L -transformasjonen. Dette gir
Z
Z Z
cos(xt) sx
sx
e
L f (s) =
f (x)e
dx =
dt dx
1 + t2
0
0
0
bytte om grensene. At integralet fortsatt tar samme verdi
Det neste steget blir a
flger fra fubinis sats siden absoluttverdien av integralet konvergerer6
Z
Z
Z
1
1
s
sx
=
cos(xt)e
dx dt =
dt
2
2 s2 + t2
1
+
t
1
+
t
0
0
0
Bruker vi delbrksoppspalting p
a den gjennst
aende integranden f
as




Z
1
1
s

1
1
s

dt =
1 =
.
=
2
2
2
2
2
1s 0
s +t
1+t
1s 2
s
2 1+s
Vi har n
a alts
a beregnet L -transformasjonen av integralet v
art
Z

cos xt
1
L
dt =
2
1
+
t
2 1+s
0
ta den inverse L -transformasjonen p
ved a
a begge sider av likningen f
as


Z
cos xt
1

dt = L 1
= e |x|
2
1
+
t
2
1
+
s
2
0
som var det som skulle beregnes. Grunnen til at vi skriver |x| er fordi vi nsker
at svaret skal holde for alle x. Siden cos x er en likefunksjon s
a er cos(xt) =
cos(xt), s
a vi f
ar samme uttrykk for negative x.
Konvolusjon
Konvolusjon er en m
ate a definere en relasjon mellom to funksjoner p
a. Akkuratt
som en definerer addisjon og subtraksjon danner en operasjon p
a de reelle
tallene. For integraler og spesielt laplace-transformasjoner er flgende definisjon
viktig
Definisjon 3.4.5. La f (t) og g(t) vre to funksjoner av t. Vi definerer operasjonen (f g)(t) som flger
Z
f g =
f (t )g( ) d

som og er en funksjon avhengig av t.


Proposisjon 3.4.3. Dersom f og g er en-sidede funksjoner, s
a er
Z t
(f g)(t) =
f (t )g( ) d
0
6

Siden | cos(xt)| < 1 har vi at

|I|
0

som komvergerer.

esx
dt dx =
1 + t2

Z
0

sx
e
2

i
0

dt dx =

sx
e
2s

i
0

2s

III

4.1 Laplace transformasjonen

145

Alts
a at f (t) = a(t)H(t) og g(t) = b(t)H(t), der H igjen betegner heavyside
funksjonen og a og b er funksjoner av t. Det interessante er hva som skjer n
ar vi
tar L -transformasjonen av f g.
Proposisjon 3.4.4. La f, g : [0, ) 7 R vre to en-sidede funksjoner, og la F (s)
og G(s) betegne deres respektive L -transformasjoner. Da er
L {(f g)(t)} = F (s)G(s) L 1 {F (s)G(s)} = (f g)(t)
Proposisjonen kan og skrives som
L (f g) = L (f ) L (g)
som noen ganger er en hendigere form. Her ser vi alts
a operasjonen gjr
L -transformasjonen til en komplett-multiplikativ funksjon7 .
skrive om F (s)G(s) som dobbeltintegralet
Bevis. Vi begynner med a
Z Z
F (s)G(s) =
es(x+y) f (x)g(y) dx dy .
0

bruke substitusjonen t 7 x + y ogs


Ved a
a y = f
ar vi at integralet kan skrives
Z Z
F (s)G(s) =
est f (t )g( ) dt d .

Herfra snur vi integrasjonsrekkeflgen, og at dette er lov flger igjen fra fubinis


sats.
Z t

Z Z t
Z
st
st
F (s)G(s) =
e f (t )g( ) d dt =
e
f (t )g( ) d dt .
0

Rt

innfrer notasjonen h(t) = 0 f (t )g( ) d har en n


Ved a
a vist
Z
F (s)G(s) =
est h(t) dt = L {(f g)(t)} ,
0

vise den andre likheten tar en invers L -transformasjon av


som nkset. For a
likningen over. Dette fullfrer beviset.
Tanken er n
a at i stedet for a beregne integralet f g kan en i stedet beregne
den inverse L -transformasjonen av produktet av F og G
Z t
f (t )g( ) d = L 1 {F (s)G(s)}
0

la oss vise et tilfellet hvor dette er nyttig.


Eksempel 3.4.4. Vi nsker a vise bevise theorem (3.3.6) ved hjelp av konvolusjonstheoremet. Alts
a at
Z 1
(x)(y)
B(x, y) =
sx1 (1 s)y1 ds =
(x + y)
0
holder for alle komplekse x,y med realdel strre enn null.
7 Alts
a

at f (n m) = f (n)f (m) holder. For tallteoretiske funksjoner har vi en tilsvarende konvolusjonen nemlig Dirichlet konvolusjonen.

146

III

4 Transformasjoner

beregne f g har vi
Bevis. Vi velger f (t) = tx og g(t) = ty . Ved a
Z t
sx (t s)y ds
(tx ty ) =
0

som nesten er definisjonen av beta-integralet. P


a den ene siden s
a er L (tx ty ) =
x
y
L (t )L (t ), slik at
L (tx ty ) = L (tx )L (ty ) =

x! y!
sx+y+2

Fra konvolusjons-theoremet har vi da at




Z t
x! y!
tx+y+1
x
y
1
s (t s) ds = L
=
x!

y!
(3.47)
sx+y+2
(x + y + 1)!
0


sette inn
hvor det ble brukt at L 1 (a + 1)/sa+1 = ta igjen fra tabell. Ved a
x = x 1, y = y 1 og t = 1 kan likning (3.47) skrives som
Z 1
(x)(y)
(x 1)!(y 1)!
sx1 (1 s)y1 ds =
=
(x + y 1)!
(x + y)
0
hvor n! = (n + 1). Dette fullfrer beviset.
1. La f vre en funksjon med periode T . Vis at
Z T
1
Lf (s) =
f (t)est dt
1 eT s 0
2. La f vre en funksjon slik at L -transformasjonen av f eksisterer. Vis at
Z t

1
L
f (w) dw = F (s)
s
0

3.4.2

FOURIER-TRANSFORMASJON

Parseval og Plancherel
Theorem 3.4.1. (Parsevals identitet) La f og g vre kvadratisk integrerbare
funksjoner f, g L2 slik at indreproduktet er definert. Da holder
Z
Z
hf, gi =
f (x)g(x) dx =
f(x)
g (x) dx = hf, gi

Korollar 3.4.2. (Plancherels Theorem) Spesielt dersom f = g i theorem (3.4.1),


s
a er
Z
Z

2
2
2
kf k2 =
|f (x)| dx =
f (x) dx = kfk22

III

4.4 Landens transformasjon

3.4.3

MELLIN

147

TRANSFORMASJONEN

Mellin transformasjonen er en multiplikativ transformasjon og er relatert til


den tosidige laplace transformasjonen. Integralet er dypt knyttet til Dirichlet
rekker, og sentral innen analytisk tallteori.
Definisjon 3.4.6. Mellin transformasjonen av f er definert som
Z
M(f (x)) = (s) =
xs1 f (x) dx
0

skrive Mf (s) eller (s) dersom det er innSom fr kommer vi enten til a
lysende hvilken funksjon vi tar transformasjonen av. Som et grunleggende og
viktig eksempel innen tallteori s
a er M-transformasjonen av f (x) = ex lik
-funksjonenen. Mellin-transformasjonen er faktisk en Fourier-transformasjon,
som kan vises ved en substitusjon. Merk at bruksomr
adene til Fouerier og Mellintransformasjonene er noe forskjellig.
Proposisjon 3.4.5. Gitt at f er en kontinuerlig funksjon p
a (0, ), at f (t)
Ct for R n
ar t 0, og at f (t) g
ar mot null raskere enn enhver potens av t
n
ar t . S
a holder flgende
1) Mellin-transformasjonen av f konvergerer absolutt for Re(s) > og definerer
en holomorf funksjon i det halvplanet.
2) La s = + iT , hvor > og la
g (t) = e2t f e2t

3) Den inverse mellin-transformasjonen er definert som et komplekst linjeintegral og konvergerer for alle > for alle x > 0
Z iT
1
f (x) =
xs Mf (s) ds
2i iT
n
ar T .

3.4.4

LANDENS

TRANSFORMASJON

Som introduksjon forklares frst hva det aritmetiske-geometriske snittet er.


Definisjon 3.4.7. (Aritmetiske-geometriske snittet) For to positive reelle tall x
og y definer som flger
1
(a + b)
2
b1 = ab

a1 =

Der a1 er den aritmetriske middelverdien av a, b og b1 er den geometriske middelverdien av a, b. Deniner s


a flgen
1
(an + bn )
2
p
= an bn

an+1 =
gn+1

Flgene konvergerer mot samme verdi og kalles det aritmetiske-geometriske snittet av a og b. Der snittet som M (a, b) eller Agm(a, b).

148

III

4 Transformasjoner

Eksempelvis s
a er Agm(1, 2) 1.45679 og Agm(1.5) 2.604008. La oss frst
vise et lemma som relaterer det ellitpiske integralet, og Agm(x, y).
Lemma 3.4.1. Integralet
Z

I(a, b) :=
0

d
p
2
2
a cos + b2 sin2

er uforandret om a og b byttes ut sitt aritmetriske og geometriske snitt





1
I(a, b) = I
(a + b), ab .
2
Bevis. Benytter substitusjonen tan = x/b s
a d = cos2 ()/b s
a
Z
d
dx
p
p
I(a, b) =
=
2
2
2
2
2
(x + a2 )(x2 + b2 )
0
0
a cos + b sin

Benyttes n
a substitusjonen x = t + t2 + ab f
as
Z
dt
r
=
2 

2 t2 + a+b
(t2 + ab)
2
Z
dt
r
=


2  
0
t2 + ( ab )2
t2 + a+b
2
Z

/2

Kvadratroten ble skrevet om som flger


s
p

(x2 + a2 )(x2 + b2 ) = 2x t2 +

a+b
2

2

og dx = (x/ t2 + ab ) dt.
Tidlig p
a 1800-tallet var Gauss interest i lengden av leminskater alts
a kurver som ligner p
a . Etter en numerisk beregning av en spesiell leminskate
observerte han at
Z
1
1
dx

og
2 0
Agm(1, 2 )
1 x4
stemte til 11 desimaler. Med forbausende klarsyn deduserte Gauss flgende
theorem
Theorem 3.4.2.
Z
I=

/2

d
p

a2

cos2

b2

2 Agm(a, b)

+ sin



a har vi alts
a
Bevis. Siden I(a, b) = I 12 (a + b), ab s
0


I(a, b) = I(a1 , b1 ) = I(a2 , b2 ) = = I M (a, b), M (a, b)

(3.48)

III

4.5 Cauchy-Schl
omilch transformasjonen

149

Setter vi dette inn i likningen f


as
Z
Z
dx
1
dy

p
=
=
I=
2
2
2
2
2
M (a, b) 0 1 + y
2M (a, b)
(x + M (a, b) )(x + M (a, b) )
0
der substitusjonen x 7 M (a, b)y ble benyttet.
At dette integralet er nrt knyttet til de elliptiske integralene er allerde hintet
til i starten, via substitusjonen b2 = a2 (1 k 2 ) f
as
I=

1
a

Z
0

/2

d
1  
1
p
= F
, k = K(k)
2
2
a
2
a
1 k sin

(3.49)

Settes likning (3.48) inn i likning (3.49) og lser med hensyn p


a K f
as
K(k) =

.
2 Agm(a, a 1 k 2 )

fordelen med dette er at iterasjonen av Agm(x, y) konvergerer kvadratisk, alts


a
tilnrme
dobbles antall rette siffer per iterasjon. Dette er alts
a srs nyttig for a
lengden av ellptiske kurver.

3.4.5

MILCH
C A U C H Y- S C H L O

TRANSFORMASJONEN

Utledningene og informasjonen her er stort sett hentet fra [? ] og mer utdyppende informasjon finnes der. Teknikken vi skal se p
a her ble popularisert
av Oscar Xavier Schl
omilch (1823-1901), men kan finnes i notater til Cauchy
tidligere, derav navnet.
Eksempel 3.4.5. Frst tar vi en titt p
a et klassisk integral fra P.Laplace.
r
Z

1 2ab
2
2
exp ax bx
dx =
e
2 a
0
beregne integralet fullfres kvadratet slik at
Bevis. For a
Z
Z

2
2
2
2 ab
exp(ax bx ) = e
e( a x b /x) dx
0

Definerer s
a siste integral som K s
a
Z

2
K :=
e( a x b /x) dx
0

p
Benyttes n
a substitusjonen t = x1 b/a f
as
r Z

2 dt
b
e( a x b /x) 2 .
K=
a 0
t
Tar en gjennomsnittet av disse to integralene f
as
Z



2
1
K=
e( a x b /x)
a + b /x2 dx .
2 a 0

(3.50)

150

III

4 Transformasjoner

Substitusjonen y 7 a x b /x gir s
a
r
Z
1
1
y 2
e
dy =
K=
2
2
2 a
og dette fullfrer beviset da
r
Z


1 2ab
exp ax2 bx2 dx = Ke2 ab =
e
.
2 a
0

Teknikken om blir brukt her ble utvidet av O.Schl


omilch til flgende theorem.
Theorem 3.4.3. (Cauchy-Schl
omilch) Gitt a, b > 0 og f en kontinuerlig funksjon
da er
Z
Z 

1
2
f (y 2 ) dy ,
(3.51)
f (ax b/x) dx =
a 0
0
gitt at integralene i likning (3.51) konvergerer.
Bevis. Benytt substitusjonen t 7 b/(ax) s
a
Z 
Z 

 dt
b
2
2
I :=
f (ax b/x) dx =
f (at b/t)
a 0
t2
0
ta gjennomsnittet av disse to integralene f
Ved a
as
 
Z 
Z

1
b
1
2
=
a + 2 f (at b/t) dt =
f (y 2 ) dy
a 0
t
a 0
Der substitusjonen y 7 at b/t ble brukt i siste overgang. Dette fullfrer
beviset.
Denne relativt enkle transformasjonen kan lse langt vanskeligere integral
enn hva dagens symbolske kalkulatorer klarer. Her sammenlikner vi med Maple
14.
Eksempel 3.4.6.
Z



1
exp (x b/x)2n =
n

1
2n

Bevis. Tar utgangspunkt i likning (3.51) med f (x) = ex slik at


Z
Z


2n
2n
exp (x b/x)
dx =
ey dy

Z
1
et t1/(2n)1 dt
=
n 0


1
1
=
n
2n
2

der substitusjonen t 7 y 2n ble brukt i andre overgang. At ex er symmetrisk


fullfre
omkring origo, og definisjonen av gammafunksjonen ble benyttet til a
beviset.

III

4.5 Cauchy-Schl
omilch transformasjonen

151

I det neste theoremet ser vi p


a en naturlig utivdelse av Cauchy-Schl
omilch
transformasjonen
Theorem 3.4.4. La s vre en kontinuerlig synkende funksjon fra R+ til R+ . Anta
at s er sin egen invers, alts
a at s1 (x) = s(x) x R+ da er
Z
Z

1
f (y 2 ) dy
(3.52)
f [ax s(ax)]2 dx =
a 0
0
der a > 0 og gitt at integralene konvergerer.
Bevis. La frst t 7 ax og bytt tilbake til x som integrasjonsvariabel da f
as
Z
Z


1
f [ax s(ax)]2 dx =
I=
f [x s(x)]2 dx
a 0
0
Videre la t 7 s(x), dette gir
Z
Z


1
1
I=
f [x s(x)]2 dx =
f [s(t) t]2 s0 (t) dt
a 0
a 0
Gjennomsnittet av disse to integralene gir
Z
Z


1
1
2
0
f [x s(x)]
1 s (t) dx =
f (y 2 ) dy
2a 0
a 0
der substitusjonen y 7 x s(x) ble benyttet i siste overgang. Dette fullfrer
beviset.
Det finnes flere funksjoner som er sin egen invers, den enkleste f (x) =
1/x har vi sett grundigere p
a fr. I flgende eksempel vises noen andre slike
funksjoner
Eksempel 3.4.7. En nyttig funksjon som er sin egen invers er
s(x) = x

1
log (ex 1)

benytte oss av likning (3.52) f


ved a
as

Z 
Z
1
2
x
log
(e

1)
dx
=
f
f (y 2 ) dy
2
0
0
Interessant! Ved n
a og sette f (x) = ex f
as



Z
Z

1
2
x
y 2
.
exp
log
(e

1)
dx
=
e
dy
=
2

2
0
0
forenkle hyresiden, dette gir
merk og at integralet kan lses ved a


Z
Z
Z
2
1
2
x
(x1/)2
exp
log
(e

1)
dx
=
e
dx
=
ey dy
2

0
0
0
via y 7 x 1/, uansett er dette en artig generalisering.

152

III

4 Transformasjoner

3.4.6

DIVERSE

TRANSFORMASJONER

I denne delen skal vi blant annet beregne


Z

I(a, b; r) :=
0

x2
x4 + 2ax + 1

r

x2 + 1 dx
xb + 1 x2

lse oppgaven innledningsvis trengs flgende


og noen spesialtilfeller. For a
omskrivning
Lemma 3.4.2.
Z 
f
0

x2
4
x + 2ax2 + 1


f

dx =
0

1
x2 + 2(a + 1)


dx

for alle funksjoner f med a > 0.


dele p
Bevis. Ved a
a x2 og faktorisere gir
Z

J=


f

x2
4
x + 2ax2 + 1


f

dx =
0

1
(x x1 )2 + 2(a + 1)


dx

La oss n
a benytte Cauchy-Schl
omilc transformasjonen fra theorem (3.4.3) til a
beregne integralet. Her settes f (t) = g 1/(t2 + 2a + 2) . Da har vi
Z


g

1
(ax b/x)2 + 2(a + 1)

dx =

1
a


g

1
2
y + 2(a + 1)


dy

sette a = b = 1.
og beviset fullfres ved a
gyve ls p
Vi er endelig klar til a
a problemet innledningsvis, n
a med barsk
og bram.
Theorem 3.4.5. (V.Molls MasterTheorem) La
r
Z 
x2 + 1 dx
x2

I1 =
x4 + 2ax2 + 1
xb + 1 x2
0
r
Z 
x2
dx
I2 =
2
4 + 2ax2 + 1
x
x
0
r
Z 
2
x
I3 =
dx
4 + 2ax2 + 1
x
0


Z 
r 
1
x2
1
I4 =
1 + 2 dx
2 0
x4 + 2ax2 + 1
x
Da er I1 = I2 = I3 = I4 og verdien er
I(a, b; c) =
der = r 1/2.



1
2(1+)
B
,
(1 + a)
2

(3.53)

III

4.6 Diverse transformasjoner

153

Bevis. Heldigvis s
a tilfredstiller

R(x) =

x2
x4 + 2ax2 + 1

r

x2 + 1
x2

funksjonallikningen R(y) = R(1/y)/y 2 , og fra theorem (2.10.1) s


a er R(x)/(xb +
vise I4 kan en enten
as I2 , b = 2 gir I3 . For a
1) uavhengig av b. Settes b = 2 f
sette b = 0 i I1 eller ta gjennomsnittet av I2 og I3 . Dette viser at integralene er
like. For a beregne integralet tar en utgangspunkt i lemma (3.4.2) med f (u) = ur
dette gir
Z
0

x2
x4 + 2ax2 + 1

r

r
1
dx
x2 + 2(a + 1)
0
Z 1
r+1/2
1
2(a + 1)
ur3/2 (1 u)1/2
2
0


1 1
21r+1/2
,
B
r

2 2
(1 + a)r1/2


(1+)
2
1
B ,
(1 + a)
2

Z
dx =
=
=
=

Der substitusjonen u = x22(a+1)


+2(a+1) ble benyttet i andre overgang. Avslutningvis
ble definisjonen av betafunksjonen benyttet og = r 1/2.
Herfra flger par eksempler p
a spesialtilfeller av dette integralet
Eksempel 3.4.8. Ved a = 7 og r = 5/4 i I2 f
as
Z
0

2 (3/4)
x

=
(x4 + 14x2 + 1)5/4
4 2

derivere med hensyn p


Et par flere eksempler f
as ved a
a variablene i lik sette a = 1/2 og derivere med hensyn p
ning (3.51). Eksempelvis ved a
ar
f
as
r
r
Z 
x2
dx
()
=

4 x2 + 1
2
x
x
4
(r)
0
r
r


Z 
2
2

x
dx
()
x
log
=
() (r)
4 x2 + 1
4 x2 + 1
2
x
x
x
4
(r)
0
(3.54)
Hvor = r 1/2 og (x) = 0 (x)/(x).
Eksempel 3.4.9. I dette eksempelet setter vi inn tre verdier for r i likning (3.54).
r=1

 


Z

x
x2
1
1
log
=
2

(
)

(1)
x4 x2 + 1
2
2
(x4 x2 + 1)3/2
0
= log 2

154

4 Transformasjoner

III

Hvor det ble benyttet at (1) = , (1/2) = 2 log 2 og (1/2) = .


Via tilsvarende regning f
as
  
r

3/4


 
Z 
x
x2
41
1
1
 1
log
=

3
4 x2 + 1
4 x2 + 1
x
x
4
4
4
4
0
r
 
1 2 1

=
2
2
4
for r = 3/4. Her ble det brukt at (1z)(z) = cot(z) (3/4)(1/4) =
og (1 z)(z) = / sin(z) 1/(3/4) = (1/4) sin(/4)/. Dette klarte
beregne.
ikke Maple 14 a

III

5 Diverse applikasjoner

3.5

155

DIVERSE APPLIKASJONER

Theorem 3.5.1. Anta f er en kontinuerlig, deriverbar8 og strengt kende funksjon


Rb
p
a [a, b]. Let m = (a + b)/2 vre midpunktet, da har g(x) = a |f (t) x| dt et
unikt minimum over x R. Minumumet er
Z b
Z m
f (t) dt
f (t) dt
m

and skjer ved x = f (m).


Bevis. Dersom x < f (a) eller x > f (b) s
a er
Z b
Z b
|f (t) x| dt >
|f (t) f (c)| dt,
a

hvor c er a dersom x < f (a) og b dersom x > f (b). Et minimum m


a alts
a skjer
i for f (a) x f (b). Fra middelverdi setningen (B.1.1) s
a eksisterer det en
x = f () hvor [a, b]. S
a
Z b
Z
Z b
|f (t) f ()| dt =
f () f (t) dt +
f (t) f () d
a

snu grensene i siste


Legg merke til at f () er konstant og kan beregnes. Ved a
integral f
ar en da
Z b
Z

= 2 (a + b) f () +
f (t) dt
f (t) dt
(3.55)

Siden vi har antatt at f er deriverbar s


a kan vi trygt derivere likning (3.55).
Z b

d
|f (t) f ()| dt = 2 (a + b) f 0 () = 0
d a
N
a siden f er strengt kende p
a intervalet medfrer dette at f 0 () > 0 for alle
[a, b]. S
a den deriverte er null n
ar = (a + b)/2. Den dobbelderiverte er
negativ ved x = slik at virkelig er et minimum.
La oss n
a vise at virkelig er et unikt minimum. Siden ingenting forandres
ved a legge til en konstant til f , eller skalere intervalet skalerer vi slik at a = a
og f (0) = 0. Da er
Z b
Z b
Z

|f (t) f ()| dt
|f (t) f ()| dt = 2 f () f () 2
f (t) dt.
b

Dersom vi velger = 0 s
a blir differansen
Z
Z
2f () 2
f (t) dt = 2
0

f () f (t) dt > 0,

og tilsvarende om vi velger = 0, s
a blir den
Z 0
Z 0
2f () 2
f (t) dt = 2
f () f (t) dt < 0,

dette viser at x = f (m) virkelig gir et unikt globalt minimum.


8 Kravet

om deriverbarhet er ikke ndvendig, men det forenkler beviset noe.

156

5 Diverse applikasjoner

3.5.1

G U LV

O G TA K - F U N K S J O N E R

3.5.2

ITERERTE

INTEGRAL

III

III

6 Derivasjon under integraltegnet

3.6

157

D E R I VA S J O N U N D E R I N T E G R A LT E G N E T

One thing I never did learn was contour integration. I had learned
to do integrals by various methods shown in a book that my high
school physics teacher Mr. Bader had given me. That book also
showed how to differentiate parameters under the integral signits
a certain operation. It turns out thats not taught very much in the
universities; they dont emphasize it.
But I caught on how to use that method, and I used that one damn
tool again and again. So because I was self-taught using that book,
I had peculiar methods of doing integrals. The result was, when
guys at MIT or Princeton had trouble doing a certain integral, it
was because they couldnt do it with the standard methods they had
learned in school. If it was contour integration, they would have
found it; if it was a simple series expansion, they would have found
it. Then I come along and try differentiating under the integral sign,
and often it worked. So I got a great reputation for doing integrals,
only because my box of tools was different from everybody elses,
and they had tried all their tools on it before giving the problem to
me.
(Feynman [? , s47-48])
Metoden som Mr. Feynman i denne passasjen refferer til g
ar ofte under navnet derivasjon under integraltegnet, derivasjon med hensyn til en parameter, eller
navngi metoden,
til og med Feynman integrasjon. Uansett hvordan en nsker a
beregne
s
a ligger appelen og skjnnheten i at metoden kan bli benyttet til a
tilsynelatende umulige integral, uten bruk av mer enn elementr kalkulus9 .
Vi skal studere hvilke situasjoner flgende likhet (som vi inderlig nsker skal
vre sann) holder
Z
Z
d

f (x, y) dy =
f (x, y) dy .
(3.56)
dx Y
x
Y
bruke definisjonen av den deriverte ser vi at utsagnet er ekvivalent med
Ved a
flgende
Z
Z
lim
f (x, y) dy =
lim f (x, y) dy
(3.57)
xn

Y xa

Flgende teorem viser n


ar utsagnet stemmer.
Theorem 3.6.1. (Elementr form). La f : [a, b] Y R vre en funksjon, der
[a, b] er et lukket interval og Y er et kompakt underrom av Rn . Da er
Z
Z
d

f (x, y) dy =
f (x, y) dy .
dx Y
x
Y
R
Dersom b
ade f (x, y) og f (x, y)/x er kontinuerlige for alle x, y og Y f (x, y) dy
en kontinuerlig, deriverbar funksjion med hensyn p
a x [a, b].
9 Om

bevise theorem (3.6.2)


en ser bort fra den lille desjen med m
alteori som er ndvendig for a

158

6 Derivasjon under integraltegnet

III

Et bevis av overnevnte ville brukt mye vanskelig notasjon og teoremer fra


funksjonalanalysen. S
a det droppes for leserens helse og velbehag. I korte
trekk vil et bevis ta i bruk at Y er kompakt, som impliserer uniform konvergens.
bytte om grenseverdien
(Selve definisjonen av kompakthet). Herfra kan det a
og integralet rettferdigjres.
Dessverre s
a er denne definisjonen ofte noe snever, da vi gjerne vil se p
a
omr
ader som ikke er kompakte. Eksempelvis halv
apne interval [x, ), eller
pne interval (, ). Heldigvis finnes det en mer generell verisjon av theoa
rem (3.6.1). Som bruker enda mer abstrakt notasjon, mer spesifikt m
alteori.
alteori utgave) La X vre et a
Theorem 3.6.2. (M
pent underrom av R og la
vre et m
alrom. Anta f : X R tilfredstiller flgende betingelser
1) f (x, ) er en Lebesgue-integrerbar funksjon av for enhver x X.
2) For nesten alle , s
a eksisterer den deriverte f (x, )/x for enhver
x X.
3) Def finnes en integrerbar funksjon : R slik at |f (x, )/x| ()
for enhver x X.
Da for enhver x X s
a er
Z
Z

d
f (x, y).d =
f (x, y) d .
dx
x
deriverre under inteDette teoremet gis oss og grunnlaget for ikke bare a
graltegnet n
ar omr
adet er Y er endelig, men og for Y = (0, ) gitt at kravene
ovenfor holder.
For leserens trygghet holder heldigvis de to frste kravene for nesten alle
integraler. Om funksjonen er kontinuerlig eller glatt, eksisterer den deriverte for
alle verdier. Om funksjonen konvergerer absolutt er den Lebesgue integrerbar,
eller om funksjonen er Riemann integrerbar p
a et endelig intervall. Det som
stort sett m
a sjekkes er at b
ade integranden, og dens deriverte er bundet p
a
intervalet. For enkelhetens skyld gis og flgende teorem som konkretiserer dette
Theorem 3.6.3. Begge sider av likningen
Z
Z
d

f (x, y) dy =
f (x, y) dy .
dx Y
x
Y
eksisterer og er like i et punkt t = t0 gitt at flgende to krav holder
1) f (x, t) og f (x, t)/t begge kontinuerlige n
ar x Y og t befiner seg i et lite
omhegn innenfor t0




2) Det eksisterer funksjoner slik at f (x, t) A(x) og f (x, t)/t B(x)
Rb
Rb
som er uavhengig av t slik at a A(x) dx og a B(x) dx konvergerer.
Teoremet noe svakere (Det gjelder ikke for Lebesgue-integral), men holder
for de funksjonene som blir studert her. Men n
a har det vrt fryktelig mye snakk,
vise hvor nyttig denne teknikken kan vre
vi sl
ar til med barsk og bram for a

III

6 Derivasjon under integraltegnet

159

bestemme flgende integral


Eksempel 3.6.1. Vi nsker a
Z 1 2
x 1
dx
log x
0
Integralet har ingen kjent antiderivert, og vi m
a derfor ty til andre metoder. Den
vante leser vil kanskje legge merke til enten at integralet kan skrives som et
dobbeltintegral, eller at det er et frullani integral i forkledning (u 7 log x). Men
derivere med hensyn
dette sparer vi til en senere annledning. Vi trenger noe a
p
a, og innfrer paramteren a > 0 som flger
Z 1 a
x 1
dx
I(a) =
log x
0
Her vil selvsagt I(2) gi oss integralet vi er ute etter. Derivasjon med hensyn p
aa
gir
Z 1 a
Z 1
Z 1
d
x 1
xa 1
1
dx =
dx =
xa dx =
I 0 (a) =
da 0 log x
a
log
x
1
+
a
0
0
Integrasjon gir oss at
I(a) = log(1 + a) + C
betrakte a = 0. Da er I(0) = 0 og log(1) = 0
Konstanten kan bestemmes ved a
slik at C = 0. Vi f
ar da alts
a at
Z 1 2
x 1
dx = I(2) = log(1 + 2) = log 3
log x
0
finne.
som var det vi nsket a
Theorem 3.6.4. (Dirichlet integral) er definert som flger
Z
sin

d =

2
0
Dette integralet vil dukke opp gjentatte ganger videre og er svrt viktig innen
signalanalyse. Ofte blir forkotelsen sinc(x) = sin(x)/x benyttet. En alternativ
frst normalisere funksjonen slik at sinc(x) = sin(x)/x, som
definisjon er a
har en rekke kjente definisjoner.


Y
sin(x)
x2
1
=
1 2 =
x
n
(1 + x)(1 x)
n=1
Har en noe kjennskap til laplacetransformasjoner eller singnalanalyse vil flgende
innfre variabelen a
utregning av integralet gi mer mening. Vi prver frst a
som flger.
Z
sin ax
I(a) =
dx
x
0
Men ved derivasjon f
as I 0 (a) = cos(ax) som ikke konvergerer! La oss heller se
p
a flgende metode

160

6 Derivasjon under integraltegnet

III

Bevis. Vi definerer flgende funksjon


Z
I(b) :=
sinc(x) ebx dx
0

Dette er faktisk laplce-transformasjonen av sinc(x) som vi skal se senere. Herfra


la b = 0. Derivasjon gir
ses at det originale integralet oppst
ar ved a
Z
Z
bx
1
I 0 (b) =
ebx sin x dx =
+C
e
sinc(x) dx =
b
1
+
b2
0
0
Hvor det siste integralet har blitt lst flere ganger fr. Enten skriv om integralet
til kompleks form, eller benytt delvis integrasjon to ganger. Integrasjon gir n
a
I(b) = C arctan(b)
Igjen gjennst
ar det a beregne konstanten C. Dersom vi lar b s
a vil I(b) 0
og arctan b /2. Som medfrer at
I(b) =

arctan(b)
2

la b = 0 f
Ved a
ar vi v
art originale integral, slik at
Z
sin x
= 2 I(0) =
x

finne.
som var det vi nsket a
Et annet forholdsvis klassisk eksempel er flgende integral

x
dx = log 2
tan x

/2

Z
0

betrakte flgende integral


som kan lses ved a
Z

/2

I(b) =
0

tan(b arctan x)
dx
tan x

Der I(1) gir oss v


art opprinnelige integral. Mellomregningene overlates til leser
skrive ut taylorrekka til tan x.
og i neste seksjon vil integralet beregnes ved a
Vi snur derimot oppmerksomheten v
ar mot et moteksempel. I p
aflgende
eksempel viser det seg at hyre og venstre side i likning (3.56) ikke ndvendigvis
er like.
Eksempel 3.6.2. For alle x, t R, s
a definerer vi flgende funksjon
(
xt3
hvis x = 0 eller t 6= 0,
(x2 +t2 )2
f (x, y) :=
0
hvis x = 0 og t = 0.
La videre
Z
F (t) :=

f (x, t) dx .
0

III

6 Derivasjon under integraltegnet

161

Integralet beregnes for t = 0 og t 6= 0 som flger


Z 1
Z 1
F (0) =
f (x, 0) dx =
0 dx = 0
0

0
1

Z
F (t) =
0

xt dx
(x2 +

2
t2 )

1+t2

=
t2

t3 du
1
t
=
2u2
2 1 + t2

Formelen er og gyldig for t = 0. Ved derivasjon f


as
d
1 1 t2
F (t) =
dt
2 (1 + t2 )2

t R

P
a den andre siden s
a er f (0, t) = 0 t, slik at f (0, t)/x = 0. N
ar x 6= 0 s
a blir
den deriverte
2


x2 + t2 (3xt2 ) xt3 4t x2 + t2
xt2 3x2 t2

f (x, t) =
=
4
3
t
(x2 + t2 )
(x2 + t2 )
Kombineres resultatet f
ar vi alts
a at
(
xt2 (3x2 t2 )

(x2 +t2 )3
f (x, t) =
t
0

hvis
hvis

x 6= 0
x=0

(3.58)

Som motbevis til likning (3.56) benyttes heller den tilsvarende definisjonen vist
i likning (3.57) hvor vi ser p
a a = 0. Da er
Z 1
Z 1
xt3
1
0
=
F
(0)
=
og
lim
lim f (x, t) = 0
t0 0 (x2 + t2 )2
2
0 t0
Alts
a er grensene ulike! Dette gjelder uansett om x = 0 eller ei. Problemet er er
at f (x, t)/t ikke er en kontinuerlig funksjon, spesifikt bl
aser den opp i origo.
Som et aller siste eksempel s
a studerer vi enda en gang det Gaussiske integralet. Det finnes enklere metoder a beregne integralet p
a enten via dobbeltintegraler se spesifikt eksempel (3.8.2) og (3.8.3). Eller ved gammafunksjonen som
vist i ??.
Eksempel 3.6.3. Vi skal vise det liknende resultatet

r
Z
2

x2 /2
e
dx =
=
2
2
0
f
og f
ar a
a det klassiske
gaussiske integralet kan resultatet ovenfor benyttes via
substitusjonen u = 2 x. Vi definerer flgende integral for t > 0
Z t
2
x2 /2
A(t) :=
e
dx
.
0

la t ogs
Hvor vi f
ar v
art nsekede integral ved a
a ta kvadratroten. Derivasjon med hensyn p
a t gir oss
Z t

2
2
2
A0 (t) = 2
ex /2 dx et /2 = 2et /2 A(t)
0

162

III

6 Derivasjon under integraltegnet

Benyttes substitusjonen x = ty n
a f
as
Z 1
Z
2
2 2
A0 (t) = 2et /2
tet y /2 dy =
0

2te(1+y

)t2 /2

dy

bestemme den antideriverte med hensyn p


Funksjonen er n
a enkel a
a t, da f
as
Z
Z 1
2
2
2 2
1 (1+y )t /2
2e(1+y )t /2
d
e
dy
=
2
dy
A0 (t) =

2
t
1
+
y
dt
1 + y2
0
0
For n
a definerer vi integralet som en ny funksjon s
a
Z 1 (1+y2 )t2 /2
e
B(t) :=
dy
1 + y2
0
Slik at vi har likningen A0 (t) = 2B 0 (t) t > 0. Integrerer vi begge sider ser vi
at det finnes en konstant C slik at
A(t) = 2B(t) + C

t > 0.

Lar vi n
a t 0+ f
ar vi at A(t) 0 mens B(t)
Alts
a s
a er

R1
0

(3.59)


dy/(1 + y 2 ) = /4.

0 = 2(/4) + C C = /2 .
Lar vi n
a t i likning (3.59) f
as
Z

x2 /2

lim

2
dx

= lim

Z

x2 /2

e
0

Z
0

e(1+y )t
1 + y2

/2

dy

(3.60)

2
Z 1
2 2

e(1+y )t /2
dx =

lim
dy
2
1 + y2
0 t

benytte oss av likning (3.57). Herfra ser vi at hyresiden g


Ved a
ar mot /2 da
eax 0 n
ar x a > 0. Slik at
r
Z

x2 /2
e
dx =
,
2
0
da integralet m
a vre positivt.
Det finnes mange flere integral som kan lses ved hjelp av derivasjon under
integraltegnet men disse blir plassert som oppgaver for leser, b
ade her og p
a
slutten av dokumentet.
1. Bruk integralet

etx dx

F (t) :=
0

vise at
til a
Z
n! =
0

hvor n N.

xn ex dx

III

6 Derivasjon under integraltegnet


2. Bestem og integralet
Z

F (t) :=
0

xa xb
dx
log x

via derivasjon under integraltegnet. Vis og at derivasjonen er lovlig.


3. Vis at

Z
J(a, b) :=

a cos2

beregne
og bruk dette til a
Z
I :=

dx =
2
x + b sin x
2 ab

1
a cos2

x + b sin2 x

2 dx

. Her er a og b reelle tall.


4. Bestem flgende integral


log 1 2 cos(x) + 2 ,

() :=
0

hvor || =
6 1 og R.
5. Vi definerer flgende integral
Z
T (u) =
0

arctan(u 2 + x2 )

dx
(1 + x2 ) 2 + x2

vise at
Bruk dette integralet til a

Z 1
arctan 2 + x2
5 2

dx =
.
2
2
96
0 (1 + x ) 2 + x
Dette integralet kalles for Ahmeds Integral [? ], og senere skal vi
og se p
a generaliseringer av integralet.

163

164

III

7 Uendelige rekker

3.7

UENDELIGE REKKER

Ofte er kan integraler forenkles betraktelig om en kan omskrive integralet


gjre dette p
skrive ut taylorrekka til hele
til en sum. En vanlig m
ate a
a er a
eller deler av integranden. For a konkretisere la oss se p
a flgende integral, som
kanskje mange kjenner igjen fra tidligere
1

Z
0

log(1 z)
dz .
z

Dette har blitt beregnet fr som Li2 (1). La oss n


a heller skrive ut taylorrekka
til logaritmen slik at vi f
ar
Z

log(1 z)
dz =
z

Z
0

X
zn

n
n=1

1
z

Z
dz =

1X

0 n=0

zn
dz
n+1

kunne n
Som vi ser hadde det vrt veldig behagelig a
a bytte rekkeflge p
a
summasjonen og integrasjonen. Flgende teorem gir oss en grunn n
ar vi kan
gjre dette
Theorem 3.7.1. (Monoton konvergens teoremet) Anta at funksjonene f1 , f2 , f3 , . . .
er positive p
a et interval (a, b) (Alts
a at fk 0 for enhver x). Da er
Z

bX

fn (x) dx =

a n=1

Z
X
n=1

fn (x) dx

som har gyldighet for negative funksjoner.


bruke teoremet f
Ved a
ar vi n
a relativt direkte at
Z
0

Z 1

X
X
log(1 z)
zn
1
2
dz =
dz =
=
2
z
n+1
n
6
n=0 0
n=1

sammenligne med verdien vi fikk tidligere for Li2 (1) kan vi anta at bytte
Ved a
vre sikker ser vi at xn /n + 1 er positiv for alle verdier, og vi
var lovlig. For a
kan puste lettet ut.
Eksempel 3.7.1. Vi ser n
a p
a et liknende integral hvor a > 0
Z
I :=


log 1 + eax dx

hvor a > 0. Vi ser herfra p


a taylorrekka til log(1 + x) som konvergerer for |x| < 1.

III

7 Uendelige rekker

165

Heldigvis s
a er eax < 1 p
a x [0, 1] n
ar a > 0. Direkte f
ar vi da
Z

I=
log 1 + eax dx
0

(1)n+1 nax
e
dx
n
0
n=1



X
enax
(1)n+1
=

dx
n
na
0
n=0
Z

1 X (1)n1
(0 1) dx
a n=1
n2


1
1
1
=
1 2 + 2 + ...
a
2
3
2

=
12a

Eksempel 3.7.2.
Z

log(1 z) log z dz = 2

I :=
0

2
6

Bevis. Skriver ut taylorrekka til log(1 z) slik at integralet kan skrives som
!
Z 1

X
zn
I=
dz
log z
n
0
n=1
Z 1

X
1
=
z n log z dz
n
0
n=1
(
)
1 Z 1

X
z n+1 log z
1
zn
dz

=
n
z+1
0 n+1
0
n=1

X
1
1
=

n
(n
+
1)2
n=1

X
1
1

n
n+1
n=1

= 1 + 1 (2) = 2

X
1
1

+
2
(0 + 1)
(n + 1)2
n=0

2
6

Her ble det brukt i andre overgang at taylorrekka til log(1 z) er analytisk p
a
(0, 1) og i siste mellomregning ble det benyttet at frste sum er en teleskoperende
rekke slik at

 


 

1
1
1
1
1
1
1
S = 1
+

+ ... +

2
2
3
k1
k
k
k+1
1
=1
k+1
slik at n
ar k s
a vil S 1. I siste sum brukte en at S(1) = a0 + S(1).

166

7 Uendelige rekker

III

I gjennom denne delen har vi gjentatte ganger benyttet oss av Basel pro bestemme flgende
blemet, mer eller mindre bevisst. Problemet g
ar ut p
aa
sum



X
1
1
1
1
=
lim
1
+
+
+
.
.
.
+
n
n2
4
9
n2
n=1
bestemme summen p
men det blir frst i neste seksjon at vi viser en m
ate a
a.
avslutte med et lemma som vil hjelpe oss senere.
Her nyer vi med a
Lemma 3.7.1.

X
4 X
1
1
=
2
n
3 n=0 (2n + 1)2
n=1

kalle summen for S og deler den i odde og like ledd


Bevis. Vi begynner a
S :=

X
X
X
1
1
1
=
+
2
2
n
(2n)
(2n + 1)2
n=1
n=1
n=0

Herfra ser vi nrmere p


a frste sum, s
a

1
1 X 1
S
=
=
(2n)2
4 n=1 n2
4

n=1

Bruker vi dette i forrige likning f


ar vi n
a at

X
X
1
1
4 X
1
1
S=
=
S= S+
2
2
4
(2n
+
1)
n
3
(2n
+
1)2
n=0
n=1
n=0

vise.
som var det vi nsket a
Oppgaver
Z
6. Bestem integralet
0


cos log x dx .

III

8 Dobbel Integraler

3.8

167

DOBBEL INTEGRALER

I denne delen skal vi fokusere hovedsaklig p


a dobbeltintegral og se hvordan
lse vanskelige integral. Metoden kan delvis bli sammenliknet
de kan hjelpe a
derivere under integraltegnet.
med a
Det kanskje viktigste theoremet n
ar det kommer til evalueringen av slike
dobbeltintegraler er flgende.
Theorem 3.8.1. (Fubinis Teorem.) Anta at X og Y er to lukkede m
albare rom.
Anta videre at f (x, y) er X Y m
albart. Da er


ZZ
Z Z
Z Z
f (x, y) d(x, y) =
f (x, y) dx dy =
f (x, y) dy dx
XY

hvis og bare hvis flgende gjelder


Z Z


f (x, y) d(x, y) <
1)
XY

Her kan gjerne X og Y eksempelvis vre relle interval X = [0, 1]. Men
theoremet holder og for mye mer abstrakte rom. Dette theoremet er langt mer
abstrakt enn hva vi trenger, og et bevis kan finnes i [? , s. 163]. Essensen er at
vi ikke kan bytte om integralgrensene n
ar det passer oss. Et enkelt eksempel p
a
dette vist under
Eksempel 3.8.1. La oss frst se p
a flgende funksjon
f (x, y) =

x2 y 2
(x2 + y 2 )2

og integralet over enhetskuben S = [0, 1] [0, 1]. Ved a regne ut absoluttverdien


av funksjonen f
as

Z 1Z 1
Z 1 Z y
Z 1
y 2 x2
x2 y 2
|f (x, y)| dx dy =
dx
+
dx
dy
2
2 2
2
2 2
0
0
0
0 (x + y )
y (x + y )

Z 1
1
1
1
=
+
2
dy
2y
2y
y +1
0
Z 1
Z 1
1
1
=
dy
dy.
2
0 y
0 1+y
Slik at absoluttverdien av integralet g
ar mot uendelig. For a g
a mellomregningene litt mer i smmene kan en se p
a likning (2.53) Her f
ar en at
Z
x2 y 2
y
dx =
(x2 + y 2 )2
x2 + y 2
Z
x2 y 2
x
dx = 2
(x2 + y 2 )2
x + y2
Dermed s
a blir
Z 1 Z
0

Z
0

0
1

Z
0


1
Z 1
y
x2 y 2

dy dx =
dx =
2 + y2
(x2 + y 2 )2
x
4
0
0



Z
0
1
x2 y 2
x

dx dy =
dy =
2
2
2
2
2
(x + y )
x +y 1
4
0

168

III

8 Dobbel Integraler

At vi f
ar to ulike verdier kommer av at absoluttverdien ikke er endelig. Akkuratt i
dette eksempelet s
a er absoluttverdien av integralet ikke endelig fordi funksjonen
forandre p
har en singularitet i origo. Ved a
a grensene forandrerer en ogs
a
hvordan en nrmer seg dette punktet med sine riemansummer.
Oppsumert kan en alts
a si at hvis
ZZ
|f (x, y)| d(x, y) =
S

(der S = [a, b] [c, d]) s


a kan godt
b

og

f (x, y) dy dx

f (x, y) dx dy

ha ulike verdier.
Som en ap`ertiff p
a det som kommer senere, tar vi atter en kikk p
a det klassiske Gaussiske integralet

ex dx = 1 ,

Som er essensiel innen blant annet sannsynlighetsregning. Fra fr har vi allerede


vist integralet ved hjelp av gamma og betafunksjonen. Den klassiske utledningen
er derimot ved dobbeltingralet og vist i det p
aflgende eksempelet
Eksempel 3.8.2. Vi definerer flgende integral, og ser p
a kvadratet.
Z

I :=

x2

Z

dx I =

 Z

x2

dx

x2


dx

si n
Variabelnavnet har ingenting a
ar vi integrer, s
a vi lar heller y vre integrasjonsvariabelen i siste integral og f
ar
2

Z

I =

x2

 Z
dx

y 2


dy

ex ey dx dy

Merk her ble integrasjonsrekkeflgen byttet, og i flge Fubinis sats er dette ikke
lov om ikke absoluttverdien av integralet konvergerer. Videre regning gir
Z

e(x

I =

+y 2 )

dx dy .

Herfra bytter vi til polarkoordinater, hvor x2 + y 2 = r2 og r dr d = dx dy


Z

r 2

re

=
0



1 r2
dr d = 2 e
=
2
0

Slik at vi f
ar I = som nsket. At vi kan bytte om grensene flger fra at
se dette s
funksjonen er absolutt konvergent. For a
a kan vi benytte at e >

III

8 Dobbel Integraler

169

0 R s
a
Z

I=

Z


x2
dx =
e

Z
dx +

1
x2

x ex dx +

ex dx
Z

ex dx

e0 dx +

dx +

x ex dx

= 2 + e1
Og siden 2 + e1 < s
a konvergerer integralet absolutt. Her ble det bla brukt
at funksjonen er symmetrisk og synkende.
I videre utledninger vil det stort sett ikke bli vist at integralet konvergerer
absolutt. Da det antas fremmover at integralene konvergerer absolutt om ikke
annet blir sagt. Men det er noe a ha i tankende, og uten tvil en god velse a vise
at integralet konvergerer absolutt.
Eksempel 3.8.3. En alternativ m
ate a beregne integralet p
a er a studere flgende
integral
ZZ


J=
x exp x2 (y 2 + 1) d(x, y)
S

hvor S = [0, ) [0, ). P


a den ene siden s
a er
Z Z


J=
x exp x2 (y 2 + 1) dx dy
0

bruke substitusjonen u = x2 (y 2 + 1) f
ved a
as
Z
 Z

1
dy
u
=
e
du
2
y2 + 1
0
0
  u 0
1
=
arctan y 0 e

2
 

1
=
0 10
2 2

=
4
ha motsatte grenser via t = xy f
Beregner vi derimot integralet ved a
as
Z Z


J=
x exp x2 (y 2 + 1) dy dx
Z0 Z0


=
exp t2 x2 dt dx
0
0
Z
 Z


 2
 2
=
exp t dt
exp x dx
0


=

I
2

Slik at I = som fr.

2

170

III

8 Dobbel Integraler

anta at et integral er et dobbelintegral, som har blitt integrert feil


Det a
veier mye brukt. Alts
a at en har integrert frst x ogs
a med hensyn p
a y i ste skrive om integralet som et dobbeltintegral
defor motsatt. Teknikken blir da a
igjen, snu grensene og beregne det forh
apentligvis enklere integralet. Dette blir
forh
apentligvis klarere i det flgende eksempelet
I forbindelse med gamma- og betafunksjonene ble flgende integral betraktet
Z
exp(ax) exp(bx)
dx
I=
xn
0
Her vil vi studere spesialtilfellet hvor n = 1, da har vi
Z
exp(ax) exp(bx)
I=
dx
x
0
Z Z b
eyx dy dx
=
a

eyx dx dy

=
a

0
b

=
a

e0
exy
lim
x
y
y


dx

dy
y
a
a
= log
b
=

Tilsvarende metode kan bli benyttet p


a en rekke integral. Faktisk har vi flgende
teorem
Theorem 3.8.2. (Frullanis Theorem). La f C 1 (kontinuerlig deriverbar), og
la f (), f (0) R. Da er
 
Z

f (ax) f (bx)
b
dx = f (0) ` log
,
a, b (0, ) ,
x
a
der limx f (x) = ` og limx0 = f (0).
Bevis. Theoremet gjelder under svakere betingelser, men gitt de ovenfor kan et
artig dobbeltintegral benyttes. Vi definerer
ZZ
Y =
f 0 (xy) dx dy
D



der D = (x, y) R2 : x 0 , a y b . Vi beregener integralet begge veier.
P
a den ene siden s
a er

0
Z b Z
Z b
f (xy)
0
Y =
f (xy) dx dy =
dy
y
a
0
a

 
Z b

f (0) `
b
=
dy = f (0) ` log
y
a
a

III

8 Dobbel Integraler

171

bytte om integrasjons rekkeflgen f


Ved a
as derimot
!
a
Z Z b
Z 
Z
f (xy)
f (ax) f (bx)
0
f (xy) dy dx =
Y =
dx
dx =
x
x
a
0
0
0
b
og vi er ferdige. For at bytte av integrasjonsrekkeflgen skal vre lovlig m
a vi
ha at |f 0 (xy)| < 0 hvor y [a, b] og x (0, ).
Flere steder blir teoremet gitt med at ` = 0, men det er ikke vanskelig a finne
eksempler p
a hvor f (0) = 0 og ` 6= 0.
Eksempel 3.8.4.
 
 
Z

q
arctan(px) arctan(qx)


p
log
= 0
=
log
x
2
p
2
q
0
Hvor f (0) = arctan 0 = 0 og ` = arctan() = /2.
N
a har vi tatt for oss Fubini og Frullani integral, s
a da er det passende at vi
tar for oss Fresnel integralene avslutningsvis.
Theorem 3.8.3. (Fresnels Integral) er definert som flger
r
Z
Z

2
2
sin x dx =
cos x dx =
8

Dette kan bevises ved hjelp av eksempelvis kompleks analyse, men her brukes
vise dette trengs et par lemma frst:
selvsagt dobbeltintegralet. Men for a
Lemma 3.8.1.
Z
2
1
1
=
etx dx

t

as direkte
Bevis. Bruk substitusjonen u = t x dx = du/ t , da f
Z
Z
2
2
1
1
1

etx dx =
eu du =

t
t

Der det ble benyttet at verdien av det gaussiske integralet over R er som ble
vist i eksempel (3.8.2) og eksempel (3.8.3).
Lemma 3.8.2.
Z
2
etx sin t dt =
0

1
1 + x4

Z
og
0

etx cos t dt =

x2
1 + x4

Bevis. Det har blitt vist i Del II (proposisjon (2.7.1)) at


Z

et 
I = et sin t dt = 2
sin t + cos t
+ 2
Z

et 
J = et cos t dt =
sin t cos t
2
2
+

172

III

8 Dobbel Integraler

Resutatet kan vises ved a skrive om p


a kompleks form eller via delvis integrasjon.
Med innsatte grenser f
as10
Z

I=
et sin t dt = 2
(3.61)
+ 2
Z0

et cos t dx = 2
J=
(3.62)

+
2
0
sette = 1 og = x2 .
og beviset fullfres ved a
Lemma 3.8.3.
Z
0

1
/n
=
1 + xn
sin(/n)

Bevis. En mer generell form er vist fr i likning (3.26), men vi gjennomg


ar
bruke substitusjonen x 7 1 + u u =
likevell beviset i korte trekk. Ved a
(x 1)1/ kan integralet skrives som
Z
Z
1
1
1
I=
(x 1)(1/n)1 dx
du
=
n
1+u
1 x
0
hvor er et reellt positivttall og substitusjonen Settes n
a x = 1/v f
as


Z 1
1
1
1
1
I=
v 1/n (1 v)1+1/n dv = B + 1,
n 0
n
n
n
Ved a benytte seg av definisjonen til betafunksjonen, og Eulers refleksjonsformel
f
as
I=

1 (1 1/n) (1/n)
/n
=
n
(1)
sin (/n)

og dette fullfrer beviset.


Alternativt er integralet lik Mellin-transformasjonen av f (x) = xn + 1 med
s = 1.
beregne Fresnel integralene, definerer som
Bevis. Vi er n
a endelig klar til a
flgende
Z
Z
F1 :=
sin x2 dx og F2 :=
cos x2 dx

Via substitusjonen t = x omskrives integralene til


Z
Z
sin t
cos t
dt ,
dt
F1 =
F2 =
t
t
0
0
Benyttes n
a lemma (3.8.1) f
as dobbeltintegralene
Z Z
Z Z
2
2
2
2
F1 =
etx sin t dx dt ,
F2 =
etx cos t dx dt
0

0
0
0
10 Disse

likningene beskriver alts


a laplace-transformasjonen av sin t og cos t.

III

8 Dobbel Integraler

173

Snus grensene og lemma (3.8.2) benyttes s


a f
as
Z Z
Z
2
2
2
dx
etx sin t dt dx =
F1 =
0
0 1 + x4
0
Z Z
Z
2
2
2
x2
F2 =
etx sin t dt dx =
dx
0
0 1 + x4
0
Det frste integralet kan beregnes, men det vises frst at
Z
Z 2
Z 2
Z
dx
x dx
x dx
dx
=

=0
4
4
4
1
+
x
1
+
x
1
+
x
1
+
x4
0
0
0
0
se dette kan vi enten bruke lemma (2.6.1) med n = 4 eller legge merke til
For a
at


Z 2
Z 0
Z 1 2
x 1
(1/u)2 1
du
u 1
dx
=

du
4
4
1 + x4
u2
1
1 1 + (1/u)
0 1+u
dele integralet i to s
Ved n
aa
a har en
Z
0

x2 1
dx +
1 + x4

x2 1
dx =
1 + x4

Z
0

x2 1
dx
1 + x4

u2 1
du
1 + u4

S
a I = 0 som nsket. Dette medfrer at F1 = F2 og
2
F1 = F2 =

Z
0

dx
2
/4
=
=
4
1+x
sin(/4)

Hvor lemma (3.8.3) ble benyttet. Dette fullfrer beviset.


Som lovet i forrige del tar vi avslutningsvis og ser p
a flgende problem.
Eksempel 3.8.5. (Basel problemet) Problemet er gammelt og ber oss beregne

summen av inversen av alle kvadrattallene. Det var frst Euler som klarte a
bestemme summen

X
1
2
=
,
n2
6
n=1

gjennom forholdsvis lange utregninger som kan vre noe vanskelig a henge med
p
a. Vi fokuserer heller p
a et bevis som kom senere av J.D Harper.
Bevis. Vi tar utgangspunkt i flgende integral
ZZ
x
J :=
d(x, y)
2
2 2
D (1 + x ) (1 + x y )


hvor D (x, y) R2 : x 0, 0 y 1 . P
a den ene siden har vi at integralet
kan skrives som
Z 1

Z Z 1
Z
1
dy
x
dy
dx
=
dx
J=
2
2 2
2
2
x(1 + x2 )
0
0 (1/x) + y
0
0 (1 + x ) (1 + x y )

174

III

8 Dobbel Integraler

det innerste integralet kan lses via substitusjonen y = x1 tan u som gir
1


Z 
Z
arctan(xy)
arctan x
arctan2 x
2
J=
dx
=
=
=
1 + x2
1 + x2
2
8
0
0
0
0
Bruker vi n
a fubinis sats og regner ut integralet ved motsatte grenser f
as
Z Z 1
x
J=
dx dy
2 ) (1 + x2 y 2 )
(1
+
x
0
0
Det innerste integralet kan lses via delbrkoppspalting som gir gir


2x
x
1
2y 2 x

=
(1 + x2 ) (1 + x2 y 2 )
2 (y 2 1) 1 + x2 y 2
1 + x2




1
d
d
2 2
2
=
log 1 + x y
log 1 + x
2 (y 2 1) dx
dx
Der y ble betraktet som en konstant. Setter vi inn og forenkler, skrives integralet
om til
 

Z 1
Z
1 + x2 y 2
log y
1
log
dy
=
dy
J=
2
2
2
2 (y 1)
1+x
0 y 1
0
0

Hvor det ble benyttet at n
ar x s
a vil log (x2 +y 2 )/(x1 +1) log(y 2 /1) =
2 log y og x 0 s
a vil log(1/1) = 0. Vi har alts
a n
a vist at
Z 1
2
log y

dy =
2
8
0 y 1
Som ikke var akkuratt det vi skulle ha... En kan n
a legge merke til at y 2 < 1 for
2
y (0, 1) slik at vi kan rekkeutvikle 1/(1 y ) som en geometrisk serie. Da f
as
Z 1
Z 1

Z 1

X
X
X
1
log y
2n
2n
dy
=

log
y
y
=

y
log
y
dy
=

2
1y
(2n + 1)2
0
0
n=0
n=0 0
n=0
benyttes n
a lemma (3.7.1) f
ar vi at

X
n=0

1
2
=
(2n + 1)2
8

4 X
1
4 2
=

3 n=0 (2n + 1)2


3
8

X
1
2
=
2
n
6
n=1

vise.
som var det vi nsket a
Oppgaver
7. Beregn flgende integral
Z
I=
0

hvor p, q R.

1
log
x

p + qeax
p + qebx


dx

III

8 Dobbel Integraler

175

8. Beregn flgende integral


Z
I=
0

sin(px) sin(qx)
dx
2

hvor p, q R.
9. Vis at
Z
0

e2x ex
dx =
x(ex + 1)(e2x + 1)

og bestem integralet.

Z
0

ex e2x
dx
x

176

3.9

9 Kompleks Integrasjon

KOMPLEKS INTEGRASJON

III

III

9.1 Typer integraler

3.9.1

TYPER

177

INTEGRALER

regne ut de ulike typene


La oss frst ta to sm
a lemma som vil hjelpe oss a
integral
Lemma 3.9.1. (M-L ulikheten) La f vre en kompleks, kontinuerlig funksjon p
a
konturen og dersom absoluttverdien |f (z)| er mindre enn en konstant M for alle
z p
a da er
Z



F (z) dz M `() .

Hvor `() betegner buelengden til og M = sup |f (z)|11 .


z

Bevis. Fra den kontinuerlige trekantulikheten har en at


Z
Z


f (x) dx |f (x)| dx


fra dette flger det at integralet kan skrives om som

Z
Z b





f (z) dz =
f
(t)
(t)

dt




C
a
Z b
Z b
Z






(t)

dt M
dt = M
dz = M L
f (t) (t)
a

hvor kurven ble parametrisert via (t), og M er den maksimale teoretiske verdien f () kan ha. I andre overgang ble det brukt direkte at |a b| |a| |b|

i
CD

CR
x
R

CR

CD
i

(a) Illustrasjonen av konturen i det vre


halvplanet

(b) Illustrasjonen av konturen i det nedre halvplanet

Figur 3.2
11 Dersom

et maksimum eksisterer s
a vil supremum og maksimum vre like. Derimot s
a er det noen
prate om en minste teoretisk
ganger maksimum ikke eksisterer, men det likevell er nskeliga
vre grense. Et klassisk eksempel er mengden S = {0 < r < 2 , r = Q}, her finnes ikke noe
maksimum siden en alltid kan finne et litt strre
ar mot

rasjonellt tall 1.4, 1.41, 1.414, . . . Flgen g


2 , men aldri helt kommer dit s
a sup S = 2 .

178

III

9 Kompleks Integrasjon

Rasjonale funksjoner
Theorem 3.9.1. La P (x) og Q(x) vre polynomer slik at deg(Q) deg(P ) + 2
da er


Z
m
X
P (x)
P (x)
dx = 2i
, zk
Res
Q(x)
Q(x)
k=1

hvor z1 , . . . , zm er singularitene til P (x)/Q(x) i det vre halvplanet y > 0.


Bevis. Her gis en kort skisse av et bevis for theoremet. Beviset deles inn i tre
deler. (1) vi viser at integralet konverger. (2) Vi lager en kontur i det komplekse
avslutte beviset brukes
planet, og viser at bue-integralet er null. (3) For a
residue theoremet.
(1) For det frste har ikke Q(x) noen nullpunkter p
a den reelle aksen, slik
at P (x)/Q(x) er definert p
a hele R. Vi ser frst p
a om integralet konvergerer
absolutt, for dette impliserer konvergens. For store verdier av x, s
a domineres
polynomene av deres frste ledd, s
a
Z
Z
P (x)
C|x|n2


dx
Q(x)
D|x|n

og siden integralet p
a hyre side konvergerer, konvergerer ogs
a integralet v
art.
vre helt formelle burde vi ha delt opp integralet i to, og vist at integralet
For a
over (0, ) og (, 0) konvergerer hver for seg.
(2) Vi integrerer funksjonen langs konturen vist i figur neger, for s
a a la R .
Da er integralet
Z
Z
Z
P (x)
P (x)
P (x)
=
+
R Q(x)
C1 Q(x)
C Q(x)
vise at integralet i det komplekse planet, som flger kurven er
M
alet er n
aa
bruke M L-ulikheten s
null. Ved a
a er
Z
P (x)
P (x)
C|z|n2
C
R sup
R
R
n
D|z|
DR2
z Q(x)
C1 Q(x)
som g
ar mot null n
ar R .
(3)

Fra Cauchys integral formel har en n


a
I

Z
Z
P (x)
P (x)
P (x)
dx = lim
dz
dz
R
Q(x)
C Q(x)
C1 Q(x)


I
m
X
P (x)
P (x)
= lim
dz 0 = 2i
Res
, zk
R C Q(x)
Q(x)
k=1

som var det som skulle vises.

III

9.1 Typer integraler

179

Integralet over P (x)/Q(x) konvergerer ogs


a dersom deg P + 1 deg Q, men
ulempen er da at integralet over halvsirkelen ikke g
ar mot null. Da m
a en beregne dette integralet eksplisitt, og det kan vre vanskeligere enn det opprinnelige
integralet.
bruke
Dersom b
ade graden til P og Q er partall, s
a er P/Q like. Ved a
symmetrien omkring x-aksen f
ar en da


Z
m
X
P (x)
P (x)
dx = i
, zk
Res
Q(x)
Q(x)
0
k=1

slik at vi ikke bare kan beregne integraler over R, men ogs


a R+ og R .
Theoremet sier at vi m
a studere polene til P/Q i det vre halvplanet, men
en kunne like gjerne studert polene i det nedre halvplanet. Da m
a en integrere
motsatt vei, og fra omlpstallet f
ar en et negativt bidrag fra hver pol, slik at


Z
n
X
P (x)
P (x)
dx = 2i
, zk
Res
Q(x)
Q(x)
k=1

hvor zn er polene i det nedre halvplanet. En kan alts


a fritt velge det halvplanet
som har frrest eller penest singulariter. Merk og at det ikke er ndvendig at
P og Q er polynomer, men at det holder at P er analytisk i det halvplanet en
studerer.
Eksempel 3.9.1. I dette eksempelet skal vi beregne flgende integral
Z 2
x +2
dx
x4 + 1
0
ved hjelp av kompleks analyse. Integralet er et spesialtilfellet av ??, men brukes
her for a vise frem kompleks integrasjon. Siden teller er av grad 4, mens nevner
er av grad 2, medfrer at integralet konvergerer. Videre s
a er integralet symetrisk
omkring x aksen, eller like s
a
Z 2
Z
x +2
1 x2 + 2
dx
=
dx
(3.63)
x4 + 1
2 x4 + 1
0
benytte seg av theorem (3.9.1) er oppfyllt. Integralet er
Slik at vilk
arene for a
dermed likt


Z 2
m
X
x +2
P (x)
dx
=
2i
,
z
Res
(3.64)
k
4
Q(x)
x + 1
k=1

Singuaritenene til funksjonen er hvor z 4 = 1, n


a kan en skrive 1 = ei(+2k)
s
a rttene blir
z0
z2

= ei/4
= ei5/4

z1
z2

= ei3/4
= ei7/4

Av disse ligger bare z0 og z1 i det vre halvplanet og i tillegg er de enkle singula bruke ?? fra ?? kan residyene skrives som
riterer. Ved a
 2

z +2
z2 + 2
1
1
Res 4
, zk = lim
=
+
zzk
z +1
4z 3
4zk
2zk3

180

9 Kompleks Integrasjon

III

dele likning (3.64) p


Ved a
a 2, og s
a benytte seg av likning (3.63) har en at





Z 2
x +2
P (x)
P (x)
dx = i Res
, z0 + Res
, z1
x4 + 1
Q(x)
Q(x)
0
 


1
1
1
1
+
= i
+
+
4z0
2z03
4z1
2z13
Merk at n
a kan en utnytte symmetrien til enhetsrttene en har at z13 = z0 og at
3
z0 = z1 . Dette gir at

 



Z 2
x +2
1
1
1
1
3
1
1
dx = i
+
+
+
=
i
+
x4 + 1
4z0
2z1
4z1
2z0
4
z1
z0
0
skrive om rttene til deres real og kompleks del ved hjelp av av eulers
Ved a
formel ei = cos + i sin f
as

 

Z 2
3
1
i
1
i
3
x +2

dx
=
i

=
4+1
x
4
2
2
2
2
2
2
0
g
som var det som skulle vises. For a
a detaljene litt i smmene s
a har en at
z13 = ei9/4 = e2i ei9/4 = e/4 = z0
og tilsvarende s
a er z03 = ei3/4 = z1 som en og kan se siden rttene er symmet regne ut rttene eksplisitt f
risk om den imaginre aksen. Ved a
ar en fra eulers
formel

1
i
1

sin
=
= ei/4 = cos
z0
4
4
2
2
i
1
3
3
1
i3/4
=e
= cos
sin
= .
z1
4
4
2
2
bestemme integralet noe enklere kunne en brukt symmmetrien av z0 og
For a
z1 .


Z 2

x +2
3
1
1
3
3

dx
=
i
+
=
i 2iImz1 =
sin
4
x
+
1
4
z
z
4
2
4
1
0
0
bestemme en trigonometrisk verdi.
som gjorde at vi bare trengte a
bestemme integralet ble det kun brukt enkel algebra, trigonometriske
For a
beregne
verdier og at i2 = 1. P
a mange m
ater er dette en enklere m
ate a
integraler over brker, enn det som ble vist i seksjon to, ulempen er selvsagt at
bruke disse nye verktyene
en trenger en dypere matematisk innsikt for a
Eksempel 3.9.2. I denne delen skal vi studere et kjent integral
Z
dx
In =
1
+
xn
0
der n N2 er et partall. Igjen tar vi utgangspunkt i den kjente konturen som er
vist i figur (3.3). Grunnen til at vi kan bruke denne konturen er at integralet er
symmetrisk omkring origo
Z
Z
dx
1
dx
In =
=
.
n
1+x
2 1 + xn
0

III

9.1 Typer integraler

181

i
CD
x
R

CR

Figur 3.3: Slyfen C = CR CD , hvor polene til 1/(1 + x10 ) i det vre halvplan
er markert.
Som flger fra at n er like. Integralet langs CR blir som fr null da graden av nevner er 2 eller hyre, mens graden til teller alltid er konstant 1. Theorem (3.9.1)
gir da at


Z
m
X
1
1 dx
In =
dx = i
Res
, zk
(3.65)
2 0 1 + xn
1 + xn
k=1

hvor vi betrakter residyene til funksjonen i det vre halvplanet. Frst m


a vi se
hvor xn + 1 = 0, og dette blir bare enhetsrttene
zk = ei(2k+1)/n ,

0k

n
1
2

regne ut en residye kan vi igjen bruke ?? fra ?? slik at


For a
Res(zk ) = lim

zzk

1
1
= ei(2k+1)/n
(1 + z n )0
n

hvor minustegnet kommer av at vi betrakter residyene i det vre halvplanet. N


a
har vi at

 n/21
m
X
X
1
ei(2k+1)/n
Res
,
z

=
k
1 + xn
n
k=1
k=0
"
#
n/2
n/21
ei2/n
1
ei/n X  i2/n k
ei/n
=
e
=
n
n
ei2/n 1
k=0
n
hvor den siste overgangen flger fra den geometriske rekken. Ved a
a forenkle
algebraen kan en skrive summen av residyene som
m
X
k=1

Res (zk ) =

ei/n
2
i2/n
n e
1

1
2i
1
1/n
i/n
=
.
i/n
in e
i sin(/n)
e

sette inn dette i


hvor bare (z + z 1 )/2i = (eiz + eiz )/2i sin z ble brukt. Ved a
likning (3.65) f
ar en n
a


Z
m
X
dx
1
/n
=
i
Res
,
z
.
(3.66)
k =
n
n
1
+
x
1
+
x
sin(/n)

k=1

som var det som skulle vises.

182

III

9 Kompleks Integrasjon

Men stemmer likning (3.66) dersom n ikke er odde? Svaret er overraskende


nok ja, men da m
a vi bruke en alternativ kontur som vi skal se p
a senere. Dette
er fordi dersom n ikke er like, s
a er ikke funksjonen symmetrisk.
Den neste type integral vi skal se p
a er p
a formen p(x)eiax /q(x), men fr det
et viktig lemma
Lemma 3.9.2. (Jordans lemma) La f vre en kontinuerlig funksjon i det komplekse planet, definert p
a flgende kontur
CR = {z : z = Rei , [0, ]}
med radius R > 0, 0 < og sentrum i origo. Dersom funksjonen f er p
a
formen
f (z) = eiaz g(z) , z CR
og a > 0, s
a er integralet over CR begrenset av
Z







sup g Rei
f (z) dz

a
[0,]
CR
med likhet kun n
ar g(z) er identisk lik null.
Merk at dersom a = 0 flger det fra ML-ulikheten at

Z






2R

f (z) dz
sup g Rei R sup g Rei

[0,]
[0,]
CR
Et helt likt argument holder for en kontur i det nedre halvplanet dersom a < 0.
Dette er p
a mange m
ater en utvidet utgave av jordans originale lemma siden da
er = .
Sett inn ref til linjeintegral

bruke substitusjonen z = Rei = R(cos + i sin ), da


Bevis. Vi begynner med a
blir grensene 0 til .
Z
Z

f (z) dz =
g Rei eiaR(cos +i sin ) iRei d
CR
0
Z

=R
g Rei eaR(i cos sin ) iei d .
0

Vi kan n
a ta
av integralet og bruke den kontinuerlige trekant R absoluttverdien
R
ulikheten f |f |
Z

Z
Z





R
g Rei eaR sin d RMR
f
(z)
dz
eaR sin d . (3.67)


CR

iz

Hvor det ble brukt blant annet at |e | = | cos z + i sin z| = | cos z| = 1. Siden
integralet er positivt s
a betrakter vi heller omr
adet fra 0 til . Vi innfrte og


MR = sup g Rei
[0,]

forenkle notasjonen litt. Da kan ulikheten skrives som


for a
Z

Z
Z /2


aR sin

RMR
f
(z)
dz
e
d

2RM
eaR sin d
R


CR

III

9.1 Typer integraler

183

hvor vi i siste overgang brukte symmetrien av sin = sin( ). Dette kan vises
dele intervalet i to eller bruke likning (2.30) fra proposisjon (2.4.5) med
ved a
m = 0, n = 1 og f (x) = exp(aRx). Funksjonen sin er konkav p
a intervalet
[0, /2]. Alts
a at funksjonen ligger over den rette linja mellom endepunktene
2
sin ,

(3.68)

vise, men kan sees intiutivt fra figur 8. Ved a


bruke
dette overlates til leser a

1 y
sin
2/

/2

Figur 3.4: Figuren viser f () = 2/, og g() = sin 2


dette f
ar vi endelig at

Z
Z




f (z) dz 2RMR

CR

/2

e2aR/ d =

1 eaR MR MR
a
a

som var det som skulle vises.


Fra dette flger det direkte at
Korollar 3.9.1. Dersom de eneste singularitetene til f (z) er poler, da er
Z
lim
g(z)eiaz dz = 0
R

CR

forutsatt at a > 0, |f (z)| 0 n


ar R .
For at linjeintegralet over p(z)/q(z) skulle g
a mot null, kreves det at p(z)/q(z)
1/R2 n
ar z . Mens her ser vi at s
a lenge p(z)/q(z) g
ar mot null, uansett
hvor sakte s
a vil linjeintegralet over p(z)eiz /q(z) g
a mot null. Dette har med
tvinge funksjonen til a
konverge raskere.
hvordan eiz bidrar til a


Proposisjon 3.9.1. La P (x) og Q(x) vre funksjoner slik at P (x)/Q(x) 0
n
ar x . Da er


Z
m
P (x) iax
a X
P (x)
e dx = 2i
Res
, zk
(3.69)
|a|
Q(x)
Q(x)
k=1

gitt at Q(x) ikke har noen singulariter p


a den reelle aksen.
Her er a R/{0} er en reell konstant ulik null og z1 , . . . , zm er singularitene
til P (x)/Q(x) i det vre halvplanet dersom a > 0 og det nedre halvplanet dersom
a < 0.

184

9 Kompleks Integrasjon

III

Dersom P og Q er polynomer m
a en alts
a ha deg(Q) deg(P ) + 1 for a
sikre konvergens. Dette er et snillere krav enn integraler p
a formen P/Q, hvor
vi trengte deg(Q) deg(P ) + 2 for konvergens.
Frst s
a kommer leddet a/|a| fra at i det nedre halvplanet integrerer vi motsatt vei, og f
ar et negativt bidrag fra hver singularitet. Dette leddet defineres
gjerne som sign a = a/|a|, og gir oss fortegnet til a. En skisse for beviset gis, og
fremmgangsm
aten er tilsvarende som i (3.9.1).
Vi kan anta at integralet langs den reelle aksen konvergerer siden her er
|eiaz | = 1, og vi f
ar
Z
Z


Z N
N P (x)

N
P (x)
P (x) ix


ix


dx
dx

e
dx







M Q(x)
M Q(x)
M Q(x)
som er samme tilfellet som i (3.9.1), og integralet konvergerer dermed. Merk
at eix = |cos x + i sin y| = |cos x| = 1. Vi studerer s
a absoluttverdien av
integralet langs sirkelbuen
Z
Z






P (z) iaz
P (z) iaz


dz R sup |P (z)| eiaz
e
dz

e




|Q(z)|
zC1
CR Q(z)
CR Q(z)
Legg merke til at |eiaz | = |ea(ixy) | = eay . Dermed s
a er

Z

1
P (z) iaz

e dz R 2 eay

R
C1 Q(z)
hvor vi som fr benyttet at P/Q 1/R2 n
ar R . For at vi skal f
a samme form
som i likning (3.69) m
a integralet langs sirkelbuen g
a mot null n
ar R 12 .
Velger vi en halvsirkel i det vre planet s
a vil y n
ar R . For her er y
den strste y-verdien til halvsirkelen.
Dersom a < 0, s
a vil eay g
a mot uendelig. Vi m
a alts
a velge det nedre
halvplanet n
ar a < 0, slik at y n
ar R .
Eksempel 3.9.3. Et av de mest klassiske integralene p
a denne formen er
Z
cos x
1
+ x2

Dette integralet kan skrives som


Z

Z
cos x
eiz
dx = Re
dz ,
2
2
1 + x
1 + z
bruke eulers formel ei = cos + i sin . M
beregne
ved a
alet vil n
a vre a
integralet p
a hyresiden og s
a ta realdelen av hva det n
a enn er vi f
ar som svar.
Siden a > 0, s
a ser vi p
a singularitene til funksjonen i det vre halvplanet.
z1 = i ,

z0 = i ,

her er det bare z1 som ligger i vre halvplan, s


a residyen blir
 iz

e
1 1
eiz
Res
, i = lim
=
e .
2
zi 2z
1+z
2i
12 Det

er ikke alltid dette er tilfellet og slike integral vil bli diskutert senere.

III

9.1 Typer integraler

185

bruke residue theoremet s


Ved a
a har vi at
"Z
#
Z
Z
R
eix
eiz
eiz
lim
dz = lim
+
.
2
2
R 1 + z 2
R
R 1 + x
CD 1 + z
I det vre halvplanet g
ar integralet over CD mot null, mens hyresiden er gitt
lse likningen f
som 2i Res(i). Ved a
ar en alts
a

eix
dx = 2i Res(i) + lim
R
1 + x2

Z
CD

eiz

dx =
.
1 + x2
e

ta henholdsvis realdelen og imainer delen av likningen f


Ved a
ar en
Z
Z
cos x

sin x
dx
=
dx = 0 ,
og
2
2
e
1 + x
1 + x
hvor det siste integralet kunne en ha forventet da funksjonen er symmetrisk
omkring origo.

Kunne vi heller ha skrevet om 2 cos z = eiz + eiz ? Faktisk ikke! For a


begrense veksten til eiaz m
atte vi taktisk velge halvplan, derimot om vi bruker
velge rett halvplan siden eiz bl
2 cos z = eiz + eiz er det umulig a
aser opp i
iz
nedre og e i vre.
Hele diskusjonen med valg av plan er noe meningsls. Anta at du skal
integrere en funksjon cos(x)P/Q, da kan en helt rett velge eiz P/Q, men
bruke at cos(x) = cos x kunne en ogs
ved a
a like gjerne ha brukt eiz P/Q.
Tilsvarende for sin x s
a er sin(z)xP/Q = sin zxP/Q siden sin x er odde.
Det a velge rett kontur, og det a kunne vite n
ar en kan velge er derimot svrt
viktig. Og for et knippe funksjoner s
a kan singularitene i det nedre halvplanet
regne ut enn i det vre.
vre enklere a
Trigonometriske integral
I denne delen skal vi igjen studere integraler p
a formen
Z

R(cos x, sin x) dz
0

beregne disse p
og vise atter en m
ate a
a.
Proposisjon 3.9.2. La R(x, y) vre en rasjonell funksjon, da er
Z

I
R(cos x, sin x) dx =

z + z 1 z z 1
,
2
2i

dz
iz

|z|=1

bestemme n
bruke
Det er noen ganger vanskelig a
ar det er fordelaktig a
Weierstrass-substitusjon eller proposisjonen ovenfor. En ls tommelfinger regel
er at dersom integralet krever delbrksoppspalting etter Weiersttrass-substitusjonen,
foretrekke. Dersom rttene til nevner har hy multiplier proposisjon (3.9.2) a
foretrekke.
sitet kan likevell Weiestrass vre a

186

III

9 Kompleks Integrasjon

bruke substitusjonen ovenfor s


Ved a
a har en
Z 2
dx
2
dz
=
2
+
cos
x
i
1
+
4z
+ z2
0
som er mer komplisert a integrere enn fremgangsm
aten i eksempel (2.5.5). Som
vanlig er dette siste metode som br testes ut for a lse slike integral, da en som
regel kan finne langt mer elegante lsninger.
Eksempel 3.9.4. La oss bestemme integralet
Z 2
sin2 x
dx
2 + cos x
0
bruke proposisjon (3.9.2) kan integralet skrives som
Ved a

2 . "
2 #
Z 2
I 
z z 1
sin2 x
dz
z z 1
2+
dx =
2 + cos x
2i
2i
iz
0
C
I
2
2
i
(z 1)
=
dz
2
2 C z (1 + 4z + z 2 )
Det neste steget er a bestemme hvor residuene til f (z) befinner seg. Vi ignorerer
alts
a faktoren i/2 frem til slutten. Konturen C er alts
a med klokken omkring
enhetssirkelen som vist i figur (3.5). Singularitene til funksjonen er

vokser

synker

synker

vokser

Figur 3.5

z0 = 0 , z1 = 2 + 3 , og z2 = 2 3
Her er det bare z1 og z3 , som ligger i enhetsdisken, og begge er enkle poler.
Residyen til z0 blir
Res(0, f ) = lim

z0

(z 2 1)(2 + 3z + 6z 2 + z 3 )
d 2
z f (z) = lim
= 4
z0
dz
(1 + 4z + z 2 )2

For litt pene notasjonen skriver vi = z2 , den frste singulariteten kan regnes
ut som

( 2 1)2
h(z)
Res(f, ) = lim 0
=
= 2 3
2
z g (z)
2(1 + 6 + 2 )

III

9.1 Typer integraler

187

g
For a
a mellomregningene litt mer i smmene har en at 1 + 6 + 2 2 = (1 +
2
a er
4 + ) + (2 + 2 ) = (2 + ), siden er en rot av f (z) = z 2 + 4z + 1, s
flgelig 2 + 4 + 1 = 0. En kan da skrive om brken som flger.
( 2 1)2
1 ( 2 1)2
1 ( 1/)2
=
=
(1 + 6 + 2 2 )
2 (2 + )
2
2+

Beregningene fullfres n
a ved a bruke at 1/ = 2Im = 2 3 og 2 + = 3 .
Totalt s
a er alts
a integralet
Z 2
I
sin2 x
i
(z 2 1)2
dx =
dz
2 + cos x
2 C z 2 (1 + 4z + z 2 )
0
h
i

i 
=
2i Res(0) + Res(2 + 3 ) = 2(2 3 ) = 2
2
beregne residuen til , kunne en frst ha
som fullfrer beregningene. For a
skrevet om funksjonen
Res(, f ) = lim

1 ( 1/)2
(z 2 1)2 /z 2
=
= 2 3
2
0
[1 + 4z + z ]
2
2+

penbar.
som fr. Klart raskere, men kanskje ikke like a
Fresnell Integralene
Z

sin(x2 ) dx =

cos(x2 ) dx =

Denne seksjonen er hovedsaklig basert p


a [? ], men fremmgangsm
aten er mye
skrive om funksjonen p
eldre. Det frste vi gjr er a
a kompleks form, slik at
Z

Z

Z
Z
2
ix2
2
ix2
sin(x ) dx = Im
e dx og
cos(x ) dx = Re
e dx

Legg merke til at vi ikke uten videre kan bruke substitusjonen z 2 = ix2 , siden
dette delegger grensene helt13 .
Integralet har ingen singulariter i det komplekse planet, slik at vi ikke kan
beregne integralet. Et annet problem er at
bruke den vante halvsirkelen til a
2
f (z) = eiz vokser mot uendelig n
ar Im(f ) > 0 og Re(f ) < 0, se figur 8. Dette
medfrer at dersom vi bruker den vante halvsirkelen vil linjeintegralet g
a mot
2
uendelig. Vi m
a alts
a integrere eix over en kvadrant hvor den synker for at
linjeintegralet skal konvergere. Valget blir derfor mellom I og III kvadrant. For
enkelhetensskyld velges I, men akkuratt samme bevis holder og for III kvadrant.
Forskjellene er at i III kvadrant m
a en holde tungen litt benere i munnen med
integrere over hele frste kvadrant
tanke p
a fortegn. Dersom vi velger a
C = {x : 0 x R} {z : z = Rei , 0
13 Se

} {y : 0 y R}
2

(3.70)

for
eksempel
http://math.stackexchange.com/questions/163946/
la
are-complex-substitutions-legal-in-integration
for en diskusjon av problemet. Ved a

arbeide med to substitusjoner


ix2 7 z 2 f
ar en z = i x, slik at en ender opp med a
a en
p
gang.
Spesielt ligger problemet i at dette blir et vei integral i det komplekse planet fra i
tolke enn v
til i som er vanskeligere a
art opprinnelige integral!

188

9 Kompleks Integrasjon

C3

III

C2

C1
2

Figur 3.6: Viser hvor funksjonen eix vokser og minker.


vil vi uheldigvis bare komme frem til at integralene er like14
Z
Z
cos(x2 ) dx
sin(x2 ) dx =

som er nyttig og interessant i seg selv, men ikke hjelper oss beregne verdien av
prve a
heller bruke
integralene. Vi m
a alts
a velge en mindre kontur. Ved a
halvparten av frste kvadrant
C = {x : 0 x R}
|
{z
}
C1

} { z : z = rei/4 , 0 r R }
{ z : z = Rei , 0
|
{z
}
4}
|
{z
C3

(3.71)

C2

beregne integralet. Konturen C er vist n


s
a vil en klare a
a i figur (3.7). Fremm-

C3

C2

C1
Figur 3.7: Viser konturen C = C1 C2 C3 fra likning (3.71), og polene til eiz
p
a omr
adet.
gangsm
aten blir som fr at vi bruker at linjeintegralet over en lukket slyfe i det
komplekse planet er lik singularitetene
I
m
X
2
ix2
e dx = 2
Res(eix , zk ) = 0
C

k=0

da funksjonen aldri er null i det komplekse planet. En har da at


I
I
I
2
2
2
eix dx +
eix dx
eix dx = 0
C1

C2

C3

da C = C1 C2 C3 , og C3 g
a mot klokken. Vi beregner n
a ett og ett integral.
14 Dette

er gitt som velse til leser, se oppgave 4.

III

9.1 Typer integraler

189

C1) Vi kan parametrisere integralet ved a bruke (t) = y 0 + t x, fra t = 0 til


t=R
I
Z R
2
iz 2
e dz =
eit dt
(3.72)
C1

som vi ser blir integralet v


art n
ar R .
C2) Dette integralet flger en kvartsirkel og det vil vre behagelig om dette
bruke ML-ulikheten direkte
integralet g
ar mot null n
ar R . La oss prve a
Z







2
2
R
R



eiz dz
sup eiR exp(i2)
sup
eR sin 2

4 [0,/4]
4 [0,/4]
2
hvor det ble brukt at z 2 = (Rei )2 = R2 ei2+ , og Eulers formel. For [0, /4]
2
s
a er sin 2 en strengt voksende funksjon. Alts
a synker eR sin 2 for [0, 4]
og den strste verdien utrykket kan f
a er alts
a n
ar = 0. Dette gir
Z



2
R
R R2 sin 20

e

eiz dz

4
4
2
Dersom vi n
a lar R g
a mot uendelig har vi alts
a vist

Z


2
R
eiz dz lim
lim

R 4
R
2
som sier oss fint lite. Dersom vi i stedet hadde studert intervalet15 (0, /4) s
a
2
penbart g
ville integralet asymptotisk g
att som ReR , som a
ar mot null. Det
vise dette. Vi trenger alts
er alts
a bare s
avidt ML-ulikehen ikke klarer a
a en
skarpere ulikhet enn hva ML-ulikheten kan gi oss. La oss heller bruke samme
teknikk som i beviset til Jordans lemma. Via trekant-ulikheten har vi
Z
Z
Z /4



2
iz2
iz 2


e
dz
|dz|

R
eR sin 2 d
(3.73)
e



C2

C2

bruke z = Rei , s
hvor atter en gang har parametrisert integralet ved a
a z2 =
2 i2
2
R e = R (cos 2 + i sin 2). Ganger en m
a med i, og tar absoluttverdien f
as
uttrykket ovenfor. Tanken er n
a igjen a bruke at sin 2 er konkav. Ved a sette inn
= 2x i likning (3.68) f
ar en
2

2x sin 2x , 0 2x

x sin 2x , 0 x

bruker vi dette i likning (3.73) f


ar en n
a
Z

Z /4


2
2

eiz dz R
eR (4x/) dx R

C2




R2
1

2
4R
4R

som heldigvis g
ar mot null n
ar R . Dette er et eksempel p
a hvor MLulikheten ikke fungerer, men integralet likevell konvergerer mot null.
15 Men

pne interval! Dette medfrer at slyfen v


vi kan ikke studere a
ar ikke hadde vrt sammenhengene og Cauchys integralformel ville ikke vrt gyldig.

190

III

9 Kompleks Integrasjon

C3) For det siste integralet bruker vi flgende parametrisering


1+i
z(t) = ei(/4)t = t ,
2

0 t t0

og dz = (1 + i) dt/ 2 . Her er t0 valgt p


a en slik m
ate at 02 + z(t0 )2 = R, alts
a
at avstanden til origo er R n
ar t = t0 . Integralet kan da skrives som
( 
2 )
Z t0
Z
Z


2
1
+
i
1 + i t0
1
+
i
iz
exp i t
dt =
exp i2 t2 dt
e dz =
2
2
2
0
0
Cr
2

sette inn 25 26
hvor det siste uttrykket selvsagt gjeennskjennes som et . Ved a
og 27 inn i 47 har vi
Z R
I
Z
1 + i t0 t2
ix2
ix2

e dx +
e dx
e
dt = 0
2
0
C2
0
Vi kan n
a la R . Da vil integralet langs C2 g
a mot null, slik at
r
r

Z
Z

1+i
1 + i t2
ix2
e dx =
=
+i
e
dt =

2
8
8
2
2
0
0
2

Siden eix = cos x2 + i sin x2 har vi n


a
r
r
Z
Z

2
2
+i
cos x dx + i
sin x dx =
8
8
0
0
som var det som skulle vises. For a vre ekstra pedantisk kan en n
a ta realdelen
av likningen og imaginerdelen og bruke at begge funksjonene er symmetrisk
omkring origo. Men dette f
ar leser klare p
a egenh
and.
beregne
Merk at vi endte opp med a bruke det gaussiske-integralet likevell. A
dette integralet med kompleks integrasjon er noe vanskelig men for et slikt bevis
kan en se seksjon 8 eller oppgave 4.
Det gaussiske integralet
I dette avsnittet skal vi studere flgende integral
I
2
ei(z1/2)
dz
2iz
DR 1 e
bruke integralet ovenfor til a
vise at
i fra Gamelin. M
alet er a
Z

2
es ds =

(3.74)

som vi har sett flere ganger fr. Konturen som vi integrerer omkring er parallelogrammet DR med sider 12 (1 + i)R som er vist i figur (3.8).
En skisse av beviset er som flger, integralet over hele konturen kan bestemmes fra Cauchys residue theorem, den eneste polen er z = 0 slik at
I
i1
f (z) dz = 2i Res [f (z), z = 0] = ei/4 =
(3.75)
2
DR

III

9.1 Typer integraler

191

1
4
3

x
1

2
1

Figur 3.8: Illustrasjon av konturen en integrerer rundt, de stiplede linjene er


henholdsvis 1 og 3 .
En kan og betrakte integralet som en sum av fire linjeintegral
I
Z
Z
Z
Z
=
+
+
+
DR

Det kan videre vises at n


ar R s
a vil integralene langs 2 og 4 g
a mot null.
bruke dette kombinert med likning (3.75) f
Ved a
ar en
Z
Z
i1

=
f (z) dz +
f (z) dz
2
1
3
Integralet langs 1 kan parametrises ved z = 1/2(1 + i)R med R < t < R
Z

f (z) dz =
R

ei(z1/2)
dz = (1 + i)
1 e2iz

e2t
dt
1 + e2i(1+i)t

og tilsvarende kan 3 parametrises ved z = 1/2(1 + i)R for R < t < R slik at
Z

f (z) dz =
3

ei(z1/2)
dz = (1 + i)
1 e2iz

legge sammen disse integralene og la R f


Ved a
ar fra at
Z

(i + 1)


2
e2t 1 + e2i(1+i)t
1+i
dt =
2i(1+i)t
1+e
2

som medfrer at
Z

2
1
e2t dt =
2

som er ekvivalent med likning (3.74).

e2i(1+i)t e2t
dt
1 + e2i(1+i)t

192

III

9 Kompleks Integrasjon

Poisson Kernel
En svrt viktig funksjon innen kompleks analyse er poisson kernel, ogs
a noen
ganger kjent som poisson kjernen og er en integral transformasjon p
a lik linje
med laplace eller fourier-transformasjonene.
Lemma 3.9.3. Poisson kernel funksjonen er definert
Pr () =

X
n=

|n| in

1 r2
=
= Re
1 2r cos + r2

1 + rei
1 rei


, r [0, 1)

Andre likhet flger fra substitusjonen og at poisson kernel funksjonen Pr () har en periode p
a 2.
Definisjon 3.9.1. Poisson kernel transformasjonen er definert som
Z 2
Z 2
d
d
i ) =
h(re
h(ei )Pr ( )
h(ei( )Pr ()
=
2
2
0
0
Dette er en transformasjon som tar inn en funksjon og dersom h er integrerbar p
a enhetssirkelen s
a er u harmonisk p
a omr
adet D = {z : |z| < 1}.
Vi skal her hovedsaklig studere ulike m
ater a bestemme poisson kernel transformasjonen av h = 1 p
a, alts
a en konstant funksjon.
Proposisjon 3.9.3. La R > r > 0 da er
Z 2
R2 r 2
1
d = 1
PR,r =
2
2 0 R 2Rr cos( ) + r2
Bevis. 1) Legg merke til at integralet ikke er akkuratt det samme som poisson
transformasjonen. Det frste vi gjr er a enten a bruke substitusjonen
se at
eller a

slik at integralet er uavhengig av . Videre la oss dele p


a R og innfre konstanten
= r/R
Z 2
1
R2 r 2
PR,r =
d
2
2 0 R 2Rr cos + r2
forenkle integranden via kompleks analyse. Da er
fremmgangsm
aten n
a blir a
1 (r/R)2
1 2
=
12 2(r/R) cos + (r/R)2
(1 ei )(1 ei )
1
ei
=
+
1 ei
1 ei

X
=
|n| ein
n=

=1+2

X
n=1

n cos(n)

III

9.1 Typer integraler

193

tanken er n
a at n
ar vi integrerer over 0 2 s
a vi alle integralene g
a mot
null. Siden vi har fra at eksempel (2.10.1) at
Z
cos nx dx = 0 , n Z/{0}
0

dermed s
a blir integralet
2

R2 r 2
d
2Rr cos + r2
0
Z 2

X
1
=
1+2
n cos(n) d
2 0
n=1
Z 2

X
=1+2
cos(n) d = 1
n

PR,r =

1
2

R2

n=1

som var det som skulle vises.


2) La oss og lse integralet p
a en litt annen m
ate, vi kan gjre flgende omskrivning
PR,r =

1
2

Z
0

R2 r 2
R2 r 2
d = 2
2
2
R 2Rr cos + r
R + r2

Z
0

dx
1
2 1 a cos x

Hvor x = og a = 2rR/(r2 + R2 ). Siste integralet har vi lst i Del II, men la oss
n
a lse det via kompleks integrasjon. Da er
I
1
dx
1
dz/zi
1
1
=
=
2 1 a cos x
2 1 a(z + z 1 )/2
i z=1 a + 2z + az 2
beregne integralet. Nullpunktene er
Vi kan n
a bruke
residue-theoremet til a
z1,2 = (1 1 a2 )/a, og det er bare z1 som ligger i enhetssirkelen, s
a
!

Z 2
1
dx
1
1 + 1 a2
=
2i Res
2 0 1 a cos x
i
a
= 2 lim

zz1

1
1
1
=
=
2 + 2az
1 + az1
1 a2

sette inn f
Ved a
ar en n
a
PR,r =

R2 r 2 1
R2 + r2 2

Z
0

dx
R2 r 2
1

= 2
=1
2
1 a cos x
R +r
1 a2

som fr. Det siste steget er bare en rekke ufine algebraforenklinger hvor en kun
bruker at a = 2rR/(R2 + r2 ), men disse stegene kan leser kan f
a kose seg med
alene.
Logaritmer
Nkkel konturer

194

III

10 Oppgavesammling III

3.10

O P P G AV E S A M M L I N G I I I

3.10.1
Z

1/2

1.
0

INTEGRAL

2.

1 e1/x dx

3.10.2

log(1/x)
dx
5.
(1
+ x2 )2
0


Z
1
dx
4.
LambertW
x2
0

log(x) log(1 x) 3.
dx
x(1 x)

Z
0

log(1 x)
dx
1+x

O P P G AV E R

1. Beregn Dirichlet integralet


Z

sin

d =

ved hjelp av dobbeltintegralet


ZZ
I=
est sin t d(s, t)
D

hvor D = [0, ) [0, ).


2. Bestem flgende sum
S=

X
n=1

1
(v u)2n+1

(x u)n (v x)n dx

hvor v > u.
3. (Tom Apostol - 10.23) Definer
Z
F (y) :=
0

sin xy
dx
x (x2 + 1)

y > 0.

a)

Vis at F (y) tilfredstiller flgende differensiallikning

F 00 (y) F (y) =
,
2
vise at
og bruk dette til a

1
F (y) = 1 ey .
2
b)

bestemme flgende tre integral


Bruk F (y) til a
Z
Z
Z
sin xy
cos xy
x sin xy
,
,
og
.
2
2
2
2
x (x + a )
x +a
x2 + a2
0
0
0

4. Vi skal i denne oppgaven studere flgende integral


Z 1
xn
In =
dx
0 x+1
hvor n N.

III

10.2 Oppgaver

195

a) Vis at lim In = 0
n

b) Bestem In+1 + In
c)

Bestem flgende sum

k=0

(1)k
k+1

5. Beregn integralet


arctan

sin x
1 cos x

dx
sin x

der , betegner den velkjente EulerMascheroni konstanten. Vis at integralet kan skrives som

log(2 + 3 )
2

benytte seg av tilnrmingen 1/ 3 .


ved a
6. Denne oppgaven handler om gamma og beta-funksjonene.
a) Vis at
1

Z
I=

(1 + x)2m1 (1 x)2n1
dx = 2m+n2 B(m, n)
(1 + x2 )m+n

hvor m, n > 0.
vise at
b) Bruk forrige oppgave til a
Z

/4

J=
/4

 cos x + sin x cos


cos x sin x

dx =

2 sin cos2

.

n
ar ikke er en multipel av ( 6= k, k N).
7. I del II viste vi at for alle n N
/2

sin(2n + 1)x

=
,
sin x
2

vise at
se proposisjon (2.10.1). Bruk dette til a
(2n+1)/2

Z
In =
0

sin x
dx
x

g
ar mot /2 n
ar n .
Z
8. Bestem flgende integral
0

xa xb
dx hvor a, b R
(1 + xa ) (1 + xb )

(3.76)

196

10 Oppgavesammling III

9. I denne oppgaven skal vi studere flgende to integral


Z
Z tx
sin(x t)
e
dx ,
I1 (t) =
dx og I2 (t) =
x
1 + x2
t
0

III

(3.77)

Vis at begge integralene tilfreldstiller differensiallikningen


y 00 + y =

1
,
t

t > 0.

vise at I
La I(t) = I1 (t) I2 (t), og bruk differensiallikningen ovenfor til a
m
a vre p
a formen
I(t) = A sin(t + B)
Bestem konstantene A og B og vis endelig at
Z
Z
sin x
dx

dx =
=
.
2
x
1+x
2
0
0

10. Beregn flgende integral


Z

/2

/2

log(1 + b sin x)
dx
sin x

hvor |b| 1.
11. Tidligere har en studert flgende integral
Z
1
In (a, b) :=
dx
2 x + b sin2 x)n
(a
cos
0
For n = 1 og n = 2. Anta n > 1 og vis at In tilfredstiller



+
In1 = In1 = (1 n)In

hvor er summen av de partiellderiverte. Bestem og et lukket uttrykk


for I
n ved hjelp av partiellderiverte. Det kan fritt benyttes at I1 (a, b) =
/(2 ab ).
12. (Putnam 2008B2) Merk, oppgaven er ikke helt den samme. La F0 (x) =
log x. For n 0, defineres
Z x
Fn+1 =
Fn (t) dt
0

Bestem flgende sum



lim log n n! Fn (1)

III

10.2 Oppgaver

197

13. La A vre definert som flger


Z x
e
e(1)x
A=
dx
x
0
Hvor og er henholdsvis minste og strste lsning av
x2 + x = 1
Bestem uttrykk for
sinh(A) og

cosh(A) ,

og skriv de s
a enkelt som mulig. Hint: + = , = 5 .
14. (Putnam 2005A5) Det integralet som kanskje har hstet mest oppmerksomhet gjennom Putnams historie er flgende er flgende
Z 1
log(1 + x)
I=
dx
1 + x2
0
bestemme integralet
i denne oppgaven ser vi nrmere p
a ulike m
ater a
p
a.
a) Ls integralet ved hjelp av substitusjonen x = tan .
b) Innfr en parameter i teller og bestem integralet.
bec) Benytt den noe uvanlige substitusjonen (1 + x)(1 + y) = 2, til a
stemme integralet.
15. I denne oppgaven skal vi se hvordan Euler kan ha kommet frem til antakelsen om at betafunksjonen var relatert til gammafunksjonen. Se p
a
flgende integral
Z 1
1

I=
dx =
2
2
1

x
0
vise hvorfor det kan vre rimelig a
anta at betafunkBruk integralet til a
sjonen kan skrives som
Z 1
(x)(y)
B(x, y) =
tx1 (1 t)y1 dt =
.
(x + y)
0

komme frem til a


komme frem an16. I forrige oppgave viste vi en m
ate a
takelsen om sammenhengen mellom betafunksjonen og gammafunksjonen. Denne sammenhengen ble vist i theorem (3.3.6) ved hjelp av Bohrmullerup theoremet. Siden den gang har vi (forh
apentligvis) lrt mange
bevise sammenhengen og vi studeflere teknikker. Disse kan benyttes til a
rer nrmere to av disse.

198

III

10 Oppgavesammling III

bevise theoa) Skriv (x)(y) som et dobbeltintegral og bruk det til a


rem (3.3.6).
b) Bruk konvolusjonstheoremet for laplace-transformasjoner med f (t) =
bevise theorem (3.3.6).
tx og g(t) = ty til a
17. Flgende likhet har tidligere blitt vist
Z 1
Z

log (x + t) dx = log 2 +
0

x log tx dx

t>0

og kan benyttes fritt i denne oppgaven. Bestem flgende integral


ZZ
B(x, y) d(x, y)
S

hvor S = [0, 1] [0, 1] er enhetsfirkanten (unit square).


18. I denne oppgaven skal en svak generalisering av Ahmeds integral

Z 1
arctan x2 + 2
dx
5 2

=
2
2
x +1
96
x +2
0
vise at
studeres. Bruk at 1/q + 1/p = (p + q)/pq til a
ZZ
ZZ
dx dy
dx dy
+
2
2
2
2
2
2
2
2
2
2
S (a + x )(2a + x + y )
S (a + y )(2a + x + y )
Z 1
Z 1
dx
dy
=
2 + y2
2 + y2
a
a
0
0
Der a er en reell konstant, og S er enhetskuben S = [0, 1] [0, 1]. Integralene p
a hyre side er like. Vis da at

Z 1Z 1
Z 1

2
2
1
dy
a2
2 arctan 2a + x

dx
=
dx
2
2
2
2
2
2
2
2a2 + x2
0
0 a + x 2a + x + y
0 a +x
endelig vise at
og bruk dette til a


2
Z 1
2a2
arctan 2a2 + x2
1
1

dx
=

arctan

arctan
.
2
2
a
2a2 + x2
2a2 + 1
0 a +x
Hvor a som fr er en reell konstant.
19. Sinus funksjonen er definert som flger
Z z
Z
Si(z) :=
sinc(x) dx =
0

sin x
dx ,
x

og vi ser at limz Si(z) = /2 gir oss Dirichlet integralet som har blitt
studert fr. I denne oppgaven skal vi studere en rekke relaterte integral.
a)

Vis at
Z

sinc(x) dx .

sinc (x) dx =
0

III

10.2 Oppgaver

199

a) Uttrykk integralene

Z
0

sin4 x
dx
x2

og
0

sin4 x
dx
x4

ved hjelp av Dirichlet integralet.


20. I denne og den neste oppgaven studerer vi nrmere Wallis integralet
Z

/2

Wn =

sin (x) dx =
0

2p

22p+1 p

2p
2p+1
2
p+1
n

n
ar n = 2p
,
n
ar n = 2p + 1

(3.78)

som vi har sett tidligere.

R /2
a) Vis frst at Wn = 0 cosn dx, og deretter at hyre og venstre side av
likheten tilfredstiller differensiallikningen
nWn = (n 1)Wn2
for alle n 2.

b) Vis flgen er synkende og at for store n s


a er Wn+1 Wn . Bruk dette
vise at for store n s
til a
a er
r

Wn
,
2n
betrakte flgen yn = (n + 1)Wn Wn+1 .
hvor det kan vre lurt a

c) Avslutningsvis viser vi noe av nytteverdien til Wallis integralet og beregningene vi har utfrt. Fra tidligere har vi vist tilnrmet at
 n n
n! C n
,
e

der C R+

(3.79)

ogs
a kjent som stirrlings formel. Tidligere har vi antatt at C = 1, men vi
kan finne en enda bedre tilnrmelse for denne konstanten. Bruk tilnrmelsen for W2n fra deloppgave b) og det eksakte uttrykket fra likning (3.78)
bestemme C. Du kan fritt benytte deg av at likning (3.79) holder for
til a
store n.
21. I denne oppgaven skal vi se nrmere p
a det gaussiske integralet. La f
vre definert som
2
f (x, t) = xt ex /

200

10 Oppgavesammling III

III

a) Vis at f (x, 0) er en sannsynlighetsfordeling. Alts


a at integralet over R
er lik 1.
For en kontinuerlig fordelig er forventingsverdien til en sannsynlighetstetthet er gitt som
Z
E(x) = hxi =
xf (x) dx

og beskriver det mest sannsynlige utfallet til funksjonen. Variansen er hvor


nrt sentrert funksjonen er omkring funksjonen gitt som
Z
2
Var(x) =
x E(x) f (x) dx

b)

Bestem forventingsverdien og variansen til f (x, 0).

Videre I denne oppgaven blir det sett nrmere p


a den moment-genererende
funksjonen til f . Den er gitt som
Z
Mf (p) =
exp f (x)dx

c)

Vis at den momentgenererende funksjonen til f (x, 0) kan skrives som


Mf (p) = ep

/4

Og vis videre at
lim Mf (p)

t0

d)

0

= E(x) og

00
lim Mf (p) = Var(x)

t0

Vis at integralet
Z

I(t) :=

f (x, t) dx

(3.80)

tilfredstiller funksjonallikningen
I(t) =

t1
I(t 1)
2

dersom t er et partall. Hva skjer dersom t er odde? Bestem integralet


I(t) og finn slik at I(t) blir en sannsynlighetsfordeling. Eksisterer det
vre en sannsynlighetsfordeling?
t-verdier slik at det er umulig for I a

IV

gi leser en pustepause og for a


la
Denne siden er med hensikt blank, for a
forfatter sl
ass mot dinosaurer.

203

Tillegg A

A.1

KONVERGENS

N
ar vi studerer integraler er sprsm
alet om integralet har en verdi, minst like
viktig som selve svaret. Hva er konvergens og hva menes med at integralet har
en verdi?
Definisjon A.1.1. Et uegentlig integral er et integral som skrives p
a formen
Z b
Z b
lim
f (x) dx ,
lim
f (x) dx ,
b

eller p
a formen
Z

Z
f (x) dx ,

lim

f (x) dx ,

lim

Hvor grenseverdiene f (x) enten er udefinert, eller g


ar mot . En sum av
uegentlige integral, er ogs
a uegentlig.
Definisjon A.1.2. Et integral kalles konvergent, dersom det kan skrives som en
endelig verdi. Alle integral som ikke er konvergent, kalles divergente.
Eksempel A.1.1. For motivasjon studeres flgende integral
Z 2
dx
x
1
0
ignorere diskusjonen omkring integralet konverger eller ikke1 og
vi velger a
regner blindt ut integralet.
Z 2
h
i2
dx
= log |x 1| = log |1| + log |1| = 0
0
0 x1
Dette virker noe rart om en studerer grafen til funksjonen vist i figur 8. Da
funksjonen klart bl
aser opp nre x = 1.
S
a funksjonen ovenfor konvergerer, men vi vil gjerne skille mellom ulike typer konvergens. Vi har for eksempel Rieman-integrerbare funksjoner, LebesgueIntegrerbare funksjoner osv. Funksjonen ovenfor konvergerer om en betrakter
prinsipial verdien av integralet, men divergerer ellers. Om vi sier at en funksjon
er integrerbar, vil betydningen alltid vre at funksjonen er Rieman-integrerbar.
1 Siden

vi ikke engang har diskutert hva konvergens er engang!

204

1 Konvergens

Definisjon A.1.3. Et uegentlig integral konvergerer hvis og bare hvis integralet


kan skrives som en sum av uegentlige integral, som alle konvergerer.
Ved a bruke denne definisjonen av konvergens p
a integralet i eksempel (C.1.1)
s
a m
a b
ade
Z 1
Z 2
dx
dx
og
x

1
x
1
0
1
bryte opp integralet i interval (a, b) hvor kun
konvergere. Teknikken er alts
aa
en av grenseverdiene limxa f (x) eller limxb f (x) ikke eksisterer. I tilleg til at
f (x) er vre begrenset p
a [a, b].
Hvorfor vi velger a definere konvergens p
a denne m
aten gir mer mening n
ar
vi studerer begrepet uendelig nrmere. Uendelig er ikke noe tall, men noe vi
reiser mot og aldri helt n
ar. For a slippe hodeverk br gjerne hastigheten p
a hvor
si, siden vi uansett aldri kommer helt frem.
raskt vi reiser ikke ha noe a
Eksempel A.1.2. La oss betrakte om flgende integral
Z
2x dx
2
1 + x
konvergerer eller divergerer. Fra diskusjonen ovenfor er m
ate a studere hvordan
integralet oppfrer seg n
ar vi g
ar mot uendelig i ulikt tempo.
Z a
h

ia
2x dx
lim
= lim log 1 + x2
= 0.
2
a a 1 + x
a
a
Derimot om vi lar funksjonen vokse dobbelt s
a raskt mot uendelig f
as
Z 2a
h

i2a
2x dx
= lim log 1 + x2
lim
= 2 log 2 .
2
a
a a 1 + x
a
vi kan dermed konkludere med at integralet er udefinert da det er avhengig av
hastigheten
Dette problemet med hastigheter lses dersom en deler opp integralet i uegentlige integral. Slik at hvert integral bare har en singularitet.
P-integral
Z
dx
Proposisjon A.1.1. Integralet
divergerer for alle relle eller komplekse p
xp
0
mens
Z 1
Z
dx
dx
,
p
<
1
og
, p>1
p
x
xp
0
1
konvergerer.
Bevis. Dersom p < 1 s
a er

1
Z 1

dx
1
1
1
p1
=
x
=
lim 1 c1p =
p
1p
1 p x0
1p
0 x
0

1 Konvergens

og likeledes s
a er
Z
1

dx
= lim
c
xp

205


dx
1
=
lim 1 c1p =
p
c
x
1p

Deretter studeres tilfellet n


ar p > 1 s
a


1

Z 1
dx
1
1
1
p1
= lim
x
lim 1 p1 =
=
p
c0 1 p
1 p c0
c
0 x
c
og likeledes s
a er
Z c
Z

dx
1
1
dx
=
lim
=
lim 1 c1p =
p
p
c
c
x
x
1

p
1

p
1
1
Siden (0, 1) konvergerer for p < 1 og (1, ) konvergerer for p > 1, s
a kan ikke
(0, ) = (0, 1) (1, ) konvergere n
ar p < 1 eller p > 1. Da gjennst
ar det bare
teste tilfellet hvor p = 1.
a
Z
Z 1
Z
dx
dx
dx
=
+
= lim ( log x) + lim log x =
x
x0
x
x
x
0
0
1
som begge divergerer. Dette fullfrer beviset.
Dette resultatet er svrt nyttig n
ar vi bruker det i kombinasjon med sammenliknings testen.
Proposisjon A.1.2. (Sammenliknings Testen) La f (x) og g(x) vre to funksjoner
definert p
a (a, b) slik at
0 f (x) g(x)
er sant for alle x [a, b]. Dersom
Z

g(x) dx
a

konvergerer s
a konvergerer integralet over f (x). Dersom integralet over g(x) divergerer s
a divergerer integralet over f (x).
Eksempel A.1.3. La oss se om flgende integral konvergerer
Z
(sin x)2
dx
x2
0
dersom x 1 s
a har vi at
(sin x)2
1
2
x2
x
dermed har vi fra p-testen at integralet ovenfor konvergerer siden p = 2 > 1.
Dersom 0 x < 1, kan vi stedet bruke flgende sammenlikning
(sin x)2
1

x2
x

206

1 Konvergens

gange med x2 , p
ved a
a begge sider s
a har en at (sin x)2 x3/2 . Dette stemmer
3/2
siden sin x x x , for x 0 som en for eksempel kan se fra taylorrekken til
sin x. Dermed konvergerer integralet fra sammenlikningstesten og vi har
Z
Z 1
Z
1
(sin x)2
1
+
0
dx
=3
2
x
x2
x
0
0
0
som er en fin tilnrming.
se p
En kunne fint ha f
att enda strammere grenser ved a
a taylorrekken til
(sin x)2 /x2 , men det er helt uviktig. Legg merke til at metoden ovenfor ikke kan
vise at (sin x)/x konvergerer, selv om integralene har samme verdi.
brukes for a
Proposisjon A.1.3. (Grensesammenliknings Testen)
La f og g enten vre to strengt positive eller strengt negative funksjoner p
a [a, b]
og la funksjonene vre udefinert i punktet a. Dersom g(x) f (x) n
ar x a s
a
konvergerer
Z b
Z b
f (x) dx hvis og bare hvis
g(x) dx konvergerer.
a

Hvor g(x) f (x) n


ar x c betyr at limxc f (x)/g(x) = 1. Begge testene
ovenfor krever at funksjonene f og g ikke skifter fortegn p
a intervalet. S
a det er
logisk a lure p
a hvordan en skal behandle integraler som ikke er strengt positive
eller negative. En m
ate a tvinge en funksjon til a vre positiv p
a er a ta absoluttverdien. Det er klart at b
ade f (x) og |f (x)| har samme singulariter s
a et naturlig
Rb
sprsm
al er hvilken konklusjon vi kan trekke om a f (x) , dersom vi vet noe om
Rb
integralet a |f (x)| dx konvergerer, eller divergerer. Et delvis svar p
a sprsm
alet
er gitt i flgende proposisjon
Proposisjon A.1.4. La f (x) vre en funksjon dersom integralet
Z b
|f (x)| dx
a

konvergerer, s
a konvergerer ogs
a integralet
Z b
f (x) dx .
a

Rb
Merk at det kontrapositive ikke stemmer! Dersom integralet a |f (x)| dx
Rb
divergerer betyr dette ikke at a f (x) dx divergerer. Et eksempel p
a dette er vist
under
Dirichlet integralet
Lemma A.1.1. La f vre definert som flger2

sin x
dersom
f (x) =
x

0
ellers
2 Det

x 6= 0 ,
R

er i utgangspunktet ingen grunn til a definere f slik, da integralet


(sin x)/x dx er Riemann0
definere f slik at funksjonen blir kontinuerlig, deriverbar.
integrerbart. Derimot er fordelen ved a

1 Konvergens
Z

207

da konvergerer integralet

f (x) dx, men ikke absolutt.

Bevis. Frst kan en legge merke til at integralet er symmetrisk omkring origo
betrakte (0, ).
f (x) = f (x), slik at det holder a
vise at funksjonen ikke konvergerer absoFrste del av bevisetRvil g
a ut p
aa

lutt, og deretter vise at 0 f (x) dx, konvergerer. Vi betrakter frst integralet av


funksjonen over et endelig interval
Z N


f (x) dx ,
0

La n
a N N, da er


Z 2N
N
1 Z 2(n+1)
X
sin x
sin x


dx =

x dx
x
0
n=0 2n
Z 2(n+1)
N
1
X
1

|sin x| dx
2(n + 1) 2n
n=0
Z 2
N
1
X
1
=
|sin x| dx
2(n + 1) 0
n=0
=

N 1
1
2 X
n=0 n + 1

som klart divergerer n


ar N . Dette viser at integralet ikke konvergerer
bruke testene ovenfor i kombinasjon med p-integralene er det som
absolutt. A
vise de aller fleste integralene konvergerer.
vil bli brukt for a

Men dette integralet krever litt mer triksing. Trikset i denne oppgaven blir a
atter en gang vende tilbake til delvis integrasjon. Vi deler igjen integralet ved
x=1
Z N
Z N
h cos x iN Z N cos x
sin x
cos N
cos x
dx =
dx
=
cos(1)

2
x
x
x
N
x2
1
1
1
1
vise at siste integral konvergerer kan en
som konvergerer n
ar N . For a
bruke p-testen med x2 siden cos x/x2 1/x2 n
ar x 1. Tilfellet N
ar x 0 s
a er
sin x x s
a (sin x)/x 1. S
a
Z 1
Z 1
sin x
dx
1 dx
x
0
0
dette flger fra grensesammenlikningstesten og medfrer konvergerens. Dette
medfrer ogs
a at integralet konvergerer.
gjre. Frst s
At integralet ovenfor konvergerer har med flere ting a
a bl
aser
ikke integralet opp nre origo da (sin x)/x 1 for |x| < 1. Faktisk s
a er
(sin x)/x 1 x R, siden 1/x drar funksjonen mot null.
Funksjonen sin x bidrar til at funksjonen oscillerer over og under x-aksen, og
det er kanseleringen av disse omr
adene som bidrar til at funksjonen konvergerer.
N
ar en ser p
a absoluttverdien av funksjonen f
ar en derimot ikke disse kanseleringene. I alle eksemplene tidligere har vi sett p
a funksjoner som g
ar mot 0 n
ar

208

1 Konvergens

x . At dette alltid er tilfellet for konvergente integral er feil. Det eksisterer


alts
a funksjoner
ar mot en verdi ulik
R hvor limx f (x) ikke eksisterer eller g
null men k f (x) dx konvergerer.
Fresnell integralene
Lemma A.1.2.
x


sin t2 dt , C(x) =

Integralene S(x) =
0


cos t2 dt konvergerer x R.

Lemma A.1.3.
Z


sin x2 dx og

Integralene


cos x2 konvergerer.

R
forst
A
a hvorfor R sin x2 dx konvergerer og ikke R sin x dx er ikke s
a veldig
vanskelig. For sin(x2 ) har funksjonen et en topp over x-aksen, ogs
a en bunn
under x-aksen. Arealet av disse toppene og bunnene blir mindre og mindre n
ar
x . Dette er ikke tilfellet for sin x som oscillerere med samme hastighet
hele tiden. Fra den alternerende rekketesten konvergerer alts
a integralet over
sin(x2 ), men ikke sin x.
bevise lemma (C.1.3) siden dette ogs
Vi kommer kun til a
a beviser lem vise at S(x) og C(x) konvergerer siden
ma (C.1.2). Selv om det er rett frem a
en integrerer en begrenset funksjon over et endelig interval.
R

betrakte intea holder det a


Bevis. Funksjonene er symmetrisk omkring origo, s
gralene over (0, ). Her vil vi bare vise at integralet over sin(x2 ) konvergerer,
men et nyaktig likt bevis kan fres for cos(x2 ). Alternativt kan en vise frst at
integralene er like store, slik at det holder a vise at ett av integralene konvergerer.


sin x2 dx =


sin x2 dx +


sin x2 dx

Funksjonen er integrerbar p
a intervalet [0, 1], den bl
aser ikke opp og har ingen
singulariterer. Vi m
a alts
a vise at integralet
Z
lim


sin x2 dx ,

bruk substitusjonen t = x2 , s
konvergerer. Ved a
a x = t og 2 t dx = dt f
ar en
Z

sin x2

2

Z
dx =

N2

sin t
cos N 2
cos 1
1
dt =
+

2
4
2 t
2 N

Z
1

N2

cos t
dt
t3/2

sammenlikne med p-integralene ser vi at


siste integralet kjenner vi igjen. Ved a
integraler konvergerer, siden 3/2 > 1 og cos t/t3/2 1/t3/2 .
-funksjonen
Z
Lemma A.1.4.
0

tx1 et dt konvergerer for alle x (0, )

1 Konvergens

Bevis. Frst s
a deler vi integralet i
Z
Z
x1 t
t
e dt =

1
x1 t

tx1 et dt

dt +
1

209

og det neste steget er a vise at de uegentlige integralene p


a hyre side konverge finne en funksjon f (t) slik at 0 tx1 et f (t),
rer. Fremmgangsm
aten blir a
og vise at integralet over f (t) er konvergerer.
For det frste integralet s
a er et 1 for t 0, slik at

 x 0

Z 1
Z 1
cx
t
1
x1 t
x1

.
0
t
e dt lim
t
dt = lim
= lim
c0 c
c0
x c c0 x
x
0
Dersom x > 0, s
a vil ax 0 n
ar a 0+ , s
a integralet konvergerer mot 1/x.
R 1 x1 t
Dette viser at integralet over 0 t
e dt konvergerer.
For det neste integralet vil vi bruke at limt tr et/2 = 0 for alle r R.
Alts
a for x > 0 eksisterer det en kx R slik at 0 tx et/2 1 for t kx . Vi
holder x konstant og deler integralet ved kx
Z
Z kx
Z
x1 t
x1 t
t
e dt =
t
e dt +
tx1 et dt .
1

kx

Det frste integralet p


a hyre side er over et endelig interval og har ingen singulariter p
a intervalet, alts
a konvergerer integralet. For det neste har vi at t kx
slik at tx1 et = (tx1 et/2 )et/2 et/2 , s
a
Z
Z


tx1 et dt
et/2 dt = lim 2ekx /2 ec/2 = 2ekx /2 .
kx

kx

Merk at uansett hva kx er s


aR konvergerer integralet. Dette viser at det andre

integralet er konvergent, s
a 1 tx1 et dt konvergerer. Dette fullfrer beviset.

Betaintegralet
Z
Lemma A.1.5.

tx1 (1 t)y1 dt konvergerer for alle x, y (0, )

Bevis. Slik som beviset for konvergens av gammaintegralet deles integralet opp
Z 1
Z 1/2
Z 1
x1
y1
x1
y1
t
(1 t)
dt =
t
(1 t)
dt +
tx1 (1 t)y1 dt .
0

1/2
x1

y1

x1

For det frste integralet merk at t


(1 t)
t
for 0 t 1/2, s
a


Z 1/2
Z 1/2
(1/x)x cx
0
tx1 (1 t)y1 dt
tx1 dt = lim
c0
x
0
0
N
a, dersom x > 0 s
a vil ax 0 n
ar a 0+ , s
a det frste integralet konvergerer
dersom x > 0. Tilsvarende for det neste integralet s
a er tx1 (1 t)y1
y1
(1 t)
for 1/2 t 1 s
a


Z 1/2
Z 1/2
(1/2)y (1 c)y
0
(1 t)y1 dt
(1 t)y1 dt = lim
x
c1
0
0

210

1 Konvergens

N
a, dersom y > 0 s
a vil (1 a)y 0 n
ar a 1 , s
a det frste integralet
R 1 x1
konvergerer dersom y > 0. Dermed s
a konvergerer 0 t
(1 t)y1 dt for alle
x, y > 0 som var det som skulle vises.

A.1.1

KONVERGENSRADIUS

A.1.2

P R I N S I PA L

VERDI

Cauchys prinsippielle verdi (Cauchy principal value) er en m


ate a gi en verdi
til integralet som i utganspunktet divergerer.

II

211

Tillegg B

B.1

A N A LY S E N S F U N D A M E N TA L T H E O R E M .

Fr vi kan vise analysens fundamentaltheorem., trengs det frst to proposisjoner.


Proposisjon B.1.1. (mellomverdi setningen)
La I = [a, b] vre et a
pent endelig interval interval, hvor a, b R. La f vre en
reell funksjon som er kontinuerlig p
a (a, b). Dersom k R er tall slik at
f (a) < k < f (b) eller

f (a) > k > f (b)

da eksisterer det en konstant c (a, b) slik at f (c) = k.


At dette stemmer er ikke vanskelig a forst
a. Theoremet sier at dersom vi har
en kontinuerlig funksjon som har verdiene f (a) og f (b) i hvert endepunkt av
intervaet, s
a tar funksjonen alle verdier mellom f (a) og f (b) i ett eller annet
punkt i intervalet. For et bevis se for eksempel [? , thm 4.23].
Proposisjon B.1.2. (mellomverdi setningen for integraler) La f vre en reell
kontinuerlig funksjon p
a det lukkede intervalet [a, b], a, b R. Da eksisterer det et
reellt tall k [a, b] slik at
Z

f (x) dx = f (k)(b a)

(B.1)

Bevis. Siden funksjonen er kontinuerlig p


a et lukket interval, s
a er den ogs
a
Rieman-integrerbar p
a [a, b]. Det flger ogs
a fra at f er kontinuerlig p
a et lukket
interval at funksjonen har et minimum og et maksimum
f (m) = max f (x)
x[a,b]

f (M ) = max f (x)
x[a,b]

Vi kan n
a definere flgende ulikhet
Z

Z
f (m) dx

Z
f (x) dx

f (M ) dx
a

212

II

1 analysens fundamentaltheorem.

Siden f (m) og f (M ) er konstaner kan en skrive


Z b
1
f (x) dx f (M )
f (m)
ba a
Fra mellomverdi setningen proposisjon (B.1.1) har vi n
a at det eksisterer en
k (a, b) slik at
Z b
1
f (x) dx = f (k)
ba a
gange likningen med b a fullfrer beviset.
A
Theorem B.1.1. (Analysens fundamentaltheorem. - Del I)
La f vre en kontinuerlig funksjon p
a [a, b]. Funksjonen g definert som
Z x
f (t) dt
g(x) =
a

er da kontinuerlig p
a [a, b], deriverbar p
a (a, b) og g 0 (x) = f (x) for alle x (a, b).
Her betegner integralet som vanlig arealet under funksjonen f (x) fra a til x.
Bevis. Siden f er kontinuerlig og vre begrenset s
a er ogs
a F kontinuerlig og
vre begrenset. La oss ta den deriverte av F , da har vi fra definisjon
g(x + h) g(x)
d
g(x) = lim
h0
dx
h
Vi kan skrive om teller som flger
Z x+h
Z
g(x + h) g(x) =
f (t) dt
a

x+h

(B.2)

x+h

f (t) dt =

f (t) dt
x

Siden vi har at
Z

x+h

f (t) dt +
a

x+h

f (t) dt =
x

f (t) dt
x

Dette er intuitivt fra at vi legger sammen to omr


ader ved siden av hverandre.
Formelt kan dette ogs
a vises via rieman-summer. Likning (B.2) kan da skrives
som
Z
d
1 x+h
g(x) = lim
f (t) dt
h0 h x
dx
skrive om
Tanken er n
a at vi bruker mellomverdi setningen for integraler til a
integralet. Direkte bruk av likning (B.1) fra proposisjon (B.1.2) gir
Z
1 x+h
1
d
g(x) = lim
f (t) dt = lim f (k)[x (x + h)] = lim f (k)
h0
h0
h0
dx
h x
h
fullfre beviset m
For a
a vi bestemme k, for husk at k er avhengig av hva x er.
Fra definjsjonen s
a er x k x + h, slik at n
ar h 0 s
a vil k x. Oppsumert
s
a er alts
a
Z
d
1 x+h
g(x) = lim
f (t) dt = f (x)
(B.3)
h0 h x
dx
vise.
som var det vi nsket a

II

1 analysens fundamentaltheorem.

213

Fra dette theoremet kan vi vise flgende korollar


a [a, b] og la F vre en
Korollar B.1.1. La f vre en kontinuerlig funkjson p
kontinuerlig funksjon p
a [a, b] slik at F 0 (x) = f (x) for alle x (a, b) da er
b

f (t) dt = F (b) F (a)


a

Mekr at korollaret krever kontinuetet p


a hele intervalet og ikke bare stykkevis
vise det
kontinuitet. Vi viser frst et kort bevis for theoremet, for deretter a
sterkere resulatet ved hjelp av middelverdi setningen.
Bevis. Anta at F er en antiderivert av f , hvor f er kontinuerlig p
a [a, b]. Vi
definerer
Z x
G(x) =
f (t) dt
(B.4)
a

fra theorem (B.1.1), s


a vet vi at G ogs
a er en antiderivert av f . Dermed s
a m
a
F og G hyst avike med en eller annen konstant c. G(x) = F (x) + c for alle
la x = a s
x [a, b]. Ved a
a har vi
Z a
F (a) + c = G(a) =
f (t) dt = 0
a

som medfrer at c = F (a). Alts


a s
a er G(x) = F (x) F (a) eller med andre
ord
Z b
f (t) dt = F (b) F (a)
a

som nsket.
Proposisjon B.1.3. (Middelverdi setningen) La f : [a, b] R vre en kontinuerlig funksjon p
a det lukkede intervalet [a, b], og deriverbar p
a det a
pne intervalet
(a, b) hvor a < b. Da eksisterer det en c (a, b) slik at
f 0 (c) =

f (b) f (a)
ba

Igjen uttelates beviset som bygger p


a Rolles sats. Men det kan finnes i de
fleste innfringsbker til Kalkulus. Deriblant [? , thm. 5.10].
Theorem B.1.2. (Analysens fundamentaltheorem. - Del II)
La f og F ta reelle verdier p
a intervalet [a, b], hvor a, b R. Slik at den deriverte av
F er f . Alts
a at F 0 (x) = f (x) for alle x [a, b]. Dersom f er Rieman-integrerbar
p
a [a, b] s
a er
Z

f (x) dx = F (b) F (a)


a

Bevis. Dette er et bevis som baserer seg p


a Riemann summer. Siden f er stykkevis kontinuerlig p
a et endelig interval s
a er f Rieman-integrerbar. La F vre den

214

II

1 analysens fundamentaltheorem.

antideriverte til f p
a [a, b]. Vi tar utgangspunkt i F (b) F (a), og la x1 , . . . , xn
vre tall slik at
a = x0 < x1 < x2 < < xn1 < xn = b
Vi antar videre at f er kontinuerlig p
a (xi1 , xn1 ) hvor i g
ar fra 1 til n. Siden
vi har definert x0 = a og xn = b har vi
F (b) F (a) = F (xn ) F (x0 )
vi kan skrive om hyresiden som en teleskoperende rekke slik at
F (b) F (a) =

n
X

F (xi ) F (xi1 )

i=1

skrive ut hyresiden og sammenlikne. Fra middelhvor det overlates til leser a


verdi setningen (B.1.3) har vi n
a at
F (xi ) F (xi1 ) = F 0 (ci )(xi xi1 )
hvor xi1 < ci < xi . Dette er lov siden f er kontinuerlig p
a (xi1 , xn1 ). Ved
innsetning i har vi n
a
F (b) F (a) =

n
X

F 0 (ci )(xi xi1 ) =

i=1

n
X

f (ci )xi

i=1

siden F (x) = f (x) for alle x (a, b). Dette kan bli sett p
a som et Riemann la
sum hvor xi beskriver lengden av partisjonene og f (ci ) er hyden. Ved a
kxi k 0 f
as
F (b) F (a) =

lim

kxi k0

n
X
i=1

Z
f (ci )xi =

f (x) dx
a

som var det som skulle vises. Den siste summen er hvordan vi opprinnelig
definerte arealet av en funksjon. Alts
a som summen av rektangler hvor bredden
g
ar mot null.

III

215

Tillegg C

C.1

KONVERGENS

N
ar vi studerer integraler er sprsm
alet om integralet har en verdi, minst like
viktig som selve svaret. Hva er konvergens og hva menes med at integralet har
en verdi?
Definisjon C.1.1. Et uegentlig integral er et integral som skrives p
a formen
Z b
Z b
lim
f (x) dx ,
lim
f (x) dx ,
b

eller p
a formen
Z

Z
f (x) dx ,

lim

f (x) dx ,

lim

Hvor grenseverdiene f (x) enten er udefinert, eller g


ar mot . En sum av
uegentlige integral, er ogs
a uegentlig.
Definisjon C.1.2. Et integral kalles konvergent, dersom det kan skrives som en
endelig verdi. Alle integral som ikke er konvergent, kalles divergente.
Eksempel C.1.1. For motivasjon studeres flgende integral
Z 2
dx
x
1
0
ignorere diskusjonen omkring integralet konverger eller ikke1 og
vi velger a
regner blindt ut integralet.
Z 2
h
i2
dx
= log |x 1| = log |1| + log |1| = 0
0
0 x1
Dette virker noe rart om en studerer grafen til funksjonen vist i figur 8. Da
funksjonen klart bl
aser opp nre x = 1.
S
a funksjonen ovenfor konvergerer, men vi vil gjerne skille mellom ulike typer konvergens. Vi har for eksempel Rieman-integrerbare funksjoner, LebesgueIntegrerbare funksjoner osv. Funksjonen ovenfor konvergerer om en betrakter
prinsipial verdien av integralet, men divergerer ellers. Om vi sier at en funksjon
er integrerbar, vil betydningen alltid vre at funksjonen er Rieman-integrerbar.
1 Siden

vi ikke engang har diskutert hva konvergens er engang!

216

1 Konvergens

III

Definisjon C.1.3. Et uegentlig integral konvergerer hvis og bare hvis integralet


kan skrives som en sum av uegentlige integral, som alle konvergerer.
Ved a bruke denne definisjonen av konvergens p
a integralet i eksempel (C.1.1)
s
a m
a b
ade
Z 1
Z 2
dx
dx
og
x

1
x
1
0
1
bryte opp integralet i interval (a, b) hvor kun
konvergere. Teknikken er alts
aa
en av grenseverdiene limxa f (x) eller limxb f (x) ikke eksisterer. I tilleg til at
f (x) er vre begrenset p
a [a, b].
Hvorfor vi velger a definere konvergens p
a denne m
aten gir mer mening n
ar
vi studerer begrepet uendelig nrmere. Uendelig er ikke noe tall, men noe vi
reiser mot og aldri helt n
ar. For a slippe hodeverk br gjerne hastigheten p
a hvor
si, siden vi uansett aldri kommer helt frem.
raskt vi reiser ikke ha noe a
Eksempel C.1.2. La oss betrakte om flgende integral
Z
2x dx
2
1 + x
konvergerer eller divergerer. Fra diskusjonen ovenfor er m
ate a studere hvordan
integralet oppfrer seg n
ar vi g
ar mot uendelig i ulikt tempo.
Z a
h

ia
2x dx
lim
= lim log 1 + x2
= 0.
2
a a 1 + x
a
a
Derimot om vi lar funksjonen vokse dobbelt s
a raskt mot uendelig f
as
Z 2a
h

i2a
2x dx
= lim log 1 + x2
lim
= 2 log 2 .
2
a
a a 1 + x
a
vi kan dermed konkludere med at integralet er udefinert da det er avhengig av
hastigheten
Dette problemet med hastigheter lses dersom en deler opp integralet i uegentlige integral. Slik at hvert integral bare har en singularitet.
P-integral
Z
dx
Proposisjon C.1.1. Integralet
divergerer for alle relle eller komplekse p
xp
0
mens
Z 1
Z
dx
dx
,
p
<
1
og
, p>1
p
x
xp
0
1
konvergerer.
Bevis. Dersom p < 1 s
a er

1
Z 1

dx
1
1
1
p1
=
x
=
lim 1 c1p =
p
1p
1 p x0
1p
0 x
0

III

1 Konvergens

og likeledes s
a er
Z
1

dx
= lim
c
xp

217


dx
1
=
lim 1 c1p =
p
c
x
1p

Deretter studeres tilfellet n


ar p > 1 s
a


1

Z 1
dx
1
1
1
p1
= lim
x
lim 1 p1 =
=
p
c0 1 p
1 p c0
c
0 x
c
og likeledes s
a er
Z c
Z

dx
1
1
dx
=
lim
=
lim 1 c1p =
p
p
c
c
x
x
1

p
1

p
1
1
Siden (0, 1) konvergerer for p < 1 og (1, ) konvergerer for p > 1, s
a kan ikke
(0, ) = (0, 1) (1, ) konvergere n
ar p < 1 eller p > 1. Da gjennst
ar det bare
teste tilfellet hvor p = 1.
a
Z
Z 1
Z
dx
dx
dx
=
+
= lim ( log x) + lim log x =
x
x0
x
x
x
0
0
1
som begge divergerer. Dette fullfrer beviset.
Dette resultatet er svrt nyttig n
ar vi bruker det i kombinasjon med sammenliknings testen.
Proposisjon C.1.2. (Sammenliknings Testen) La f (x) og g(x) vre to funksjoner
definert p
a (a, b) slik at
0 f (x) g(x)
er sant for alle x [a, b]. Dersom
Z

g(x) dx
a

konvergerer s
a konvergerer integralet over f (x). Dersom integralet over g(x) divergerer s
a divergerer integralet over f (x).
Eksempel C.1.3. La oss se om flgende integral konvergerer
Z
(sin x)2
dx
x2
0
dersom x 1 s
a har vi at
(sin x)2
1
2
x2
x
dermed har vi fra p-testen at integralet ovenfor konvergerer siden p = 2 > 1.
Dersom 0 x < 1, kan vi stedet bruke flgende sammenlikning
(sin x)2
1

x2
x

218

III

1 Konvergens

gange med x2 , p
ved a
a begge sider s
a har en at (sin x)2 x3/2 . Dette stemmer
3/2
siden sin x x x , for x 0 som en for eksempel kan se fra taylorrekken til
sin x. Dermed konvergerer integralet fra sammenlikningstesten og vi har
Z
Z 1
Z
1
(sin x)2
1
+
0
dx
=3
2
x
x2
x
0
0
0
som er en fin tilnrming.
se p
En kunne fint ha f
att enda strammere grenser ved a
a taylorrekken til
(sin x)2 /x2 , men det er helt uviktig. Legg merke til at metoden ovenfor ikke kan
vise at (sin x)/x konvergerer, selv om integralene har samme verdi.
brukes for a
Proposisjon C.1.3. (Grensesammenliknings Testen)
La f og g enten vre to strengt positive eller strengt negative funksjoner p
a [a, b]
og la funksjonene vre udefinert i punktet a. Dersom g(x) f (x) n
ar x a s
a
konvergerer
Z b
Z b
f (x) dx hvis og bare hvis
g(x) dx konvergerer.
a

Hvor g(x) f (x) n


ar x c betyr at limxc f (x)/g(x) = 1. Begge testene
ovenfor krever at funksjonene f og g ikke skifter fortegn p
a intervalet. S
a det er
logisk a lure p
a hvordan en skal behandle integraler som ikke er strengt positive
eller negative. En m
ate a tvinge en funksjon til a vre positiv p
a er a ta absoluttverdien. Det er klart at b
ade f (x) og |f (x)| har samme singulariter s
a et naturlig
Rb
sprsm
al er hvilken konklusjon vi kan trekke om a f (x) , dersom vi vet noe om
Rb
integralet a |f (x)| dx konvergerer, eller divergerer. Et delvis svar p
a sprsm
alet
er gitt i flgende proposisjon
Proposisjon C.1.4. La f (x) vre en funksjon dersom integralet
Z b
|f (x)| dx
a

konvergerer, s
a konvergerer ogs
a integralet
Z b
f (x) dx .
a

Rb
Merk at det kontrapositive ikke stemmer! Dersom integralet a |f (x)| dx
Rb
divergerer betyr dette ikke at a f (x) dx divergerer. Et eksempel p
a dette er vist
under
Dirichlet integralet
Lemma C.1.1. La f vre definert som flger2

sin x
dersom
f (x) =
x

0
ellers
2 Det

x 6= 0 ,
R

er i utgangspunktet ingen grunn til a definere f slik, da integralet


(sin x)/x dx er Riemann0
definere f slik at funksjonen blir kontinuerlig, deriverbar.
integrerbart. Derimot er fordelen ved a

III

1 Konvergens
Z

219

da konvergerer integralet

f (x) dx, men ikke absolutt.

Bevis. Frst kan en legge merke til at integralet er symmetrisk omkring origo
betrakte (0, ).
f (x) = f (x), slik at det holder a
vise at funksjonen ikke konvergerer absoFrste del av bevisetRvil g
a ut p
aa

lutt, og deretter vise at 0 f (x) dx, konvergerer. Vi betrakter frst integralet av


funksjonen over et endelig interval
Z N


f (x) dx ,
0

La n
a N N, da er


Z 2N
N
1 Z 2(n+1)
X
sin x
sin x


dx =

x dx
x
0
n=0 2n
Z 2(n+1)
N
1
X
1

|sin x| dx
2(n + 1) 2n
n=0
Z 2
N
1
X
1
=
|sin x| dx
2(n + 1) 0
n=0
=

N 1
1
2 X
n=0 n + 1

som klart divergerer n


ar N . Dette viser at integralet ikke konvergerer
bruke testene ovenfor i kombinasjon med p-integralene er det som
absolutt. A
vise de aller fleste integralene konvergerer.
vil bli brukt for a

Men dette integralet krever litt mer triksing. Trikset i denne oppgaven blir a
atter en gang vende tilbake til delvis integrasjon. Vi deler igjen integralet ved
x=1
Z N
Z N
h cos x iN Z N cos x
sin x
cos N
cos x
dx =
dx
=
cos(1)

2
x
x
x
N
x2
1
1
1
1
vise at siste integral konvergerer kan en
som konvergerer n
ar N . For a
bruke p-testen med x2 siden cos x/x2 1/x2 n
ar x 1. Tilfellet N
ar x 0 s
a er
sin x x s
a (sin x)/x 1. S
a
Z 1
Z 1
sin x
dx
1 dx
x
0
0
dette flger fra grensesammenlikningstesten og medfrer konvergerens. Dette
medfrer ogs
a at integralet konvergerer.
gjre. Frst s
At integralet ovenfor konvergerer har med flere ting a
a bl
aser
ikke integralet opp nre origo da (sin x)/x 1 for |x| < 1. Faktisk s
a er
(sin x)/x 1 x R, siden 1/x drar funksjonen mot null.
Funksjonen sin x bidrar til at funksjonen oscillerer over og under x-aksen, og
det er kanseleringen av disse omr
adene som bidrar til at funksjonen konvergerer.
N
ar en ser p
a absoluttverdien av funksjonen f
ar en derimot ikke disse kanseleringene. I alle eksemplene tidligere har vi sett p
a funksjoner som g
ar mot 0 n
ar

220

III

1 Konvergens

x . At dette alltid er tilfellet for konvergente integral er feil. Det eksisterer


alts
a funksjoner
ar mot en verdi ulik
R hvor limx f (x) ikke eksisterer eller g
null men k f (x) dx konvergerer.
Fresnell integralene
Lemma C.1.2.
x


sin t2 dt , C(x) =

Integralene S(x) =


cos t2 dt konvergerer x R.

Lemma C.1.3.
Z


sin x2 dx og

Integralene


cos x2 konvergerer.

R
forst
A
a hvorfor R sin x2 dx konvergerer og ikke R sin x dx er ikke s
a veldig
vanskelig. For sin(x2 ) har funksjonen et en topp over x-aksen, ogs
a en bunn
under x-aksen. Arealet av disse toppene og bunnene blir mindre og mindre n
ar
x . Dette er ikke tilfellet for sin x som oscillerere med samme hastighet
hele tiden. Fra den alternerende rekketesten konvergerer alts
a integralet over
sin(x2 ), men ikke sin x.
bevise lemma (C.1.3) siden dette ogs
Vi kommer kun til a
a beviser lem vise at S(x) og C(x) konvergerer siden
ma (C.1.2). Selv om det er rett frem a
en integrerer en begrenset funksjon over et endelig interval.
R

betrakte inteBevis. Funksjonene er symmetrisk omkring origo, s


a holder det a
gralene over (0, ). Her vil vi bare vise at integralet over sin(x2 ) konvergerer,
men et nyaktig likt bevis kan fres for cos(x2 ). Alternativt kan en vise frst at
integralene er like store, slik at det holder a vise at ett av integralene konvergerer.


sin x2 dx =


sin x2 dx +


sin x2 dx

Funksjonen er integrerbar p
a intervalet [0, 1], den bl
aser ikke opp og har ingen
singulariterer. Vi m
a alts
a vise at integralet
Z
lim


sin x2 dx ,

bruk substitusjonen t = x2 , s
konvergerer. Ved a
a x = t og 2 t dx = dt f
ar en
Z

sin x2

2

Z
dx =

N2

sin t
cos N 2
cos 1
1
dt =
+

2
4
2 t
2 N

Z
1

N2

cos t
dt
t3/2

sammenlikne med p-integralene ser vi at


siste integralet kjenner vi igjen. Ved a
integraler konvergerer, siden 3/2 > 1 og cos t/t3/2 1/t3/2 .
-funksjonen
Z
Lemma C.1.4.
0

tx1 et dt konvergerer for alle x (0, )

III

1 Konvergens

Bevis. Frst s
a deler vi integralet i
Z
Z
x1 t
t
e dt =

1
x1 t

tx1 et dt

dt +
1

221

og det neste steget er a vise at de uegentlige integralene p


a hyre side konverge finne en funksjon f (t) slik at 0 tx1 et f (t),
rer. Fremmgangsm
aten blir a
og vise at integralet over f (t) er konvergerer.
For det frste integralet s
a er et 1 for t 0, slik at

 x 0

Z 1
Z 1
cx
t
1
x1 t
x1

.
0
t
e dt lim
t
dt = lim
= lim
c0 c
c0
x c c0 x
x
0
Dersom x > 0, s
a vil ax 0 n
ar a 0+ , s
a integralet konvergerer mot 1/x.
R 1 x1 t
Dette viser at integralet over 0 t
e dt konvergerer.
For det neste integralet vil vi bruke at limt tr et/2 = 0 for alle r R.
Alts
a for x > 0 eksisterer det en kx R slik at 0 tx et/2 1 for t kx . Vi
holder x konstant og deler integralet ved kx
Z
Z kx
Z
x1 t
x1 t
t
e dt =
t
e dt +
tx1 et dt .
1

kx

Det frste integralet p


a hyre side er over et endelig interval og har ingen singulariter p
a intervalet, alts
a konvergerer integralet. For det neste har vi at t kx
slik at tx1 et = (tx1 et/2 )et/2 et/2 , s
a
Z
Z


tx1 et dt
et/2 dt = lim 2ekx /2 ec/2 = 2ekx /2 .
kx

kx

Merk at uansett hva kx er s


aR konvergerer integralet. Dette viser at det andre

integralet er konvergent, s
a 1 tx1 et dt konvergerer. Dette fullfrer beviset.

Betaintegralet
Z
Lemma C.1.5.

tx1 (1 t)y1 dt konvergerer for alle x, y (0, )

Bevis. Slik som beviset for konvergens av gammaintegralet deles integralet opp
Z 1
Z 1/2
Z 1
x1
y1
x1
y1
t
(1 t)
dt =
t
(1 t)
dt +
tx1 (1 t)y1 dt .
0

1/2
x1

y1

x1

For det frste integralet merk at t


(1 t)
t
for 0 t 1/2, s
a


Z 1/2
Z 1/2
(1/x)x cx
0
tx1 (1 t)y1 dt
tx1 dt = lim
c0
x
0
0
N
a, dersom x > 0 s
a vil ax 0 n
ar a 0+ , s
a det frste integralet konvergerer
dersom x > 0. Tilsvarende for det neste integralet s
a er tx1 (1 t)y1
y1
(1 t)
for 1/2 t 1 s
a


Z 1/2
Z 1/2
(1/2)y (1 c)y
0
(1 t)y1 dt
(1 t)y1 dt = lim
x
c1
0
0

222

1 Konvergens

III

N
a, dersom y > 0 s
a vil (1 a)y 0 n
ar a 1 , s
a det frste integralet
R 1 x1
konvergerer dersom y > 0. Dermed s
a konvergerer 0 t
(1 t)y1 dt for alle
x, y > 0 som var det som skulle vises.

C.1.1

KONVERGENSRADIUS

C.1.2

P R I N S I PA L

VERDI

Cauchys prinsippielle verdi (Cauchy principal value) er en m


ate a gi en verdi
til integralet som i utganspunktet divergerer.

III

C.2

2 Funksjonalanalyse

223

F U N K S J O N A L A N A LY S E

Lemma C.2.1. Anta fn er en funksjonsflge definert p


a en a
pen delmengde D
av C. Dersom fn konvergerer uniformt p
a ethvert kompakt (lukket og begrenset)
undermengde av D til grensefunksjonen f , da er f analytisk p
a D. Videre s
a er
sekvensen av deriverte fn0 konvergerer ogs
a uniformt mot f 0 p
a enhver komptakt
undermengde av D.
Lemma C.2.2. (Derivasjon under integraltegnet) La D vre en a
pen mengde
og la vre en kontur av endelig lengde L(). Anta : {} D C er en
kontinuerlig funksjon, og definer g : D C ved
Z
g(z) =
(w, z) dw

Da er g kontinuerlig. Videre dersom /z eksisterer og er kontinuerlig p


a {}D
da er g analytisk med derivert
Z
d
0
(w, z) dw
g (z) =
dz
Korollar C.2.1. La D vre en a
pen mengde og : [a, ] D C vre en
funksjon med kontinuerlig partiellderiverte /z. Dersom integralet
Z
g(z) =
(w, z) dw

konvergerer uniformt p
a en kompakt delmengde av D, da definerer det en analytisk
funksjon p
a omr
adet og har deriverte
Z
d
g 0 (z) =
(w, z) dw
dz
Lemma C.2.3. (Produkt representasjonen av (s))
n
Z 
t
ts1 dt
(s) =
1
n
0
dersom s = it + , hvor > 0 og t R.
Bevis. Vi innfrer funksjonen
Z
fn (s) =
0

t
1
n

n

ts1 dt

vise at F (s) fn (s) g


og nsker a
ar mot null, n
ar n . Vi kan skrive
Z n
Z
 n  s1
t
(s) fn (s) =
e
1t n
t
dt +
et ts1 dt ,
(C.1)
0

og ser at siste leddet g


ar mot null n
ar n vokser. Det virker rimelig at frste ledd
og g
ar mot null siden frste ledd blir mindre og mindre. Tanken er n
a at vi
vise at
nsker a

n
t
0 et 1
n1 t2 et .
(C.2)
n

224

III

2 Funksjonalanalyse

Dersom denne ulikheten stemmer s


a kan likning (C.1) skrives som
Z n


1
(s) fn (s) 1
et t+1 dt < ( + 2)
n 0
n
som flger fra ulikheten ovenfor. Dette uttrykket g
ar mot null n
ar n siden
( + 2) er endelig. Alts
a konvergerer fn (s) uniformt mot (s) som nsket.
vise likning (C.2) tar vi utgangspunkt i at for 0 y 1 s
For a
a har vi
1 + y ey (1 y)1 . For store n kan vi sette y = t/n, slik at


t
1
n

n
e

t
1
n

n

hvor vi ogs
a opphyde ulikheten i n, s
a (et/n )n = et . Vi skriver om venstresiden av ulikheten ytterligere



n
n 
t
t
= et 1 et 1 +
0 et 1 +
n
n


n 
n 
t
t
et 1 et 1 +
1
n
n


n 
2
t
= et 1 et 1 2
n
Det neste steget blir a bruke at dersom 0 a 1 s
a er (1 a)n 1 na, gitt at
2
2
na < 1. Lar vi a = t /n s
a har vi for store n

n
t2
t2
1 1 2

n
n
Dermed s
a har vi
0e

t
1
n

n

n1 t2 et

og dette fullfrer beviset.


Proposisjon C.2.1. -funksjonen definert som
Z
(s) =
et ts1 dt ,
0

er analytisk for alle > 0.


R
Bevis. Frst legger vi merke til at for a > 0 s
a er funksjonen a et ts1 dt ana vise dette m
lytisk. For a
a vi bare vise at funksjonen konvergerer unfomormt
p
a enhver kompakt undermengde av D. Deretter kan vi bruke korollar (C.2.1)
siden alle de andre vilk
arene er tilfredstilt. Som forventet s
a dominerer eksponensialen integralet for store n
Z n

Z
Z
Z




t s1
t s1
t s1

=

e
t
dt

e
t
dt
e
t
dt
et t1 dt




a

III

2 Funksjonalanalyse

225

derivere og se
Funksjonen et/2 t1 er vre begrenset av, som vi kan se ved a
p
a ekstrema. Merk at ekstrema ikke befinner seg i noen av endepunktene. Dette
medfrer at vi kan skrive

Z
t/2 1
|fn f | max e
t
et/2 dt 2C()en/2
n

Hvor vi ikke bryr oss om hva C er, bare at den avhengier av og ikke n. Slik at
n
ar n s
a vil |fn f | = 0, som er selve definisjonen p
a uniform konvergens.
vise at funksjonen er analytisk for
Dette holder bare for n 1, og vi nsker a
(0, ). Vi definerer flgende funksjon
Z
fn (s) =
et ts1 dt
1/n

Ved argumentet ovenfor s


a er hver fn analytisk. Anta at c 0. For 0 < t 1
s
a har vi et < 1 og t1 tc1 . Alts
a for n > m,
Z
Z


1/m

1/m
1
1
1


t s1
c1
e
t
dt

t
dt
=



1/n

c
ma
na
1/n
Vi m
a n
a vise at uttrykket ovenfor g
ar mot null. Gitt >
a kan vi alltid velge
0 s
0 1 slik at (ma na ) /c  n
ar m1 n1 . Alts
a tilfredstiller
fn Cauchys betingelse for uniform konvergens p
a en kompakt undermengde av
det positive halvplanet > 0. Fra lemma (C.2.1) flger det at -funksjonen er
analytisk for > 0.

226

C.3

III

3 Bohr-Mullerup Theoremet

BOHR-MULLERUP THEOREMET

Beviset her baserer seg p


a Rudins bevis [? , thm. 8.19] og hele kapitel 4 fra
[? ] se spesielt theorem 4.4. Fr vi beviser theoremet trengs et lite lemma
Lemma C.3.1. La f : (a, b) R vre en konveks funksjon, da holder
f (t) f (s)
f (u) f (s)
f (u) f (t)

ts
us
ut
for alle u, t, s (a, b) slik at a < s < t < u < b.
Bevis. La a < s < t < u < b da er
f (s + (1 )) f (s) + (1 )f (u)
siden f er konveks. Vi lar =

ut
us

(C.3)

slik at t = 0, = 0 og t = s, = 1. Da er


s + (1 )u = (s u) + u =

tu
us


(u s) + u = t

Fra dette flger det at likning (C.3) kan skrives som


f (t) f (u) +
bruke at
Ved a

ut
us

=1+

st
us


ut
f (s) f (u)
us

(C.4)

i likningen ovenfor f
ar vi

f (t) f (s) +


st
f (s) f (u)
us

(C.5)

dele p
Ved a
a s t kan likning (C.4) skrives p
a formen
f (u) f (s)
f (u) f (t)

us
ut

(C.6)

dele ulikheten p
Tilsvarende s
a kan vi skrive om likning (C.5) ved a
a s t,
f (t) f (s)
f (u) f (s)

ts
us

(C.7)

alts
a s
a er
f (u) f (s)
f (u) f (t)
f (t) f (s)

ts
us
ut
vise.
som var det vi nsket a
Theorem C.3.1. (BohrMollerup) Gitt en funksjon f : (0, ) (0, ) som
tilfredstiller
1) f (1) = 1
2) f (x + 1) = xf (x)
3) log f er konveks.

III

3 Bohr-Mullerup Theoremet

227

Da flger det at f (x) = (x) x (0, ).

Bevis. Vi vet allerede at (s) tilfredstiller punkt (1) til (3), og det gjennst
ar a
vise at f (x) er unikt definert utifra disse restriksjonene. I tillegg er det fra (2)
se p
nok a
a tilfellet hvor x (0, 1).
Vi innfrer funksjonen = log f . Punkt (1) sier at (1) = 0, punkt (2) gir at
(x + 1) = (x) + log x

(C.8)

mens punkt (3) betyr at er konveks. La x (0, 1), og la n N. Vi bruker oss


n
a av frste del av lemma (C.3.1), og setter s = n, t = n + 1 og u = n + 1 + x i
likning (C.6)
(n + 1) (n)

(n + 1 + x) (n + 1)
x

sette inn s = n + 1, t = n + 1 + x og u = n + 2 i likning (C.6)


Tilsvarende ved a
gir
(n + 1 + x) (n + 1)
(n + 2) (n + 1)
x
Likning (C.8) sier n
a at (n + 1) (n) = log n, og i tillegg ved a bruke (2) har
vi (n + 2) (n + 1) = log(n + 1). Vi kan kombinere ulikehetene til
log n

(n + 1 + x) (n + 1)
log(n + 1)
x

(C.9)

bruke ??, n + 1 ganger p


Ved a
a (n + 1 + x) s
a f
ar vi
(x + n + 1) = (x + n) + log(x + n)
= (x + n 1) log(x + n) + log(x + n 1)
..
.


= (x) + log (x + 1)(x + n 1) (x + 1)x
Ved a sette x = 0 i utledningen ovenfor v
ar vi ogs
a (n + 1) = log n! siden n N.
Vi kan n
a skrive om likning (C.9)


(x) + log (x + 1)(x + n 1) (x + 1)x log n!
log(n + 1)
log n
x
gange ulikheten med x, som er gyldig siden x (0, 1) og
Det neste steget blir a
trekke fra log(nx ) fra hvert ledd
a


0 (x) + log (x + 1)(x + n 1) (x + 1)x log n! log nx
log(n + 1)x log nx
forenkle likningen ovenfor f
Ved a
ar en




n!nx
1
0 (x) log
x log 1 +
x(x + 1) (x + n)
n

228

3 Bohr-Mullerup Theoremet

n
Dette uttrykket holder for alle n og ved a
a la n f
ar vi


n!nx
(x) = lim log
n
x(x + 1) (x + n)

III

(C.10)

siden log(1 + 1/n) log 1 = 0. Ved a ta exp p


a begge sider av likning (C.10) f
as


n!nx
f (x) = lim
n x(x + 1) (x + n)
siden e(x) = f (x). Dette viser at funksjonen f har en unik representasjon, det
eksisterer alts
a bare en funksjon som tilfredstiller punkt (1) til (3). Siden vi
vet at -funksjonen ogs
a tilfredstiller punkene, m
a f (x) = (x). Dette fullfrer
beviset.

230

1 Kortsvar

5.1

K O R T S VA R
1.3.3

1.2
1. 2
2. /2
1. /3

3.


log 2
0

a 6= 0
ellers

log 2
0

a 6= 0
ellers

I=
2. /3
4.
1.3.2 hello
I=

1. x + 3 ln |x| + C
5.
6. log 5

27. 2
7. log 5

2 x 4 ln x + 2 + C

49.

8. 2
9. 6
10. 6

70.

11. /2
13. log 5

72.

x3
log |x + 1|
3

14. log 2 > 1/ 2

8.1 Oppgaver

2.3

1. like f (x) = f (x)


2. 0
3. 1/2
4. 0
6.
2.4

8.
9. /2
10. 1 + 2 log 2
11. /2
2.5

12.

2cd

13. log

14. Del opp og la u = ex


15.

2a

16. Del opp og la u = ex

231

232

1 Kortsvar

2.5.1

2.8

1.
1. /3
2.6.2
2.
2
1.
3 3

cos a
2

4.

2. arctan

x
+2

x2

5. 2 /4
3
3.
2 2

2.10.2

2.7.1

1. log

1. ex sin x + C

2. /2

2. log
3. c = 3, k = 2


4. log

5. log

2.7.2

1. (x 1)4 ex + C

3. 0

4. log

2.10.3

1. 4I = sin 2x + 2x

2. (x 1)4 ex + C
3. (x 1)4 ex + C
4. (x 1)4 ex + C

2. 4I = sin 2x + 2x

3. 4I = sin 2x + 2x

14.1 Oppgaver

2.11.1

233

18. 2

1. arcsin(2x 1) + C
2. ln (cos x + sin x) + C

19.
20. 2
21. arctan x +

3. 2

1
arctan(x3 )
2


3/2  

 22. 1Roger
1
1
1+
3 1+
5 +C
x
x
1
23.
2013
1
1
5.
+ arctan x2
24. 0
2x
2

2
4.
15

6.

1
e
2



xa
1 arctan( x ) + C
7. ln
x+a
a
a
8. ln x ex + C

25.

1
2

26. 2

27.
2 ab
r

9. /4
10. /4

29.

1
x log x
1
x
log
+ arctan
4
x + log x
2
log x

30.

1
ln(6)
6

11. 4 1 + x
12. n
13.

1 2x
e +C
2

q
p
14. 2 x + x2 + 1 + C



1
1
15.
log 1 6 + C
6
x
16.

3 2
16

17. 2
18.

1 (x + 2)3 (x + 22)
+C
300
(x + 7)4


1
+ 2 log x + 1 + x
x

28. 2 1 +

31. 1/2
32.

2
2 log2 (2)

1
33.
log
2

e1
e+1

34.

log 5
2 log(2/3)

36.

37.

x
x5 + x + 1

234

1 Kortsvar

38. 2 log 2 1

39.
2 ab

40.

 3
2

14.2 Oppgaver

235

2.11.1

2.11.2

41. /4

1. I1 = /4
I2 = /4
2. I1 = /4
I2 = /4

42. log(5/4)C



p
3. arcsinh(x) = log x + x2 + 1
43.
4. 1/r n
ar r > R > 0,
1/R n
ar R > r > 0

44. 2

5. I1 = /4
I2 = /4


45. 2x sin x + C

6.
46. log

x2 1 + x

x2 + 1 + x

!
+C

7. I() = /4
8. = a2 /5
2ab
cos =
25

47. 0

9. t =
48. 1

10. / 3

49. 2 /4

11.
4 2

50. 42 /32

12. t =

1
+ n n N
2

13.

51.
3 6

52. 1/2 log

1
+ n n N
2

14. 1 + 2 log 2
p
3/2

15. 1 + 2 log 2
16. A = B = 1, C = 0, /3

53. 2

17. 2
18. 2

236

1 Kortsvar


19. 47

41.

20. 2 2
42.
21. I = /2

4
3

2

26
15

log 3
3
2

43. xe2x

22. I = positiv = 22/7


44. hint: u = x2 ,

23.
4


4
3

24.

29.

4
3

u=t

45.
46.

2

47. a = 3, b = 4 s
a ab + ba = 12

25. 2
26.

5
49

48. B = 1/4, A = 1/4.

a ab + ba = 12
49. a = 3, b = 4 s

2

2
22n+1

 
2n
n

3.3.2
3. arcsin(2x 1) + C

30. /4
4.
31. /4

a b
(1 )

5. C = log 2 .

32. Hint: La integralene f


a samme
grenser.
3.3.3
33.

1.

34. a = 3, b = 4 s
a ab + ba = 12
35. I =

x(x 2)
, a = 0, b = 16/9
2

/n
sin /n
2. / sin n

3.3.4

36. x = tan tan

3. I(m) =

37. 2

3.3.9

38. a = 3, b = 4 s
a ab + ba = 12

1.

39. 2
40. a = 0, b = 1 max(I) = 1/2

2.

sign m, I(0) = 0.

1 Kortsvar

3.6

3.10.2

1. arcsin(2x 1) + C

1. /2

2. arcsin(2x 1) + C
1
2

3.

1
1
+
a
b

2 ab

4. arcsin(2x 1) + C
5. arcsin(2x 1) + C
3.7

237

2
2. 1
3 3
3. /2
4. /2
5. arctanh
6. arctanh
8. Hint xa xb = (xa + 1) (xb + 1)

3.8

7. log

8. log

9. log

10. arcsin b
(1)n
n!

11.

12. /2

14. /2

1. (3)

15. /2

3.

4. (3)
5.

1
2
log2 (2)
2
12

13. 2 5

3.10.1

2.

16. /2

3
17. log 2 +
4
18. /2

3
19. log 2 +
4
20. /2

n

ab

238

5.30

30 Langsvar

L A N G S VA R

1.2

benytte proposisjon (2.8.1)


1. Det logiske her blir a

benytte proposisjon (2.8.1) siden uttrykket er p


2. Det logiske her blir a
a
formen xR(sin x, cos2 x). Dermed s
a er
Z
Z

(cos x)2 sin x dx .
I=
x(cos x)2 sin x dx =
2 0
0
bruke substitusjonen u 7 cos x med du = sin x. Videre
Det neste steget blir a
s
a blir grensene u = cos 0 = 1, u = cos = 1.

=
2

1
2

u dx =
1

1
2

x dx =
0

x3
3

1
,
0

hvor vi snudde grensene og brukte at x2 er symmetrisk omkring origo. Dermed


Z

x cos2 x sin x dx =
,
3
0
som var det som skulle vises.
1.3.2 hello
1. Vi legger merke til at vi kan dele opp integralet i to, som vist under
Z 2
Z 2
x + 3x
x
3x
dx =
+ 2 dx
2
2
x
x
x
Z 2
Z
x
3x
=
dx +
dx
x2
x2
The first integral kan forkortes til 1, mens i det andre kan vi sette konstantleddet
utenfor, og faktorisere ut en x i teller og nevner.
Z
Z
1
= 1 dx + 3
dx
x
= x + 3 ln |x| + C
Hvor vi tilslutt brukte at (ln x)0 = 1/x

70. Dette integralet kan se noe vanskelig ut, men lses relativt greit ved
substitusjon. Vi kan for eksempel velge u 7 ex eller t 7 ex/2 . Dersom vi bruker

3.3 Oppgaver

239

y = x + 2
y = sin x
x = sin y

D
x

Figur 5.1: Omr


adet D avgrenset av de tre funksjonene.
frste substitusjon f
ar vi at du == ex dx = u dx. Videre s
a blir ex/2 = (ex )1/2 =

u.
Z
Z
Z
ex/2
u du
du

dx =
=
x
1
+
e
1
+
u
u
u
(1
+ u)

0
0
integrere. La oss heller prve den andre
som ikke virker spesielt mye enklere a
substitusjonen. Da er dt = 12 ex/2 dx = 2t dx, og ex = (ex/2 )2 = t2
Z
Z
Z
2 dt
ex/2
t
du
dx
=
=
2
=
x
1 + t2 t
1+t
1 + e
0
0
Mye enklere. Siden t = ex s
a vil t n
ar x og t 0 n
ar x siden
t = lim ex = lim ex = lim
x

1
=0
ex

I aller siste overgang ble integralet av arctan funksjonen brukt, se.


1.3.3
2.
bruke delvis integrasjon to ganger. Vi ser
5. Integralet kan beregnes ved a
frst p
a det ubestemte integralet og f
ar
Z
Z
1 2x
1
2
2x
2
(x 1)e dx = (x 1) e 2x e2x dx
2
2


Z
1
1
= (x2 1)e2x x e2x e2x dx
2
2


1 2
1
1
=
x x
e2x + C
2
2
4

240

30 Langsvar

gjre ting
I frste delvise integrasjon ble u = x2 1 brukt og v = e2x /2. For a
litt enklere skrives integralet om til
Z


22 (x2 1)e2x dx = 2x2 2x 1 e2x + C = 2 x(x 1) 1 e2x + C
Setter vi inn grensene v
are f
ar vi n
a
Z
h

 i
22 x2 1 e2x dx = 2 x(x 1) 1 e2x



= 2 ( 1) 1 e2 2 ( 1) 1 e2


= 2 1 1 e2 2 1 1 e2
= e2 e2
I den andre overgangen ble det brukt at og lser likningen x(x 1) = 1.
beregne og direkte for a
beregne integralet.
Merk det var ikke ndvendig a
slippe den delvise integrasjonen kan vi gjre flgende antakelse
For a
Z


x2 1 e2x dx = Ax2 + Bx + C e2x
Dette virker logisk for n
ar vi deriverer er polynom ganget med en eksponensialfunksjon, har polynomet i svaret like hy grad. [xex ]0 = [1ex + xex] = [1 + x]ex
for eksempel. Da derivasjon og integrasjon er motsatte opperasjoner br dette
holde her og. Derivasjon gir


(x2 1) e2x = 2A x2 + 2(A + B) x + (2C + B) e2x
sammenlikne koeffisientene s
Ved a
a er 2A = 1, 2(A + B) = 0 og 2C + B = 1.
Slik at A = 1/2, B = A = 1/2 og C = (B 1)/2 = 1/4 og


Z

1 2
1
1
2
2
2x
2
x x
e2x = 2 x(x 1) 1 e2x
2
(x 1) e dx = 2
2
2
4
som fr.
6. 5
7. Vi har via substitusjonen u 7 1/t, s
a er du = dt/t2 dt = du/u2 .
Z
f (1/x) =
1

1/x

log t
dt =
1+t

log 1/u du
=
1 + (1/u) u2

Z
1

1 log u
du
u 1 + u2

Legger vi sammen funksjonenene f


ar vi totalt sett
Z x
Z x
Z x
log t
1 log t
t log t + log t
f (x) + f (1/x) =
dt +
dt =
dx
1
+
t
t
1
+
t
t(1 + t)
1
1
1
Z x
Z x
(1 + t) log t
log t
(log x)2
=
dt =
=
t(1 + t)
t
2
1
1
Dermed s
a blir f (e) + f (1/e) = (log e)2 /2 = 1/2. Som nsket.

30 Langsvar

241

8. Vi beregner frst de to frste integralene


2

Z
d




sin d = 2 cos 0 = 2 cos + cos 0 = 4

beregne det siste


Siden cos = 1 og cos 0 = 1. Det enkleste en kan gjre for a
integralet er delvis integrasjon. Vi har
Z
Z
2r 2 du
2r
1
1
dr
=
u
= =
2
2
1
2r
u
1
+
r2
(1 + r )
via substitusjonen u = 1 + r2 , s
a er du = 2r dr slik at dr = du/2r. Vi bruker n
a
delvis integrasjon med
r
2
1
u0 =
2

v0 =

og

u =

2r
2

(1 + r2 )
1
v =
1 + r2

og

kan integralet skrives som


Z

I = 4

2r
(1 +

2
r2 )

r
dr = 4
2

r
2 (1 + r2 )

Z
4

1 dr
2 1 + r2

Den frste delen av integralet g


ar mot null slik at

Z
= 4

r2

Z
2

(1 + r2 )

dr = 2
0





dr

0
=
2
arctan
r
=
2
0
1 + r2
2

der det siste integralet er den deriverte av arctan r. Oppsumert s


a er alts
a
Z

sin d

d
0

beregne.
som var det vi nsket a

9. 7

10. 8

13.

14.

r2
(1 +

2
r2 )

Z
dr = 2
0

dr
= 2
1 + r2

242

30 Langsvar

2.3

vise dette integralet p


2. Det er flere ulike m
ater a
a


Z 2
(1 + sin x)1+cos x
log
dx = 0 .
1 + cos x
0
Merk at integralet bl
aser opp n
ar x = og x = 3/2 slik at det er naturlig a dele
inn integralet som flger.
Z

/2

Z
f (x) dx =

|0

Z
Z 3/2
Z 2
f (x) dx +
f (x) dx +
f (x) dx +
f (x) dx

3/2
{z
} | /2 {z
|
{z
}
}
|
{z
}
I

III

II

IV

En kort skisse av lsningene er gitt under


Lsning a) Alle integralene kan mappes til intervalet (0, /2), deretter fra
symmetri er integralet null.
Lsning b)

Integalene kan beregnes direkte


I = 1 + 2 log 2
II + IV = 2 log 2
III = 1

hvor en ser med en gang at summen er 0.


Lsning c) En viser at
Z 2
Z
(1 + cos x) log(1 + sin x) dx =
0

log(1 + cos x) dx

Alle disse metodene fungerer utmerker. Her blir a) og c) studert nrmere. De


individuelle integralene vil bli studert senere i heftet.
mappe hvert integral p
a) Begynner a
a (0, /2).
Z
II =
(1 + cos x) log(1 + sin x) log(1 + cos x) dx
/2
/2

(1 sin y) log(1 + cos y) log(1 sin y) dy

=
0

Der substitusjonen x 7 y + /2 ble brukt.


Z

3/2

(1 + cos x) log(1 + sin x) log(1 + cos x) dx

III =

Z /2

(1 cos y) log(1 sin y) log(1 cos y) dy

=
0

30 Langsvar

243

Der substitusjonen x 7 y + ble brukt. Siste interval blir


Z 2
(1 + cos x) log(1 + sin x) log(1 + cos x) dx
IV =
3/2
/2

(1 + cos y) log(1 sin y) log(1 + cos y) dy

=
0

legge sammen II + III + IV f


Med bruk av x 7 2 x. Ved a
ar en
Z /2
II + III + IV =
log(1 sin y) log(1 cos y) sin y log(1 + cos y)
0

bruke
De to frste leddene kanselerer hverande, som kan bli sett siden ved a
z 7 /2 p
a en av de, slik at
Z /2
Z /2
II + III + IV =
log(1 + cos y) sin y dy =
log(1 + sin y) cos y dy
0

Igjen via enten z 7 /2 y eller siden integralvet over f (x) er likt integralet
legge dette sammen med
over f ( x) (proposisjon (2.3.1)). Ved a
Z /2
II =
(1 + cos x) log(1 + sin x) log(1 + cos x) dx
0

f
ar en det koselige resultatet
Z
I + II + III + IV =

/2

log(1 + sin x) log(1 + cos x) dx = 0

se at integralet er null
Siden v
art originale integral var delt opp i I til IV. For a
kan en igjen benytte seg av enten z 7 /2 y eller proposisjon (2.3.1).
Z /2
Z 0
Z /2
log(1 + sin x) =
log(1 + sin(/2 z)) dz =
log(1 + cos z) dz .
/2

som nsket.
c)

Integralet kan skrives som


Z 2
(1 + cos x) log(1 + sin x) log(1 + cos x) dx
0

M
alet blir n
a a vise at (1 + cos x) log(1 + sin x) = log(1 + cos x). Kall integralene
for henholdsvis A og B. Ved a bruke substitusjonen x 7 y kan frste del av
integralet skrives som
Z 2
A=
(1 + cos x) log(1 + sin x) dx
0
Z
=
(1 cos y) log(1 + sin y) dy

(1 cos x) log(1 + sin x) dy

=
0

244

30 Langsvar

ta
Der en benyttet substitusjonen x 7 y + 2 ble brukt i siste overgang. Ved a
gjennomsnittet av frste og siste integral f
ar en
1
2

A=

2 log(1 + sin x) dx =

log(1 + cos x) dx = B

Dermed s
a er
Z 2
Z
(1 + cos x) log(1 + sin log(1 + cos x) dx =
0

A B dx = 0

siden A = B 1.
dele integralet som
3. En mulig fremgangsm
ate er a

dx
=
1 + x2

dx
+
1 + x2

dx
1 + x2

(5.1)

Frste integralet kan beregnes via (2.3.2) s


a
Z

dx
=
1 + x2

1
1
+
dx = 2
1 + x2
1 + (x)2

dx
1 + x2

(5.2)

bruke substitusjonen y 7 1/x slik at


Siste integralet kan vises ved a

Z
1

dx
=
1 + x2

dx
dy
=
2
1 + (1/y) y 2

Z
0

dy
1 + y2

(5.3)

sette inn likning (5.2) og (5.3) i (5.1) f


Ved a
ar en som nsket at

dx
=2
1 + x2

Z
0

dx
+
1 + x2

dx
1 + y2

(5.4)

vise siste likhet kan en ta utgangspunkt i


For a
Z
0

dx
=
1 + x2

Z
0

dx
+
1 + x2

Z
1

dx
=2
1 + x2

dx
1 + x2

gange likheten
Hvor igjen likning (5.2) ble benyttet i siste overgang. Ved a
ovenfor med 3/2 s
a er
3
2

Z
0

dx
=3
1 + x2

Z
0

dx
=
1 + x2

dx
1 + x2

som var det som skulle vises. I siste overgang ble (5.4) brukt.
5. La oss benytte oss av den gamle og kjente identiteten
Z a
Z a
f (x) dx =
f (a x) dx
0
1 Takk

til Rob Johnson for denne lsningen

30 Langsvar

245

slik at integralet kan skrives om som flger


Z
( x)R(sin x, cos2 x) dx

(5.5)

Der det ble brukt at sin( x) = sin x og at cos2 ( x) = ( cos x)2 = cos2 x.
N
a er
Z
Z
Z
xR(sin x, cos2 x) dx =
R(sin x, cos2 x) dx
xR(sin x, cos2 x) dx
0

dele likning (5.5) i to. Ved a


legge til det
Der det ikke ble gjort annet enn a
opprinnelige integralet p
a begge sider f
ar en
Z
Z
2
2
xR(sin x, cos x) dx =
R(sin x, cos2 x) dx
0

dele likningen p
Beviset fullfres n
a ved a
a 2. Det samme gjelder ikkefor

R(cos x, sin2 x) siden


ved
a
bruke

x
f
a
r
en R cos(x ), sin2 ( x) =

2
R cos x, sin x . Minustegnet delegger dessverre for bruken av denne teknikken.
2.4
7.
a)

vise dette er rett frem


A
Z T
Z nT
Z T
1
f (nx) dx =
f (u) du =
f (x) dx
n 0
0
0

Hvor substitusjonen u = nx dx = du/n. Grensene blir u = n 0 = 0,


og u = nT . I siste overgang ble det benyttet at (2.18) er gyldig siden f (x) er
periodisk.
b) Denne og neste deloppgave flger samme fremmgangsm
ate. Vi bruker frst
substitusjonen y 7 2x, slik at
Z /2
Z
Z /2
1
f (sin 2x) dx =
f (sin y) dy =
f (sin x) dx
2 0
0
0
Hvor proposisjon (2.4.5) ble brukt i siste overgang med m = 0 og n = 1. Dette
dele integralet i to, og benytte u 7 y p
kan og vises ved a
a den siste delen
Z
Z 0
Z /2

f (sin y) dy =
f sin( u) du =
f (sin u) du
/2

/2

vre
Siden sin( u) = sin cos u cos sin u = 0 (1) sin u = sin u. For a
pinlig nyaktig. Dette medfrer
Z
Z /2
Z
Z /2
f (sin y) dy =
f (sin y) dy +
f (sin y) dy = 2
f (sin y) dy
0

som vist fr.

/2

246

30 Langsvar

c) Denne oppgaven vises p


a samme m
ate som likning (2.18). Merk at vi ikke
kan benytte oss av a), siden sin x har periode 2 og ikke /2. Derimot s
a er
/2

Z
0

1
f (sin nx) =
n

n/2

f (sin x)
0

bruke u 7 xn. Siden n Z/{0} er et heltall kan integralet deles inn i


ved a
perioder, slik at
Z
0

/2

 

n Z
n Z
1 X /2
k
1 X k/2
f (sin x) =
f sin
f (sin nx) =
y
n
n
2
(k1)/2
0
k=1

k=1

Hvor substitusjonen x 7 k/2 y ble benyttet i siste overgang.


variabelen ble slyfet grunnet plassmangel, men er dx, dx og dy.
argumentet f
ar en

cos y ,



 
 

k
k
k
sin y ,
sin
y = sin
cos y cos
sin y =

cos
y ,

2
2
2

sin y ,

IntegrasjonsSkriver en ut
k
k
k
k

= 4m + 1
= 4m + 2
= 4m + 3
= 4m

Hvor m Z. Tanken er at argumentet alltid reduseres til enten sin x eller


cos y. Dermed kan summen n
a skrives som
/2

Z
0

n Z
X

/2

 

k
f (sin nx) =
f sin
y du
2
k=1 0
Z /2
Z /2
=
f (cos y) dy +
f (sin y) dy
Z

0
/2

0
/2

f ( sin y) dy +

f ( cos y) dy +

+
0

Siden f (x) = f (x) s


a er f ( sin x) = f (sin x) og f ( cos x) = f (cos x). Vi kan
se at integralene over sin x og cos x er like
Z

/2

/2

f (sin y) dy =
0

f (cos y) dy

(5.6)

Ra
Ra
bruke y 7 /2 w, 0 f (x) dx = 0 f (a x) eller lemma (2.4.1).
ved enten a
sette inn dette kan integralet skrives som
Ved a
Z

n/2

/2

f (sin x) =
0

/2

f (sin y) dy +
0
Z /2

f (sin y) dy
0
Z /2

f (sin y) dy +

f (sin y) dy +

+
0

0
/2

f (sin y) dy

=n
0

dele begge siden p


som nsket. Beviset fullfres ved a
a n. Litt mer formelt s
a er
alle tall heltall 0, 1, 2 eller 3 tall unna 4 gangen. Slik at alle heltall kan skrives

30 Langsvar

247

p
a formen 4m, 4m + 1, 4m + 2 eller 4m + 3 hvor n Z. Dette gir at vi kan dele
opp summen som flger
n
X

14mn

k=1

14m+1n
k
2


summere over f sin
Ved a


 
n
X
k
y
=
f sin
2

k=1



14m+2n

(5.7)

14m+3n

f
ar en da
X

f ( sin y) +

14mn

f (cos y)

14m+1n

f (sin y) +

14m+2n

(5.8)
f ( cos y)

14m+3n

Integrerer vi likning (5.8) fra 0 til /2 og bruker (5.6) f


ar vi
n Z
X

I=

k=1

/2

f (sin y) dy +

14m+2n

f (cos y) dy
0

14m+1n

/2

/2

f (sin y) dy +

14mn

/2

 

k
f sin
y dy
2

/2

f (cos y) dy

14m+3n

14mn

14m+1n

X
14m+2n

X
14m+3n

/2

f (sin y) dy

Summene i parentesen er bare summen fra k = 1 til n se likning (5.7), slik at


integralet reduseres til
Z

/2

 

n Z
1 X /2
k
f sin
y dy
n
2
k=1 0
Z
Z /2
n
1 X /2
=
sin y dy =
sin y dy
n
0
0

f (sin nx) dx =
0

k=1

R /2
som var det som skulle vises. Beviset for 0 f (cos x) dx fres
samme m
ate. Her vil en dele opp argumentet som

cos y ,


 
 

k
k
k
sin y ,
cos
y = cos
cos y + sin
sin y =

cos
y ,

2
2
2

sin y ,

akkuratt p
a
k
k
k
k

= 4m
= 4m + 1
= 4m + 2
= 4m + 3

og argumentere p
a samme m
ate som fr. Dette fullfrer beviset.

jukse en smule og ikke eksplisitt regne ut I(1) her. Derimot


8. Vi velger a

248

30 Langsvar

henvises det til oppgave (1) fra seksjonen om substitusjoner.


r
Z /2
d
2
21
2

arctan
=
=
=
2
2 + sin
2+1
3 6
3 3
2 1
0
Z
d
2
2
=
=
3
22 1
0 2 + sin

(5.9)
(5.10)

Fra likning (2.42) og (2.43) med a = 2. Ved a bruke substitusjonen n = y s


a er
Z /2
Z n/2
dy
d
1
=
.
2
+
sin
n
n
2
+
sin y
0
0
Herfra benyttes samme tankegang som i oppgave (7). Vi vet at n alltid er p
a
formen 4k, 4k + 1, 4k + 2 eller 4k + 3 og drfter hvert tilfellet. La n = 4k + r,
hvor 0 r 3
Z /2
Z 2k+r/2
d
1
dy
=
2 + sin(4k + r)
4k + r 0
2 + sin y
0
!
Z 2
Z r/2
1
dy
dy
=
k
+
4k + r
2 + sin y
2 + sin y
0
0
Hvor likning (2.19) fra theorem (2.4.1) ble brukt i andre overgang. Bruker vi
n
a likning (5.10) f
as
Z /2
Z r/2
1
k
2
dy
d
=
+
.
2 + sin(4k + r)
4k + r
4k + r 0
2 + sin y
3
0
Siden vi har k like integral. La n
a frst n = 4k + 0,r = 0 da forsvinner siste
integral slik at
Z /2
d
k
2

=
+0=
2
+
sin
4k
4k
+
0
3
2
3
0

Som aldri er likt /3 3 . Dersom n = 4k + 1, s


a er
"
#
Z /2
Z /2
1
2k
dy
3k + 1
d
+

=
=
2
+
sin(4k
+
1)
4k
+
1
2
+
sin
y
2k
+1 3 3
3
0
0

Igjen se (5.9). Som bare er likt /3 3 hvis k = 0. Om n = 4k + 2, s


a er
Z /2
Z
d
k
2
1
dy
=
+
2
+
sin(4k
+
2)
4k
+
2
4k
+
2
2
+
sin y
3
0
0
k
2
2

3k + 1
+
=
,
=
4k + 2 3
4k + 2 3 3
2k + 1 3 3

R /2
Som bare er likt /3 3 dersom k = 0. Siste overgang flger fra med 0 f (sin x) dx =
R
1
2 0 f (sin x) dx. For eksempel (2.30) fra proposisjon (2.4.5) med n = 1, m = 0.
Tilslutt om n = 4k + 3 s
a er
Z /2
d
k
2
1
4
6k + 4
+
=
,
=
2
+
sin(4k
+
3)
4k
+
3
4k
+
3
4k + 3 3 3
3
3
3
0

30 Langsvar

249

Dette viser at integralene bare er like for n = 1 og n = 2


/2

d
=
2 + sin

/2

Z
0

=
2 + sin 2
3 3

Som fullfrer frste del av oppgaven. Regningen ovenfor viser og at


Z

/2

=
2 + sin 4k
2 3

Alts
a kun dersom n = 4k, k N.

9. Merk at
Z b+nT

b+(nm)T

f (x) dx =

b+(nm)T

f (u + mT ) du =

a+mT

f (x) dx
a

Der substitusjonen x = u + mT ble benyttet. Integralet kan n


a skrives som
b+(nm)T

Z
f (x) dx =

b+(nm)T

f (x) dx +

a
b

f (x) dx
b

(nm)T

f (x) dx +

=
a

f (x) dx
0

f (x) dx + (n m)

=
a

f (x) dx
0

og dette fullfrer beviset. I andre overgang ble likning (2.17) brukt, og i siste
(2.18).

bruke likning (2.19) s


10. Ved a
a er
Z

3/2+19(2)

I=

3/2

f (x) + (19 11)

f (x) dx =
+11(2)

f (x) dx
0

Det det siste integralet er null


Z

dx
dx = 0 ,
(1 + 2sin x ) (1 + 2cos x )

Dette har blitt vist i en tidligere oppgave, se oppgave (2). N


a gjennst
ar bare det
frste integralet som kan beregnes relativt smertefritt. Siden integralet over f (x)
og f (a + b x) er like store s
a er
I=
=

1
2
1
2

3/2

f (x) + f (5/2 x) dx

Z 3/2

log(1 + sin x) cos x + log(1 + cos x) sin x dx

250

30 Langsvar

bruke u 7 sin x og det andre leddet kan


Det frste integralet kan lses ved a
bruke u 7 cos x s
lses ved a
a
 Z 1
Z 1
Z 0
1
log(1 + u) = 1
log(1 + u) du =
I=
log(1 + u) du
2
0
1
0
oppsumert har en alts
a utrolig nok at
Z

71/2

(1 + sin x)1+cos x
1 + cos x


log

23


dx = 1

som var det som skulle beregnes.

11.
a er h(x) h(x), og dersom de er like s
a er
a) Dersom funksjonene er odde s
h(x) = h(x) direkte regning gir
x

Z
f (t) dt =

G(x) =
0

f (u) du = G(x)
0

siden f er en likefunksjon. Dermed s


a er G odde. Tilsvarende s
a er
Z

Z
g(t) dt =

F (x) =

g(u) du = F (x)

en likefunksjon, der det ble brukt at g er odde.


bestemme G(2nL) og F (2nL) benyttes selvsagt (2.18). S
b) For a
a
Z

2L

Z
f (t) dt og

G(2nL) = n

2L

F (2nL) = n

g(t) dt ,
0

Videre benyttes akkuratt samme fremmgangsm


ate p
a begge funksjonene For
enkelhetensskyld studeres G(2L) frst.
Z

G(2L) =

Z
f (t) dt +

f (t) dt
L
Z L

0
L

f (t) dt

2L

f (t + 2L) dt
0

= G(L) G(L)
= 2G(L)
Hvor det ble brukt at G er odde, f er periodisk og at
Z

2L

Z
f (t + 2L) du =

f (t) dt =
L

f (t + 2L) du
0

30 Langsvar

251

ble brukt i frste overgang med substitusjonen t 7 x 2L. P


a samme m
ate kan
F (2L) beregnes
L

Z
F (2L) =

2L

Z
g(t) dt +

g(t) dt
L
L

0
L

Z
g(t) dt

g(t + 2L) dt

= F (L) F (L)
=0
siden F er like. Oppsumert s
a er alts
a
G(2nL) = 2nG(L)
F (2nL) = 0
Merk at her kunne en ogs
a benyttet lemma (2.3.1) direkte med likning (2.13).
Dette gir
L

f (x) + f (2L x) dx =

G(2L) =

f (x) + f (x) dx = 2G(L)

0
L

0
L

g(x) + g(2L x) dx =

F (2L) =
0

g(x) g(x) dx = 0
0

c) For at en funksjon skal vre periodisk kreves det at F (x + 2L) = F (x), ved
sette inn verdier s
a
a er
Z x+2L
Z x
Z 2L
F (x + 2L) =
g(t) dt =
g(t) dt +
g(t) dt = F (x) + F (2L).
0

Som selvsagt er periodisk siden F (2L) = 0, og andre overgang kan vises ved a benytte (2.19). For at G(x) skal vre periodisk flger en samme fremmgangsm
ate
x+2L

Z
G(x + 2L) =

f (t) dt =
0

2L

f (t) dt +
0

f (t) dt = G(x) + 2G(L).


0

Igjen se (2.19). Alts


a trengs kravet om at G(L) = 0 for at G skal vre periodisk.

2.5

benytte seg av substitusjonen x t1 .


12. Det enkleste blir a
Z

J=
a

dt
p
=
t (t a)(t x)

1/b

=
1/a

dx
p

(1 xa)(xb 1)

1/b

x
1
p
dx
1
1
(x a)(b x ) x2

1/a

1/a

=
1/b

dx
p

ab(x

b1 )(a1

x)

252

30 Langsvar

Her brukte vi at b1 < a1 , og snudde grensene. Herfra innfrer vi = 1/b og


= 1/a slik at
Z

1
J=
ab

=
ab

dx
p

(x 1 )( 1 x)

hvor vi benyttet oss av eksempel (2.5.1) i siste overgang.

13. For a forenkle regningen noe innfres g(x) = f (x + a2 /x). En nsker da


vise at
a
Z a
Z a

dt
2 dt
g t
=
.
g (t)
t
t
1
1
benytte seg av den logiske substitusjonen y 7 t2 p
Ved a
a venstre side f
as
1
2

a2

g(y)
1

dy
.
y

(5.11)

Hvor dy = 2t dt dt/t = dy/2y. Der siste likhet f


as ved a dele p
a t2 = y. Dette
integralet kan n
a deles opp p
a en smart m
ate
1
2

Z
1

a2

dy
1
g(y)
+
y
2

dy
,
y

g(y)
a

(5.12)

og det siste integralet p


a hyre side kan n
a skrives om via y 7 a2 /t.
Z

a2

dy
g(y)
=
y

g(t)
1

dt
.
t

(5.13)

a dy/y = dx/t. Likning (5.14) skrives da om til


Hvor dy = a2 /t2 dx s
1
2

g(y)
1

dy
1
+
y
2

dt
t

g(t)
1

(5.14)

Fra dette flger det direkte at


Z

g t2

som fullfrer beviset.

 dt
1
=
t
2

a2

g(y)
1

dy
=
y

g(y)
1

dy
y

30 Langsvar

253

14. Vi deler opp integralet som flger



Z 
1
f x
dx
I=
x

Z 0
Z


1
f x x1 dx
f xx
dx +
=

0


Z   y
Z   y
e ey
e ey
y
=
f 2
e dy +
f 2
ey dy
2
2

Z
Z
y
=
f (2 sinh y) e dy +
f (2 sinh y) ey dy

 y

Z
e + ey
f (2 sinh y) 2
=
dy
2

Z
=
f (2 sinh y) 2 cosh y dy

Z
=
f (x) dy

Den frste substitusjonen ble x = ey benyttet, i det andre integralet ble x = ey


brukt. I aller siste overgang ble x = 2 sinh y slik at dx = 2 cosh y dy, og det
fullfrer beviset.

15.
Lsning 1

Legg merke til at funksjonen er symmetrisk, slik at


Z
Z
dx
1
dx
=


1 2
1 2
2
2
2
0
a + x
a + x x
x

Ved n
a og bruke resultatet
Z


f

1
x
x

f (x) dx

dx =

som ble vist i en tidligere oppgave har en alts


a at
Z
Z
1
dx
dx
=


2
1
2 a2 + x 1 2
0
a2 + x x
x
Z
1
dx
=
2 a2 + x2

 x 
1
=
arctan
a
a 0

=
2a

254

30 Langsvar

Lsning 2

Z
I :=
0

1
2

1
=
2

dx
2

a2

+ (x 1/x)


1 + 1/x2 + 1 1/x2

dx
2
a2 + (x 1/x)
Z
1 + 1/x2
1 1/x2
1
dx
dx
+
2
2 0 a2 + (x 1/x)2
a2 + (x 1/x)
|
{z
}
{z
}
A

For det frste integralet benyttes substitusjonen u = x1/x s


a du = (1+1/x2 ) dx.
Da er
Z
Z
h1
 u i
du
1 + 1/x2

dx
=
A=
=2
=
arctan
2
2
2
2 + (x 1/x)
a
+
u
u
a
a
0
a

0
Videre s
a er B = 0 da integranden er odde. Alternativt la x = u i B s
a
du = dx,
Z
Z
1 1/(u)2
1 1/(u)2
(
du)
=

B=
2
2 du = B .
a2 + (u 1/u)
a2 + (1)2 (u 1/u)
0
0
Siden B = B s
a er 2B = 0 og resultatet flger. Oppsumert har en n
a
Z
A+B

dx
=
I=
=
2 + (x 1/x)2
2
2a
a
0
som er relativt pent svar.
16.
2.5.1
1. Ved a bruke den kjente a kjre substitusjonen t 7 tan(x/2) og theorem (2.5.3)
f
ar en
Z
Z
Z
d
1
2 dt
2a dt
=
=
(5.15)
2t
2
a + b sin
(at + b)2 + a2 b2
a + b 1+t2 1 + t
Siden tan(/4) = 0, tan(0/2) = 1, f
ar en

Z 1
Z /2
Z
d
2a dt
a2 b2 dy
=
=
2
2
2
2
a + b sin
(a b2 )(1 + y 2 )
0
0 (at + b) + a b

Via y a2 b2 7 at + b. Det siste integralet kjenner vi igjen som arctan y, s


a


1
Z /2
d
2
at + b
=
arctan
(5.16)
a + sin
a2 b2
a 2 b2
0
0





2
a+b
b
=
arctan
arctan
a2 b2
a2 1
a2 1

9.2 Oppgaver

255

xy
bruke arctan x arctan y = arctan 1+xy
forenkle
Det neste steget er a
. For a

regningen
a x y/(1 + xy) med x = (a + 1)/ a2 b2 og
noe ser vi frst p
2
2
y = b/ a b
r

.
a+1
ab
a
a
ab
ab

1+ 2

=
=
=
2
2
2
2
2
a 1
a
a+b
a b
a 1
a b

bruke
Siden (a + b)/(a2 b2 ) = 1/(a 1) forenkles nevner til a/(a b). Ved a
xy
forrige likning og arctan x arctan y = arctan 1+xy
p
a likning (5.16) f
ar vi n
a
Z
0

/2

d
=
a + sin

Z
0

2a dt
2
=
arctan
2
(at + b)2 + a2 b2
a b2

a+b
ab

xy
som var det som skulle vises. Overgangen arctan x arctan y = arctan 1+xy
q
a+b
er gyldig, siden xy = ab
> 1 n
ar |a| > |b|. Tilfellet hvor a = b m
a drftes

for seg selv, og har blitt gjort tidligere. Bruker vi igjen likning (5.15) s
a kan det
neste integralet skrives som

Z
Z
Z 2
d
2a dt
a2 b2 dt
=
=
2
2
2
2
2
2
a + b sin
0
(at + b) + a b
(a b )(1 + y )

Igjen via y a2 1 7 at + 1. N
a forandres ikke grensene og vi f
ar
Z 2
Z
2
2
d
dy
=
=
2
2
2
2
a
+
b
sin

1
+
y
a b
a b2
0

vise.
som var noe enklere a
2.6.2
1. Det logiske her blir a benytte lemma (2.6.1) med n = 3 fra dette flger det at
integralene er like, og at
Z
Z
Z
dx
x
1 1+x
=
dx
=
dx
1 + x3
1 + x3
2 0 1 + x3
0
0
En kan faktorisere teller som x3 + 1 = (x + 1)(x2 x 1), som medfrer at
Z
Z
Z
1 1+x
1
dx
2 dx
dx =
=
2 0 1 + x3
2 0 x2 x 1
(2x

1)2 + 3
0

n
Ved a
a bruke substitusjonen 2x 1 = 3 tan y kan integralet forenkles til
Z
Z /2
1 1+x
1
sec2 y dy
1 


dx
=
=
+
2 0 1 + x3
2
6
3 /6 1 + (tan y)2
3
Siden sec2 y = 1 + tan2 y som har blitt vist fr. Oppsumert har en alts
a
Z
Z
dx
x
2
=
dx =
3
3
1
+
x
1
+
x
3
3
0
0

256

30 Langsvar

beregne
som
det som skulle vises. Denne m
aten er noe raskere enn a
R var
dx
direkte.
1+x3
0

dele integralet i to slik at


3. a) En fremmgangsm
ate er a
Z

2 + x2
dx =
1 + x4

1 + x2
dx +
1 + x4

dx
1+2
=
1 + x4
2

1 + x2
1 + x4

siden 1 + 1/2 = (1 + 2/2). Siste overgang flger direkte fra lemma (2.6.1) siden

1
2

1 + x42
dx =
1 + x4

Z
0

dx
1 + x4

Verdien av siste integralet flger fra likning (2.51) i eksempel (2.6.7). Vi f


ar da

Z
0

2 + x2
3
dx =
4
1+x
2

1 + x2
3
dx =
4
1+x
2 2

som var det som skulle bestemmes. En kunne og ha delt inn integralet i

2 + x2
dx =
1 + x4

x2
dx +
1 + x4

Z
0

2 dx
=3
1 + x4

Z
0

dx
1 + x4

For deretter a ha brukt likning (2.52), uten at det ville ha gjort utregningen mer
spennende eller kortere. b) Proposisjonen sier at
Z

a + bxn2
a+b
dx =
n
1+x
2

Z
0

1 + xn2
dx ,
1 + xn

(5.17)

sette inn a = 0, b = 1 og b = 0, a = 1 f
ved a
as henholdsvis
Z
0

xn2
dx =
1 + xn

Z
0

1 + xn2
dx
1 + xn

og
0

1
dx =
1 + xn

Z
0

1 + xn2
dx.
1 + xn

bevise det motDette viser at lemma (2.6.1) flger fra proposisjonen. For a
satte tilfellet, alts
a at proposisjonen flger fra lemmaet tar vi utgangspunkt i
venstresiden av likning (5.17).

Z
0

a + bxn2
dx = a
1 + xn

Z
0

xn2
dx + b
1 + xn

Z
0

dx
dx
1 + xn

bruke likning (2.48) fra lemmaet f


ved a
ar vi at
Z
0

a + bxn2
a
dx =
1 + xn
2

som var det som skulle vises.

Z
0

1 + xn2
b
dx +
1 + xn
2

Z
0

1 + xn2
dx
1 + xn

10.1 Oppgaver

257

2.7.1
1. Dette er et svrt enkelt eksempel hvor delvis integrasjon kommer til nytte.
Z
I = ex sin x + ex cos x dx

 Z
Z
x
x
= e sin x e cos x dx + ex cos x dx
= ex sin x + C
Hvor den den delvise integrasjonen ble utfrt p
a ex sin x. Med v 0 = ex , v 0 = ex
og u = sin x, u = cos x. Helt tilsvarende kunne en og brukt delvis integrasjon p
a
ex cos x, men n
a m
atte en ha valgt u = ex , u0 = ex og v 0 = cos x, v = sin x.
Z
I = ex sin x + ex cos x dx


Z
Z
x
x
x
= e sin x dx + e sin x e sin x dx
= ex sin x + C
hamre inn poenget kunne en og ha brukt produktregelen baklengs
Bare for a
Z
Z
Z
ex sin x + ex cos x dx = (ex )0 sin x + ex (sin x)0 dx = (ex sin x)0 dx
Som gir akkuratt det samme som fr.
2. Dette problemet er hakket mer utfordrende enn forrige problem. Rett frem
2
delvis integrasjon vil ikke fungere her. Vi kan ikke velge v 0 = ex siden en da
x2
0
finne v. En mulighet er da a
velge u = e
ikke klarer a
og v = x2 . Dette gir
da
 3

Z
Z
2 4 x2
x x2
2 x2
x e
dx =

e
x e
dx
3
R
R 3

Hvor forkortelsen (, ) = R ble benyttet. For det frste konvergerer ikke


uttrykket i klammene, og for det andre er ikke det siste integralet noe enklere.
2
velge v 0 = xex ,
Det m
a alts
a finnes en annet valg for u og v. Trikset blir n
aa
2
v = ex /2, og u = x, u0 = 1. Dette gir det mye enklere resultatet

Z 
Z
0
h x
i
1

x2
x2
x2
I=
x e
/2 dx = e
+
e
dx =
2
2 R
2
R
R
vise grenseverdien
Hvor det oppgitte integralet ble brukt i siste overgang. For a
2
kan en se at ex g
ar mye raskere mot null enn x g
ar mot uendelig. Alternativt
lh
opital
lim

x x2
1
e
=
2
2

lim

x
1
2 =
x
4
e

lim

1
=0
xex2

bruke mot en s
som blir litt meste laget a
a st og liten grenseverdi.

258

30 Langsvar

3. Anta at vi kan skrive c summen av to reelle tall a + b. Da kan en skrive


integralet som
Z
Z
Z

(x + c) ex x dx = (x + a) ex x dx + b
ex x dx
(5.18)

N
a kan
delvis integrasjon p
a siste leddet med
u = ex x , u0 = ex (1 +

en bruke
x)/2 x og v 0 = 1, v = x. Dermed kan en se at u0 v = xex (1 + x)/2 og
Z
Z

b
b ex x dx = bx xex
xex (1 + x) dx
2
sette dette inn i likning (5.18) f
Ved a
ar en at
Z
Z



b
(x + c) ex x dx = bx xex +
(x + a) (1 + x) xex dx
2

For at integralet skal skulle skrives p


a formen k xex m
a det siste integralet vre
null. Problemet er at en vil ende i en evig runddans med delvis integrasjon som
aldri vil forenkle integralet. Eneste mulighet for at siste integralet blir null, er
dersom a = 1 og b = 2 Siden a, b ikke kan avhengige av x. En har alts
a at
Z

(x + 3) ex x dx = 2 ex x dx
med valgene c = 3 og k = 2.
5. Dette er et av de morsomste integralene jeg vet om, og deler av integralet
frst dele integralet i to
er vist p
a forsiden av dokumentet. Det enkleste er a
biter
Z
Z
1
1
1
dx +

dx
I = log(log x) +
log x
log x
(log x)2
For det frste integralet kan en bruke delvis integrasjon p
a log(log x) med v 0 = 1,
v = x og u = log(log x), u = 1/(x log x) s
a

 Z
Z
Z
dx
x dx
dx
log(log x) +
= x log(log x)
= x log(log x)
+
log x
x log x
log x
Hvor konstanten ble unnlatt med vilje. Helt tilsvarende kan en bruke delvis
integrasjon p
a det andre integralet n
a med u = 1/ log x, u0 = 1/[x log2 x] og
0
v = 1, v = x.
 Z

Z
Z
1
dx
x
x dx
dx
x

=
+

=
2
2
2
log x
(log x)
log x
log x
x log x
log x
hvor igjen konstanten ble utelatt med vilje. Ved a kombinere disse to resultatene
s
a kan integralet skrives som
Z
2
1
x
log(log x) +

dx = x log(log x) +
+C
2
log x
(log x)
log x
Noe som er spesielt siden ingen av integralene individuelt har en elementr
antiderivert.

30 Langsvar

259

2.7.2

1. Begynner som fr med antakelsen om at svaret er et polynom av samme grad


ganget med eksponensialfunksjonen. Alts
a at
Z
(x4 6x2 + 8x 3)ex dx = (ax4 + bx3 + cx2 + dx + e)ex + C
derivere kan hyre side skrives som
Ved a
 4

ax + (4a + b)x3 + (3b + c)x2 + (2c + d)x + e + d ex
sammelikne koeffisienter p
Ved a
a samme m
ate som fr f
as liknignssystemet.
1=a
0 = 4a + b
6 = 3b + c
8 = 2c + d
3 = e + d
Nest verste verste likning gir b = 4a = 4, neste gir c = 12 6 = 6. S
a f
ar
en fra neste at d = 8 2c = 4, og tilslutt s
a er e = 4 3 = 1. Oppsumert s
a
har en alts
a at
Z
(x4 6x2 + 8x 3)ex dx = (x4 4x3 + 6x2 4x + 1)ex + C
= (x 1)4 ex + C
Der siste likhet kan ses fra binomialformelen, og eventuelt Pascals trekant. Merk
at selv strre polynom lses relativt raskt. Delvis integrasjon hadde her blitt
langt mer kronglete.

2. Begynner som fr med antakelsen om at svaret er et polynom av

3. Begynner som fr med antakelsen om at svaret er et polynom av

4. Begynner som fr med antakelsen om at svaret er et polynom av

260

30 Langsvar

2.8
a formen
1. Det logiske her blir a benytte proposisjon (2.8.1) siden uttrykket er p
xR(sin x, cos2 x). Dermed s
a er
Z
Z

I=
x(cos x)2 sin x dx =
(cos x)2 sin x dx .
2 0
0
bruke substitusjonen u 7 cos x med du = sin x. Videre
Det neste steget blir a
s
a blir grensene u = cos 0 = 1, u = cos = 1.
=

u2 dx =

Z
0

x2 dx =

x3
3

1
,
0

hvor vi snudde grensene og brukte at x2 er symmetrisk omkring origo. Dermed


Z

x cos2 x sin x dx =
,
3
0
som var det som skulle vises.
2. Legg frst merke til at via y 7 1/x s
a kan en skrive
Z
Z 1
 arctan x
 arctan 1/y
f xn + xn
dx =
dy
f y n + y n
x
y
1
0
dele opp
Hvor det ble ble brukt at dx = dy/y 2 dx/x = dy/y. Ved a
integralet ved x = 1 og benytte seg av forrige resultat kan integralet skrives som
Z 1
 arctan x + arctan 1/x
f xn + xn
dx
(5.19)
x
0
benytte seg av ??. Merk at en kunne ha f
Beviset fullfres n
a ved a
att lik ha brukt proposisjon (2.5.2), med S = 1.
ning (5.19) direkte ved a
bestemme
ater a
dette integralet p
a. Vi begynner med
3. Det er mange m

substitusjonen x = sin , slik at 1 x2 = | cos | og dx = cos d.

Z 1
1 x2
J=
dx
2
2
0 1 x (sin )
Z /2
Z /2
dt
| cos | cos
d
=
d
=
2
2
2 () tan2 () sin2 ()
1

sin
()
sin
()
sec
0
0
I frste overgang ble grensene = arcsin(0) = 0 og = arcsin(1) = /2. Videre
s
a er cos positiv p
a intervalet 0 /2 s
a | cos | = cos .
Siste overgang delte vi teller og nevner p
a cos2 og brukte at 1/ cos() =

sec() og sin / cos = tan . Videre kan vi bruke at sin2 () = 1 cos2 (), til a
skrive om nevner. Legg merke til at
1 + tan2 x =

cos2 x
sin2 x
sin2 x + cos2 x
1
+
=
=
= sec2 x
2
2
2
cos x
cos x
cos x
cos2 x

30 Langsvar

261

Dette medfrer at 1 = sec2 x tan2 x, dermed s


a har vi
Z /2
Z /2
d
d

J=
=
1 + tan2 () cos2 ()
sec2 () tan2 () 1 cos2 ()
0
0
Herfra benyttes substitusjonen u 7 tan x og du = 1 + tan2 (x) dx = 1 + u2 dx.

Z
Z
du/ 1 + u2
1
1
cos2 ()

=

=
du
1 cos2 () 0 1 + u2
1 + u2 cos2 ()
1 + u2 cos2 ()
0
I siste overgang ble vanlig delbrksoppspalting benyttet. Begge disse integralene
er elementre og beregne. Merk at 1 cos2 () = sin2 () slik at





1
1

arctan
u

cos
a
arctan
cos(a)

u
=
=

cos()
2
2
2
sin ()
sin () 2
0
Dette medfrer at vi kan skrive integralet v
art som

Z 1

1 x2
dx =
=
2 (sin )2
2 (/2)
1

x
2
1
+
cos()
4
cos
0
som var det som skulle vises. I den frste overgangen ble det kun brukt at
(1cos )(1+cos ) = 1cos2 = sin2 = (1cos )/ sin2 = 1/(1+cos ).
Tilsvarende i siste overgang ble dobbelidentiten cos(2x) = cos2 (x) sin2 (x) =
2 cos2 (x) 1 brukt. Som medfrer at 1 + cos(2x) = 2 cos2 (x), dette er akkuratt
se delbrksoppspaltingen kan en for
det samme som vi har med x = /2. For a
eksempel gjre det som flger
1
1
1 a2 + (a2 u2 a2 u2 )
=
(1 + u2 )(1 a2 u2 )
1 a2
(1 + u2 )(1 + a2 u2 )
1
(1 + u2 a2 ) a2 (1 + u2 )
=
1 a2
(1 + u2 )(1 + a2 u2 )


1
a2
1
=

1 a2
1 + u2
1 + a2 u2
hvor forkortelsen a = cos2 ble innfrt.
4. Dette integralet benytter en rekke av resultatene vi allerede har sett p
a.
dele opp integralet f
Ved a
as
Z
Z 0
Z
arctan x dx
arctan x dx
arctan x dx
=
+
2
2
2
x 2x sin a + 1
R x 2x sin a + 1
x 2x sin a + 1
0
benytte substitusjonen x 7 y kan det integralet skrives som
ved a
Z
Z
arctan x dx
arctan x dx

2 2x sin a + 1
2 + 2x sin a + 1
x
x
0
0
dele dem opp i (0, 1) (1, )
Begge disse integralene kan forenkles ved a
Z 1
Z
Z 1
arctan x dx
arctan x dx
arctan x + arctan(1/x) dx
+
=
2 2x sin a + 1
2 2x sin a + 1
x
x
x2 2x sin a + 1
0
1
0

262

30 Langsvar

hvor substitusjonen ble brukt x 7 1/y i den siste overgangen. Herfra brukes
(??) med arctan x + arctan 1/x = /2 slik at
Z
Z
arctan xdx
1
1
1
=
2
dx
2 2x sin a + 1
2 2x sin a + 1
x
2
x
x
+
2x
sin a + 1
R
0
Resten er n
a er elementr algebra. Merk at x2 2x sin a+1 = (xsin a)2 +cos2 a
dermed s
a er
Z 1
Z 1
dx
dx
J=
=
2 2x sin a + 1
2 + cos2 a
x
(x

sin
a)
0
0
sette u cos a 7 x sin a og holde fortegne bent i munnen forenkles
Ved a
integralet til





1
sin a
sin a 1
1 
a
1
1
J =
tan
tan
=

cos a
cos a
cos a
cos a 4
2
Via enten sumformelen for arctangens eller diverse trigonometriske identiteter.
Settes alt sammen f
ar en at


Z

1 
a
a
arctan xdx
1 
a
=
+

=
2 2x sin a + 1
x
2
cos
a
4
2
cos
a
4
2
2 cos a
R
som var det som skulle beregnes.
2.10.2

1. log


2. log

3. Legg merke til at xa xb = (1 + xa ) (1 + xb ). En kan dermed skrive


(1 + xa ) (1 + xb )
1
1
xa xb
=
=

.
(1 + xa )(1 + xb )
(1 + xa )(1 + xb )
1 + xb
1 + xa
Integralet kan n
a skrives p
a flgende form
Z
Z
dx
1
dx
xa xb
=

.
a
b
2
b
2
a
(1 + x )(1 + x ) 1 + x
(1 + x )(1 + x )
(1 + x )(1 + x2 )
0
0
R

Fra eksempel (2.10.2) og spesielt likning (2.78) har vi 0 (1+xadx


)(1+x2 ) = 4 for
alle a. Alts
a er
Z
xa xb
dx

= 0,
a
b
2
(1 + x )(1 + x ) 1 + x
4
4
0
som var det som skulle vises.
 
2
4. log

14.1 Oppgaver

263

2.10.3

2.11.1

1. Begynn med substitusjonen u = 2x 1 da er du = 2 dx og




1
1
1 u2
2
x x = (u + 1)
(u + 1) 1 =
2
2
4
Innsatt f
ar en da direkte at
Z
Z
Z r
1
dx
4

dx =
I=
=
du = arcsin u = arcsin(2x 1) + C
2
2
1u
xx
1 u2
fullfre kvadratet i teller. Da
Om denne lsningen virker noe rar, hjelper det a
ser en at



2
1
1
1
1
x2 x +
+
=
x
4
4
4
2
Og dette hinter kraftig til at substitusjonen x 1/2 = u/2 u = 2x 1
vil virke. Det siste integralet
p er standard, men alternativt kan substitusjonen
u = sin y benyttes. Siden 1 sin2 y = cos y og du = cos dy.

2. Legg merke til at (cos x + sin x)0 = cos x sin x slik at






Z
Z
d
(cos x + sin x)0
(cos x + sin x)0
(cos x + sin x)0
I=
dx =
log
dx = log
+C
cos x + sin x
dx
cos x + sin x
cos x + sin x
hvor det ble benyttet at
du

Z z 0}| {
Z
f (x)
du
dx =
= log f (x) dx
f (x)
u
|{z}
u

holder for alle funksjoner.

4. En kan skrive integralet som flger


Z
J=

x+1
=
x7/2

x + 1 dx
=
x1/2 x3

1
x

r
1+

1 dx
x x2

264

30 Langsvar

Ved hjelp av substitusjonen t2 = 1 + (1/x) s


a er 2t dt = dx/x2 og 1/x = t2 1
innsatt f
as da
Z

J =
t2 1 ( t2 )2t dt
2 2 2
2
t t t4 t2 + C
3
5

2 2 2
=
t t 3t2 5 + C
15

3/2  


2
1
1
=
1+
3 1+
5 +C
15
x
x

5. Ved faktorisering f
as
Z
I=

1
=
3
x + x7

1 + x4 x4
dx =
x3 (1 + x4 )

x dx

1 + x4

dx
x3

Sistnevnte integral er rett frem, og frstnevnte kan lses via u = x2 s


a du = 2x dx
Z

2
1
1
1
du
+
=
+ arctan x2 + C
I=
2
2
2
2
1+u
2x
2x
2

6. Dette integralet har blitt lst i eksempel (2.10.3) s


a
Z

sin(log x) dx =
0

ie
1h
1
x sin log x x cos log x
= e
2
2
0

Alternativt kan integralet skrives ved substitusjonen u = log x til


Z
I=
eu sin u du

Hvor en har lagt merke til at eu = x og at du = dx/x dx = eu du. Dette er et


standard integral somkan beregnes p
a samme m
ate som fr ved a innfre hjelpe skrive det om p
integralet eu cos u, via delvis integrasjon eller ved a
a kompleks
form. Legg merke til at
Z

Z
 Z
u iu
u
u
Im
e e du = Im
e cos u + i e sin u du = eu sin u du
Der Eulers identitet
eix = cos x + i sin x

14.1 Oppgaver

265

benytte seg av at
ble benyttet. P
a den andre siden s
a er Dette oppn
as ved a
gange med den konjugerte.
i2 = 1 og a
Z



1
i 1 iu u
Im
eu eiu du = Im

e e
i+1 i1


1
(1 i) (cos u + i sin u)eu
= Im
2
 h
i
i 
1
1h
= Im
sin u + cos u eu + i
sin u cos u eu
2
2
i
1h
=
sin u cos u eu
2
sammenlikne de reelle delene i stedet ville en f
Ved a
att integralet av eu cos u.
En f
ar n
a at integralet kan skrives som
Z
  h
Z
i 
1
1
sin u cos u eu
= e
eu sin u du = Im
eu eiu du =
2
2

som vist fr.


ater a angripe dette integralet p
a. Det letteste er nok a bruke
9. Det er flere m
korollar (2.3.3) med f (x) = sin x, a = 0 og b = /2. Da f
ar en direkte
Z /2
Z /2
sin x
sin x
/2

dx =
dx =
=
,
cos
x
+
sin
x
sin(/2

x)
+
sin
x
2
4
0
0
bruke theorem (2.3.1) siden
Som nsket. Integralet kan og beregnes ved a
g(x) + g(/2 x) er konstant. Helt elementrt kan en og ta utgangspunkt i
Z a
Z a
f (x) dx =
f (a x) dx .
0

Slik at integralet kan skrives som


Z /2
Z /2
Z /2
sin x dx
sin (/2 x) dx
cos x dx
=
=
.
cos x + sin x
cos (/2 x) + sin (/2 x)
sin x + cos x
0
0
0
Dobbler en integralet v
art f
ar en at
Z /2
Z /2
sin x
sin x
2I =
dx +
dx
cos
x
+
sin
x
cos
x
+ sin x
0
0
Z /2
Z /2
sin x
cos x
=
dx +
dx
cos
x
+
sin
x
sin
x
+ cos x
0
0
Z /2
Z /2
sin x + cos x

=
dx =
dx =
cos x + sin x
2
0
0
Slik at en f
ar da direkte at
Z /2
Z /2
sin x

cos x
dx =
=
dx
cos x + sin x
4
cos x + sin x
0
0
Som vist fr.
14.
angripe denne sauen i f
Det er flere m
ater a
areklr.

266

30 Langsvar

Lsning 1 Vi prver seg p


a den virkelige v
agale substitusjonen
p

0

q
2+1
p
x
+
x
1
x
+
x2 + 1
du

=
u = x + x2 + 1 ,
= p

dx
2
x2 + 1
2 x + x2 + 1
Alts
a ser en at

q
Z p
p
x + x2 + 1

dx = 2 x + x2 + 1 + C
x2 + 1
Alternativt kan en benytte seg av substitusjonen da har en videre at
p

1 x + x2 + 1
1
u

du =
dx =
dx
2
2
2
2
x +1
x +1
Alts
a bli integralet v
ar flgende
p

Z p
Z
x + x2 + 1
1 x + x2 + 1

dx =2
dx
2
x2 + 1
x2 + 1
q
Z
p
=2 du = 2u = 2 x + x2 + 1 + C
Moralen er ikke vr redd for a benytte v
agale substitusjoner. For kompletthet s
a
g
ar en derivasjonen av u nrmere i smmene. Ved bruk av kjerneregelen s
a er
0
0
g(x)1/2 = 1/2g(x)1/21 g 0 (x) = 12 g (x) s
a
g(x)

d
dx

q
p
1
x + x2 + 1 =
2
=

1
2

1
2

1
2


p
d 
2x
x + x2 + 1
1 1 + 2x2 +1
dxp
p
=

2 x + x2 + 1
x + x2 + 1

p
2

x+
x +1
x + x2 + 1
x2 +1
p
p

x + x2 + 1
x + x2 + 1
p


x + x2 + 1
x + x2 + 1


x2 + 1 x + x2 + 1
p

x + x2 + 1

x2 + 1

vise.
Som var det en nsket a
Lsning 2 Vi benytter seg av den litt mindre v
agale substitusjonen

p
du
2x
x + x2 + 1
du
dx
=1+
=

=
u = x + x2 + 1 ,
2
2
dx
u
2 x +1
x +1
x2 + 1
S
a integralet v
art kan bli omformet som flger

Z p
Z
Z
du
x + x2 + 1
1 du

I=
dx =
u
=2
2
u
2
u
x +1
q
p

= 2 u + C = 2 x + x2 + 1 + C

14.1 Oppgaver

267

Her ble det implisitt brukt at


0
1 1
u
u u
1
= og

=
x =
u
u
2 x
u
u

15. Integralet kan bli lst p


a flere ulike metoder, den antakeligvis mest kreative metoden er vist under.
Z
Z
dx
dx

I=
=
7
7
x x
x 1 x16
1
du
6
6
u=1 6 ,
= 7 du = 7 dx
dx
x
Zx
Z x
1
1
1
1
1
6
I=
du =
log |u| + C
7 dx =
6
x
6
u
6
1 x16



1
1
I=
log 1 6 + C
6
x

frst bestemme integralet av cos4 (x) for


16. Den slaviske metoden blir a
bruke delvis integrasjon.
deretter a

her vil vi heller benytte proposisjon (2.8.1)


Z
Z

xR(sin x, cos2 x) dx =
R(sin x, cos2 x) dx .
2 0
0
Heldigvis kan integralet v
art skrives p
a denne formen siden cos4 (x) = (cos2 (x))2 .
Dermed s
a er
Z
Z
Z /2

I=
x cos4 (x) dx =
cos4 (x) dx =
cos4 (x) dx
2 0
0
0
Hvor en kan se siste overgang enten fra symmetri, eller ved a dele integralet ved
beregne det siste integralet p
/2. Det finnes uttallige m
ater a
a. Det raskeste er
gjennskjenne det som et av Wallis integral
nok a
Z

/2

cos (x) dx =

22n+1

2n
n

vise integralet p
som gir oss svaret direkte. Vi velger dog a
a et mer elementrt
vis. Via symmetri eller substitusjonen x 7 /2 x har vi
Z
0

/2

cos4 (x) dx =

Z
0

/2

sin4 (x) dx

268

30 Langsvar

Dermed s
a kan integralet n
a skrives som
Z
Z
Z



cos4 (x) + sin4 (x) dx =
3 + cos(4x) dx =
3 dx
I=
2 0
6 0
6 0
regne ut. Dette gir dermed
Siste integralet er enkelt a
Z
Z

3
4
x cos (x) dx =
3 dx =
6 0
16
0
som var det som skulle vises. Vi g
ar s
a mellomregningene litt nrmere i smmene.
se den trigonometriske omskrivningen har vi
For a
2 
2
1 + cos 2x
1 cos 2x
+
2
2


1
1
1 1 + cos 4x
3 + cos 4x
1
+ cos2 (2x) =
+
=
=
2
2
2
2
2
4

cos4 (x) + sin4 (x) =

se at det siste integralet er null kan enten sees ved direkte regning eller symA
metri. Vi velger dog (som vanlig) en litt annen vri. La f (x) = cos(x) eller
f (x) = sin x da er
Z

2n

f (x) dx = 0 ,

nN

bruke substitusjonen x = 2n s
siden vi integrerer over hele perioder. Ved a
a har
vi
Z
f (2nu) du = 0
0

f
som nsket. For a
a integralet v
art setter vi f (x) = cos x og n = 2.
benytte seg av delvis integrasjon med
17. Lsning: 1 Her velger en a
1
log x
1
, u0 = 2
, v=
x2
x
x

u = log(x)2 , v 0 =
Dette gir seg at
Z 

log x
x

2
dx =

1
log(x)2 + 2
x

log x
dx
x2

Igjen bruker en delvis integrasjon hvor


u = log x , v 0 =

1
1
1
, u0 =
, v=
x2
x
x

S
a dette gir seg at det ubestemte integralet kan skrives som
Z 

log x
x

2
dx =

log(x)2
1
2 log x
x
x

1
dx
x2

14.1 Oppgaver

269

Med innsatte grenser gir dette seg



2

Z 
log x
log(x)2
log x
2
dx =
2

=2
x
x
x
x 1
1
Den siste grensen kan for eksempel vises via rekkeutvikling, eller Lh
optial. La
f (x) = log(x)a /x hvor a > 1 da har en
log(x)a1
log(x)aa
a!
log(x)
x
x
L = lim
= lim
= = lim
=0
x
x
x
x
1
1
Hvor en benyttet seg av Lh
optial, a antall ganger da en har et / uttrykk
hver gang.
a

Lsning: 2 Uten videre lar en t = log x s


a x = et og dt = x1 dx innsetning gir
da
2
Z 
Z
Z
log x
(log x)2 dx
et t2 dt = 2!
dx =
=
x
x
x
1
0
0
Den siste likheten kommer fra gammafunksjonen, som er definert som flger. La
n
a z vre et positivt heltall, da er
Z
et tz dt = z!
0

Denne funksjonen er bevist i Del I, og gjr en sterk tilbakekomst i Del III.


18.
Z

uttrykke
N
a trenger en a
=
=
=
=
=
Z

(x + 2)

(x + 7)

dx =

(x + 2)

Z 

x+2
x+7

2

1
dx
5
(x + 7)3
(x + 7)
x+2
du
5
La n
au=
,
=
x+7
dx
(x + 7)2
2 

Z 
x+2
1
5
1
dx
=
5
x+7
x + 7 (x + 7)2
I =

dx =

1
x+7

5
via u, Legg merke til at 1 u = x+7
, slik at


Z
1
1u
u2
du
5
5


1 3
1
1
u u4 + C
25 3
4
1 1 3
u [4 3u] + C
25 12

3 


1
x+2
x+2
43
+C
300 x + 7
x+7

3 

1
x+2
22 + x
+C
300 x + 7
x+7

1 (x + 2)3 (22 + x)
+C
300
(x + 7)4

270

30 Langsvar

20. Legg merke til at


/2



2

Z /2

1
cos 1 x dx
dx = 2
x


2
2
0
0


/2
i

h 
1
sin(0) = 2
= 2 2 sin
x
= 2 2 sin
2
4
0
/2

1 + cos x dx =

2 cos

Den vanskeligste overgangen er nok den frste, den er vist i strre detalj under.
Vi har

cos(2x) = cos2 x sin2 x = cos2 x 1 cos2 x = 2 cos(x)2 1
Legger en til 1 p
a begge sider, og setter x = t/2 f
ar en

cos t + 1 = 2 cos

1
t
2

2

vise. Den neste tingen som kan virke uklart er hvorfor


som var det en nsket a
en fjerner absoluttegnet og dette er enkelt og greit fordi cos(x/2) > 0 n
ar x
0
det
kun
brukt
kjente
ting
som
at
(sin
x) =
[0, /2]. Videre i utregningen blir

cos x, sin(0) = 0, sin(/4) = 1/ 2 osv.


23. Fra tidligere vet en at
Z

Z
f (x) dx =

f (x) + f (x) dx
0

Ved innsetning f
ar en da at
Z

f (x) dx =
1

=
0

=
0
1

(x)2012
x2012
+
dx
x
1+e
1 + ex
1006
(x)2
x2012
ex
+
dx
x
x
1+e
1+e
ex
x2012
ex x2012
+
dx
x
1+e
ex + 1

x2012 (1 + ex )
dx
1 + ex
0

1
Z 1
1
1
2012
2013
=
x
dx =
x
=
2013
2013
0
0
Z

24.

(5.20)

14.1 Oppgaver

271

Lsning 1 Vi legger merke til at integralet er odde omkring x = /2, og derfor


null.
bruke substitusjonen u = x. Vi
Lsning 2 En helt tisvarende metode er a
har vist tidligere at
Z a
Z a
f (x) dx =
f (a x) dx
0

Dette gir seg p


a v
art integral at
Z
I=
sin x sin 2x sin 3x dx
Z0
=
sin( x) sin(2( x)) sin(3( x)) dx
Z0
=
( sin x)( sin 2x)( sin 3x) dx
0
Z
=
sin x sin 2x sin 3x dx
0

Dobbler en integralet v
art f
ar en at
Z
Z
2I =
sin x sin 2x sin 3x dx
0

sin x sin 2x sin 3x dx

2I = 0
I=0
N
a var en svrt nye i utledningen v
ar her. Men det er nok a si at dersom I = I
s
a er I = 0.
28. Sm
akjip oppgave, legg merke til at
r
r
r
1+x
1
1+x
1
1
=

=
1+
x3
x2
x
x
x

(5.21)

Herfra benytter en seg av substitusjonen


1
1
, x=
x
u1
1
1
1
x du = dx
du = dx
x
u1
x
u=1+

du
1
= 2
dx
x

Integralet v
ar kan dermed skrives som
Z r
I=

x+1
dx =
x3

Z
Z

1
1+
x

!

1
x


dx

(5.22)

u
du
u1

(5.23)

272

30 Langsvar

29. Legg merke til at en kan skrive om integranden som flger




x (log x 1)
= 
x4 (log x)4

2

x
log x
4
x
log x

log x 1
log2 x

Dette hinter kraftig til at substitusjonen = x/ log x vil virke. Da f


ar en fra
kvotientregelen
=

x
log x 1
dx
d =
log x
log2 x

Innsatt f
ar en da direkte at integralet kan skrives som
Z 2
x (log x 1)
dx
I=
x4 (log x)4

2
x
Z
log x
log x 1
=
dx


4
log2 x
x
1
log x

Z
=
Z
=
1
4
1
=
4
=

2
d
1


1
1
1
1
1

+
4 1
+1
2 1 + 2


1
1
log
+ arctan() + C
+1
2




x log x
x
1
log
+ arctan
+C
x + log x
2
log x
4

Og en er ferdige. Oppspaltingen av brken kan eksempelvis gjennomfres slik


x2
1 (x2 1) + (x2 + 1)
1
1
1
1
=
=
+
1
2 (x2 1)(x2 + 1)
2 x2 1
2 x2 + 1
1 (x + 1) (x 1)
1
1
1 1
1 1
1
1
+
=

+
=
4 (x 1)(x + 1)
2 x2 + 1
4 x1
4 x+1
2 x2 + 1

x4

32. Begynn med substitusjonen z = 2x 1 s


a x = log(1 + z)/2. Integralet
blir da
Z
Z
x1
1
log(z + 1)
x
dx =
dz
I=
2
1/2 (1 + z) log(z)
x 1)
z
2

1
log(2
log (2) 0
0
Deler n
a integralet opp fra (0, 1) til (1, ). I det siste integralet blir variabelskifte
z 1/y benyttet s
a
I=

1
log2 (2)

Z
0

log(1 + z)
1
dz +
1/2
z (1 + z) log(z)
log2 (2)

Z
0

log(1 + 1/y)
dy
+ 1/y)( log(y)) y 2

y 1/2 (1

14.1 Oppgaver

273

Ved a bytte tilbake fra dummy-variablenen y tilbake til z i det siste integralet og
forenkle integranden kan uttrykket forenkles til
Z 1
log(1 + z)
log z log(1 z)
1
dz +
dz
2
1/2
(1 + z) log(z)
log (2) 0 z 1/2 (1 + z) log(z)
0 z
Z
Z 1
1
dr

dz
1
=
=
=
z 1/2 (1 + z)
log2 (2)
log2 (2) 0 1 + r2
2 log2 (2)

1
=
log2 (2)

Hvor i nest siste linje ble substitusjonen r 7 z 1/2 og z = r2 .


35. Hej Hej
37. Et forholdsvis tft integral om en ikke ser cruxet. Legg merke til at
4x5 1 = x (x5 + x + 1)0 x0 (x5 + x + 1)
ved n
a og bruke kvotientregelen som en har sett nyere p
a fr
 u 0
v

u0 v uv 0
uv 0 u0 v
=

v2
v2

baklengs. Kan integralet skrives som


Z
Z
x (x5 + x + 1)0 x0 (x5 + x + 1)
4x5 1
dx
=
dx
5
2
(x + x + 1)
(x5 + x + 1)2
0
Z 
x
=
dx
x5 + x + 1
x
= 5
+C
x +x+1
og en er ferdige.
38. Vi studerer frst det ubestemte integralet og legger merke til at frste del
kan enkelt integreres.
r
Z
Z r
x
x1
x1

dx = x log x
dx
x+1
x+1
x+1
Hvor det ble brukt at x/(x + 1) = 1 1/(x + 1). Det siste integralet kan enten
frst sette u2 = (x 1)/(x + 1) eller via delvis integrasjon
lses ved a
r
Z r
Z
x1
x1
x+1

dx = (x + 1)

dx
x+1
x+1
(x + 1) x2 1
p
Hvor u = (x 1)/(x + 1) og v = x + 1 ble
brukt. det siste integralet har blitt
regnet ut fr, og frste del kan forenkles til x2 1 , oppsumert har en
Z r


p
p
x1
dx = x2 1 log x + x2 1
x+1

274

30 Langsvar

Ved a bruke dette resultatet og sette inn grenesene bir det opprinnelige integralet
r
Z
h

i
p
p
x
x1

dx = x log(x + 1) x2 1 + log x + x2 1
x+1
x+1
1
1
sette inn x = 1 gir ingen problemer ovenfor og blir 1 log 2, mens tilfellet
A
hvor x m
a behandles med strre omhu. Heldigvis kan grensen deles opp
og en kan vise flgende


p
p
lim x x2 1 = 0 og lim log(x + 1) + log x + x2 1 = log 2
x

vise frste grenseverdi ganger vi med den konjugerte


For a

p
x + x2 1
1
2

lim (x x 1 )
= lim
=0
2
x
x x + x2 1
x+ x 1
vise siste
hvor konjugatsetningen (a b)(a + b) = a2 b2 ble brukt. For a
bruke lh
grenseverdi kombineres logaritmene, for deretter a
opital
!
!

x + x2 1
x + x2 1
lim log
= log lim
x
x
x+1
x+1
!

1 + x/ x2 1
= log lim
= log 2
x
1
p
p

siden x/ x2 1 = x2 /(x2 1) = 1 + 1/(x2 1) 1 n


ar x . Oppsumert har en alts
a at
r
Z
x
x1

dx = log 2 (1 log 2) = 2 log 2 1


x
+
1
x
+1
1
som var det som skulle beregnes.

dele opp integralet i to


40. Dette er et noe langt integral. Begynner med a
deler
Z 0
Z
x sin x arctan ex
x sin x arctan ex
I=
dx
+
dx
1 + cos2 x
1 + cos2 x

0
|
{z
}
|
{z
}
A

sette x 7 t i integralet A f
Ved a
as
Z

A=

Z
=

Z
=

x sin x arctan ex
dx
1 + cos2 x
(t) sin(t) arctan et
( dt)
1 + cos2 (t)
t sin t arctan et
dx
1 + cos2 t

14.1 Oppgaver

275

legge sammen A og B og benytte at arctan x + arctan 1/x = /2 kan det


Ved a
opprinnelige integraler skrives som
Z
t sin t (arctan ex + arctan 1/ex )
I=
dt
1 + cos2 t
0
Z
t sin t

=
2 0 1 + cos2 t
Rb
Rb
Husk at a f (x) dx = a f (a + b x) dx, eksempelvis via x 7 a + b u. Slik at
Z
Z
t sin t
( t) sin( t)
I=
=
2 0 1 + cos2 t
2 0 1 + cos2 ( t)
Z
Z

( t) sin t
sin t
=
=
dt I
2 0 1 + cos2 t
2 0 1 + cos2 t
Dermed s
a er
I=

Z
0

sin t
2
=
2
1 + cos t
4

Z
0

 3
dy
3
=
=
1 + y2
8
2

Som var det som skulle beregnes.


2.11.1
a at cot x er definert som cot x =
41. Fr vi begynner med lsningen minner vi p
1/ tan x. Som vil bli mye brukt fremmover, detteRfrer direkte til at
R a1 = cot x tan x.
a
bruke subtitusjonen x 7 /2 u eller ( 0 f (x) dx = 12 0 f (a) + f (a
Ved a
x) dx) f
as
Z
1 /2
1
1
1
1
+
+
+
dx .
I=
2 0
log tan x
1 tan x
log cot x
1 cot x
sin(/2x)
x
= cos
Hvor det kun ble brukt at tan(/2 x) = cos(/2x)
sin x = 1/ tan x = cot x.
kombinere logaritmene
Tanken er n
a at vi kan forenkle uttrykket ved a

1
1
log cot x + log tan x
log(cot x tan x)
+
=
=
=0
log tan x
log cot x
(log tan x)(log cot x)
(log tan x)(log cot x)
siden cot x tan x = 1 og log 1 = 0. Tilsvarende for de to siste leddene f
ar vi
1
2 cot x tan x
1
+
=
=1
1 tan x
1 cot x
(1 tan x)(1 cot x)
Siden (1 tan x)(1 cot x) = 2 cot x tan x + cot x tan x = 2 cot x tan x.
Integralet forenkles dermed ned til
Z /2
1
1
I=
+
dx
log
tan
x
1

tan x
0
Z
1
1
1
1
1 /2
+
+
+
dx
=
2 0
log tan x
1 tan x
log cot x
1 cot x
Z
1 /2

=
0 + 1 dx =
2 0
4

276

30 Langsvar

som var det som skulle vises.


se at telleren danner et perfekt kvadrat
42. Integralet kan forenkles ved a
2

e2x 2 + e2x = (ex ) 2ex ex + ex

2

= ex ex

2

Integralet kan da skrives som


q
Z log 2 (ex ex )2
Z log 2 x
Z log 2 x
|e ex |
e ex
dx
=
dx
=
dx
x
x
x
x
e +e
e +e
ex + ex
0
0
0
siden ex ex > 0 n
ar x > 0. Heretter kan substitusjonen u 7 ex + ex brukes.
x
x
Da er du = [e e ] dx og grensene blir e0 + e0 = 2 og elog 2 e log 2 = 5/2.
Z

log 2

ex ex
dx =
ex + ex

5/2

Z
2

du
= log(5/2) log(2) = log(5/4)
u

som nsket.
43. Integralet minner nesten om enhets identiteten sin2 x + cos2 x = 1. Dessverre har vi sin2 u + cos2 v, u = sin x og v = cos x. Vi har derimot Bruker vi
x 7 /2 y f
as
Z

sin2(sin /2 y) + cos2(cos /2 y) dy

I=
/2

Z
=

/2

sin2(cos y) + cos2(sin x) dy

ta gjennomsnittet av
Siden cos /2 y = sin y og sin /2 y = cos y. Ved a
dette integralet med v
art originalet f
as
1
I=
2

1
2

/2

sin2(sin x) + cos2(sin x) + sin2(cos x) + cos2(cos x) dx

0
/2

1 + 1 dx =
0

Siden argumentene er like flger resultatet fra enhetsformelen.


47. 0
48. Vi definerer de ubestemte integralene
Z
Z
sin x dx
ex cos x
J=
og
I
=
dx.
x
x
e sin x cos x
e sin x cos x
Der valget av I er valgt slik at
Z
ex sin x cos x
J I = x
dx = x
e sin x cos x

14.1 Oppgaver

277

beregne
og heldigvis s
a er ogs
a J + I rimelig pen a
Z
(ex + sin x + cos x)0
J +I =
dx = log |ex + sin x + cos x|
ex + sin x + cos x
lse systemet s
Ved a
a er alts
a
Z
x
1
sin x dx
= + log |ex + sin x + cos x|
x
e sin x cos x
2
2
Setter en n
a inn grensene kan integralet skrives som
2

Z 2
e 1
sin x dx


= + log (e 1)
= + log

x sin x cos x
e
2
e
+
1
2

som var det som skulle bestemmes. Tilslutt noteres det at nevneren er positiv
n
ar x > 0 slik at det ikke er noen problem med singulariter. I siste overgang ble
det benyttet at log(x2 1) = log(x 1) + log(x + 1).
bruke substitusjonen x2 7 t s
52. Ved a
a er 2x dx = dt slik at integralet
forenkles til
Z
1 log 3
sin t dt
I=
2 log 2 sin t + sin(log 6 t)
bruke proposisjon (2.3.1), eller substitusjonen y 7
Ved n
a og bruke enten a
a + b x har en at
Z b
Z b
f (x) dx =
f (a + b x)
a

bruke dette p
Hvor a + b x = log 6 x. Ved a
a v
art integral f
as
Z log 3
1
sin(log 6 t) dt
I=
2 log 2 sin(log 6 t) + sin t
Gjennomsnittet av disse to integralene blir flgelig
Z
sin t
sin(log 6 t)
1 log 3
I=
+
dt
4 log 2 sin t + sin(log 6 t)
sin(log 6 t) + sin t
Z
1 log 3 sin t + sin(log 6 t)
=
dx
4 log 2 sin t + sin(log 6 t)
p

1
1
log 3 log 2 =
log 3/2
=
4
2
Merk at etter substitusjonen x2 7 t kunne theorem (2.3.1) blitt benyttet direkte
siden f (x) f (a + b x) er konstant p
a intervalet.
53. La f (x) = x7 3x5 +7x3 x+1. Da er f (x) = (x7 3x5 +7x3 x+1)+2,
som er det samme som at f (x) + f (x) = 2. Interesant! For a forenkle integralet
benyttes substitusjonen n
a y 7 x, s
a dx = dy, x = /4 y = /4.
Z /4
Z /4
Z /4
f (x)
f (y)
f (x)
dx
=

dy
=
dx
2
2
2
cos (y)
/4
/4 cos x
/4 cos x

278

30 Langsvar

ta gjennomsnittet av disse to uttrykkene for integralet f


Ved a
as da
I=

1
2

/4

/4

f (x) + f (x)
dx =
cos2 x

/4

/4

h
i/4
dx
=
2
tan
x
=2
cos2 x
0

Som var den en nsket a bestemme. Hvor det ble brukt at f (x) + f (x) = 2, og
at (tan x)0 = 1/ cos2 x. Pent!

2.11.2

1. Ve

2.
d) Vi nsker a vise at integralet over (, ) er null. Ved a bruke at integranden er symmetrisk, og substitusjonen x 7 yt s
a er
Z

x2 y 2
dx = 2
(y 2 + x2 )2

(yt)2 y 2
y dt = 2y
(y 2 + (yt)2 )2

t2 1
dt
(t2 + 1)2

1 t2
dx
(1 + t2 )2

Legg n
a merke til at integralet over (1, ) kan skrives som

Z
0

t2 1
dt =
((1/x)2 + 1)2

dx
t2 1
=
((1/x)2 + 1)2 x2

Z
0

via substitusjonen t 7 1/x. Dette medfrer at


Z

f (x, y) dx = 2

f (x, y) dx + 2
0

f (x) dx
1

Z
f (x, y) dx 2

=2
0

f (x, y) dx = 0
0

som var det som skulle vises. Her ble forkortelsene f (x, y) = (x2 y 2 )/(x2 +y 2 )2
og f (x, 1) = f (x) innfrt.
Z
2
1

e)

x2 y 2
dx = 2
(y 2 + x2 )2

Z
1

(1/u)2 y 2
du
= 2
(y 2 + (1/u)2 )2 u2

Z
0

u2 y 2
du
(y 2 + u2 )2

14.2 Oppgaver

279

dele opp integralet og bruke inverf) Den frste likheten kan etableres ved a
sjonen u 7 1/x p
a siste integral. Vi har
Z
Z 0
Z 1
log x
log 1/u
du
u2
dx
=
=

log
u
du
1 2 2
2
(1 + x2 )2
(1 + u2 )2
1
1 (1 + [ u ] ) u
0
via u 7 1/x. Deler vi opp integralet og bruker likningen ovenfor f
as
Z
Z
Z
1

1 x2
1 x2
1 x2
log
x
dx
=
log
x
dx
+
log x dx
2 2
(1 + x2 )2
(1 + x2 )2
0
0 (1 + x )
1
Z 1
Z 1
x2
1 x2
log
x
log
x
dx

dx
=
2 2
(1 + x2 )2
0
0 (1 + x )
Z 1
1 x2
=
log x dx
2 2
0 (1 + x )
som var det som skulle vises.
lse integralet som innvolverer logritmer har vi ikke s
a mange verkty
b) For a
tilgjengelig. Senere vil vi f
a flere, men s
a langt har vi stort sett bare delvis
integrasjon. Vi velger
v0 =

1 x2
(1 + x2 )2

og

u = log x

Husker vi tilbake har en allerede regnet ut v 0 , se lemma (2.7.1) med x = 1.


v=

x
1 + x2

og u0 =

1
x

Alternativt s
a kan vi beregne integralet som flger
Z
Z
(1/x + x)0
1 x2
1
x
dx
=

dx =
= 2
(1 + x2 )2
(1/x + x)2
1/x + x
x +1
sette inn har vi n
Via u 7 1/x + x siden (1/u2 )0 = 1/u. Ved a
a

1 Z 1
Z 1
1 x2
x
x
dx
I=
log
x
dx
=
log
x

2 )2
2
2
(1
+
x
1
+
x
1
+
x
x
0
0
0
Frste del beregnes til null siden x log x 0 n
ar x 0. Siste integral er
elementert
Z
Z 1
Z 1
log x
1 x2
dx

dx
=
log
x
dx
=

=
2 )2
2 )2
2
(1
+
x
(1
+
x
1
+
x
4
0
0
0
som var det som skulle beregnes.
3. Vi beregner frst det innerste integralet relativt enkelt og f
ar

Z h
ix+ x2 +a2
K=
log t
dx
1
Z


p
= log x + x2 + a2 dx

280

30 Langsvar

bruke delvis integrasjon


hvor det ble brukt at log
1 = 0. En lsning videre er a
med v = 1 og eu = x + x2 + a2 . Slik at

2 0
)

 Z
p
1 + 2(x
2
2
x +a dx
= x log x + x2 + a2 x
x + x2 + a 2

 Z
p
x
2
2

= x log x + x + a
dx
2
x + a2
 p

p
= x log x + x2 + a2 x2 + a2 + C
som nsket. Den siste algebra-transformasjonen kan en se eksempelvis slik

(x2 )0
x2 + a2
x

1+
+
2
2
2
2
2
2 x + a
x + a
x + a2
=
x + x2 + a2
x + x2 + a2

2
x + a2 + x
1

2
2
x +a
x + x2 + a2
1
=
x2 + a2
og endelig s
a ble det siste integralet bergnet via substitusjonen u = x2 + a2 s
a
du/2 = x dx
Z
Z
p

x dx
1

du = u + C = x2 + a2 + C
=
2 u
x2 + a2
vise siste likheten bestemmer en frst et uttrykk for arcsinh(x). Vi har
For a
ex ex
2
x
x 2
2ye = (e ) 1
y=

0 = u2 2yu 1
p
2y (2y)2 4(1)(1)
u=
2
p
2
u=y y +1
Siden u = ex og sinh(x) > 0 x s
a er


p
x = log y + y 2 + 1


p
arcsinh(x) = log x + x2 + 1
Til slutt legger en merke til at
arcsinh

x
a

!
x 2
+ log a = log
+ 1 + log a
a


1
1p 2
2
= log
x+
x +a
+ log a
a
a


p
= log x + x2 + a2 + log a1 + log a
x
+
a

r

14.2 Oppgaver

281

p
p

Hvor en benyttet seg av at x2 /a2 + 1 = x2 /a2 + a2 /a2 = x2 + a2 / a2 og


at log a1 +log a = log a+log a = 0 eller log a1 +log a = log a1 a = log 1 = 0.
Uansett, en n
a skrive integralet som
K=

ix+x2 +a2

Z h

log t
dx
1
 p

p
= x log x + x2 + a2 x2 + a2 + C
i p
h
x
+ log a x2 + a2 + C
= x arcsinh
 ax 
p
= x arcsinh
+ x log a x2 + a2 + C
a

vise. Alternativt kan en benytte omskrivningen fr en


som var det en nsket a
beregner integralet. Da f
ar en nesten tilsvarende
K=

Z h

log t

ix+x2 +a2

dx

Z
=



p
log x + x2 + a2 dx

Z
=

arcsinh

x
a

+ log a dx

Frste integralet beregnes ved delvis integrasjon hvor u = 1 og sinh v 0 = x/a,


og siste integralet er trivielt.

= x log a + x arcsinh

x
a
Z

= x arcsinh + x log a
= x arcsinh + x log a

x
a

x2

x q

x
a

0

2

dx

+1

x
dx
+ a2

x2 + a2 dx

som nsket.

4. Velger a frst lse det ubestemte ved a prve den noe v


agale substitusjonen
u=

r2 + R2 2Rr cos

du
sin
=
d
2Rr
2 r2 + R2 2Rr cos

Dermed s
a kan integralet skrives om til
Z
I=

sin d
=
r2 + R2 2Rr cos

du
=
Rr

r2 + R2 2Rr cos
Rr

282

30 Langsvar

det bestemte integralet kan n


a skrives som
"
#
Z
sin d
r2 + R2 2Rr cos

=
Rr
r2 + R2 2Rr cos
0
0
p
p
2
2
(R + r)
(R r)
=

Rr
Rr
R r
R+r
=

Rr
Rr
Hvor fortegnet p
a |R r| avhengier av om R > r eller om R < r. Siden |x| = x
n
ar x > 0 og |x| < x n
ar x < 0. For R > r f
as


R r
R+r
R+r
Rr
2

=
Rr
Rr
Rr
Rr
r
Og for R < r f
as




R r
R r
R+r
2
R+r

=
+
=
Rr
Rr
Rr
Rr
R
Integralet blir dermed

Z
sin d
2/r

=
2
2
2/R
r + R 2Rr cos
0

n
ar R r
n
ar R < r

5. Ve
6.
g)

Uten videre s
a har en at
Z

log 1 =
1

1
dt = 0 ,
t

beregne. Frste del avanalysens fundamentalteorem


da det ikke er noe areal a
sier at la f : [a, b] R vre en kontinuerlig funksjon. La F vre funksjonen
definert for x i [a, b] ved
Z x
F (x) =
f (x) dt
a

da er
F 0 (x) = f (x)
bruke dette s
ved a
a f
ar en direkte at
d
d
log x =
dx
dx

Z
1

1
1
dt =
t
x

14.2 Oppgaver

283

Rimelig standard. Neste integral er ikke like bent frem her har en
r

ar

1
dt =
t

log a =
1

1
r du = r
u

Z
1

1
du = r log a
u

Via substitusjonen t 7 ur . Derivasjon gir


dt = r ur1 du = r ur u1 du
dt = r t du/u
dt/t = r du/u
For grensene har en at t = ar
ur = ar u = a og tilsvarende for nedre
r
r
grense t = 1 u = 1 u = 1 = 1. For det siste integralet f
ar en
Z
log(ab) =
1

ab

dt
=
t

Z
1

dt
+
t

Z
a

ab

dt
=
t

Z
1

dt
+
t

du
= log a + log b
u

hvor t [1, ab] = t [1, a] [a, ab] og substitusjonen t 7 au ble benyttet p


a siste
dele p
integralet. Da er dt = a du og ved a
a t f
as dt/t = a du/au = du/u.
bruke
h) Vi begynner med spesialtilfellet = 1. En frekk lsning2 blir a
lhopital s
a
lim

x0

log(1 + x)
1/(1 + x)
= lim
=1
x0
x
1

Alternativt kan vi lse oppgaven som flger. La f (x) = log(x), da er


lim

x0

log(1 + x)
log(1 + x) log(1)
= lim
x0
x
x
= lim

x0

f (1 + x) f (1)
x

= f 0 (1) = 1

Hvor kun definisjonen av den deriverte ble brukt. En siste metode fr vi g


ar
videre
Z
Z
Z 1
Z 1
1 1+x dt
1 x dt
log(1 + x)
du
= lim
= lim
= lim
=
lim
du = 1
x0 x 1
x0 x 0 t + 1
x0 0 1 + xu
x0
x
t
0
se p
Via t 7 xu. For > 1 s
a deriveriverer vi begge sider for a
a vekstraten
d
d
log(1 + x )
(x)
dx
dx
x1

1 + x
x1 x + 1
x1 (1 x) 1
2 Begge

definere den
metodene vist her bruker nesten sirkulr logikk. Da grensen ofte brukes til a
deriverte av logaritmefunksjonen. Vi kan alts
a ikke bruke den deriverte til a bestemme grensen. Dog
g
ar det fint siden v
ar definisjonen av logaritmen var gitt som et integral, alts
a flger den deriverte
direkte fra analysens fundamentalteorem. Dette er en av fordelene med integraldefinisjonen.

284

30 Langsvar

ta stegene i
Dersom 1 og x > 1 s
a holder den siste ulikheten trivielt. Ved a
motsatt rekkeflge har vi vist
d
d
log(1 + x )
(x)
dx
dx
hvorp
a integrasjon gir oss ulikheten v
ar. Konstanten bestemmes av spesialtilfellet
= 1.
7. I utgangspunktet et heselig integral, men forenkles raskt ned til noe mer
h
andterlig. La f (x) betegne integranden da er


1
1 + (1/x)
1
1
f (1/x) = 2 log
2
x
x
2
log 1/x(1 + (1/x)2 )



1+x
1
= log

2x
log x(1 + x2 )
Fra proposisjon (2.5.2) eller ved a dele integralet ved x = 1 og benytte x 7 1/u
R
R1
p
a siste integral har vi at 0 f (t) dt = 0 f (t) + f (1/t)/t2 dt. Dermed s
a er
1

 



1 + x
1 + x
dx
I() =
log
log

2
2x
log x(1 + x2 )
0
Z 1
Z 1
dx

dx
=
=
=
[log x ]
2
log x(1 + x2 )
4
0 1+x
0
Z

Siden log

1+x
2x

= log

1+x
2

log x . Mycket pent svar!

i) Legg merke til at








1 + xb
1 + xa
1 + xa

log
log
=
log
1 + xb
2
2
Dette medfrer at vi kan skrive


Z log 1+xab
1+x
dx

J(a, b) =
= I(a) I(b) = (a b)
2
1
+
x
log
x
4
0
sette inn verdiene v
Ved a
are i likningen over f
ar vi

!
1 + x4+ 15



q
Z log

1 + x2+ 3


dx
=
2
+
15

3
=
2
+
6
3

5
.
(1 + x2 ) log x
4
4
0
se den siste algebraovergangen er noe vanskelig, men kan for eksempel fres
A
r

2
15 3 =
15 3
q
q
q
3

2

=
15 2 3 15 + 3 = 18 6 5 = 6 3 5

vise.
Siden 3 15 = 3 3 5 = 3 5 . Som var det vi nsket a

14.2 Oppgaver

285

j) Nei, integralet konvergerer ikke. Det bl


aser opp ved x = 1, faktoren 1/2
sikre
(eller en annen vilk
arlig konstant strre enn 1) er alts
a helt sentral for a
seg konvergens.

frst finne cos 2. Legg merke til at


8. Begynner med a
sin2 =

1 cos 2
1 + cos 2
og cos2 =
2
2

dele disse p
Ved a
a hverandre f
as
sin2
1 cos 2
1 tan2
=
cos 2 =
2
cos
1 + cos 2
1 + tan2
Uttrykket for tan2 kan regnes ut eksakt som
2

tan () =
a

2ab
x

dx
3
x
ab

!2


=

ba
x2
+
x2
2ab

a
=
b

1
4

b2
a2

2
+
a2
b2

Ved n
a og sette inn verdier kan cos 2 skrives som
1 tan2
1 + tan2
1 (b/a a/b)2 /4
=
1 + (b/a a/b)2 /4
4 (b/a a/b)2
=
(b/a + a/b)2
 2 2

4b a
(a2 b2 )2
a2 b2
=

2
2
2
2
2
b a
a b
(a + b2 )2
h
 i
1
2
2
2
2 2
=
4a
(25

a
)

(25

a
)
252
8 2
8 4
a +
a 1 = 8 2 + 8 1
=
625
25

cos 2 =

hvor vi i siste overgang innfrte = a2 /5. Tar frst noen hjelperesultater fr en


finner cos uttrykt via. En har
tan2 (x) =

sin2 (x)
1 cos2 (x)
1
=
=
1
2
cos (x)
cos2 (x)
cos2 (x)

og


b
2a

2




2
 a 2  b 2
b  a   a 2
a
b
(a2 + b2 )2
+1+
=
+2
+
=
+
=
2b
2a
2a
2b
2b
2b
2a
4a2 b2
|
{z
}
1

286

30 Langsvar

Dermed s
a kan en skrive om integralet som
!2

2
2ab
x
1
b
a

=
tan () =
dx

x3
ab
4 a
b
a

2
 a 2
1
b
1

1=
+
2
cos ()
2a
2
2b
2
2 2
1
(a + b )
=
cos2 ()
4a2 b2

2ab
2ab
2a 25 a2
cos = 2
=
=
a + b2
25
25
2

bestemme.
som var det en nsket a
9. Fr en gjr noen antakelser om t prver en a benytte seg av delvis integrasjon. Med u = ex og v 0 = sin x f
as
Z t
I=
ex sin x dx
0
Z t
0
= [ex cos x]t +
ex cos x dx
0
Z t
t
= 1 e cos t +
ex cos x dx
0

Herfra br en legge merke til at

Rt
0

1/t dx = 1 s
a

= et cos t +

Z
0

ex cos x +

1
dx
t

Til slutt s
a m
a et cos t = 0 for at uttrykkene skal vre like. Dette skjer hvis og
bare hvis t kan skrives som t = n 2 for alle n Z. Alts
a m
a t vre periodisk.

se frste
10.R For a
R a likhet kan substitusjonen x 7 u benyttes, eller identia
teten 0 f (x) dx = 0 f (a x) dx. Begge metodene gir at
Z
Z
Z
cos2 x dx
cos2 ( u) dx
cos2 x dx
=
=
0 1 + cos x sin x
0 1 + cos( u) sin( u)
0 1 cos x sin x
Helt tilsvarende kan en gjre med sinus integralene. Neste likhet kan vre noe
se, men det finnes flere fremmgangsm
vanskeligere a
ater. For eksempel s
a kan
vi vise at
Z
Z
(cos x)2
(sin x)2
cos(2x)
J=

dx =
dx = 0
1
1 + cos x sin x
0 1 + cos x sin x
0 1 + 2 sin(2x)
Via u 7 2x og du/2 = dx f
ar en
Z 2
Z 2
cos u
du
cos u
J ==
=
du
1
2 + sin u
1 + 2 sin u 2
0
0

14.2 Oppgaver

287

Herfra kan en enten benytte seg av wiesrtrass substitusjonen t 7 tan(u/2)


(theorem (2.5.3)) eller kompleks analyse for a beregne intgralet.3 Frste metode
gir da
Z

J=

1t2
1+t2
2t
+ 1+t
2

2 dt
=
1 + t2

Z
=

(t2

1 t2
dt
+ 1 + t)(t2 + 1)

1 + 2t
2t
2
dt = 0
2
1+t+t
t +1

bruke substitusjonen 2t 7 2u + 1 p
Siste overgang kan vi enkelt se ved a
a siste
integralet.
Alternativt. Siden begge integrande har periode holder det a vise at identiten holder p
a et vilk
arlig intervall med lengde . Via u 7 x /2 kan integralet
betrakte
skiftes til [/2, /2]. Siden funksjonene er symmetriske holder det a
vise dette flger via x 7 /2 u.
identiteten over [0, /2]. A
beregne selve integralet er rett frem
A
I=

1
2

Z
0

(cos x)2
(sin x)2
+
dx =
1 + cos x sin x
1 + cos x sin x

1+

dx/2
1
2 sin(2x)

Benytter s
a u 7 2x og t 7 tan(u/2)
Z
I=
0

dx
=
2 + sin u

1
2t dt
=
2
2 + 2t/(1 + t ) 1 + t2

dt
1 + t + t2

En kan fullfre kvadratet og f


a t2 + 1 + t = (t + 1/2)2 + 3/4. S
a hefra bruker en
p
3/4 u 7 t + 1/2 s
a

(cos x)2
=
1 + cos x sin x

Z
=

dt
(t + 1/2)2 + 3/4

Z
2
du
2
1
3
du =
=
3
2
2
2
1
+
u
3
3
4 (u + 1)

og vi er ferdig. Siden verdien av det siste integralet er .

11.
3 Ved

bruke de komplekse formene (se avsnitt (3.9.1)) kan integralet skrives som
a

Z
0

cos u
i
du =
2 + sin u
4

z + z 1
1+

|z|=1

1
4i

dz
=
(z z 1 ) z

dz
z2 + 1
z z 2 + 4iz 1

|z|=1

Polenene til integranden befinner seg ved z = 0 og z = (2 3 )i, med henholdsvs residue 1
og 1. Det flger da fra residue theoremet (??) at integralet er null.

288

30 Langsvar

benytte den oppgitte substitusjonen, da er


Lsning 1 Velger a

1 + x4
2x(1 x2 )
cos =

dx = sin d
2
1+x
1 + x4 (1 + x2 )2
bruke at sin2 + cos2 = 1 kan en skrive om sin som flger
Ved a

p
1 x2
2x
d
2

sin = 1 cos =
dx =
2
2
4
1+x
2
(1 + x ) 1 + x
bestemme grensene har en at
For a

= arccos

1 + x4
1 + x2

S
a = arccos(0) = 0 og = arccos(1/ 2 ) = /4. Integralet forenkles dermed
dramatisk til
Z 1
Z /4
d

1 x2
=

dx =
2
4
2
4 2
0
0 (1 + x ) 1 + x
og dette fullfrer regningen.
dele teller og nevner p
Lsning 2 Ved a
a x2 kan integralet skrives om til
Z
0

1 x2

dx =
(1 + x2 ) 1 + x4

Z
0

1 1/x2
q
dx

(x + 1/x) (x + 1/x)2 ( 2 )2

n
Ved a
a sette y 7 x + 1/x og dt(1 1/x2 ) dx blir integralet
Z
0

1 x2

dx =
(1 + x2 ) 1 + x4

1
1
q
=

2
1 y y 2 ( 2 )2
1 


= .
2
4
2
4 2



0
x + 1/x

arcsec
2
1

som er det samme som fr.


12. Merk at det spiller ingen rolle hvordan en uttrykker vinkelen. Her velger
benytte oss av arccos slik at
vi a
 x 
(x) = arccos
H
Alts
a hosliggende over hypotenusen. For a finne hyptenusen benyttes pytagoras,
men frst m
a hyden fra x til sirkelenp
beregnes. Formelen for vre halvdel med
radius r og sentrum c er gitt som y = r2 (x c)2 . Slik at
s
2 
2
p
ab
a+b
y=
x
= x2 + x(a + b) ab ,
2
2

14.2 Oppgaver

289

dermed kan hypotenusen uttrykkes som


H=

x2 + y 2 =

p
x(a + b) ab .

Integralet blir n
a flgelig
I(a, b) :=

arccos

x(a + b) ab

dx .

Men merk helt tilsvarende kunne en vist at


s
!


(x a)(b x)
1p
= arcsin
(x a)(b x)
= arctan
x(a + b) ab
x
Men vi holder oss som sagt til arccos, samme fremmgangsm
ate funger uansett
begynne a
beregne dette beistet benyttes
hvordan en definerer vinkelen. For a
delvis integrasjon
p med
u = arccos(x/ (a + b)x ab ) og v = x ab/(a + b) som i hintet.
Z

h
ib Z
(x) dx = v (x) +
a

(a + b)x 2ab
p
dx
2(a + b) (b x)(x a)

Fra figur s
a er vinkelen null n
ar x = a og x = b. Dette medfrer at frste del

kollapser til null, som en og kan se via innsetning siden arccos(1) = 0. For a
beregne det gjennst
aende integralet brukes flgende substitusjon
2x (a + b)
ba
p

S
a x = a w = 1 og x = b w = 1, og dx/ (b x)(x a) = dw/ 1 w2 .
Substitusjonen mapper [a, b] til [1, 1] og er generelt en nyttig greie. Integralet
blir dermed
Z 1
Z 1
Z 1
dw
w dw
(b2 a2 )w + (b a)2

dw = A
+B
2
2
4(a + b) 1 w
1w
1 w2
1
1
1
w=

2
2
Hvor A = (b a)2 /4(a + b) og B = (b
a )/4(a + b) er konstanter. Legg merke
til at siste integralet er null da w/ 1 w2 er en odde funksjon. Det frste
integralet kan eksempelvis

lses ved a sette w = sin x slik at dw = cos x dx. Med


1 w2 = cos x. S
a dw/ 1 w2 = dx.

A
1

dw
= 2A
1 w2

/2

dx = A .
0

Da integranden
ar problemet med
er symmetrisk omkring origo. Dette unng
fortegnet av 1 w2 . Ved og sette inn f
ar en endelig at
1
ba

(x) dx =
a

1 1 (b a)2
ba 4 a+b

dw
ba
=

2
4 b+a
1w

290

30 Langsvar

Som var det som skulle vises. Omformingen av integranden kan eksempelvis
gjres slik


(a + b)x 2ab
x
ab
1
p
p
=

2
a+b
2(a + b) (b x)(x a)
(b x)(x a)


1
ab
1
p
=
(b + a w(b a))
4
a+b
(1 w2


(b + a)2 4ab w(b2 a2 )
1
p
=
4(a + b)
(1 w2
=

(b a)2 w(b2 a2 )
p
4(a + b) (1 w2

I frste overgang ble det brukt at x = [b + a w(b a)]/2, i andre at at (b + a)(b


a er (b + a)2 4ab = b2 2ab + a2 = (b a)2 .
a) = b2 a2 . I aller siste overgang s
La oss avslutningsvis se nrmere p
a den den delvise integrasjonen.


g 0 (x)
x(a + b) ab
0 (x) v = p
.
(5.24)
a+b
1 g(x)2
p

Hvor g(x) = x/ (a + b)x ab . Hvor det ble brukt at [arccos u]0 = u0 / 1 u2 .


Herfra har en
"
#
p
1
x(a + b)
0
g (x) =
(a + b)x ab p
(a + b)x ab
2 (a + b)x ab
=

x(a + b) 2ab
p
2(a + b)x ab) (a + b)x ab

(5.25)

Nevneren kan skrives om som flger


1
p
1 g(x)2


= 1

x2
(a + b)x ab

1/2

p
(a + b)x ab
= p
(x a)(b x)

(5.26)

sette inn likning (5.26) og (5.25) inn i (5.24) kan 0 (x)v uttrykes som
Ved a
! p
!

(a + b)x ab
(a + b)x ab
x(a + b) 2ab
p

p
a+b
2 (a + b)x ab (a + b)x ab
(x a)(b x)
Etter en serie dramatiske forkortningen forenkles utrrykket ned til
0 (x) v =

(a + b)x 2ab
p
2(a + b) (b x)(x a)

Dette fullfrer sagaen om det delvis vanskelige integralet.


13. Viser at funksjonen er en konstant, slik at integralet blir trivielt. Velger a
bestemme den deriverte av y = arcsin x frst
y = arcsin x
sin y = x

14.2 Oppgaver

291

Deriverer begge sider implisitt med tanke p


a y, og f
ar
dx
dy
1
1
dy
=
=
dx
cos y
cos(arcsin y)

cos y = 1

Herfra har en fra enhetssirkelen og pytagoras at cos2 x + sin2 x = 1, bruker



2 
2
en x = arcsin y f
as cos arcsin y + sin arcsin y = 1 slik at cos(arcsin y) =
p
1/ 1 y 2 . Dermed s
a blir den deriverte
y = arcsin x y 0 =

1
1 x2

Tilsvarende kan det vises at


y = arccos x y 0 =

1
1 x2

flgelig s
a er f 0 (x) = 0. Siden den deriverte er null, betyr dette at f (x) er en
konstant. Eksempelvis s
a er f (0) = /2. Dermed s
a er f (x) = /2 for alle x.
Videre s
a er 1 cos x 1 og tilsvarende for sin x, slik at definisjonsmenden
til f er D = [1, 1]. Integralet v
art er dermed hyde gange bredde eller J =
h b = (/2) 2 = . Noe mer formelt kan det og fres som
Z

J=

dx =
+
=
2
2
2

f (x) dx =
1

som nsket.
frst se at p
a intervalet (0, /2) er b
ade sin x og cos x
14. Det enkleste blir a
dele opp integranden.
strengt positive. Vi st
ar alts
a fritt til a
/2

(1 + cos x) log (1 + sin x) log(1 + cos x) dx ,

I=
0

bruke substitusjonen x 7 2 x eller symmetrien mellom sin x og cos x


Ved a
har en ogs
a at I kan skrives p
a formen
/2

(1 + sin x) log(1 + cos x) log(1 + sin x) dx ,

I=
0

ta gjennomsnittet av disse to integralene f


Ved a
ar en
1
I=
2

/2

sin x log(1 + cos x) + cos x log(1 + sin x) dx ,


0

Dermed s
a har en
Z /2
Z
I=
log(1 + cos x) sin x dx =
0

h
i2
log(x + 1) dx = (y log y) y
1

292

30 Langsvar

I regningen ovenfor ble frst symmetrien mellom sin x og cos x brukt se . Begge
beregne ett av dem. Dette kan og
integralene er like store, og vi trenger bare a
dele integralet i to og bruke substitusjonen v 7 1 + cos x p
sees ved a
a frste
integral og w 7 1 + sin x p
a andre.
I andre overgang Rble substitusjonen u 7 cos x brukt og i siste ble y 7 x + 1
brukt. Sammen med log x dx = x log x x + C. Alts
a har en
/2


log

(1 + sin x)1+cos x
1 + cos x

log(x + 1) dx = 1 + 2 log 2

dx =
0

som var det som skulle bestemmes.

16.
a) Legg merke til x6 + 1 og x2 + 1 deler samme nullpunkt, nemlig x = i.
Polynomdivisjon gir da
x4 x2 + 1
2

x +1

x
+1
x6 x4
x4
x4 + x2
x2 + 1
x2 1
0

Dermed s
a er x6 + 1 = (1 + x2 )(x4 x2 + 1). Ved innsetning ser en da at
Z
0

x4 + 1
dx =
x6 + 1

Z
0

(x4 x2 + 1) + x2
dx =
(1 + x2 )(x4 x2 + 1)

Z
0

dx
+
1 + x2

x2
+1

x6

Frste integralet er trivielt, og siste kan lses ved a sette u 7 x3 du = 3x2 du.
Videre s
a blir x = 0 7 u = 0 og x = 1 7 u = 1 s
a grensene blir som fr.
Z
0

dx
1
+
1 + x2
3

Z
0

du
4
=
u2 + 1
3

Z
0

dx
=
1 + x2

Z

1 + x2 dx

 Z

dx
1 + x2

se omskrivningen av 4/3 kan en eksempelvis tenke p


For a
a flgende m
ate

1 Z 1
1
x3
4
=1+
= x+
=
1 + x2 dx
3
3
3 0
0
beregne selve integralet blir n
Der cruxet er a se andre overgang. A
a enkelt siden
Z
0

x4 + 1
4
dx =
x6 + 1
3

Z
0

i1
dx
4h
4

=
arctan
x
=

=
1 + x2
3
3 4
3
0

14.2 Oppgaver

293

b) Integralene er like store som flger fra lemma (2.6.1) med n = 6. Alternativt
vise via substitusjonen x 7 1/u.
er det like enkelt a
Z
0

x4
dx =
1 + x6

(1/u)4 du
=
1 + (1/u)6 u2

dx
1 + x6

som nsket. Her en i siste overgang byttet tilbake til x som integrasjonsvariabel.
Legger en sammen integralene eller bruker lemma (2.6.1) har en
Z
0

x4
dx =
1 + x6

Z
0

dx
1
=
1 + x6
2

1 + x4
dx =
1 + x6

Z
0

1 + x4
dx
1 + x6

I den siste overgangen ble likning (2.74) fra theorem (2.10.1) brukt. At integranden tilfredstiller likning (2.73) kan sees ved direkte utregning
 
1 + (1/x)4 1
1 + x4
1
1
=
=
= R(x)
R
2
6
2
x x
1 + (1/x) x
1 + x6
eller ved a bruke at 1 + x4 er symmetrisk med indeks 4 og 1 + x6 er symmetrisk
bruke a) har vi da
med indeks 6 (proposisjon (2.10.3)). Ved a
Z

 Z 1
Z
Z

x4
dx
dx
2
dx =
=
=
1 + x dx
6
6
2
1
+
x
1
+
x
1
+
x
3
0
0
0
0
som var det som skulle vises.
17. Dette integralet faller direkte ut fra Riemann-Lebesgue lemmaet (2.4.2).
Lemmaet kan benyttes siden 1/(1 + cos2 x) er en periodisk, kontinuerlig funksjon
og funksjonen sin x er og kontinuerlig, og deriverbar.
 Z
 Z

Z
sin t
1
dt
lim
dt
=
sin
tdt
x 0 1 + cos2 (tx)
0 1 + cos2 t
0
Dette integralet kan for eksempel lses ved Weiestrass substitusjon, settes u 7
tan(t/2) s
a er
Z
Z
Z
dt
dt
2 du
1 + u2
=
=
du

2
2
2
2
1+u
1 + u4
0 1 + cos t
0
0
1 + 1u
1+u2
som kommer i fra theorem (2.5.3). Dette integralet ble studert i eksempel (2.6.7),
s
a
Z
Z
1 + u2

dt
=
du =
2
4
1+u
2
0
0 1 + cos t
Dette integralet kan lses noe mer intuitivt via substitusjonen u = tan t. Siden
tan x g
ar mot uendelig n
ar x /2 benytter en at funksjonen er like omkring
x = /2, s
a
Z
0

dt
=
1 + cos2 t

Z
0

/2

2 dt
1 + cos2 t

294

30 Langsvar

bruke substitusjonen s
ved a
a er cos2 t = 1/(1 + u2 ) og dt = du/(1 + u2 ) slik at
Z
Z
Z dy
du

2
du

=
=
=
2
2
2
2
2
1 + 1/(1 + u ) 1 + u
1+y
1 + (u/ 2 )
2
0
0
0

I andre
overgang ble substitusjonen y = u/ 2 brukt, og deretter at 2 /2 =

1/ 2 . Det neste integralet er mye enklere


Z
h
i0
sin t dt = cos t = cos 0 cos = 2

Slik at
Z
lim

sin t
1
dt =
2
1 + cos (tx)

2= 2

18. Siden gjennomsnittsverdien av f og g er 1, la oss for enkelhetensskyld sette


h g f et yeblikk s
a
Z a
Z a
1
h(x) dx
h(x) dx = 8
1=
80 0
0
Dette vil komme til nytte snarlig, forenkling av integralet gir
Z 4
Z 4
I=
8f (x) 4g(x) dx +
4g(x) 8f (x) dx
8

bruke at
Ved a

Rb
a

f (x) dx =
Z

Ra
b

f (x) dx bytter grensene i siste integral plass.

Z
8 f (x) 4g(x) dx +

8f (x) 4g(x) dx ,

Integralene kan n
a sl
as sammen. Siden g(x) er odde, er integralet over (a, a)
null
Z 8
Z 8
Z 8
I=
8f (x) 4g(x) dx =
8f (x) dx = 16
f (x) dx = 128
8

Slik at I = 27 . Dermed s
a er a = 7 som var det som skulle bestemmes.
19. Benytt substitusjonen f 1 (x) = y slik at f (y) = x. Da funksjonen er
strengt voksende, kontinuerlig og deriverbar s
a er dx = f 0 (y). Dermed s
a er
Z 10
Z 5
f 1 (x) dx =
yf 0 (y) dt
0

Benyttes n
a delvis integrasjon med u = y og v = f (y) blir integralet
h

Z 5
Z 5
5

0
yf (y) dt = yf (y) 1
f (y) dy = 5f (5) f (1) 7 = 47 .
1

Tislutt s
a er f (5) = 10 og f (1) = 0.

14.2 Oppgaver

295

kalle integralene for henholdsvis J og K.


20. La oss begynne med a
/2

(sin x)a+1 dx og K =

J=

/2

(sin x)a1 dx ,

Hvor a = 2 . M
alet blir n
a a finne en sammenheng mellom integralene, noe som
se p
kan gjres ved eksempelvis delvis integrasjon. Ved a
a J med u = (sin x)a
og v = cos x har en
/2

h
i/2 Z
J = cos x sin(x)a
+
0

a cos2 x sin(x)a1 dx

/2

a 1 sin2 x sin(x)a1 dx


=
0

/2

=a

sin(x)a1 dx a

/2

sin(x)a+1 dx

Hvor en kan gjennkjenne de to siste integralene som henholdsvis K og J. Lik finne slik
ningen J = aK aJ kan lses for J/K , det er jo dette vi nske a
at
J
a
2a a2
=
=
K
2+a
4 a2
sette inn har en alts
Der teller og nevner ble ganget med 2 a. Ved a
a

(sin x) 2 +1
0

R /2
(sin x) 2 1
0

R /2

dx

dx

2 2 ( 2 )2

=2 2
4 ( 2 )2

og dette fullfrer utregningene.

21. En kan frst forenkle integralet noe via y 7 x2 . Da er dy = x dx og


Z
a

2x dx
p

(x2

a2 )(b2

x2 )

b2

=
a2

dy
p

(y

a2 )(b2

y)

=
u

dy
p

(y u)(v y)

vise at
Hvor i siste overgang variablene u = a2 og v = b2 ble innfrt. For a
integralet er uavhengig av a og b, beregnes
Z
K=
a

dx
p

(x a)(b x)

eksplisitt. Her kan en for eksempel benytte substitusjonen


x 7 a cos2 + b sin2 ,

296

30 Langsvar

med dx = 2(b a) cos sin . N


ar x = a blir = 0 og n
ar x = b blir = /2.
Innsetning gir da
Z /2
2(b a) cos sin d
q
K=


0
a cos2 + b sin2 a b a cos2 b sin2
Z /2
2(b a) cos sin d
q
=


0
b sin2 a(1 cos2 ) b(1 sin2 ) a cos2
Z /2
2(b a) cos sin d
p
=
sin2 cos2 (b a) (b a)
0
Hvor det ble brukt at sin2 x + cos2 x = 1, to ganger. Legg merke til at b
ade
sin og cos er positive p
a intervalet s
a fortegnet blr positivt n
ar vi tar roten.
Tilsvarende er selvsagt (b a)2 positivt siden b > a.
Integralet forenkles da dramatisk til
Z b
Z /2
Z /2
dx
2(b a) cos sin d
p
=
=
2 dx =
(b a) sin cos
(x a)(b x)
a
0
0
Oppsumert s
a er alts
a
Z b
Z b
2x dx
dx
p
p
=
=
2
2
2
2
(x a )(b x )
(x a)(x y)
a
a
som var det som skulle vises. Merk integralet kunne og vrt vist ved en litt
gange ut teller f
mindre vill substitusjon. Ved a
ar en
Z b
Z b
dx
dx
p
q
K=
=
2
2
(x (a + b)x + ab)
a
a
(a b)2 /4 x (b + a)/2
bruke substitusjonen u 7
Hvor kvadratet ble fullfrt i andre overgang. Ved a
x (b + a)/2 n
a blir integralet
Z b
Z (ba)/2
dx
2 du
p
p
=
(x

a)(b

x)
(a

b)2 4u2
a
(ba)/2
Utregningen fullfres ved en siste substitusjon u 7 (a b) sin(v)/2 som gir
Z b
Z /2

(a b) cos v dv
dx
p
p
=
=
+
=
2
2
2
2
(x a)(b x)
(b a) cos v
/2
a
som fr.
22. Legg frst merke til at integralet er positivt siden 1 + x2 > 0 n
ar x > 0.
Videre s
a er teller positiv da x4 > 0. Tislutt er (1 x)4 > 0 om enn synkende p
a
x [0, 1]. Vi noterer frst at x4 1 = (x2 1)(x2 + 1) og skriver ut teller
x4 (1 x)4 = x4 4x5 + 6x6 4x7 + x8
= (x6 4x5 + 5x4 ) + (5x6 4x7 + x8 ) 4x4 + 4 4
= x4 (x2 4x + 5) + x4 x2 (x2 4x + 5) 4(x4 1) 4
= x4 (x2 4x + 5)(1 + x2 ) 4(x2 + 1)(x2 1) 4

14.2 Oppgaver

297

n
Ved a
a dele begge sider av likningen p
a x2 + 1 f
as endelig
x4 (1 x)4
4
= x4 (x2 4x + 5) 4(x2 1)
2
x +1
1 + x2
Alternativt s
a gir tidenes lengste polynomdivisjon
x6 4x5 + 5x4 4x2 + 4
x2 + 1

x8 4x7 + 6x6 4x5 + x4


x8
x6
4x7 + 5x6 4x5
4x7
+ 4x5
5x6
5x6

+ x4
5x4
4x4
4x4 + 4x2
4x2
4x2 4
4

Uansett s
a kan integralet n
a skrives som
Z

x4 (1 x)4
dx
1 + x2

x4 4x5 + 6x6 4x7 + x8


dx
1 + x2

I=
0

Z
=
0

Z
=

x6 4x5 + 5x4 4x2 + 4

4
dx
1 + x2

x7
2x6
4x3

+ x5
+ 4x 4 arctan x
7
3
3
1
2
4
=

+1
+4
7
3
3


1

Siden arctan(1) = /4 og arctan(0) = 0


=

22
.
7

Siden integralet v
art er positivt flger det at
22
22
>0
>
7
7

I>0

som nsket. Siden 2 < 1 + x2 < 1 n


ar x (0, 1) har en at
Z
0

x4 (1 x)4
dx <
1

Z
0

x4 (1 x)4
dx <
1 + x2

Z
0

x4 (1 x)4
dx
2

298

30 Langsvar

opphye ulikheten i 1. Integralet kan beregnes forholdsvis enkelt


Ved a
Z

x4 (1 x)4 dx

S=
0

Z
=

x4 (1 4x + 6x2 4x3 + x4 ) dx

1
2
6
1
1

+
5
3
7
2
9
1
=
630
=

Dermed s
a er
1
630

22
1

7
630

<

22

<

<

<

1 1
2 630
22
1

7
1260

23. Siden er en lsning av 1 = x2 x medfrer dette at 1 = 1/.


Dermed s
a kan en skrive
Z
Z
x arctan x
x arctan x dx
I=
dx
=
2
2

x
(1
+
x
)
(1 + x )
0
0
substituere u = 1/x f
Ved a
as
Z

(1/u) arctan 1/u


(1 + (1/u) )

Z
(u) du =
0

x arctan 1/x dx
2
x
(1 + x )

Ved a ta gjennomsnittet av disse integralene og bruke at arctan x + arctan 1/x =


/2 forenkles integralet betraktelig.
1
I=
2

Z
0

x (arctan x + arctan 1/x) dx


=
2

x
4
(1 + x )

Z
0

x
(1 +

2
x )

dx
x

benytte seg av substitusjonen y 7 x som gir


Neste steg blir a


Z
dy

I=
=

=
4 0 (1 + y)2 y
4
1+y 0
4
g
For a
a den siste substitusjonen litt nrmere i smmene. Derivasjon gir
dy = x1 dx

1
dx
1 dy
dx
dy = x

x
y
x

Hvor det ble benyttet at xa+b = xa xb og i siste overgang at x = y. Dermed s


a
er
Z
x arctan x

I=
dx =
2

4
(1 + x )
0

14.2 Oppgaver

299

vise at 2 < 6I < 3 eller


Som var det som skulle vises. N
a gjennst
ar det bare a
1
3

<

2
(1 + 5 ) <
3

4 1+ 5

<

<

1
2

1+ 5

vise ulikheten til venstre, alts


bruke at
Velger
a at 2(1 + 5 ) < 3. Ved a
frst a
1 + 5 = 4 sin(3/10) f
as direkte at.

2(1 + 5 ) = 8 sin(3/10) < 8 sin(5/10) = 8


se frste ulikheten legg merke til
Siden 8 < 3 3 < 3 stemmer ulikheten. For a
at sin x stiger p
a x (0, /2) s
a er sin(3/10) < sin(5/10). For a vise den neste
ulikheten krever litt mer spissfindighet. La oss frst benytte oss av at 22/7 > ,
som ble vist i forrige oppgave. Fra taylorutvikling s
a er

1+x 1+

1
x2
x
+
2
8

S
a lenge |x| < 1. Siden rekka konvergerer s
a er frste ledd x/2 et overslag andre
ledd x2 /8 et underslag for den egentlige verdien osv. Dermed s
a er
r



1
1 1
1 1
15
5 +1=1+2 1+
>1+2 1+

=3+
4
2 41
8 42
64
Da er en egentlig ferdig siden

15
22
5 +1>3+
>
>
64
7
frskt skrive om 22/7 til 3 + 1/7
Den midterste ulikheten kan en enkelt se ved a
trekke fra 3 fra begge sider av ulikheten f
. Ved a
as 15/64 > 1/7 105 > 64.
Via kryssmultiplikasjon. Oppsumert har en alts
a vist at
1

2
1
<
<
,
3
4 1+ 5
2
som nsket.
24. Vi noen raske omskrivninger kan integralet skrives p
a formen
  2 
Z 
x 1
(1 x2 ) ln(1 + x2 ) + (1 + x2 ) (1 x2 ) ln(1 x2 )
I=
xe x2 +1 dx
4
2
(1 x )(1 + x )



1 x2
2
2


Z
(1

x
)
ln

(1
+
x
)

1 x2
1 + x2

=
x exp 1 + x2 dx
(1 x2 )(1 + x2 )(1 + x2 )


1 x2
2


Z
(1

x
)
ln

(1
+
x
)

2x
1
1 x2
2 dx
1 + x2

=
exp

2
2
2
4
(1 x )
1+x
1+x
1 + x2

1
4





Z  
1 x2
1 + x2
2x
1 x2
2 dx
ln

exp

(5.27)
1 + x2
1 x2 1 + x2
1 + x2
1 + x2

300

30 Langsvar

Videre s
a benyttes weierstrass-substitusjonen fra theorem (2.5.3).
x = tan

t
1 x2
2 dx
2x
, cos t =
, og dt =
.
, sin t =
2
2
1+x
1 + x2
1 + x2

Dette gjr at likning (5.27) skrives om til



Z 
1
1
I=
ln (cos t)
sin t e cos t dt.
4
cos t
Videre s
a kan vi la y = cos t dy = sin t dt slik at

Z

Z 
Z y
1 y
1
e
1
ln y
e dy =
ey ln y dy
dy .
I=
4
y
4
y

(5.28)

(5.29)

Dette integralet kan lses via delvis kanselering, avsnitt (2.7.1). Med u = ey
du = ey dy og
Z y
Z
e
dy = ey ln y + ey ln y dy
(5.30)
y
Setter vi n
a likning (5.30) inn i (5.29) kan integralet skrives som
Z


Z
1
1
I=
ey ln y dy ey ln y + ey ln y dy
= ey ln y + C.
4
4
Oppsumert vil integralet v
art dermed bli

 2

Z t
x 1
(1 x2 ) ln(1 + x2 ) + (1 + x2 ) (1 x2 ) ln(1 x2 )
x exp
dx
(1 x4 )(1 + x2 )
x2 + 1
0




1 t2
1
1 t2


= exp
ln
1 + t2 + C
4
1 + t2
vise.
som var det vi nsket a
25. Begynner med a legge merke til at omr
adet en integrerer over kan skrives
om som flger
x2 + y 2 4x 0 (x 2)2 + y 2 22

(5.31)

Alts
a beskriver D en disk med sentrum i (2, 0) og radius 2. Dette kan en og se
tegne x2 + y 2 4x = 0.
fra a

1
x
1
1
2

14.2 Oppgaver

301

Vi kan da beskrive omr


adet via dobbeltintegralet
Z

4xx2

4xx2

arctan exy dy dx

Vi integrerer alts
a frst langs y-aksen fra bunnen av sirkelen til toppen. Deretter
integrer vi langs x-aksen som g
ar fra 0 til 4. Integralet kan n
a deles opp i to
deler
Z 4Z 0
Z 4 Z 4xx2
xy
arctan e dy dx +
arctan exy dy dx

4xx2

bruke substitusjonen y 7 y i det frste integralet f


Ved a
ar en
4

4xx2

arctan e
0

xy

4xx2

arctan exy dy dx

dy dx +

Dette forenkler integranden betraktelig da en kan sl


a sammen integralene og
benytte ??
Z
0

4xx2

arctan exy + arctan exy dy dx =


2

4xx2

dy dx
0

Alts
a at arctan x + arctan 1/x = /2. Integralet beskriver n
a bare arealet av en
halvsirkel med radius 2 slik at arealet blir


ZZ

22
arctan exy dy dx =
= 2
2
2
S
Som nsket. Her ble det brukt at arealet av en halvsirkel er r2 /2. Merk en
kunne og ha beregnet integralet direkte, med noe mer grisete regning.
ZZ

arctan exy dy dx =
2
S

4xx2

dy dx
0

Z
4p
4 (x 2)2 dx
=
2 0
Z
/2 p
=
2 4 4 sin2 u cos u du
2 /2
Z /2
=
cos2 u du = 2
/2

samme som fr. I andre overgang ble substitusjonen x2 7 2 sin u benyttet


med

dx = 2 cos u du. Videre s


a er cos2 x positiv p
a x (/2, /2) slik at cos2 x =
cos x. Integralet over cos2 x er som fr lik halve omr
adet s
a 1 (/2 + /2)/2.
Alternativt
Z /2
Z /2
Z /2


2 cos2 u du =
cos2 u du +
sin2 u du =
+
=
2
2
/2
/2
/2
Siden omr
adet over cos2 x og sin2 x er like store. Dette st
ar det mer om p
a
slutten av avsnitt (2.3).

302

30 Langsvar

28. For enklere notasjon byttes


til , og vi kaller integralet for I. Vi har at
2 cos(/3) = 1 og 2 sin(/2) = 3 slik at



3 cos sin = 2 sin(/2) cos() 2 cos(/3) sin() = 2 sin


3
(5.32)
Hvor identiteten 2 sin(A + B) = cos A sin B + cos B + sin A ble brukt. Via flere
identiter s
a har vi




3 cos sin sin = 2 sin
cos
3





1
2 cos
=
2 sin2
= cos
3
3
2
6
bruke disse frekke omskrivingene f
Ved a
ar vi
Z /3
1/2
I=
( 3 cos sin ) sin
cos d
0

/3 

=
0

1
=
2

1
2 sin2
2

1/2
cos d

/6

1 4 sin2

1/2

/6



cos +
d
6

Hvor likning (5.32) ble brukt i frste overgang og substitusjonen + /6


i siste overgang. Tanken er videre at vi kan bruke cos(A + B) = cos A cos B
sin A sin B p
a siste leddet.


Z /6
1/2
1
3
1
2
I=
1 4 sin ()
cos() sin() d
2
2
2 /6
r Z /6
1/2
1 3
=
1 4 sin2 ()
cos() d
(5.33)
2 2 /6
bruke 1 sin2 = cos2 , da dette vil kanselere
Tanken er n
a at vi nsker a
gjre dette benyttes enkelt og greit sin x 7 21 sin u. Selve
bort rottegnet. For a
integralet blir da
Z /6
Z /2
1/2
1/2
2
1 4 sin ()
cos() d =
1 sin2 ()
cos u du
/6

/2

1
=
2

/2

1
cos (u) du =
4
/2
2

/2

du =
/2

sette inn /4 i likning (5.33) f


Ved a
ar en som nsket
r
r
Z /3
1/2
3
1 3

=
( 3 cos sin ) sin
cos d =
2 2
4
8
2
0
Dette fullfrer beviset.
29. En frekk lsning her blir a benytte to trigonometriske substitusjoner. som
var det som skulle vises.

14.2 Oppgaver

303

benytte to trigonometriske substitusjoner.


30. En frekk lsning her blir a
Z a
dx

I=
2
2
0 x+ a x
la x = a cos u, og dx = a sin u du. Grensene blir s
Ved a
a x = a u = 0 og
x = 0 u = /2. Dermed s
a er
0

Z
I=

/2

1
a sin u du

=
2
2
2
a
a cos u + a a cos u

/2

Z
0

a sin u du
cos u + sin u

Hvor det blant annnet ble benyttet at a2 a2 cos u = a2 (1 cos2 u) = a2 sin2 u.


Videre s
a ble grensene snudd og sin u er positiv fra 0 til /2. Derimot om substitusjonen x = a sin u f
as
Z

/2

I=
0

a cos u du
1
p
=
2
2
2
a
a sin u + a a sin u

/2

a cos u du
cos u + sin u

Hvor dx = a cos u du. Grensene blir s


a x = a u = /2 og x = 0 u = 0. Ved
samme argument som fr, velger den positive roten siden cos u er positv for alle
ta gjennomsnittet av disse to integralene f
u (0, /2). Ved a
as
1
2a

Z
0

/2

a cos u
a sin u
1
+
du =
cos u + sin u
cos u + sin u
2

/2

cos u + sin u

du =
cos u + sin u
4

som var det som skulle vises.


31.
a)

Vi har at for alle a, b s


a er
Z b
Z
f (x) dx =
a

f (a + b x) dx

bruke dette s
Ved a
a er
Z /2
Z /2
Z /2
sin x dx
sin(/2 x) dx
cos x dx
=
=
3
3
3
3
3
sin x + cos x
sin (/2 x) + cos (/2 x)
cos x + sin3 x
0
0
0
Slik at K(1, 3) = K (1, 3). Integralene kan dermed skrives som
K(1, 3) =

K(1, 3) + K (1, 3)
1
=
2
2

Z
0

/2

sin x + cos x
dx
sin3 x + cos3 x

Merk at enten fra pascals trekant eller binomialformelen s


a er
an + bn
= an1 ban2 + . . . bn2 a + bn1
a+b
Slik at
sin3 x + cos3 x
1
= sin2 x sin x cos x + cos2 x = 1 sin 2x
sinx + cos x
2

(5.34)

304

30 Langsvar

sette dette inn i likning (5.34) f


Ved a
as
Z
Z
1 /2
dx
1
dx
K(3, 1) =
=
2 0
2 0 2 sin x
1 21 sin 2x
Der substitusjonen u 7 2x ble brukt i siste overgang4 . Dette integralet kan lses
for eksempel via Weiestrass-subtitusjonen t = tan(x/2) s
a
Z
Z
Z
1
dx
dy
2 dy
1
=
=
2
2 0 2 sin x
2 0 y y+1
(2y 1)2 + 3
0
I andre overgang ble theorem (2.5.3) benyttet og litt opprydning i algebra.
benytte seg av substitusjonen 2y 1 =
Tilslutt
s
a fullfres kvadratet. Ved a

3 tan t blir s
a integralet
Z /2
3 sec t dt

1 

=
+
=
2
2
6
3
/6 3 tan t + 3

Her ble det blant annet brukt at sec x = 1 + tan2 x, arctan(1/ 3 ) = /6 og


arctan x /2 n
ar x .
Z
0

/2

sin x dx
=
sin3 x + cos3 x

/2

Z
0

b) Vi har at for alle a, b s


a er
Z b
Z
f (x) dx =
a

cos x dx
2
=
sin3 x + cos3 x
3 3

f (a + b x) dx

slik at
Z
K(n, m) =
0

/2

sinm (/2 x) dx
= K (n, m)
sinn (/2 x) + cosn (/2 x)

Dermed s
a er K(n, m) = K (n, m) og
K(n, m) + K (n, m)
1
K(n, m) =
=
2
2

/2

sinm x + cosm x
dx
sinn x + cosn x

for alle n, m. Dersom n = m har en at


Z

1 /2 sinn x + cosn x
dx =
K (n, n) = K(n, n) =
2 0
sinn x + cosn x
4
c) Dette integralet kan frst forenkles ved a benytte seg av proposisjon (2.3.2)
eller dette sprsm
alet. Siden K(2014, 2014) er en likefunksjon. Slik at La n
a for
enkehletensskyld
f (x) =
4 Det

sin2n x
sin x + cos2n x
2n

er ikke noe problem a benytte seg av Weiestrass-substitusjonen direkte, men regningen blir noe
se hva jeg mener.
mer grisete. Prv selv for a

14.2 Oppgaver

da er
Z

/2

f (x)
dx =
1 + x

/2

/2

305

f (x)
f (x) x
dx +
dx =
x
1+
1 + x x

/2

f (x) dx
0

Siden f (x) = f (x) da f er en likefunksjon. Dermed s


a er
Z

/2

/2

f (x)
dx =
1 + x

/2

f (x) dx = K(2n, 2n) =


0

Spesielt s
a er
Z

/2

/2

dx

(sin x)2014
=
2014
x
+ (cos x)
1+
4

(sin x)2014

numerere ekvivalensene som flger


32. Velger frst a
(1)
Z

na

ma

log(x a)
dx
x2 + a2

a/n

(2) m

log 2a2
arctan n arctan m
2a

a/m

(3)

log(x + a)
dx
x2 + a2

m (4)
log 2a2
1
1
arctan
arctan
2a
m
n
,

vise (1). Ved a


skrive
Da dette vil gjre utregningen noe ryddigere. Begynner a
om nm = n + m + 1 f
as n = (m + 1)/(m 1). I frste integral, benyttes
substitusjonen u = x/a. S
a du = dx/a.
Z n
Z
Z na
log(ax a)
1 n log a + log(u 1)
log(x a)
dx =
a du =
du
I1 =
2
2
x2 + a2
a m
1 + u2
m (ax) + a
ma
For v
art andre integral benyttes substitusjonen u = a/x, s
a du = a dx/x2 .
2
2
Dette kan skrives om til dx = (x /a) du = a du/u .
Z

a/m

I2 =
a/n

1
a2

log(x + a)
dx
x2 + a2
a/m

a/n

log a + log(1 + x/a)


2

(x/a) + 12

dx =

1
a

log a + log(1 + 1/u)


du
u2 + 1

vise at I1 I2 = 0, addisjon gir


Vi nsker n
aa
Z
1 n log(u + 1) log u log(u 1)
I1 I2 =
du
a m
u2 + 1


u+1
m+1
Z m1
log u1
log u
1
=
du
2
a m
u +1

306

30 Langsvar

Skriver om uttrykket via substitusjonen y = (u + 1)/(u 1), videre s


a er dy =
2 du/(u 1)2 = (y 1)2 dy/2. Mer regning gir da


y+1

Z
log y 
log y1
1 m
dy
2
=


2
m+1
a m1
(y 1)2
y+1
+
1
y1


y+1
m+1
Z m1
log y1
log y
1
=
dy
2
a m
y +1
= (I1 I2 )
Og n
a siden I1 I2 = (I1 I2 ) s
a er I1 I2 = 0 som nsket.
vise ekvivalens (2) n
Velger a
a, alts
a
Z a/n
log(2a2 ) 
1
1
log(x + a)
=
arctan
+ arctan
I2 =
2
2
2
x +a
a
m
n
a/m
faktorisere litt s
Ved a
a kan en se at substitusjonen
x=a

t+a
ta

og

dx =

2a2
dt
(a t)2

utfre litt mer mellomregninger s


Ved a
a er
 2

t + a2
2
2
2
x + a = 2a
(a t)2
Som gjr at uttrykket dx/(x2 + a2 ) = dt/(t2 + a2 ). En annen ting som gjr at
denne substitusjonen ble forskt, er at den er sin egen invers. Ved a sette inn f
as
 2

Z ma
Z na
Z ma
log 2a /(t a)
log(x a) dx
log(2a2 )
I1 =
=
dt =
dt I1
2
2
2
2
x +a
t +a
t2 + a2
na
ma
na
Legg merke til at vi ender opp med v
art opprinnelige integral p
a hyresiden og
lse for I1 forenkles integralet til
ved a
Z
Z
log(2a2 ) na
dt
log(2a2 ) n du
I1 =
=
2
2
2
2
2a
ma (t + a )
m u +1
Der substitusjonen t 7 au ble brukt. Siste integralet er bare den antideriverte
av arctan x, s
a en f
ar
Z ma

log 2a2
log(x a)
dx =
arctan n arctan m
2
2
x +a
2a
na
gange begge sider med
som var det som skulle vises. Vi viser s
a (3), ved a
2a/ log(2a2 ) f
as
arctan n arctan m = arctan

1
1
arctan
m
n

som selvsagt holder da en kan skrive den om som


arctan n + arctan

1
1
= arctan
+ arctan m
n
m

14.2 Oppgaver

307

og b
ade hyre og venstre side er n
a lik /2 fra ??. Alternativt kan en og trekkke
bruke
sammen begge sidene ved a


x+y
arctan x + arctan y = arctan
1 xy
som er sum formelen for tangens, og deretter bruke at nm = n + m + 1 men det
vise (4) g
overlates til lese. For a
ar en frem p
a samm m
ate som i (1)
Z

a/m

I2 =
a/n

log(x + a)
dx
x2 + a2

lse dette integralet s


For a
a kan flgende substitusjon benyttes
x=a

at
a+t

dele integralet og f
Tanken er igjen a
a I2 p
a hyre side. Som fr s
a er
a2

dx
dt
= 2
2
+t
a + t2

og n
ar x = a/m s
a er t = a(1 + m)/(1 + m) = a/n og tilsvarende n
ar x = a/n
s
a er t = a(1 + n)/(1 + n) = 1/m. Endelig s
a kan integralet skrives som
Z

a/m

I2 =
a/n

log(x + a)
dx
x2 + a2

a/n

=
a/m

log[2a2 /(a + t)]


dt =
t2 + a2

a/m

a/n

log(2a2 )
dt I2
t2 + a 2

Dette er en likning som kan lses med hensyn p


a I2 slik at
I2 =

log(2a2 )
2

a/m

a/n

dt
log(2a2 )
=
2
2
t +a
2a

1/m

1/n

du
1 + u2

Via den kjre substitusjonen t 7 au. Integranden er den deriverte av tangens,


slik at en f
ar
Z a/m
log(x + a)
log(2a2 ) 
1
1
dx =
arctan
+ arctan
2
2
x +a
2a
m
n
a/n
som var det som skulle vises. Dette fullfrer den noe lange oppgaven. Merk at
en ogs
a kunne ha brukt theorem (2.9.1) fra begynnelsen, men dette ville kanskje
ikke gitt den samme insikten.

33. Noe problematisk oppgave om en ikke holder tungen bent i munnen. Ved
gange med 4/4 og s
a
a fullfre kvadratet f
as
1
4
4
=
=
x2 + bx + c
(2x + b)2 + 4c b2
(2x + b)2 + (/10)2

308

30 Langsvar

sette inn i
Hvor det i siste overgang ble benyttet at 4c b2 = 2 /100. Ved a
integralet kan en n
a benytte substitusjonen u 7 2x + b s
a du = 2 dx.
Z a/2
dx
I=
2 + bx + c
x
b/2
Z a/2
2 dx
=2
2
2
b/2 (2x + b) + (/10)
Z /10
du
=2
2
u + (/10)2
0


/10
2
x
=
arctan
/10
/10
0
20
[arctan(1) arctan(0)]
=

i
20 h
=
0 =5
4
Hvor det ble benyttet at
Z

x
1
dx
arctan
=
x2 + 2

Som er et kjent integral og kan eksempelvis vises via x = arctan u.


34. Oppgaven er egentlig algebra og elementr faktorisering i forkledning.
P
a samme m
ate som en kan se at
q
q

3 2 2 = 12 2 2 + ( 2 )2 = 1 2 ,
kan en og forenkle rttene i integranden. Faktorisering av kvadratrttene gir
som flger


2

x + 2 2x 4 = ( x 2 )2 + 2 2x 4 + ( 2 )2 =
x2 + 2


2

x 2 2x 4 = ( x 2 )2 2 2x 4 + ( 2 )2 =
x2 2
.
sette dette inn i det ubestemte integralet f
Ved a
as da
r
Z q
q

x + 2 2x 4 + x 2 2x 4 dx
I=
s
r
Z r
2
2

=
x2 + 2
+
x2 2
dx
Z r


=
x 2 + 2 + x 2 2 dx .

Herfra ser en at x 2 2 < 0 n


ar x [2, 4] og positiv n
ar x > 4. S
a




2 x
n
ar x [2, 4]

2
x 2 2 =
x2 2
n
ar x > 4

14.2 Oppgaver

309

Siden |x| = x n
ar x > 0 og |x| = x n
ar x < 0 naturligvis. Ser frst p
a tilfellet
n
ar x [2, 4] da er
Z r

Z q

 

x2 + 2 +
2 x 2 dx =
2 2 dx = 23/4 x + C1

N
ar x > 4 s
a kan integralet skrivet som
r
Z 
Z
 


4
x2 + 2 +
x 2 2 dx =
2 (x 2)1/4 dx =
2 (x 2)5/4 + C2
5
For at funksjonen skal vre kontinerlig s
a m
a konstantene vre like i punktet
x = 4 s
a

8 3/4
8 2 
4
8 2 + C1 = 2 + C2 C2 =
2 5 + C1
5
5
legge sammen alt dette f
Ved a
as endelig
r
Z q
q

x + 2 2x 4 + x 2 2x 4 dx
I=


42 (x 2)5/4
ar x 4
 /5 + C1 n
=
23/4 x + 8 2 4 2 5 /5 + C1 n
ar x (2, 4)
beregne. Det bestemte integralet blir s
som var det en nsket a
a
Z 4 rq
Z 4
q

x + 2 2x 4 + x 2 2x 4 dx =
23/4 x dx = 27/4
2

Som er et stt lite svar p


a et stort stygt beist.

35.
k)


Legg merke til at vi har x2 x = x ( x )3 1 . Ved a bruke at a3 1 =

(a 1)(a2 + a + 1) s
a kan vi gjre flgende omskrivning

.

 3
x +1
1
x +1

x
(
x
)

1
x x +x+ x
x2 x
x x +x+ x

x +1
( x 1) x x + x + 1
=
x x +x+ x


Ved a bruke at x x + x + 1 = x x + x + x forenkles n


a integralet ned til
Z

Z
.



x +1
1

dx =
x +1
x 1 dx
x x +x+ x
x2 x
Z
1
x(x 2)
= x 1 dx = x2 x =
+C
2
2

310

30 Langsvar

l) Helt tilsvarende som i forrige deloppgave f


ar vi
Z
a

Z b
.


x
1

x
x 1 dx
dx =
x x +x+ x
x2 x
a
 
b
Z b

2
1
=
x x dx = x
x
x
2
3
a
a

Vi ser at uttrykket i parentesen er null n


ar x = 0, en mulighet er alts
a at a = 0
eller b = 0. Den andre muligheten for at uttrykket i klammene er null, da er
2
4
16
1
1 2
x
x x=0 x
=0
x =0
2
3
4
9
9
s
a x = 16/9. Alts
a er
Z

16/9

.
x
1

dx = 0
2
x x +x+ x
x x

og a = 0, b = 16/9 siden a > b.

36. Dette er en velse i delvis integrasjon. Telleren hinter om at delvis


velge
integrasjon vil hjelpe med a
v=

1
.
x arctan x

(5.35)

For at dette skal g


a m
a det finnes to funksjoner u og dv slik at

u dv =

arctan x
x arctan x

2
(5.36)

finne. Ved a
derivere likning (5.35) f
Disse er heldigvis ikke vanskelige a
as en
mulig dv, og ved a dele denne funksjonen p
a begge sider av likning (5.36) f
as u
dv =

x2
1
1 + x2
og
u
=

(arctan x)2 .
1 + x2 (x arctan x)2
x2

Overraskende nok s
a blir ogs
a u0 relativt pen
u0 =

2 arctan x
)(arctan x x) .
x3

bruke den kjre formelen for delvis integrasjon


Ved a
Z
Z
u dv = uv u0 v ,
kan integralet I skrives som
Z 

arctan x
x arctan x

2
dx =

(1 + x2 )(arctan x)2
+
x2 (x arctan x)

2 arctan x
dx . (5.37)
x3

14.2 Oppgaver

311

sette u = arctan x og dv = 2x3 s


Det siste integralet kan igjen lses ved a
a
Z
Z
arctan x
2 arctan x
dx
dx =
+
3
2
2
x
x
x (1 + x2 )
Z
Z
arctan x
dx
dx
=
+

x2
x2
1 + x2
arctan x
1
=

arctan x + C .
2
x
x
Delbrksoppspaltingen kan for eksempel bli sett ved a legge til og trekke fra x2 i
teller. For a f
a det endelige svaret er resten bare faktorisering. La oss frst skrive
gange med x arctan x i teller og nevner
om det siste integralet, ved a


Z
arctan x
1
2 arctan x
x arctan x
dx =

arctan x
x3
x2
x
x arctan x
.

arctan x2
2
+ (arctan x) 1 x arctan x
(x arctan x)
=
x2
(1 + x2 )(arctan x)2
1 + x arctan x
=

.
2
x (x arctan x)
x arctan x
sette dette uttrykket
Hvor konstanten ble droppet av praktiske grunner. Ved a
for integralet inn i likning (5.37) f
ar en endelig at
2
Z 
arctan x
1 + x arctan x
+C.
(5.38)
dx =
x arctan x
arctan x x
vise. Ved a
snu likning (2.81) p

som er nesten det vi nsket a


a hodet, alts
aa
opphye begge sider i 1 s
a er sumformlene for tangens er gitt som
1
1 + tan a tan b
=
.
tan(a b)
tan a tan b
For at hyresiden i denne likningen skal vre lik hyresiden av likning (5.38) s
a
m
a vi ha at tan a = x og at tan b = arctan x. Dette gir som nsket
2
Z 
1
arctan x
dx =
,
x arctan x
tan( tan )
hvor x = tan tan eller = arctan(arctan x).
37. Teknikken en bruker her blir selvsagt delvis integrasjon. Vi velger her
u=x
u0 = 1
Slik at en f
ar
Z


x

v 0 = f 00 (2x)
1
v = f 0 (2x)
2

0
hx
i1 Z 1 1
1 0
0
f (2x) dx =
f (2x)
f 0 (2x) dx
2
2
0
0 2

1
1
1
= f 0 (2)
f (2x)
2
4
0

1 0
1
= f (2)
f (2) f (0) = 2
2
4

312

30 Langsvar

I siste linje ble det brukt at f (0) = 1 , f (2) = 3 og f 0 (2) = 5.


39. At f /g har et kritisk punkt betyr at den deriverte er null, alts
a
 
d
f
f 0 g g0 f
= 0,
=
dx
g
g2
for x = 5 og x = 7. Siden g 2 > 0 medfrer dette at
f 0 g g0 f = 0 ,

for x = 5, 7 .

(5.39)

Dette blir lagt litt i bakhodet, mens en tar fatt p


a selve integralet. For enkelhetens
skyld blir det videre skrevet f (x) = f og g 00 (x) = g. Frst benyttes delvis
integrasjon to ganger som gir
Z

h i7 Z
f g 00 dx = f g 0
5

h
i7 Z
f 0 g 0 dx = f 0 g g 0 f +
5

f 00 g dx

benytte (iii) og (iv) s


En kan skrive om f 00 g ved a
a
(iv)

z
}|
{ z(iii)
}| {


2
f 00 g = 2g 00 f g 00
1/g 00 = 2 f g 00


Innsetning gir n
a at
Z

h
i7 Z
f g 00 dx = f 0 g g 0 f +
5

f g 00 dx = 0 +

2x f g 00 dx

5
7
00

f g dx = 2

2
5

f 00 g dx

x dx
5

f g 00 dx = 2

I frste overgang ble likning (5.39) benyttet og at f 00 g = 2 f g 00 .


40.
Legg merke til at arccos(x) bare er definert for x [1, 1] og siden en

har x , s
a m
a 0 a b 1. Da integralet v
art er positivt, og en nsker a
maksimere I, m
a en la integralet v
art g
a over det maksimale intervallet.
Alts
a
er

a = 0 og b = 1. Vi har at sin(u)2 + cos(u)2 = 1, la n


a u = arccos( x ) da har en

2 
2
sin arccos x
+ cos arccos x
=1

2
sin arccos x
=1x
Alts
a er
Z
I=
0


2
sin arccos x
dx =

Z
0


1
1
1
1 x = x x2 =
2
2
0

14.2 Oppgaver

313

se at
41. Det er ikke videre vanskelig a

2
g(x)2 = sin(log x) + cos(log x) = 1 + sin(2 log x)
Hvor en i siste uttrykk benyttet seg av sin2 x+cos2 x = 1 og sin(2x) = 2 cos x sin x.
vise at
Vi nsker alts
aa
Z 1
Z 1
cos2 (log x) dx =
1 + sin(2 log x) dx
0

stadfeste denne likheten benytter en seg av delvis integrasjon med u =


For a
cos2 (log x) og v 0 = 1
1

f (x)2 dx =

cos2 (log x) dx

h
i1 Z
= x cos(log x)
0

1
sin(2 log x) dx
x

sin(2 log x) dx

=1+
0
1

1 + sin(2 log x) dx

=
0
1

g(x)2 dx

=
0

vise. Legg for ordens skyld merke til at det ikke var
Som var det en nsket a
vise likheten. For a
fylle p
ndvendig og beregne integralene for a
a litt mer
detaljer: I den delvise integrasjonen ble det benyttet at v 0 = 1, v = x og
u0 =

2
d  2
2
1
cos (log x) =
cos(log x)[ sin(log x)] = log(2 sin x)
dx
x
x

og grenseverdien kan eksempelvis vises slik


h
i1
L = lim x cos(log x) = 1 lim n cos(log n)
n0

n0

vise den siste grensen p


Siden 1 cos(log 1) = cos 0 = 1. Det er flere m
ater a
a. En
legge merke til at limx0 xf (x) = 0, dersom det eksisterer en K
metode er a
tenke p
slik at |f (x)| K. En m
ate a
a dette som er at om f ikke vokser mot
uendelig, s
a vil x fr eller siden dra grensen mot null. I dette tilfellet er K = 1,
siden cos x oscillerer mellom 1 og 1 for alle x.
Alternativt s
a kan en definere L = x sin(log x) og R = x cos(log x). Da er L2 0
og R2 0 for alle x. Videre s
a er L2 + R2 = x2 , slik at n
ar x g
ar mot null s
a m
a
L og R og g
a mot null.
bruke substitisjonen x 7 1 y, alts
a at
R a 42. Det frste
R a steget vi gjr blir a
f
(x)
dx
=
f
(a

x)
0
0
Z


(1 y) log

J=
0





Z 1
2+ 1y
2+ x

x log
dx =
dx
2 x
2 1y
0

314

30 Langsvar

Vi kan s
a dele J inn i flgende to integral
1

Z
J=


x log 2 + x dx


x log 2 x dx

Vi kan bruke substitusjonen y 7 2 + x p


a frste integralet og y 7 2 + x p
a
andre integralet slik at
Z

3
3

2(y 2) log y dy

J=
2

2(y 2)3 log y dy

dette medfrer som nsket at





Z 1
Z 3
2+ 1y

(1 y)
dy = 2
(y 2)3 log y dy
2

y
0
1
beregne integralet n
A
a er ikke spesielt vanskelig.
44.
Lsning 1 Det fine med dette integralet er at det kan lses ved hjelp av en
frst bruke
serie mer eller mindre normale substitusjoner. Vi velger her a
u = x2 du = 2x dx
Z
x29
I=
17 dx
(5x2 + 49)
0
14
Z
x2
1
=
2x dx
2 0 (5x2 + 49)17
Z
1
u14
=
du
2 0 (5u + 49)17
forenkle nevner, og bruker derfor substitusjonen
Videre nsker en a
49
49
t du =
dt
5
5



Z
14
49
1
49
5 t
=
dt

17
2 0
5
5 49
5 t + 49

14 
Z
1 49
49
t14
=
dt
17
2
5
5
49 (t + 1)17
0
Z
1
t14
=
dt
15
2
2 5 49 0 (t + 1)17

u=

ta det igjen. Vi skriver


Det siste integralet har en vrt borte i fr, men velger a
om

14
t14
t
1
=
(t + 1)17
t+1
(t + 1)3

14.2 Oppgaver

Vi lar y =

315

t
1
og legger merke til at 1 y =
s
a
t+1
1+t
t
t+1

1
dy
1
dy =
dt
=
dt
(t + 1)2
(t + 1)2
Z
t14
1
dt
=
15
2
2 5 49 0 (t + 1)17

y=

=
=

1
2

515

x29
(5x2 + 49)

17

492

1y
dy
}|
{
}|
{
14 z }| { z
1
1
t
dt
t+1
t+1
(t + 1)2

1
515

492

z


y 14 (1 y) dy

1 15
1 16
y
y
15
16

2 515 492
1
=
2 515 492 15 16
14!
dx =
15
2 5 492 16!

1
dy
0

vise =)
Som var det en nsket a
beregne T (1) frst, da har en
45. Det enkleste er a
T (1) =


h
f (a) + f (b)
f (a) + f (b) + 0 = (b a)
.
2
2

Siden f er kontinuerlig og integralet er begrenset betyr dette at T (n) konvergerer


mot A. S
a
Z b
f (x) dx = lim T (`)
`

La oss regne ut hyresiden eksplisitt. Ved a sette inn uttrykket for trapesmetoden
f
as
!
n1
X

h
lim T (n) = lim
f (a) + f (b) + h
f (xk )
n
n
2
k=1

= lim

= lim

ba
n

n1
X

f (xk )

k=1

n1
ba X
f (xk ) + f (xnk )
2n
k=1

I siste overgang legger vi sammen siste elementet med frste, nest siste med
andre osv. Virker dette ulogisk, s
a bare skriv ut et par ledd og sammenlign. En
har ogs
a at


a+b
f (x) + f (a + b x) = f (a) + f (b) = 2f
(5.40)
2

316

30 Langsvar

sette x = a eller
Siden f (x) + f (a + b x) er konstant for alle x [a, b]. Ved a
sette x = (a + b)/2 f
bruke
x = b f
as frste likhet, og ved a
as andre. M
alet er a
forenkle summen, dette kan bli gjort ved a
skrive om
dette for a

ba
(n k)
n

ba
=a+b a+
k
n

xnk = a +

= a + b xk

bruke dette i summen v


Ved a
ar f
ar en n
a at

lim T (n) = lim

= lim

= lim

n1
ba X
f (xk ) + f (xnk )
2n
k=1

ba
2n

n1
X

f (xk ) + f (a + b xk )

k=1

n1
ba X
f (a) + f (b)
2n
k=1

n1
1 X
ba
(f (a) + f (b)) lim
1
n n
2
k=1


f (a) + f (b)
= (b a)
2

Der frste del av likning (5.40) ble benyttet. Videre ble f (a) + f (b) satt utenfor
summen siden dette er en konstant. Tilslutt s
a ble grensen beregnet som flger



n1

1
1 X
1
1
lim
1 = lim
1 + + 1 = lim
(n 1) = lim 1 +
{z }
n n
n n |
n n
n
n
k=1

n1 ganger

som g
ar mot 1 n
ar n . Dette viser punkt 1 og punkt 3 siden

f (x) dx = lim T (`) = T (1)


a

da gjennst
ar det bare a vise punkt 2. Merk at selv om T (n) konvergerer mot T (1)
kan det vre at T (10) > T (1), T (15) < T (1) osv. For a vise det for alle k kan en

14.2 Oppgaver

317

enten benytte seg av induksjon, eller eksplisitt regning


n1

T (n) =

X

h
f (a) + f (b) + h
f (xk )
2
k=1

n1

h X
h
f (a) + f (b) +
f (xk ) + f (x2n1k )
2
2
k=1

n1

h
h X
=
f (a) + f (b) +
f (xk ) + f (a + b xk )
2
2
k=1


h
h
=
f (a) + f (b) + (n 1)[f (a) + f (b)]
2
2



hn 
f (a) + f (b)
=
f (a) + f (b) = (b a)
2
2
Siden h = (b a)/n s
a hn = b a. Videre ble de fleste triksene fra forrige del
benyttet.
bruke antagelsene kan en skrive om som flger
46. Ved a
dp
S
n lf /T
=
=
dt
V
Vg
Deler en n
a begge sider av likningen p
a p f
as
n lf /T
1 dp
=
p dT
pVg

n lf /T
Lf
d 
log p =
= 2
dT
nRT
T R
som nsket.
b) Ved innsetning f
as n
a

p0


d 
T
log p =
dT
T 2R
Z T



d
1

dT
log p dT =
2
dT
R T0 T
T





p
1
T
1
1
log
=
log
+

p0
R
T0
T
T0
(

/R )
 

T
1
1
p = p0 exp log
exp

T0
T0
T

/R



T0

1
1
p(T ) = p0
exp

T
R T
T0

se at b
47. a) Frste metode er a
ade x og ex er strengt voksende funksjoner,
x
slik at xe ogs
a m
a vre strengt voksende. Alts
a er f (x) = xex injektiv. Siden

318

30 Langsvar

x og ex kontinuerlige, s
a er f kontinuerlig. Tilslutt legges det merke til at
f (0) = 0e0 = 0, og f (1) = e > 2. S
a dermed finnes det en unik konstant
0 < < 1 slik at f () = 1. P
a bakgrunn av at funksjonen er kontinuerlig,
strengt voksende.
Alternativt anta at likningen xex = 1 har to lsninger 1 og 2 . Siden n
a
e = 1/x s
a er
x

e1 = e2

og

1
1
=
1
2

og begge likningene gir at 1 = 2 , s


a igjen s
a er unikt definert.
lse likningen f = g, da f
b) nsker a
as
x = x2 log x 0 = x(1 x log x)
Slik at enten s
a er x = 0 eller s
a er x log x = 1, en ser videre p
a siste lsning.
1 = x log x
1/x = log x
e1/x = elog x
1 1/x
e
=1
x
Sammenlikner en n
a med definisjonen av konstanten ser en at
x=

1
x

= s
a

1
= e

Hvor den siste likheten ble tatt fra definisjonen av Omega, alts
a e = 1

e = 1/.
beregne integralet
c) Her nsker en a
Z

exp()

A=

1 dy dx
f

exp()

x x2 log x dx
)

exp() (
exp() Z exp()
1 2
1 3
1 3 1
=
x

x log x

x
dx
2
3
3
x
0
0
0
=

1 2
1
e e3 log e +
2
3
1 2
1
= e e2 e +
2
3
1 2
1
= e + e3
6
9
1
1
=
+
2
6
93
=

1 3
e
9
1 3
e
9

14.2 Oppgaver

319

vise. I siste overgang ble igjen egenskapen at e = 1


som var det en nsket a
b+c
benyttet, og at a
= ab ac .
1+0
1+1

d) Frste iterasjon med 0 = 0 gir seg 1 =


gir 2 =

1+1/2
1+e1/2

1
2

. Den neste iterasjonen

. Slik at

2
1
=
1+ e
2
3

Innsatt i nederste uttrykk for arealet f


as da
1
1
+
2
6
93
 2 

1
1
1 1
=
+

6
9

2 



1 2
2
1
1+ e
1+ e
+

3
6
9 3


2 13
8
1+ e
+ e

243
4

A=

Dette er bare en halvgod tilnrming da 2 e2 6= 1 alts


a stemmer ikke tilnrmingen som ble benyttet i c). konstanten lengre. En kan bruke et tidligere
uttrykk for arealet, for hyere nyaktighet. Men det f
ar vre opp til leser.
Benytter i stedet initialverdien 0 = log 2 f
as
1 =

1
(1 + log 2)
3


og

exp (1 ) = exp

1
log 2e
3


=

2e

Setter en inn dette i det andre uttrykket en har for arealet f


as


1
1 1
1
1
+
A = e2 + e3 = e3
6
9
6 e
9




1
1
1
1
1/3

= 2e
(2e)
+
+
= 2e
9
6
9
6 3 2e
Benytter en seg n
a av tilnrmingen e 8/3 f
as
23 1
=

3
3

24
3

1/3

8 1
+

=
3 9

3
2

5/3
+

16
27

For en endelig tilnrming kan en se at (3/2)5 = 32/243 30/240 = 1/8 s


a
(3/2)5/3 1/2 og da blir A = 1 + 5/54.

48.
x2 + yey = 1 ,

y > 1

(5.41)

320

30 Langsvar

Velger frst benytte delvis integrasjon slik at integralet kan skrives som

b
Z b
Z
1 2
1 b 2 0
xf (x) dx =
x f (x) dx
x f (x)
2
2 a
a
a
sette inn henholdsvisx = a og x = b i likning (5.41) og lse med hensyn
Ved a
p
a yey , kan b
ade f (a) og f (b) bestemmes. Dette gir
1
1
log .
2
2
bruke
Slik at f (a) = 0 og f (b) = log 1/2, som kan sjekkes via innsetning. Ved a
den anbefalte substitusjonen u = ef (x) s
a er f (x) = log u, og
f (a)ef (a) = 0 og

f (b)ef (b) =

du = f 0 (x)ef (x) dx

du
= f 0 (x) dx .
u

bruke
Tilslutt s
a kan x2 skrives som x2 = 1 f (x)ef (x) = 1 u log u, ved a
likning (5.41). Integralet kan n
a skrives som

b
Z
1 2
1 b 2 0
I=
x f (x)
x f (x) dx
2
2 a
a
Z
1
1 exp(f (b))
du
= b2 f (b)
(1 u log u)
2
2 exp(f (a))
u
h
i
1/2
1
1
(u 1) log(u) u
= b2 f (b) +
2 
2

 1

1
1
1 1
1
=
1 + log 2 log 2
log 2 (1)
2
2
2 2
2
1
1
= log(2) (1 + log 2) +
4
4
Som var det som skulle vises. Her er B = 1/4 og A = B.
49. a) Frste del s
a har en at
V (r) =

2Ze2
2Ze2 r2
r2
=
=
E
40 r
40 r2 r
r

som nsket. Videre s


a vil integralet kunne skrives som


Z q


2 r2
T
exp

2m
V
(r)

E
dr
=
} r1
Z r2 r
i
hr
2
2
log T
E E dr
2m
=
} r1
r
Z r2 r

r2
2 2mE
log T =
1 dr
}
r
r1
Herfra benyttes substitusjonen r = r2 x s
a dr = r2 dx.
r
Z
1
2
1
log T

2mE
r
1 dx
=
2
}
x
r1 /r2
som nsket.

17.2 Oppgaver

321

3.3.2
5. Via substitusjonen y 7 x + t kan integralet skrives p
a formen
Z t+1
Z 1
log (y) dy
log (x + t) dx =
t

bruke
For enkelhet byttes det n
a tilbake til x som integrasjonsvariabel. Ved a
likning (3.19) kan den deriverte med hensyn p
a t skrives som
Z t+1

d
d
log (y) dy = log (1 + t) (t + 1) log (t) t
t t
dt
dt
Ved n
a og anvende x(x) = (x + 1) kan den deriverte forenkles til
Z t+1


log (y) dy = log (t + 1)/(t) = log t


t t
derivere hyresiden f
Tilsvarende ved a
as
Z t
i
h

log x dx =
t log t 1 = log t
t 0
t
Siden de deriverte er like m
a f og g avike med hyst en konstant. Anta at f og
g aviker med mer en enn konstant s
a f kan skrives som f (x) = g(x) + h(x) hvor
h0 (x) 6= 0. Derivasjon gir at f 0 (x) = g 0 (x) + h0 (x), som er en motsigelse siden
f 0 (x) = g 0 (x), dermed s
a m
a h0 (x) = 0 og h = C.
bestemme konstanten kan vi se p
For a
a grensetilfellet n
ar t 0. Da er
Z 1
Z t
lim
log (x + t) dx = C + lim
log x dx
t0

t0

Integralet p
a hyre side g
ar mot null, og p
a venstre side st
ar en igjen med det
klassiskelog (x) integralet som ble vist i proposisjon (3.3.9). Dermed s
a er
C = log 2 og
Z 1
Z t

log (x + t) dx = log 2 +
log x dx
0

som nsket.
La oss n
a ta utgangspunkt i Raabes formel, en liten omskrivning gir
Z u+1

log (x) dx = u log u u + log 2

(5.42)

som er formen som vil bli vist her. Ved a dele opp integralet kan det skrives som
Z u+1
Z u+1
Z u
log (x) dx =
log (x) dx
log (x) dx
u

0
u+1

log (x) dx +
log (x) dx
log (x) dx
0
1
0
Z
Z
u
u

= log 2 +
log (x + 1) dx
log (x) dx.

322

30 Langsvar

Hvor det i siste overgang ble brukt proposisjon (3.3.9) og i midterste integralet
ble substitusjonen x 7 y + 1 brukt. Legg n
a merke til at siden x(x) = (x + 1)
s
a er x = (x + 1)/(x). Ved innsetning s
a er
Z

u+1

Z0 u

log (x) dx = log 2 +


u

= log 2 +


log (x + 1)/(x) dx

log x dx = u log u u + log 2

trekke fra logaritmen p


Ved n
aa
a begge sider f
ar en som nsket Raabes formel.

3.3.3

benytte seg av substitusjonen som fr. Her ble korol1. Det enkleste blir a
lar (3.3.3) brukt i andre overgang. i andre overgang.
benytte seg av substitusjonen t 7 ex da f
2. Det enkleste blir a
as
Z
Z
n1
t

enx
dx
=
dx = B(n, 1 n) = (n)(1 n) =
x
1
+
e
1
+
t
sin
n

0
Her ble Eulers refleksjonsformel theorem (3.3.4) brukt i siste overgang. Alternativt kan og substitusjonenu 7 ex benyttes, da er
Z
Z
1
1 /(1/n)

enx
du
dx =
=
=
x
1/n
1
+
e
n
n
sin
n
sin
n
1
+
u

0
som fr. Her ble korollar (3.3.3) brukt i andre overgang. i andre overgang.
definere flgende funksjon
3. Vi velger a
(x) :=

B(x, y)(x + y)
(y)

vise at (x) = (x), ved a


benytte seg av Bohrog m
alet med beviset blir a
mullerup teoremet. Vi g
ar igjennom punkt for punkt 1. (1) = 1.
Her har vi fra gammafunksjonen at (1 + y) = y(y). For betafunksjonen f
as
Z
B(1, y) =

t11 (1 t)y1 dt =

uy1 du =

uy
y

Ved innsetning f
as n
a at
(1) =

B(1, y)(1 + y)
=
(y)

1
y

y(y)
(y)

=1

1
=
0

1
y

17.3 Oppgaver

323

som nsket. Videre s


a m
a vi vise at 2. (x + 1) = x(x). Ved a ta utgangspunkt
i B(x + 1, y) og delvis integrasjon f
as
Z 1
B(x + 1, y) =
tx (1 t)y1 dt
0
x
Z 1
t
=
(1 t)x+y1 dt
1

t
0
0 Z 1
 x
(1 t)x+y
xtx1
t (1 t)y

=
x+y
x+y
(1 t)x+1
0
1
Z 1
x
=
tx1 (1 t)y1 dt
x+y 0
x
=
B(x, y)
x+y
Ved innsetning ser vi n
a at
(x + 1) =

(x + y)(x + y)
x
(x + y + 1)
B(x + 1, y) =

B(x, y) = x(x)
(y)
(y)
x+y

vise at log er konveks. Ved a


bruke blant annet lemVi mangler n
a bare a
ma (3.3.1) som fr. La p (0, ) og velg q slik at p1 + 1q = 1. Da har vi
at






up + vq
u
v
u
v

+
=
B
+ ,y
p
q
(y)
p
q
For betafunksjonen har vi at

 Z 1
u
v
B
+ ,y =
t(u/p+v/q1 (1 t)y1 dx
p
q
0
Z 1
u
1/p  v
1/q
=
t (1 t)y1
t (1 t)y1
dt
0

Z

1/p Z

1
u

t (1 t)

y1

y1

t (1 t)

dt

1/q

dt

= B(u, y)1/p B(v, y)1/q


gjennomfre akkuratt den samme utregningen f
Ved a
as ulikheten


v
u
+
+ y (u + y)1/p (v + y)1/p

p
q
Dermed f
ar vi at flgende ulikhet gjelder


1/p
1/q
v
{(u + y)B(u, y)} {(v + y)B(v, y)}
u
+

p
q
(y)

1/p 
1/q
(u + v)
(v + v)
=
B(u, y)
B(v, y)
(y)
(y)

1/p 
1/q
= (u)
(v)

324

30 Langsvar

Dersom vi n
a lar = 1/p og dermed 1 = 1/q. Hvor (0, 1) f
ar vi

u + (1 )v (u) (v)1

log u + (1 )v log (u) + (1 ) log (v)
for alle par u, v (0, ). Dermed s
a er log er konveks og da flger det fra
BohrMollerup theoremet at (x) = (x), flgelig s
a er
(x) =

B(x, y)(x + y)
(x)(y)
(x)(y)
B(x, y) =
=
(y)
(x + y)
(x + y)

vise.
som var det vi nsket a
3.3.4
3.3.9
1.
Bevis. Gjennom hele beviset antas det at vi deriverer med hensyn p
a x uten tap
av generalitet. og et identitsk bevis kan fres hvor en deriverer med hensyn p
a
y. Her har vi frst at
1xy
1xy
1xy
1u=
,
1v =
,
1 uv =
1y
1x
(1 x)(1 y)
derivere polylogaritmen f
Ved a
as
[Li2 (f )]0 = L02 (f ) f 0 =

log(1 f )
f 0 = log(1 f ) log0 (f )
f

der det ble brukt at f var en kontinuerlig funksjon. Den deriverte av polylogaritmen fikk vi fra analysens fundamentalteorem. N
a regner vi frst ut de enkle
logaritmisk deriverte s
a
1
1
,
log0 (v) =
, og
x
1x
For klarhet regner vi ut `en og `en derivert da er
log0 (u) =

log0 (u v) =

1
1
+
x
1x

log(1 x)
x
log(y x)
= Li2 (y) =
0=0
y
= Li2 (x) =

log(1 x y) log(1 y)
x
log(1 x y) log(1 x)
= Li2 (u) = log(1 u) log(u)x =
1x
= Li2 (u) = log(1 u) log(u)x =

= Li2 (uv) = log(1 uv)(log u + log v)0




1
1
+
x
1x

log(1 y)
=
Li2 (u) +
Li2 (v)
Li2 (x)
.
x
x
x
1x


= log(1 x y) log(1 x) log(1 y)

18.1 Oppgaver

325

Addisjon gir oss n


a endelig at den deriverte kan skrives som
=
=

(Li2 (u) + Li2 (v) Li2 (uv) Li2 (x) Li2 (y))
x

log(1 y)
1x

Deriverer vi n
a venstresiden ser vi at

log(1 y)
log(1 x) log(1 y) =
x
1x
slik at hyre og venstre side i likning (3.35) er ulike med hyst en konstant C.
Alts
a
log(1 x) log(1 y) = Li2 (u) + Li2 (v) Li2 (x) Li2 (y) Li2 (uv) + C ,
settes x = 0, y = 0 s
a er u = 0 og v = 0 slik at
log2 (1) = Li2 (0) + Li2 (0) Li2 (0 0) Li2 (0) Li2 (0) + C
som medfrer at C = 0. Dette fullfrer beviset.

2. Velger a vise frst Landens identitet. Derivasjon med hensyn p


a Li2 [z/(z 1)]
gir
d
Li2
dz

z
z1

z/(z1)

log(1 t)
dt
t
0



z
log 1 z1
1

= log(1 z) + log(1 z)
=
(z 1)2
z/(z 1)
1z
z
d
=
dz

Integrasjon gir n
a direkte at


1
z
Li2
= log2 (1 z) Li2 (z) + C
z1
2
sette z = 0 s
la z = z f
konstanten bestemmes ved a
a C = 0. Ved a
as


1
z
= log2 (z + 1)
Li2 (z) + Li2
1+z
2
som vi kjenner igjen som Landens identitet.

1. Det vi nsker er a dele inn funksjonen i perioder p


a T siden f (x+T ) = f (x)
for alle x. Vi har alts
a
Z
Z (n+1)T
X
st
Lf (s) =
e f (t) dt =
est f (t) dt
0

n=0

nT

326

30 Langsvar

bruke substitusjonen t 7 nT + w har en


Ved a
Lf (s) =

Z
X
n=0

s(nT +w)

f (w + nT ) dw =

nT s

n=0

esw f (w) du

se at f (x + nT ) = f (x) kan en for eksempel bruke samme induksjonsarFor a

gument som i beviset for theorem (2.4.1) p


a side
(32). Beviset fullfres ved a
P
bruke sumformelen for den geometriske rekken n=0 xn = 1/(1 x)
!Z
Z T

T
X

1
n
T s
sw
f (t)est dt
Lf (s) =
e
e
f (w) du =
1 eT s 0
0
n=0
som var det som skulle vises.

sette inn definisjonen av L -transformasjonen f


2. Ved a
as
Z t
 Z Z t

Lf (s) = L
f (w) dw =
f (w) dw est dt
0

bruke delvis integrasjon, med


Tanken er n
aa
Z
du = f (t) dt

u=

f (w) dw
0

dv = est dt

v=

1 st
e
s

Dette frer til at integralet blir



0
Z t
Z 
est
est
Lf (s) =
f (t) dt
f (w) dw

s
s
0
0



Z
Z t
1
1 st
=
lim est
f (w) dw +
e f (t) dt
s t
s 0
0


R0
Siden 0 f (w) dw = 0 s
a forsvinner frste del. N
ar t s
a vil est dominere
integralet s
afremt s > 0. Dermed vil grensen g
a mot null for alle s hvor L transformasjonen av f er definert. Siste integralet kjenner vi igjenn og vi f
ar
Z t

1
Lf (s) = L
f (w) dw = 0 + F (s)
s
0
som var det som skulle vises.
3.6

3.7

24.1 Oppgaver

327

3.8

3.10.1

1. La I betegne integralet og benytt substitusjonen x = 1 u, s


a dx = du s
a
1/2

Z
I :=
0

log(x) log(1 x)
dx =
x(1 x)

1/2

log(1 u) log(u)
du .
(1 u)u

Ved n
a og ta gjennomsnittet av disse integralene f
as
I=

1
2

Z
0

log(x) log(1 x)
dx .
x(1 x)

En kan n
a eksemeplvis legge merke til at
Z
0

log(1 x) log x
dx =
x

Z
0

log(1 x) log x
dx ,
1x

(5.43)

Rb
Rb
bruke at a f (x) dx = a f (a + b x) dx, eller substitusjonen
ved enten a
1 x = u. Uansett vender vi tilbake til v
art opprinnelige integral, og bruker
delbrkoppspalting f
as


Z
Z
1 1 log(x) log(1 x)
1 1
1
1
dx =
+
log(x) log(1 x)
dx
2 0
x(1 x)
2 0
x
1x
Z 1
log(x) log(1 x)
=
dx
x
0

I=

lse
hvor vi benyttet oss av likning (5.43). N
a er integralet relativt simpelt a
Enten kan vi huske p
a definisjonen av dilogaritmen Li2 (z) og benytte oss av
delvis integrasjon Dette gir at
h
i1 Z
I = log x Li2 (x) +
0

X
Li2 (t)
1
dt = Li3 (1) =
= (3)
t
k3
n=1

finne. I den delvise integrasjonen ble v = Li2 (z) og


Som var det vi nsket a
u = log(x) brukt videre s
a ble den rekursive definisjonen av polylogaritmen
benyttet og at Lin (1) = (n). Alternativt kan en og lse siste integral uten bruk

328

30 Langsvar

av dilogaritmer
1

log(x) log(1 x)
dx
x
0


Z 1
log x
x2
x3
x+
=
+
+ dx
x
2
x
0


Z 1
x
x2
log x 1 +
=
+
+ dx
2
3
0

Z 1

X
1
log(x)xn dx
=
n+1 0
n=0




X
1
1
=

n+1
(n + 1)2
n=0
Z

I=

X
n=0

1
(n + 1)3

hvor vi kjenner igjen siste sum som Apierys konstant (3).

2. Vi bruker substitusjonen x = t1 slik at


Z

1/2

1 e1/x dx =

1 et
dt
t2

Herfra defineres flgende funksjon


Z

I(a) =
0

1 eax
dx
x2

hvor vi legger merke til at I(1) er integralet vi nsker. Derivasjon under integraltegnet gir
I 0 (a) =

d
da
Z

=
Z0

Z
0

1 eax
dx
x2
2

eax
dx
a x2
2

eax dx
0
Z
2
1
ey dy
=
a
r 0
1
=
2 a
=

Hvor i siste overgang ble det klassiske gaussiske integralet benyttet. Et bevis for
dette finnes flere steder i heftet, blant annet ??. Integrasjon gir n
a at

I(a) = a + C

24.1 Oppgaver

329

siden I(0) = 0 er konstanten null, og vi f


ar dermed at
1/2

2
1 e1/x dx = I(1) =

Og dette fullfrer oppgaven.


3. Integralet her kan regnes ut utelkukkende ved hjelp av en serie elemen komme p
tre substitusjoner og omskrivninger. Likevell er det a
a disse s
apass
krevende at integralet havner i denne delen. Frste steg er a dele opp intervalet
(0, ) til (0, 1) og (1, ). Deretter mappes (1, ) p
a (0, 1) via x 7 1/x, som er
blitt gjort mange ganger fr

log x dx
=
(1 + x2 )2

log x dx
+
(1 + x2 )2

Z
1

log x dx
=
(1 + x2 )2

1 x2
log x dx
(1 + x2 )2

Legg n
a merke til at
Z
Z
1 x2
(1/x + x)0
1
x
dx
=

dx =
= 2
2
2
2
(1 + x )
(1/x + x)
1/x + x
x +1
Via u 7 1/x + x siden (1/u2 )0 = 1/u. Siden frste del er integrerbar virker det
benytte seg av delvis integrasjon. Dette gir
svrt fristende a
Z
0

h x log x i
1 x2
log x dx =

2
2
(1 + x )
x2 + 1 0

Z
0

dx

x
=0
x2 + 1 x
4

Siste integralet er bare arctan x, og frste leddet g


ar mot null. Nevner g
ar mot
null og teller er begrenset. Alternativt via Lhoptial s
a er
lim

x0

log x h i
1/x
lim 0
= x
x0 1/x2
1/x

Der [/] betyr at b


ade teller og nevner g
ar mot og en kan derivere. Merk at
den deriverte av teller vokser proposjonalt med log x, og teller synker proposjonalt med x3 s
a n
ar x vil uttrykke g
a mot null.
4. Begynn med substitusjonen
5. En kan legge merke til at vi kan skrive log(1 x) om til et integral som
flger
x

Z
log(1 x) =
0

du
=
1+u

Z
0

dm
m1

via substitusjonen u 7 m. Integralet kan n


a skrives om som flger
Z
I=
0

log(1 x)
dx =
1+x

Z
0

Z
0

dm dx
=
(m 1)(1 + x)

Z
0

Z
0

x du dx
(ux 1)(1 + x)

330

30 Langsvar

der substitusjonen m 7 ux ble benyttet i siste overgang. Siden x = (x + 1) 1


skrives integralet om til
Z 1Z 1
Z 1Z 1
Z 1Z 1
du dx
du dx
x du dx

=
(ux 1)
(ux 1)(1 + x)
0
0 (ux 1)(1 + x)
} |0 0
{z
}
| 0 0 {z
A

Siden vi har flgende delbrkoppspalting


1
u
1
=

(ux 1)(1 + x)
(ux 1)(u + 1)
(x + 1)(u + 1)
s
a kan B skrives som
Z 1Z
B=

Z 1Z 1
u du dx
du dx

0 (ux 1)(1 + u)
0 (1 + u)(1 + x)
0
0
Z 1Z 1
du dx
=I
0 (1 + u)(1 + x)
0
Z 1
 Z 1

du
dx
=I
0 1+u
0 1+x
= I log2 (2)

For A s
a har en
Z 1Z 1
Z 1
Z 1

X
du dx
log(1 u)
log(1 + u)
1
A=
=
du =
du =
u
u
n2
0
0 (ux 1)
0
0
n=1
skrive ut maclaurinrekka til log(x + 1) som konSiste overgang kan sees ved a
vergerer da x (0, 1). Siste sum har vrt beregnet gjentatte ganger og er
(2) = 2 /6. Eventuelt kan en legge merke til at integralet er polylogaritmen
Li2 (1), som en og har sett og regnet p
a fr. Legger vi sammen alt dette f
as
I =AB =


2
2
log2 (2) I 2I = log2 (2)
6
6

og dette fullfret beviset da


Z 1
log(1 x)
1

dx =
log2 (2)
1
+
x
2
12
0
som var det som skulle beregnes.
3.10.2
1. Vi beregner integralet begge veier
Z Z
Z
h
i
ds

st
I1 =
e
sin t dt ds =
=
arctan
s
=
2
1
+
s
2
0
Z0 Z0
Z0
Z
Z
sin t
I2 =
est sin t ds dt =
sin t
est ds dt =
dt
t
0
0
0
0
0

24.2 Oppgaver

331

vise. Hvor da selvsagt


som var det vi nsket a
Z
est sin t dt = L {sin t} (s) =

1
1 + s2

skrive om integralet via t 7 x + (u + v)/2 s


2. En begynner med a
a
Z v
Z a
(x u)n (v x)n dx =
(a t)n (a + t) dt
a

Hvor forenklingen a = (u + v)/2 ble innfrt. Benyttes n


a substitusjonen t 7
(1 2y)a s
a blir integralet
Z v
Z 1
(x u)n (v x)n dx =
(2a)n y n (2a)n (1 y)n 2a dy
u

= (u + v)2n+1

y n (1 y)n dy

Dette medfrer at vi kan skrive om summen som flger


Z v

X
1
S=
(x u)n (v x)n dx
2n+1
(v

u)
u
n=1
Z 1
X
=
y n (1 y)n dy
n=1

Siden y [0, 1) s
a vil og y(1 y) [0, 1) og en kan benytte seg av summen av
den geometriske serien.
Z 1
y(1 y)
=
dy
1

y(1 y)
0
Z 1
1
dt 1
=
1

y(1
y)
0
Z 1
1
=
2 dy 1
0 3/4 + (y 1/2)
Z 1/2
1
=
dy 1
2 + 3/4
t
1/2


2
1
= 2 tan1
1
3
3
r
4
1
=
3
3
Hvor det ble brukt blant annet at
 
Z b
dx
2
b
=
arctan
2
2
a
a
b a + x

4.

og

X
n=1

xn =

x
x [0, 1)
1x

332

30 Langsvar

m) Legg merke til at for x [0, 1] s


a er
Z
Z 1



1 xn
1
xn



dx
dx

x
n
0
0 x+1
Siden 1/n 0 n
ar n har vi fra skviseteoremet at
Z 1
xn
lim
dx = 0
n 0 x + 1
som var det som skulle vises. Den frste ulikheten kan for eksempel vised ved
kryssdele f
at xn x xn (x + 1) og ved a
ar en xn /(x + 1) xn /x.
n)

Elementrt
Z
In+1 + In =

xn+1
dx +
x+1

xn
dx =
x+1

xn (x + 1)
1
dx =
x+1
n+1

o) Ved en smart omskriving har vi


n
X
(1)k
1
1
1
1
=

+
+
k+1
0+1
1+1
2+1
n+1

k=0

= (I0 + I1 ) (I1 + I2 ) + (I2 + I3 ) + (In + In+1 )


= I0 + In+1
la n har vi
Hvor resultatet fra b) ble brukt In + In+1 = 1/(n + 1). Ved a
n
X
(1)k
lim
= lim I0 + In+1 = I0 + I = I0
n
n
k+1
k=0

Hvor resultatet fra a) ble brukt. Siden I0 =

X
(1)k
=
k+1

k=0

R1
0

x0
dx + lim
n
x+1

dx/(x + 1) = log 2 har vi


Z

xn
dx = log 2
x+1

som var det som skulle vises. Dette fullfrer oppgaven.


5. Begynn med substitusjonen
6. Tanken er a bruke en fiffig substitusjon for a vise identiten. La y 7
da er 1 y =

(1x)2
2(1+x2

og dy =

1x2
(1+x2 )2

= 2y 1/2

(1+x)2
2(1+x2 )

dx
1+x2

(1 + x)2m1 (1 x)2n1
dx
(1 + x2 )m+n
1
Z 1
n 21
1
1
1
1 1
1
2y 2 (1 y) 2
dy
=
(2y)m 2 2(1 y)

2
1+x
1 + x2
0
Z 1
= 2m+n2
y m1 (1 y)n1 dy
Z

I=

= 2m+n2 B(m, n)

24.2 Oppgaver

333

Begynner med substitusjonen x = tan u og husker p


a at 1 + tan2 u = sec2 u.
Z 1
(1 + x)2m1 (1 x)2n1
dx
2m+n2 B(m, n) =
(1 + x2 )m+n
1
Z /4
(1 + tan u)2m1 (1 tan u)2n1
=
sec2 (u) du
sec2(m+n) (u)
/4
Z /4
(cos u + sin u)2m1 (cos u sin u)2n1 du
=
/4

/4

=
/4

cos x + sin x
cos x sin x

cos

Hvor vi i siste overgang satt m = 1+cos


og n = 1cos
. Dermed s
a er s
a kan
2
2
integralet skrives som


cos
Z /4 
1+cos

1cos
cos x + sin x
1
2
2
dx
=2
J=
cos x sin x
(1)
/4

 

1 + cos
1 + cos
= 21
1
2
2

1

=2

sin 1+cos
2


=
2 sin cos2 2

som var det som skulle vises.


bruke substitusjonen x = (2n + 1)t s
7. Ved a
a kan In skrives som
Z

(2n+1)/2

In =
0

sin x
dx =
x

Z
0

/2

sin(2n + 1)t
dt
t

skrive om
Tanken er n
a at vi betrakter /2 In , og bruker likning (3.76) til a
/2. Alts
a har vi

Z /2 

1
1
Hn =
In =

sin(2n + 1)x dx
2
sin x
x
0
Videre bruker vi at f (x) = 1/ sin x 1/x, g
ar mot null n
ar x 0, slik at f (0) 0.
Funksjonen f har alts
a en kontinuerlig begrenset derivert p
a intervalet (0, /2)
og vi kan trygt bruke delvis integrasjon



Z /2
1
1
1
Hn =

cos(2n + 1)x +
f 0 (x) cos(2n + 1)x dx
sin x
x
2n + 1 0
Frste del av uttrykket g
ar mot null siden limx0 f (x) = 0 og at cos(2n+1)/2 =
la n s
a ser en at Hn 0, siden integralet er
sin n = 0, n N. Ved a
begrenset. Dette medfrer at In /2 og
Z
lim In = lim

(2n+1)/2

sin x

dx =
x
2

334

30 Langsvar

vise at Hn faktisk konvergerer kan vi for


som var det som skulle vises. For a
eksempel se at


Z /2
1



0
|Hn | =
f (x) cos(2n + 1)x dx
2n + 1 0

Z /2
1

|f 0 (x)| |cos(2n + 1)x| dx


2n + 1 0

Z /2 
1
2

1
1 dx
2n + 1 0

/2 1

2n + 1
som g
ar som O(1/n) n
ar n .
bruke substitusjonen x = (2n + 1)t s
9. Ved a
a kan In skrives som
10. Integralet kan lses p
a flere metoder, den enkle ben hinter mot at derivasjon under integraltegnet kan benyttes. Vi definerer
Z

/2

I(b) =
/2

log(1 + b sin x)
dx .
sin x

Derivererer en begge sider av denne likningen med hensyn p


a b f
as

I (b) =
b
0

/2

log(1 + b sin x)
dx =
sin x

/2

/2

/2

dx
.
1 + b sin x

(5.44)

Det siste integralet er langt enklere a beregne, og kan lses med a bruke weierstrasssubstitusjonen t = tan x/2. Da f
as
Z
Z
1
2 dt
2 dt
I 0 (b) =
=
.
2
2
2
2
1 + 2bt/(1 + t ) 1 + t
(t + b) + 1 b

bruke substitusjonen t + b = 1 b2 tan y s


Ved a
a blir integralet
Z /2
Z /2
1 b2 sec2 y dt
1

0
I (b) =
=
dy =
.
2
2
2
1b
1 b2
pi/2 (1 b )(tan y + 1)
/2
integrerer begge sider av likning (5.44) fra 0 til b
Neste steg blir a
Z

b
0

I (b) db =
0

db ,
1 b2

og siden I(b) = 0 kan en skrive venstresiden som I(b) I(0) = I(b). Hyresiden
blir s
a
Z b

I(b) =
db = arcsin b ,
1

b2
0

24.2 Oppgaver

335

siden arcsin 0 = 0. Dette var det som skulle vises, men vi har n
a vist integralet
for |b| < 1, siden substitusjonenene vi benyttet bare er gyldige for |b| < 1. Det
vise at resultatet stemmer i grensetilfellet b = 1, dersom b = 1
som gjenst
ar er a
s
a er
beEn kunne og ha brukt kompleks integrasjon og proposisjon (3.9.2) til a
regne integralet over 1/(1 + b sin x). Siden vi integrerer over [/2, /2] og ikke
(, ), s
a blir integralet halvparten av integralet over enhetssirkelen5 .
I(b) =



m
X
dx
dz
2
=
i
,
z
Res
k
1 + b(z z 1 )/2i iz
bz 2 + 2iz b

1
2

k=0

|z|=1



Nevner har to singulariter, men bare z0 = i + b2 1 /b ligger i enhetsdisken.
I(b) = i Res(z0 ) = i lim

zz0

2
i

=
=
(bz 2 + 2iz b)0
i + bz0
1 b2

som vist fr. Den


men flger direkte fra
siste overgangenvirker kanskje noe rar,
at 2bz0 = i + b2 1 = i(1 + 1 b2 ), s
a i + bz0 = i 1 b2 .

regne ut de partiellderiverte til In f


11. Ved a
as
d
In =
d

d
In =
d

n cos2 x
( cos2 x + sin2 x)n+1

n sin2 x
( cos2 x + sin2 x)n+1

Addisjon av de partiellderiverte gir da




I1 = In = n
0

(sin2 x + cos2 x)dx


= nIn+1
( cos2 x + sin2 x)2

(5.45)

hvor operatoren ble brukt. Dette gir


In1 = (1 n)In ,
skrive om likning (5.46) f
som var det som skulle vises. Ved a
as
In =
5A

1
In1
n1

vise at

/2

/2

dx
1
=
1 + sin x
2

Z
0

dx
1 + b sin x

spare plass a
gi leser en meningsfyllt fritidssyssel.
overlates til leser, b
ade for a

(5.46)

336

30 Langsvar

Bruker vi denne likningen n ganger med initalverdien I1 = /(2 ab ) f


ar en
flgende relasjon



I2 = I1 = 2
+

ab
1
I3 = I2
2 

2
2
1
2
2

+
2
+
=
1 2 2
2
2
ab

2

1
2

=
+
1 2

ab
1
I4 = I3
3

 3
3
3
1

3
2

+3
+
+ +3
=
3
2
2
3
1 2 3

ab

3

2
1

+
=
1 2 3

ab
..
.
 n


n
n
1
2

+
n
+

+
In =
n
n1
n
(1)2(3)(4)

ab

n
2

=
(1)n
+
n!

ab
for teste om dette faktisk er en lsning, kan en se om uttrykket oppfyller likning (5.46), men dette overlates til leser. En kan og uttrykke differensialoperatorene som en sum

n X
 k  X
n  
n  

n nk

n
n
+
=
=

k nk
k
k nk k
k=0

k=0

uten at dette gjr regningen noe enklere. For alle praktiske hensyn er det enklere
benytte seg av likning (5.46) for a
regne ut In . Men litt algebramagi har aldri
a
skadet noen.
13. Legg merke til at integralet er et frullani integral og fra theorem (3.8.2)
har en at
Z
f (ax) f (bx)
b
dx = f (0) log
x
a
0
Dermed s
a kan A skrives som


Z x
e
e(1)x
1
A=
dx = log
x

0
forenkle
Med f (x) = exp(x) og f (0) = 1. Fr vi setter inn kan det vre lurt a
uttrykket for A. Siden x2 + x = 1 s
a er 1/x = x + 1. Dermed s
a er 1/ = 1 + .






1

A = log
= log 1 +
= log (2 + ) = log 2 + 5

30 Langsvar

337

Der det ble benyttet at / = 1 og at = 5 . Frste likhet kommer fra


dele p
+ = , ved a
a og isolere / p
a hyre side f
as likheten. Husk at
de hyperbolske funksjonene er definert som
sinh x =

ex + ex
2

og

cosh x =

ex ex
2

legge sammen og trekke fra likningene og sette inn x = A f


Ved a
as flgende
likningsett

sinh A + cosh A = eA = 5 + 2

sinh A cosh A = eA = 5 2
ta gjennomsnittet av likningene f
Ved a
ar en et uttrykk for sinh A, subtraksjon
gir cosh A s
a

sinh A = 2 og cosh A = 5
Hvor exp(A) ble regnet ut p
a flgende m
ate

exp(A) = [exp(log 2 + 5 )]

1
5 2
5 2

=
=
54
5 +2 5 2

sette rett inn i de hyberbolske funksjonene, men


Det er selvsagt fullt mulig a
undertegnede er dessverre allergisk mot brker.
regne ut I er ikke spesielt vanskelig, og kan gjres via substitusjonen
15. A
x = sin u. Vi viser frst at


21 12


=
=
sin(/2)
12 + 12
omforme integralet bruker vi u = 1 x2 s
Dette kan vi sette inn i I, for a
a
du = 2x dx som gir
Z

dx =
2
2
1x
0


Z 0
1 12 21
1

=
du =

2 12 + 12
1 2 u 1u


Z 1
21 12
11/2
11/2

=
u
(1 u)
du =
12 + 12
0

I=

anta at
p
a bakgrunn av dette s
a kan det vre rimelig a
Z
B(x, y) =
0

tx1 (1 t)y1 dt =

(x)(y)
.
(x + y)

gi leser en pustepause og for a


la
Denne siden er med hensikt blank, for a
forfatter sl
ass mot dinosaurer.

BIBLIOGRAFI

339

Bibliografi
[] Zafar Ahmed, Knut Dale, and George Lamb. Definitely an integral: 10884.
The American Mathematical Monthly, 109(7):670671, 2002.
[] T. Ambderberhan, M. L. Glasser, V. H. Moll M. C. Jones, R. Posey, and
D. Varela. The cauchy-schl
omilch transformation. http://129.81.170.
14/vhm/papers_html/schlomilch-sub.pdf, 1999.
[] James Bonnar. The Gamma Function. CreateSpace Independent Publishing
Platform, 2 edition, Nov 2013.
[] J. W. Brown. An extension of integration by parts. The American Mathematical Monthly, 67(4):327, 1960.
[] Philip J. Davis. Leonhard eulers integral: A historical profile of the gamma
function. The American Mathematical Monthly, 66(10):849869, 1959.
The development of the laplace transform,
[] Michael A. B. Deakin.
17371937 i. euler to spitzer, 1737-1880. Archive for History of Exact
Sciences, 25(4):343390, jan 1981.
The development of the laplace transform,
[] Michael A. B. Deakin.
17371937 ii. poincar to doetsch, 18801937. Archive for History of
Exact Sciences, 26(4):351381, 1982.
[] Richard P. Feynman.
Company, Inc, 1985.

Surely Youre Joking, Mr.Feynman! WW.Norton

[] I. S. Gradshteyn and I. M. Ryzhik. Table of Integrals, Series, and Products.


Academic Press, 7 edition, 2007.
[] Victor. H. Moll. The integrals in gradshteyn and ryzhik. part 2: elementary
logarithmic integrals. Scientia, (14):715, 2007.
[] Roger Nelsen. Symmetry and integration. The College Mathematics Journal,
26(1):3941, jan 1995.
[] Erin Pearse. Math 209d - lecture notes. http://www.math.cornell.edu/
erin/analysis/lectures.pdf, 1999.
[] J.L. Raabe. Angen
aherte bestimmung der factorenfolge 1.2.3.4.5....
n=...(1+n)=...xne-xdx, wenn n eine sehr grosse zahl ist. Journal f
ur
die reine und angewandte Mathematik, 25:146159, 1843. URL http:
//eudml.org/doc/147170.

340

BIBLIOGRAFI

[] Walter Rudin. Principles of Mathematical Analysis. International Series in


Pure and Applied Mathematics. McGraw-Hill Higher Education, 3 edition,
Sep 1976.
[] Walter Rudin. Real and Complex Analysis. International Series in Pure and
Applied Mathematics. McGraw-Hill Science/Engineering/Math, 3 edition,
May 1986.
[] David Sirajuddin. Fresnel integrals. http://www.itcanbeshown.com/
integrals/Fresnel%20Integrals/fresnel_integrals.pdf, 2008.
[] Steve Smith.
Integration by parts, the tabular method, ii.
http:
//chelseas-roost.co.uk/resources/maths/DIS_II.pdf, 1999.
[] Daniel Zwillinger. The Handbook of Integration. A K Peters/CRC Pres, 1
edition, Nov 1992.

You might also like